Is project management a profession? - PTMC/APMX Building Project ...

0 downloads 192 Views 3MB Size Report
In Greece around the 5th century B.C. there was a rise in a new class of healer. ...... Within 15 minutes, the auto repa
Is project management a profession? If yes, where does it fit in and if not, what is it?

A thesis submitted for the degree of Doctor of Philosophy in Strategy, Programme & Project Management

June, 2003 to August 2007 by

Paul D. Giammalvo CDT, PMP, CCE, MScPM, PhD Candidate [email protected] ESC-Lille University Under the Supervision of Professor Christophe N. BREDILLET PhD, D.Sc., MBA, Ingénieur EC Lille, CPD IPMA Level A, PRINCE2 Practitioner, CCE Professor of Strategy, Programme & Project Management Dean Postgraduate Programmes ESC Lille – Lille School of Management Avenue Willy Brandt 59777 Euralille – France Phone: +33 3 20 21 59 72/73 – Fax: +33 3 20 21 59 74 Email: [email protected]

Table of contents List of exhibits List of tables List of figures Acknowledgments Declaration Abstract

viii vix x xii xiii xiv

CHAPTER 1

Introduction

1

1.1 1.2 1.3 1.4

1 2 3 4

Introduction Context of the study The nature of the problem Statement of the research problem

1.5 The objectives of the study 1.6 Methodology 1.6.1 1.6.2 1.6.3

5 6

Research design overview Professions used for benchmarking Project phases

1.7 Stakeholder analysis 1.7.1 1.7.2 1.7.3 1.7.4 1.7.5 1.7.6 1.7.7 1.7.8

15

Stakeholder meetings/discussions Consumers Society Professional organizations Employers Academics/institutions of higher learning Consultants Practitioners

1.8 The significance of the study 1.9 Definitions of the key terms 1.10 Organization of the thesis

22 23 30

ii

CHAPTER 2

Literature review and preliminary research

33

2.1 Introduction: Considering the nature of professions 2.2 The concept of a profession: An historical and semantic perspective

33 34

2.2.1 2.2.2 2.2.3 2.2.4

The concept of professional project managers in the professions Evolution of the concept of professionalism Researching words and their meanings General and dictionary definitions

2.3 The concept of a profession: Legal definitions 2.3.1 2.3.2 2.3.3

51

Legal definitions Fiduciary responsibility defined and explained ‘Legal no man’s land’ in defining a profession

2.4 The concept of a profession: Sociological definitions 2.4.1 2.4.2

66

Sociological definitions Sociological models

2.5 The concept of a profession: Economic definitions 2.6 The concept of a profession: Semantic definitions 2.6.1 2.6.2

72 74

Semantic definitions Equitable estoppel as a special case definition

2.7 Definitions summarized

79

CHAPTER 3A

Case study 1: Construction project management

85

Introduction to the case studies 3A.1 Case Study 1: Construction project management

85 86

3A.1.1 The concept of construction project management (CM) 3A.1.2 History of construction project management 3A.1.3 Construction project management: Is it a profession?

3A.2 Summary of case study 1: Construction project management 3A.3 Recommendations to improve the standing 3A.4 Summary

120 121 126

CHAPTER 3B

Case study 2: Commercial airline piloting

127

3B.1 Commercial airline pilot: A project manager? 3B.2 Comparison and preliminary evaluation: Commercial aircraft pilot

127 154

3B.2.1 Cruciality: Need is important/need is immediate 3B.2.2 Dénouement: Practitioner is trusted to produce positive results relatively quickly 3B.2.3 Long period of training including higher education 3B.2.4 Subscribe to a code of ethics 3B.2.5 Service to the public, including pro bono work 3B.2.6 Publishing in learned journals 3B.2.7 Advertising not permitted or restricted 3B.2.8 Use of title restricted by law

iii

3B.3 Comparison and preliminary evaluation: Pilot and construction project manager comparison summarized

156

3B.3.1 3B.3.2 3B.3.3 3B.3.4 3B.3.5 3B.3.6 3B.3.7

Fiduciary responsibility to the public Professional association defines best practices Identify with the occupation and not the employer Held in high esteem by the community Serving a formal apprenticeship or internship License required to fly a plane Has a body of knowledge which is complicated, secret, abstruse, esoteric or unique 3B.3.8 Requires liability insurance to practice 3B.3.9 Uses symbolic costumes or uniforms 3B.3.10 Requires highly ritualistic processes CHAPTER 4

Survey development and analysis, Part 1

164

4.1 Purpose and design of the survey 4.2 The participants: A demographic survey

164 165

4.2.1 4.2.2 4.2.3 4.2.4 4.2.5 4.2.6 4.2.7 4.2.8 4.2.9

Geographic area of origin Age Gender Education Licenses/certifications held Experience Salary Industry sector Job title

4.3 Other survey questions: Towards a professionalization index 4.3.1 4.3.2 4.3.3

173

The foundation questions General survey questions Summary: The purpose and development of the survey questions

4.4 Summary of Chapter 4

186

CHAPTER 5

Survey development and analysis, Part 2

187

5.1 Analysing answers to the key questions

187

5.1.1 5.1.2

5.1.3 5.1.4 5.1.5 5.1.6 5.1.7 5.1.8

Key question 1 Is project management a profession? Key question 2 Is there a continuum and where on that continuum of professionalization; if so, where does project management lie on the continuum? Key question 3 What is the actual numeric value for each attribute by respondents? Key question 4 Is the project management body of knowledge unique? Key question 5 How much experience/apprenticeship/education is associated with the term profession? Key question 6 Is project management a calling? Key question 7 How do project managers perceive the practice of individual ethics? Key question 8 How do project managers compare against other professionals in terms of pro bono work?

iv

5.1.9 5.1.10 5.1.11 5.1.12 5.1.13 5.1.14 5.1.15 5.1.16 5.1.17

5.1.18 5.1.19

5.1.20 5.1.21

5.1.22

5.1.23

Key question 9 How do organizations representing practitioners compare against other organizations in terms of ethics? Key question 10 Do project managers publish papers in peer reviewed journals? Key question 11 Does advertising by project managers conform to other professional norms? Key question 12 Is (or should) use of the title ‘project manager’ be restricted by law? Key question 13 Are symbolic costumes, uniforms or other identifying insignia necessary to define a profession? Key question 14 Should project managers be licensed? Key question 15 Should project managers be required to carry professional liability insurance? Key question 16 Given autonomy in decision making is a key attribute of a profession, do project managers have autonomy in making decisions? Key question 17 Given identifying with the occupation and not with an employer is a key attribute of a profession, who do project managers identify with? Key question 18 Those in existing professions enjoy high community esteem. What is the community esteem of project managers? Key question 19 Those in existing professions earn higher than average salaries. What is the perception of project managers in terms of the salaries they earn? Key question 20 A leading intrinsic attribute for a profession is the mystique which surrounds it. Does project management have mystique? Key question 21 A leading intrinsic attribute for a profession is the cruciality (criticality) of the services. Does project management have criticality? Key question 22 A leading intrinsic attribute for a profession is the dénouement or the ability to produce positive results very quickly and without limited involvement of the consumer of the services. Does project management have dénouement? Key question 23 How does the fiduciary responsibility of project managers compare against that of established professions?

5.2 Summary of key question results

275

CHAPTER 6

Discussion

292

6.1 6.2 6.3 6.4 6.5

292 292 295 301 305

Introduction Summary of quantitative results Summary of qualitative results compared against case studies Weight adjusted professionalization index Considering the attributes on the whole Is project management a profession? What is it?

v

CHAPTER 7

Conclusion

309

7.1 Introduction 7.2 Limitations of this research

309 311

7.2.1 7.2.2 7.2.3 7.2.4 7.2.5

Lack of significant demographic differences Bias in the key word analysis of the body of knowledge More research on the effectiveness of licensing Development of a true ratio scale Data mining to research ethics questions

7.3 Reflections 7.3.1 7.3.2 7.3.3 7.3.4 7.3.5

313

Project management: Not a profession Observations from the literature For those stakeholders who believe project management is NOT a profession For those who believe project management IS a profession For those who believe it doesn’t matter

References

322

Appendices

336

vi

Dedicated to my parents Evelyn Meldrum Giammalvo and the late Peter Giammalvo

Thanks for everything, Mom and Dad.

vii

List of exhibits Exhibit 2.1

Examination of words used to define the concept of profession

47

Exhibit 3A.1 Primary traits of a profession

93

Exhibit 3A.2 CMAA requirements for certification

98

Exhibit 3B.1 The Airline Pilot

134

Exhibit 3B.2 The Aloha Airlines incident, 1988

152

Exhibit 5.1

Generation of survey question 28

187

Exhibit 5.2

Generation of survey questions 30/31, 39-/42, and 47.

192

Exhibit 5.3

Generation of survey question 45/46

208

Exhibit 5.4

Generation of survey question 15 to answer key question 4

210

Exhibit 5.5

Generation of survey questions 34, 35, 36 and 43 to answer key question 5

211

Exhibit 5.6

Generation of survey questions 16, 20 and 24 to answer key question 6

217

Exhibit 5.7

Generation of survey question 27 to answer key question 7

220

Exhibit 5.8

Generation of survey question 29 to answer key question 8

222

Exhibit 5.9

Key question 9

224

Exhibit 5.10 Generation of survey question 32 to answer key question 10

234

Exhibit 5.11 No survey question for KQ11

236

Exhibit 5.12 Key word analysis to answer key question 12

239

Exhibit 5.13 Generation of survey question 17 to answer key question 13

241

Exhibit 5.14 Generation of survey questions 37, 38 and 44 to answer key question 14

242

Exhibit 5.15 Generation of survey questions 18 and 26 to answer key question 15

245

Exhibit 5.16 Generation of survey questions 19, 25, 27 to answer key question 16

249

Exhibit 5.17 Generation of survey question 27 to answer key question 17

253

Exhibit 5.18 Generation of survey question 27 to answer key question 18

255

Exhibit 5.19 Generation of survey question 22 to answer key question 19

258

Exhibit 5.20 Generation of survey question 15 to answer key question 20

261

Exhibit 5.21 Generation of survey question 33 to answer key question 21

264

Exhibit 5.22 Generation of survey questions 19, 25 and 26 to answer key question 22

266

Exhibit 5.23 Generation of survey questions 18, 19, 25 and 26 to answer key question 23

271

Exhibit 6.1

Summary of average scores from survey respondents

294

Exhibit 6.2

The 22 attributes of a profession as identified during the literature research

295

Exhibit 6.3

Comparison between survey responses, construction project managers and airline pilots

296

Weight adjusted professionalization index score

304

Exhibit 6.4

viii

List of tables Table 2.1

Summary of legal definitions of a profession

64

Table 2.2

Impacts of restrictions on QUALITY of services

73

Table 2.3

Impacts of restrictions on COST of services

74

Table 2.4

Attributes of a profession ranked by source and relative scores

82

Table 2.5

22 attributes commonly associated with a profession

84

Table 3A.1 Time line of the history of construction management

89

Table 3A.2 Typical CM curriculum

95

Table 3A.3 Summary of CM attributes

120

Table 3B.1 Summary of commercial airline pilot ranking mapped against attributes of a profession

153

Table 3B.2 Ranking of CM and commercial aircraft piloting compared

157

Table 4.1

Foundation questions in the survey

174

Table 4.2

22 attributes commonly associated with a profession (based on the review of the literature)

177

Table 4.3

Survey question 45/46

178

Table 4.4

Development of the survey questions, designed to generate data for answering the research questions (demographics excluded)

179

Table 5.1

Attributes rank ordered showing actual values (weight) from survey

207

Table 5.2

Source of the major elements of the project management BoK

210

Table 5.3

Frequency of key words as a percentage of total words in the code of ethics

225

Table 5.4

Key words as a percentage of total words in codes of ethics

226

Table 5.5

Comparison of codes of ethics

238

Table 5.6

Codes of ethics sorted by frequency of words and ratio of key words to total words

270

Summary of interpreted qualitative and quantitative results from survey data

276

Table 5.7

ix

List of figures Figure 1.1

Survey wording offering incentives for participating

11

Figure 1.2

Geographical roots of the respondents

12

Figure 3A.1

Esteem question results from the survey by - project managers

115

Figure 3A.2

Salary survey by sector

116

Figure 3A.3

Job satisfaction by salary

116

Figure 4.1

Results from survey question 14

166

Figure 4.2

Results of survey question 6

167

Figure 4.3

Results from survey question 7

167

Figure 4.4

Results from survey question 8

168

Figure 4.5

Results from survey question 9

168

Figure 4.6

Results from survey question 10

169

Figure 4.7

Results from survey question 11

170

Figure 4.8

Graph showing salary as a function of work experience (AACE, 2005)

171

Figure 4.9

Results from survey question 12

171

Figure 4.10

Results from survey question 13

172

Figure 4.11

Normalized salaries by job function (ACCE, 2005)

173

Figure 4.12

Generation of a survey question (excluding demographics)

178

Figure 5.1

Results from survey question 28

189

Figure 5.2

Survey question 28, responses plotted on an interval scale

190

Figure 5.3

Results of survey question 30/31; relative ranking of occupations compared against recent college graduate responses plotted showing mean, and range, +/1 sigma 193

Figure 5.4

Professionalization continuum developed from data in response to survey question 30/31; interval scale showing relative relationships between licensed occupations and project managers 194

Figure 5.5

Question 39-/42, professionalization continuum: Interval scale showing relative relationships between licensed professions and project managers, by applying Thurstone’s Law of Comparative Judgement

197

Applying the bias shift from SQ39-/42 against the results of SQ47, illustrates the interim step to eliminate bias from the results of both SQ 30/31 and SQ 47.

197

Figure 5.7

Question 47, responses plotted showing mean, and range, +/-1 sigma

198

Figure 5.8

Question 47 illustrating bias adjustment; professionalization continuum: Interval scale showing relative relationships between licensed occupations and project managers, adjusted for bias using Thurstone’s Law. (reading left to right, least professional 21.0 to most professional at 1.0)

196

Figure 5.9

Questions 30/31, 39-/42 and 47 compared BEFORE adjustment for bias

201

Figure 5.10

Questions 30/31, 39-42 and 47, compared AFTER adjustment for bias

203

Figure 5.11

Results for survey question 45/46, attributes rank ordered showing +/-

206

Figure 5.12

Results, survey question 34

213

Figure 5.13

Results, survey question 43

213

Figure 5.14

Results, survey question 35

214

Figure 5.15

Results, survey question 36

214

Figure 5.16

Results, survey question 16

219

Figure 5.17

Results, survey question 20

219

Figure 5.18

Results, survey question 24

219

Figure 5.6

x

Figure 5.19

Results, survey question 27

220

Figure 5.20

Results, survey question 29

223

Figure 5.21

Results, survey question 32

236

Figure 5.22

Results, survey question 17

242

Figure 5.23

Results, survey question 44

243

Figure 5.24

Results, survey question 37

243

Figure 5.25

Results, survey question 38

244

Figure 5.26

Results, survey question 18

247

Figure 5.27

Results, survey question 26

248

Figure 5.28

Results, survey question 19

250

Figure 5.29

Results, survey question 25

250

Figure 5.30

Results, survey question 26

251

Figure 5.31

Results, survey question 16

254

Figure 5.32

Results, survey question 20

254

Figure 5.33

Results, survey question 24

254

Figure 5.34

Results, survey question 21

256

Figure 5.35

Results, survey question 22

258

Figure 5.36

Salary by industry sector

259

Figure 5.37

Salary by project type

260

Figure 5.38

Job satisfaction by salary, all respondents

260

Figure 5.39

Job satisfaction by salary, project managers only

240

Figure 5.40

Results, survey question 23

263

Figure 5.41

Results, survey question 33

266

Figure 5.42

Results, survey question 19

268

Figure 5.43

Results, survey question 25

269

Figure 5.44

Results, survey question 26

269

Figure 5.45

Results, survey question 19

273

Figure 5.46

Results, survey question 25

273

Figure 5.47

Results, survey question 26

274

Figure 5.48

Three phases of the research

275

xi

Acknowledgments Despite the quest for a PhD being more or less an individual effort, no great undertaking can be accomplished without the support and encouragement from a broad spectrum of players. First thanks go to my parents, who not only gave me the genetic capabilities, but also instilled in me a love of learning for learning’s sake. Thanks Mom and Dad!! Took me awhile, but it looks like I may have finally made it!! As a PhD is an undertaking not everyone does and, among those who do, is usually a journey undertaken only once; and having a guide or mentor to show the way is a must. I have depended on the guidance, mentoring and advice of my Supervisor, Professor Christophe Bredillet and Research Advisor, Professor Philippe Ruiz. Thanks, Christophe and Philippe!! Your guidance, wisdom, advice and patience was sincerely appreciated!! Even with the best intentions and highest enthusiasm, there are days when it just doesn’t seem worth the effort. For keeping my motivation and spirits up, and for never losing faith in my ability to complete this task, I owe a debt of gratitude to my wife, Cheryl Carl. Cheryl, looks like I no longer have an excuse to postpone that book you’ve been wanting. Writing a PhD consumes a lot of time and money. And a good portion of this time came at the expense of potentially ‘billable time’. For her patience and support, I owe a debt of gratitude to my boss, Ms. Yani Suratman, President Director of PT Mitratatata Citragraha. Yani, thanks for providing the funding for this project and I hope having my PhD will generate a fair return on your investment, in terms of both time and money. With all those kids to educate, I can’t very well let you down. Last, but certainly not least, is my editor, Ms. Barbara Brougham. Wow!!! You took an ugly duckling and made it into a swan. A true “miracle worker” with words. If I get my PhD, it will because of your talents with words as much as my research. Others who played key supporting roles: „

PTMC Office Manager, Ms. Sustu C. Inastu, (Ms. Tutu) who helped with all the logistics

„

Mr. Ramadhani Achdiawan, (Dhani) who was my mentor and advisor in using SPCC

„

R. Max Wideman, who read over the original works and who’s Glossary I relied on so heavily

To one and all, and anyone I may have forgotten, a most sincere THANK YOU!

xii

Certificate of authorship/originality I certify that the work in this thesis has not previously been submitted for a degree nor has it been submitted as part of requirements for a degree except as fully acknowledged within the text. I also certify that the thesis has been written by me. Any help that I have received in my research work and the preparation of the thesis itself has been acknowledged. In addition, I certify that all information sources and literature used are indicated in the thesis.

Paul D. Giammalvo

Dated: 1 July, 2007

xiii

Abstract The research described in this thesis has been a journey of exploration and discovery. Using a combination of positivist and post positivist approaches, both traditional and non-traditional definitions of a profession and professional have been identified; various hypotheses related to these definitions have been tested using a survey instrument; data collected from over 400 respondents has been analysed in order to understand the nature of the practice of construction management, which has failed to professionalize itself after almost 50 years of formalized practice. In conducting the research on what it means to be a ‘professional’, commercial aircraft piloting, which has in less than 100 years been accepted as a profession, was closely examined to determine what set it apart from project management. From this and other disparate explorations, the research has identified the areas of agreement, the areas of divergence, and the areas still in question regarding the past and future of project management, and from this examination distilled answers or recommendations and provided a glimpse into the future of project management. Amongst the highlights of the findings is that project management is not, by and large, regarded to be a profession. It is considered by the majority of its practitioners to be a process, methodology or system. The key attributes required to professionalize project management are not licensing, but creating a methodology, system or process which, done correctly and appropriately, can consistently deliver ‘successful’ projects and build trust in the consuming public in the competencies of the practitioners to apply these methodologies, systems or processes to deliver projects on time, within budget while substantially meeting the needs, wants an expectations of the stakeholders. From the research, a Professionalization Index has been developed that can provide a snapshot of where project management, as perceived by the practitioners, lies on the path to professionalization, when compared against other occupations. Key words: profession, professional, project, management

xiv

Is project management a profession? If yes, where does it fit in and if not, what is it?

CHAPTER 1

Introduction 1.1

Introduction

As practitioners of any knowledge based occupation transition into an integrated and virtual world, the question of their occupation being considered a ‘profession’ is becoming increasingly important, as every practitioner wants to shed the image of the work they do being ‘just another job’ and reap the prestige, benefits and potential rewards, if not the responsibilities, liabilities and risks, associated with belonging to a recognized and established profession. This trend towards ‘professionalization’ of many basic occupations is readily observed (Wilensky, 1964), evidenced by garbage collectors wanting to be called sanitary engineers, housewives referring to themselves as domestic engineers, librarian’s seeking licensure, nurses and other medical care specialists seeking equal footing with doctors, and construction project managers seeking equal standing with architectural and engineering professionals. Fortune magazine called project management ‘the career choice of the 90’s and beyond’ (Stewart, 1996). The largest professional organization representing project managers, the US based Project Management Institute (PMI), unabashedly refers to the ‘profession’ of project management. And PMI is not alone. Other organizations representing the practice also refer to project management as a profession. Should we take the approach of Dorothy, in the Wizard of Oz, however, and believe that simply closing our eyes and wishing hard will turn the practice of project management into a profession? Or is there some standard or criteria which need to be met in order to legitimize this claim? J. Rodney Turner and Ralf Muller, writing in the International Journal of Project Management (2003), noted: If project management were recognized as a profession, and the use of the title Project Manager restricted to those individuals in possession of related professional certification, it would help in the selection of project managers, giving greater confidence in their competence and trustworthiness.

This research has been designed to explore these paradigms and discern the difference between rhetoric and fact.

1 Chapter 1: Introduction

1.2

Context of the study

Project management is an integral part of mankind’s evolution: the great pyramids of Egypt and the seven wonders of the ancient and modern world; the construction of massive cathedrals in the UK and Europe by the Freemasons; the Panama Canal; putting a man on the moon; 100+ story buildings. The list of unique, one time events linked to achieving a long term objective or goal is almost endless, providing persuasive evidence that initiation, planning and execution of projects may some how ‘hard wired’ into the human psyche. Further evidence of the human inclination to organize, to plan, to put things in order can be seen in our individual development. To any parent, toilet training of their young child is a major project, the completion of which is much anticipated and greeted with joy and relief. Being able to tie one’s shoes is generally considered a prerequisite to enter elementary school. Other major ‘projects’ inherent in most people’s lives include: graduating from high school; obtaining one or more university degrees; getting married; purchasing one or more houses; and, consistent with the concept of life cycle (which plays such an important role in project management), the final project, one’s funeral. Citing these examples as representative of the idea of ‘project’ means there are over six billion ‘project managers’ in the world today. But certainly we can’t compare learning to tie one’s shoes to putting a man on the moon, which means that some differentiator must exist which separates those who are ‘professional’ project managers from those who are ‘amateurs’ or even marginally competent. Can the differentiator be salary? Status in the community? Size of the project managed? Importance of the project to society? Complexity of the project? What attributes, factors, combinations or permutations differentiate the merely good from the great project manager? Furthermore, since projects must be managed in virtually all occupational specialties, and exist within all sectors, how do we differentiate between the expertise in the underlying technical specialty (i.e. medicine, engineering, biology or science) and the ability to initiate, plan, execute, control and close projects? Is competency in project management the differentiator or does the ability derive from or inherent in the mastery of the underlying technical specialty?

2 Chapter 1: Introduction

1.3

The nature of the problem

Coming from a background in construction management, I had always considered my profession to be civil engineering and the discipline or specialty within the field of civil engineering to be the management of technical and tradespeople to complete projects in the built environment. To belong to or read about professional organizations claiming that project management was or should be considered a separate profession simply didn’t feel right. Preliminary investigation prior to formally starting this research, in fact, indicated a cacophony of mixed and conflicting messages about the concept of project management, coming not only from practitioners but, more importantly, from those organizations purporting to represent the voice of the practitioner. The largest of these organizations, the US based Project Management Institute (PMI), states in their Policy governance manual that ‘Project management is a profession’ (PMI, 2000). Yet in a study partially funded by PMI, Bill Zwerman, Janice Thomas, Susan Haydt and Terry Williamson (2004, p. 195) concluded in part that project management is ‘unlikely to ever become recognized as a stand alone profession’ (Zwerman, Thomas, Haydt & Williamson, 2004, p. 195). No less an authority than Peter Drucker (1999) states ‘management is the distinguishing organ of any and all organizations’ (pp. 7, 8), meaning that the fundamental skills and principles of managing people to produce goods and services are necessary regardless of the application. And Henry Fayol (1916), in his Administration industrielle et generale (Industrial and general administration), almost perfectly describes today’s project management body of knowledge, yet does so in the context of general management, not project specific. To complicate the picture more, the highly regarded International Council on Systems Engineering (INCOSE) (n.d., online) describes systems engineering (which in large part forms the basis for project management) as: an engineering discipline whose responsibility is creating and executing an interdisciplinary process to ensure that the customer and stakeholder’s needs are satisfied in a high quality, trustworthy, cost efficient and schedule compliant manner throughout a system’s entire life cycle. (INCOSE, online)

3 Chapter 1: Introduction

To illustrate further just how divided the practicing community is, consider the comment from Peter Morris (2004, n.d., online) on ‘Re-thinking project management’ where he describes project management as a discipline which remains ‘stuck in a 1960s time warp’. At the same time the debate rages about whether project management deserves to be recognized as a stand alone profession, one only has to look in newspapers or listen to the nightly news to see evidence that project management, as practiced today, is far from deserving of anything but derision and scorn. The larger and more complex the project, it seems the more likely it is to run into problems. Boston’s ‘Big Dig’, Ford Motor’s attempts to computerize their procurement process, the Hubble telescope, all stand as evidence that something is wrong. While one may say that the news industry thrives on ‘bad news’ and is only reporting those projects which have run amok, academically rigorous research by the Standish Group, Gartner Group, FMI and others seems to support claims that project management, as it is currently being practiced, is far from as effective as professional project management organizations would like potential clients to believe. Certainly the kind of consistent and predictable performance which would justify or earn the right of an occupation to be called a profession appears to be lacking. Upon what basis, therefore, could or should project managers or those organizations representing them, base any claims that the practice of project management qualifies for or deserves recognition as a profession? A clear lack of consensus in response to this question on the part of some practitioners, as well as those organizations claiming to represent their views, along with highly respected leaders in the field of management, raise uncomfortable questions about project management as the best way to manage a project, much less as worthy of status as a ‘profession’.

1.4

Statement of the research problem

The stage was set for the research reported in this thesis by the apparent discrepancies between what is being touted by professional organizations (that project management is a profession); some practitioners (who honestly believe themselves to be professional practitioners); and the perception of the consuming public, (especially in the public sector), that project management simply is not delivering value for money. 4 Chapter 1: Introduction

This perception is supported at least in part by credible qualitative and quantitative research. The current study was designed to explore these apparent discrepancies and reach some conclusions about whether or not project management is or should be a considered a profession, and if not, what can or should be done to raise the credibility of the practice and professional image of the practitioner.

1.5

The objectives of the study

The five primary objectives of the research were to: 1) answer the question Is project management a profession? 2) determine what project management is, if not a profession 3) ascertain where project management stands in relation to other emerging occupations claiming to be professions 4) determine what can be done, if anything, to enhance the image of the occupation 5) decide what, if anything, can be done to improve the delivery of successful projects. Answers to these questions will provide a guide as to where the practice of project management is vis a vis other occupations, and will serve as a beacon or guiding light to help practitioners identify what needs to be done to improve both the success and the image of project managers. While much research in this area is qualitative, one of the intended contributions of the current study was to create a model which can translate qualitative attributes or perceptions into numerical values, not only to examine project management, but to gauge the professional quotient of other occupations when compared to one another. This research was not intended to be prescriptive, although it does conclude with some recommendations. The aim was to highlight or illuminate issues for further research and consideration, and to develop a methodology to quantify relative professionalization.

5 Chapter 1: Introduction

1.6

Methodology

1.6.1 Research design overview The underlying philosophy driving the research is based on the understanding that practitioners are moving from a constructionist to a post-constructionist world. As with any period of great change, the zone of transition is beset by seemingly chaotic events. Recognizing the implication and limitations chaos has in applying the scientific method alone, the research methodology was designed to take advantage of not only qualitative and quantitative research. Thus, although planned, this research project also evolved, very much as the very occupational specialty being researched has evolved. Stated another way, the methodology chosen to do the research was designed to mirror the subject of the research as closely as possible. As with any other project, ‘scope creep’ was inevitable. In this case, the increased scope was caused by the identification early in the data collection of a reality characteristic of project management that ultimately had to be addressed when considering the evidence for accepting the occupation as a stand alone profession: unlike other, more identifiably independent occupations, project management exists as an integral part of most, if not all, existing occupations considered to be ‘professions’. It is an essential life skill (PMI, n.d., online) which everyone must master to some degree in order to survive as a human being. Therefore, how can project management ever be considered as a profession unto itself? Would any data gathered during this research support project management as an independent and singularly identifiable profession? To answer these questions first required that the attributes normally associated with established professions be identified. Once identified, the tactic was to quantify where project management stood in relation to the benchmarks of the attributes of the established professions. Every effort was made to look at project management from the perspective of both those practitioners who identify themselves as project managers as well as those who perform the same functions as project managers yet do not recognize or consider themselves to be project managers. The method behind this current study was influenced by the overlapping juncture between the sociologic (Shimberg, 1982; Young, 1987; Kimball, 1995; Maister, 6 Chapter 1: Introduction

1997; Zwerman & Thomas, 2004), legal (Polelle, 1999; Spaulding, 2003; Gawley, 2002; Matter of Freeman, 34 NY2d 1, 9-10 [1974]), economic (Cox & Foster, 1990; Donaldson, 1982; ‘An economic review and analysis of the implications of occupational licensing’, 1999), and semantic (Marutello, 1981; Haga, 1974, pp. 3-10; Goode, 1969) perspectives that were used to determine if and how a true ‘profession’ can be differentiated from an ordinary occupation. 1.6.2 Professions used for benchmarking It was complicated by the fact the research had the potential to include just about all existing professional occupations. To simplify the research, therefore, and yet make it relevant in the global context, the only occupations targeted for inclusion in the research were those requiring professional licensure in the top third of OECD countries. The basis for selecting the top third of OECD countries lay in the premise that in the developing nations, ‘licensure’ is often used as a means of extortion that has little or nothing to do with protecting the public or other attributes associated with defining a profession, and everything to do with collecting ‘economic rents’. Initial qualitative research identified nine occupations that are licensed in all or near all of the top third of OECD countries (OECD, n.d., online). These are the professions or trades targeted for comparative research: „

commercial airline pilots

„

medical doctors

„

real estate agents

„

lawyers

„

architects/engineers

„

accountants

„

building contractors

„

electricians

„

plumbers

The reason for including several trades in the mix is due to the fact that historically, the professions evolved from the trades (guilds). As one of the tests of professionalizm involved ascertaining whether professions are static or evolve over time, it was important to quantify just where project management lies on an interval 7 Chapter 1: Introduction

scale between what is generally accepted to be trades and those occupations already considered to be professions. As will be discussed in the section on Methodology, it was anticipated that a meaningful zero point could be established, turning the interval scale into a valid ratio scale. Another important reason for choosing occupations normally and customarily licensed by the top third of OECD countries lay in the fact that while licensing in the developed nations tends to be done (at least ostensibly) to protect the health, safety and well being of the consuming public, in many of the developing nations, occupational licensing is a means to extort economic rents from practitioners with little or no regard for anything but lining the pockets of government officials. This matter is of considerable concern, for in many of the developing nations, credentials such as the PMP are often manipulated by local officials as a means to extort money, a form of economic rent seeking (Naritomi, J., Soares, R. & Assunçäo, J., 2007), through monopolistic control of the training, application and/or testing process. 1.6.3 Project phases Given research by most definitions qualifies as a project, consistent with the core tenets of project management; this project was executed by taking a phased approach. This research project was broken down into four phases: 1 qualitative research in the form of literature review to determine the attributes common to professions 2 quantitative research to determine the relative professionalization of project management as an occupation compared against other, more established occupations already considered to be professions 3 qualitative and quantitative research to better define what project management is 4 interpreting the data and writing the thesis. PHASE 1

Extensive literature reviews established that while research existed on the professionalization of various occupations from a sociologic, economic or legal perspective, there was little or no research that tried to combine all three perspectives into a single, comprehensive set of metrics or attributes against which the ‘degree of

8 Chapter 1: Introduction

professionalization’ of an occupation could be quantified, measured and evaluated. The literature search led to the distillation of 19 extrinsic attributes common to those occupations currently considered in socio-economic and legal terms to qualify as professions (see Table 2.4, Chapter 2). However, the literature research turned up yet another perspective that provided an interesting alternative method of evaluating or determining a profession. Solely extrinsic attributes could be mediated by intrinsic factors that cause people to accord any given occupation the status of being a profession. Further literature search turned up another set of factors, less mainstream but certainly having validity, that argued that for any occupation to be considered as a profession, practitioners must be perceived by their fellow men and women as: „

having a certain ‘mystique’ acquired by having mastered a complex, unique, difficult or obscure body of knowledge

„

offering services that were crucial in terms of immediacy of need as well as the importance of the services being rendered

„

acting always in good faith to apply a mastery of a complex body of knowledge to effect immediate, significant and measurable relief or improvement in the situation (Marutello, 1981).

Thus the intrinsic attributes of a profession (see Table 2.4, Chapter 2) provide a counterbalance to the more ‘traditional’ or extrinsic metrics against which professions are determined and measured. This current research appears to be unique in that it is the first attempt to move beyond the extrinsic factors of project management, and look to the intrinsic factors as well. For while it could be argued that having all the extrinsic attributes would more than likely result in a given occupation being accorded status and recognition as a profession, without the intrinsic factors, the extrinsic factors become mere trappings – window dressing. For in the end, it is how people perceive any occupation as adding value to their lives under stressful or difficult conditions that determines whether or not that occupation has earned the respect and trust of the general public sufficiently to be accorded the respect and advantages normally associated with a profession.

9 Chapter 1: Introduction

The research for both extrinsic and intrinsic factors resulted in a combined total of 22 factors or attributes against which to evaluate an occupation to determine how professional it may be at any given point in time vis a vis other occupations. The preliminary validation of these 22 attributes as indicative of the professionalism of an occupation involved an internet search conducted using two representative occupations – commercial aircraft pilots and construction project managers. The case studies of these occupations represented preliminary attempts to validate and test an evolving survey instrument against two occupations with which I was particularly familiar. Initial theories were tested in the context of these occupations before more extensive research was undertaken. PHASE 2

A survey instrument (Appendix A) was developed, tested and run using Free OnLine Surveys1. This survey instrument represented several iterations of refinement, including conducting pilot surveys at the various paper presentations, conducting oneon-one interviews with various categories of stakeholder, all with the objective of refining the survey questions to answer the fundamental research questions: Is project management a profession, and if not, what is it? To attract as many respondents as possible, the link to this survey2 was published on various webs, including the Project Management Forum3 and other commercial webs, including the researcher’s corporate web4. Attempts were made to publish the survey in other forums to maximize exposure and obtain the broadest possible range of input. These forums included the Construction Management Association5 and the Association for the Advancement of Cost Engineering International6. The total population having access to the survey instrument was estimated to be at least 170,000 people from all around the world, with a maximum of 300,000 and a minimum of 150,000. From this base population, the survey elicited some 400 responses over a period of nine months, running from March of 2005 through December, 2005. 1 2 3 4 5 6

http://www.freeonlinesurvey.com/ http://www.freeonlinesurvey.com/viewresults.asp?surveyid=76670 www.pmforum.org www.getpmcertified.com CMAA. www.cmaanet.org AACEI, www.aacei.org 10 Chapter 1: Introduction

In order to entice participation in a survey which took about an hour to complete, 2 (two) professional development units (PDUs) or .2 continuing education units (CEUs) were awarded to participants. The offer of PDUs was cleared by PMI (Dr. Mike Price) so long as they were claimed under Category 2 – Self Directed Learning. One hour or PDU was awarded for participating in the survey. Another hour was awarded for reading and reflecting on the content as part of self-directed learning. The only requirement made by PMI was to make this information clear on the survey (Figure 1.1).

Figure 1.1

Survey wording offering incentives for participating

To compensate other participants for participating in this long survey, free tee-shirts were offered. It was clear that unless some incentive or compensation was offered, few people would respond, given the length and complexity of the survey. Only ~7% of respondents opted to participate anonymously, with 46% opting for PDU/CEUs and 47% opting for tee-shirts. As can be seen in Figure 1.1, statistically meaningful samples were received from all major areas of the world, except for Russia. Of particular importance is the relative heavy response from the Asia Pacific (39.7%). Given China, India and Indonesia

11 Chapter 1: Introduction

represent the world’s first, second and fourth largest countries (Asian Development Bank, 2005), and that Indonesia is the world’s largest Muslim country, the results of this study take on significant importance as this geographical area represents the most potential future growth in the application and use of project management, however it may be defined.

Figure 1.2

Geographical roots of the respondents

This survey instrument was designed and tested to measure not only the perceived extrinsic and intrinsic attributes of project management, but also with the aim of providing a quantifiable answer to the question: What is project management? PHASE 3

Preliminary responses to the pilot survey indicated that while project managers were making progress in their attempts to professionalize the occupation, project management is ‘unlikely to ever become recognized as a stand alone profession’ (Zwerman & Thomas et al., 2004). Phase 3 of the research, therefore, considered the question: ‘If project management is not a stand alone profession, then what is it?’ There were two additional or follow on hypotheses put forward and researched to answer this question. Hypothesis 1: Project management is a soft, complex dynamic system, a methodology, not a profession. Hypothesis 2: There is a difference between practitioners engaging in an occupation being able to earn their living from applying a methodology, system or approach and that occupation being a profession. 12 Chapter 1: Introduction

Qualitative research to date would indicate clearly that project management is in fact a soft, complex, dynamic system – a method, approach or discipline. But a very special one. Research by the leading commentators on project management, including Kerzner (2005), Lewis (1999), Cleland (1999) and Frame (2002) consistently refer to project management in those terms. Early theoretical work by Jay Forrester (1961) George Richardson and Alexander Pugh, III (1981) and more recently, research in computerized modeling by John Sterman (1996), all from MIT’s Sloan School of Management, indicate clearly that project management is a complex, dynamic system. These researchers have, in fact, developed validated models of applied project management. However, as project management is almost purely people driven, practitioners need to look beyond mechanistic models based on Taylor’s (1911) scientific method, and look to ‘soft systems’ where the human element is emphasized and featured as the core element for success. This is known collectively as soft systems engineering INCOSE, n.d., online) and appears to be the most likely future area of growth for

(

project management. Having sufficient grounded research indicating that project management is a methodology, the remaining question to be answered was whether becoming sufficiently competent in the mastery and execution of a given methodology constitutes being a ‘professional’. Based on research to this point, and experience, it was hypothesised that doing something well enough to be able earn a living from it is insufficient justification for calling an occupation a profession. There needs to be recognition that the noun, ‘profession’ is context-sensitive and potentially very ambiguous when used in speaking or writing, as is the adjective ‘professional’. An excellent example of the potential for confusion can be found in the way in which these words are used in sport. It is common in all sports to refer to those who make their living playing a game as ‘professionals’, implying that the game itself is a ‘profession’. Yet very few people would actually argue that soccer or basketball, cycling or snooker are ‘professions’, although the individuals who succeed at making their living from them are termed ‘professional’ sportsmen or women.

13 Chapter 1: Introduction

Likewise, project managers may do their job with a great deal of skill and ‘professionalism’ without project management being a profession in itself, a fact that project managers should not overlook with debating the status of their occupation. One of the key deliverables from this research was quantifying, measuring and evaluating the professional status of various occupations in order to ascertain the professional status of project management. PHASE 4

Phase 4 was the final compilation of the data, the analysis of that data and the writing of the thesis. The remainder of this document is the culmination of the fourth and final phase.

Outline summarising the methodology of the research. Method was both qualitative and quantitative, using content analysis in developing and testing hypotheses; and statistical analysis of responses to a an online survey to test hypotheses step 1 Formulated the initial research objectives

Phase 1

¾

Identified stakeholders ƒ solicited perspectives outside of North America ƒ validated the need for the research

step 2 Conducted an extensive literature review (content analysis) ¾

¾ ¾

Extracted 19 extrinsic and 3 intrinsic attributes of established professions ƒ Electronic (key word) searches ƒ Manual searches Content Analysis > a hypothesis for each research question/attribute Content Analysis > a survey question to test each hypothesis

step 3 Development of the survey instrument ¾

Phase 2

¾ ¾

47 Question- 15 demographics/32 Research based ƒ ~45-60 minutes to complete ƒ Incentives – PDU/CEU’s or T-Shirt Control Groups Identified/Targeted ƒ Teachers, Nurses, MBA’s, Doctors, Commercial Airline Pilots Multiple approaches to test key hypothesis ƒ Triangulation used to determine the relative ranking of project management vis a vis other occupations

step 4 Tested the initial survey instrument ¾ ¾

Using 12 ESC Lille grad students/13 commercial clients ƒ Bias/ESL interpretation Pilot Survey ƒ Pre-tested against Construction Project Management ƒ Pre-tested against Commercial Airline Piloting

14 Chapter 1: Introduction

step 5 Published the refined survey instrument online Phase 2

¾ ¾

Phase 3

step 6 Statistical analysis of the data ¾ ¾ ¾ ¾

Pearson Product Moment Correlation t-Test (identifying possible bias w/ p-values for the t-tests >0.05) One Way ANOVA Thurstone’s Law of Comparative Judgment (Factor Analysis)

step 7 Reflect on the analysis and draw conclusions

Phase 4

1.7

Thin research on how to develop & conduct on-line surveys March 2005 – December 2005 ~400 Respondents ƒ North America 28% ƒ Central/South America 06% ƒ European Union 08% ƒ Middle East 03% ƒ Africa 16% ƒ Asia/Pacific 39%

Stakeholder analysis

Keith MacDonald, in ‘The sociology of the professions’ identified the stakeholders as ‘social actors’ (Macdonald, 1995, p. 6) and performing a stakeholder analysis is an essential step early in the planning process. To appreciate the importance of stakeholders in the world of project management, the PMBOK guide, 2004 mentions stakeholder over 150 times in 400 pages. At the early stages of the research, the following stakeholders were identified, understanding that as the research progressed, additional stakeholders might be added. The original list included: „

consumers

„

society

„

professional organizations

„

employers

„

practitioners

1.7.1 Stakeholder meetings/discussions To ensure that all stakeholders had been identified, stakeholder meetings were organized, enabling the stakeholders to be identified and providing them the opportunity to offer input. As this was global research, every attempt was made to

15 Chapter 1: Introduction

elicit input from stakeholders from around the world, giving special emphasis on those outside of North America, including the EU, Africa and Asia. In order to ensure exposure and to solicit this global input and evaluation from interested stakeholders, this research has been presented at the following seminars/symposia: Pan-Pacific Conference XXI International Business and Global Project Management May 26-28, 2004 Anchorage, Alaska

Annual Seminar/Symposia ESC-Lille University August 26-28, 2004, Lille, France

PMSA/PMI Joint Congress Global Project Management August 11-13, 2004, Johannesburg, South Africa

Pan-Pacific Conference XXII International Business and Global Project Management June 7-9, 2005, Shanghai, China

Annual Seminar/Symposia ESC-Lille University August 28-31, 2005, Lille, France

PMSA/PMI Joint Congress Global Project Management June 11-13, 2006, Johannesburg, South Africa

Annual Seminar/Symposia ESC-Lille University August 13-15, 2006, Lille, France

By exposing and sharing these ideas before North American, European, African and Asian audiences, and capturing the comments, suggestions and inputs from both within and external to the world of project management, it was intended that the current study should represent a fair and balanced global perspective of the practice of project management, rather than primarily one coming from North America alone. 1.7.2 Consumers The first and foremost stakeholders of any profession are those individuals or organizations who directly employ the practitioners. Historically, the rise of the professions came about largely based on the models of the guilds of the 15th – 18th centuries. According to several notable sociologists, economists and semanticists, like the guilds of old, the professions are nothing more than monopolies (Haga, 1974; Friedman, 1970; Maynard, n.d.) operating under the guise of ‘protecting the consuming public from quacks, charlatans and incompetents’ (Kimball, 1995; Polelle, 1999). 16 Chapter 1: Introduction

There are many examples where the consuming public has been exposed to bodily, mental or financial harm by incompetent practitioners. There is a question, therefore, of whether classifying an occupation as a profession and affording protection to the consuming public via the right of the practitioners to be self-policing is the most effective method. Extensive prior research indicates at best neutral and at worst, negative benefit: cost ratios resulting from licensing and other restrictions associated with an occupation being ‘professionalized’ (Pew, 1990; Cox & Foster, 1990; ETB, 2003). 1.7.3 Society Second to the direct consumers of professional services, is society in general. Above all else, extensive review of relevant literature, particularly from legal and socioeconomic perspectives indicates that the only meaningful difference between ordinary business and the professions lies in their commitment to making society better. This is exemplified by Eliot Freidson, who describes professions as operation within the framework of ‘an ideology, serving some transcendent value and asserting more devotion to doing good work than to economic reward’ (Freidson in Macdonald, 1995, p. 180). Recent examples of failed projects – Chunnel, Ford Motor Company’s Supply Chain ERP Software debacle, Australian Customs House Software System, Boston’s ‘Big Dig’ and the US Internal Revenue’s (IRS) failed database implementation system – represent but a few of the indicators that something is seriously amiss with project management as it is being executed today. This is supported by reputable research for the IT, telecommunications and construction sectors (Gartner, 2000; Standish, 2004; FMI 2005, 2006). As projects (applying almost any definition) (Wideman, 2006) are a part of mankind’s history going back to the taming of fire, inventing the wheel, the pyramids of Egypt and the Great Wall of China, project management must be acknowledged as being an integral part of human evolution and destiny. Accordingly, the current rate of real or apparent failures should not be tolerated, otherwise there is the risk that the contributions project management has made over the years will become devalued. This is especially true as projects become more, not less, complex and more, not less, expensive and critical. It is not unthinkable that in the foreseeable future, mankind will be faced with a life threatening event, such as bird flu, terrorism

17 Chapter 1: Introduction

or an asteroid collision, which will require individuals with project management skills to save the world as we know it or to reform aspects of society in order to adjust to new circumstances. 1.7.4 Professional organizations Looking to the professional organizations which represent those involved with project management in its various forms and applications, practitioners represented by these societies are sending a cacophony of mixed messages. The largest of these organizations, the US based Project Management Institute (PMI) states in its Policy governance manual that ‘project management is a profession’ (PMI, 2000, para 1.1). Yet in a study partially funded by PMI, Zwerman, Thomas et al. (2004) concluded in part that (project management) is ‘unlikely to ever become recognized as a stand alone profession in the traditional sense’ (p. 195). Some appreciation of the confusion associated with the evolution of project management can be found by reading the way in which the highly regarded International Council on Systems Engineering (INCOSE) (INCOSE, n.d., online) describes systems engineering (which in large part forms the basis for project management): [Systems engineering is] an engineering discipline whose responsibility is creating and executing an interdisciplinary process to ensure that the customer and stakeholder’s needs are satisfied in a high quality, trustworthy, cost efficient and schedule compliant manner throughout a system’s entire life cycle.

The Construction Management Association of America defines construction project and program management to be: Construction Management is a professional service that applies effective management techniques to the planning, design, and construction of a project from inception to completion for the purpose of controlling time, cost and quality. Construction Management is a discipline and management system specifically created to promote the successful execution of capital projects for owners. These projects can be highly complex. Few owners maintain the staff resources necessary to pay close, continuing attention to every detail--yet these details can ‘make or break’ a project.

18 Chapter 1: Introduction

The American Institute of Constructors mission statement explains: The purpose of The American Institute of Constructors (AIC) is to promote individual professionalism and excellence throughout the related fields of construction

To illustrate further just how divided the practicing community is in defining exactly who or what project managers are, consider the comment from Professor Peter Morris (2004) on ‘Re-thinking project management’ where he describes project management as a discipline which remains ‘stuck in a 1960s time warp’ (online). This clear lack of consensus on the part of leading practitioners supports the need for further investigation into the status of project management as an occupation. So what is project management? A profession? Not a profession? Engineering discipline? A professional service? Disciplined management system? Discipline stuck in a time warp? At the South African PMSA/PMI meeting in June of 2006 in Johannesburg, Iain Frazer, Chairman of the PMI Board of Directors claimed, ‘It doesn’t really matter’. If it doesn’t matter, then why all the efforts to claim project management is a profession? 1.7.5 Employers Living, working and providing project management related professional services in a region of developing nations, the importance of the ‘professionalization’ of the workforce is both real and urgent, and can best be summed up by a statement made by Mr. James J. Mulva, CEO and Chairman of ConocoPhilips Petroleum that, ‘The faces of the people working for ConcocoPhilips should be a mirror-image of the faces in the countries in which we operate’. The use of a predominately expatriate workforce in the developing nations is generally not desirable financially or politically correct situation. Consistent with the Millennium Development Goals (n.d., online), it is the obligation of today’s corporations, guided by the principles of corporate social responsibility (CSR) to raise the level of skills, knowledge and competencies of the work force in the developing nations to international standards through the process of technology transfer7.

7

Corporate social responsibility is the continuing commitment by business to behave ethically and contribute to economic development while improving the 19 Chapter 1: Introduction

This becomes the driver behind the need to develop credible means to measure the ‘professionalism’ of people from different cultures and religions, speaking different languages. Thus, the process of developing professionalism in fulfilment of the principles of CSR and the Millennium Development Goals requires that we fully understand what exactly what the definition of a professional is before we can possibly hope to produce them. 1.7.6 Academics/institutions of higher learning There is big money to be made in project management for academics and their institutions. Since the 1990s, as project management came into vogue as a panacea for all corporate and organizational ills, and project management as a delivery system became the latest darling of the business schools, the opportunity for academics and their alter egos, business consultants, to make a lot of money selling repackaged and reworked business concepts has proven too much to pass up. As Freidson (2001) notes, one of the requirements for an occupation to become a profession is ‘an occupationally controlled training program which produces the credentials defined by the practitioners, in a way segregated from general labor markets; which is associated with ‘higher learning’ and the development of new knowledge’ (p. 180). This opens up the door to the dangers of crass commercialization and potential exploitation, especially of those seeking the credentials as a means to achieve higher paying jobs or to obtain jobs outside of their country of origin. 1.7.7 Consultants Consultants play an interesting role in the professionalization of project management. On one hand, assuming all practitioners become professionals, then what is the need for a consultant? On the other hand, consultants are needed to guide the practitioner on the road to professionalization. This dichotomy is exemplified in Christopher McKenna’s recent book, The world’s newest profession – management consulting, in which he explains that in the age of knowledge, the rate of change is increasing so quickly that it makes more sense economically to outsource the creation of new knowledge than it does to create it in quality of life of the workforce and their families as well as of the local community and society at large. 20 Chapter 1: Introduction

house. Dr. McKenna (McKenna, 2006) concludes his book with the statement that, ‘The world’s newest profession (management consulting) will have to wait until the 21st Century’ (p. 251). Clearly, management consulting is not a profession, however, despite big salaries and attempts by McKenna and others to try to professionalize the image. 1.7.8 Practitioners Practitioners have the most to gain (or lose) by the professionalization of any occupation. Studies by Gartner (2003), Marco (2004; 2000), Standish (1995), Engineering News Record (ENR, n.d., online), Schoppman, Greg et al. (2006) and others (Morris & Hough, 1987; Chan & Kumarasawamy,1999) serve as broad indicators that project management, as it is being practiced today, is resulting in failure rates of anywhere between 30% – 70%. This wide inconsistency and such excessively high failure rates are paradoxical with the image of an occupation having earned the right to call itself a profession. Failure rates of this magnitude are more reminiscent of medicine in the 17th century than in an occupation attempting to earn the right to be recognized as a profession. Project managers owe it to themselves to improve their image not through hype, marketing and misplaced emphasis on restraint of trade through licensing, but by focusing their efforts on delivering successful projects consistently and reliably. The American Bar Association has summed it up rather nicely, saying, ‘The only fruitful use of the term profession today relates to individuals, not groups’ (Rotunda, 1997). As the American Bar Association has admitted: Some lawyers are professional and some are not…[and]…it is neither useful nor appropriate to label the practice of law or any other occupation as a profession, if that label is then used to justify restraints of trade that would not be accepted for all occupations. (ABA Working Group on Civil Justice, Blueprint for Improving the Civil Justice System, 1992, cited in Polelle, 1999, online)

From these meetings, stakeholders could be classified into three groups, according to their school of thought: „

those who believe project management is not a profession This group generally views project management as a delivery system or methodology, holding their ‘profession’ to be the underlying technical specialty from which they entered project management. Usually, but not always, a

21 Chapter 1: Introduction

Bachelors degree in architecture, engineering or some scientific specialty, or having worked their way up through the trades or some other experiential based career path. „

those who believe project management is a profession This group seems most often to consist of practitioners coming from the software sector (IT/IS). While the organization which represents many of them – IEEE – is a well established professional organization, consistent with Abbot (1988), by ignoring the needs of this constituency, it would appear the IEEE may have subordinated their jurisdiction of the project management functions of IT/IS practitioners, thus forcing their members in that category to seek out an alternate representation for them and their needs as they move from being technicians into management roles (p. 2).

„

those who believe it doesn’t matter whether it is a profession or not This group seems to consist of two sub-groups. The first are those who subscribe to the ideas put forth in several studies by the American Bar Association indicating, as Ronald Rotunda (1997) argues, ‘the only fruitful use of the term ‘profession’ today relates to individuals and not groups’ (online). , while the second seem to believe that simply saying that project management is a profession will make everyone believe it is one.

1.8

The significance of the study

The research was originally designed to answer the questions outlined previously. However, after the research for this study was well underway, Dr. Bill Zwerman, Dr. Janice Thomas, Susan Haydt and Terry Williamson, researchers at the University of Calgary and Athabasca University, published Professionalization of project management – Exploring the past to map the future (Zwerman, Thomas, Haydt, & Williamson, 2004). To a large degree, their work served as foundation and to some extent, a template for this research. As their research established a precedent, this study was modified mid point to validate or refine their findings. The primary differences between their work and this research are: „

In this research, the attributes or traits of a profession were dealt with in more detail, and were expanded somewhat to include more focus on intrinsic and semantic aspects of professionalization.

22 Chapter 1: Introduction

„

While the Zwerman, Thomas et al. (2004) study focused primarily on North America, the current research explored how global practitioners viewed themselves, hoping the data would serve as a baseline for future analysis to see if project management were becoming more or less professionalized.

„

Where Zwerman, Thomas et al. (2004) focused on the professionalization efforts of nursing, social work and teaching, the focus of this dissertation was the analysis of two occupations more associated directly with project management: ⎯

commercial airline piloting as an existing and recognized profession, with particular focus on the fact that a commercial air transport pilot represents a uniquely global approach to defining a profession (Due to the international nature of piloting, and the inherent need for safety, the piloting of commercial aircraft represents perhaps the most standardized of any occupational specialty, even more than medicine.)



construction project management, firstly because it is one of the older and more mature manifestations of project management, and secondly because of the (largely unsuccessful) efforts over the past 40 or 50 years to raise construction project management to be on equal standing with the already well established architectural and engineering professions.

Taken together, these two examples provide ‘outliers’ in that commercial aircraft piloting does not meet many of the fundamental or traditional attributes of a profession, yet is perceived to be one; while construction project management, which evolved directly from architecture and engineering, has never been able to achieve equal professional standing with either.

1.9

Definitions of the key terms

Project management is generally recognized as a cross functional, multi-disciplined delivery vehicle for both products and services, and lack of standardization thus far has resulted in a polemic and often schismatic assemblage of terms, many of which are confusing at best and, at worst, outright conflicting. One of the challenges encountered in this research was the lack of standardized terminology. Max Wideman, PMP, PMI Fellow and highly regarded practitioner and author, has compiled a respected, web-based compendium or glossary of some 20,000 terms common to project management (online). In his glossary, Wideman lists

23 Chapter 1: Introduction

some 23 definitions for the word project, 13 for project management and 14 for project manager. While there are some similarities, the definitions are different enough to make any meaningful research difficult. As much of the surge in the popularity of project management comes as a result of the computing sector, the following statement by the Computing Sciences Accreditation Board (CSAB) (which is the Computing Accreditation Commission member of ABET) has relevance when it states: Any discussion of names of the computing sciences professions must recognize the semantic imprecision and confusion that plagues practically every aspect of computers, computer studies, and the range of traditional academic disciplines that increasingly use computers in instruction and in academic research. (CSAB, 1997, online)

Given that project management, at least in North America, is a ubiquitous and imprecise term in the computing, as well as other sectors, the journey of determining whether project management is or is not a profession began by acknowledging the importance of semantics. As the key words in this research are profession, professional, project and management, the objective was to first define these terms as they would be used throughout the study. (Other terms have been defined at future junctures as necessary.) The rationale behind the semantic approach was to discover the most ‘normal and customary’ English usage and meanings for these words, thus minimizing or avoiding the impact of ‘spin doctors’, who change or distort meanings and definitions to market particular points of view or perspectives or who use semantics as a method to obfuscate and dominate a specific knowledge base. As can be appreciated from a review of the myriad of dictionaries available, there exist many nuances of meaning for the key words, even between dictionaries coming from the same country, including those where English is the primary language. As this research was designed to be global in nature, whenever practical, reliance has been on those definitions offered by Webster’s third new international dictionary (unabridged) (Merriam-Webster, 2002), with secondary or supplemental reference to the Concise Oxford English dictionary (OUP, 2004). Some specific definitions have come from the Encarta dictionary online (Microsoft Corporation, n.d.). Webster’s third new international dictionary (Merriam-Webster, 2002) defines profession thusly:

24 Chapter 1: Introduction

Profession: a calling requiring specialized knowledge and often long and intensive preparation including instruction in skills and methods as well as in the scientific, historical, or scholarly principles underlying such skills and methods, maintaining by force of organization or concerted opinion high standards of achievement and conduct, and committing its members to continued study and to a kind of work which has as its prime purpose the rendering of a public service. Learned professions: theology, law and medicine

In order to develop a full and complete understanding of these terms, let’s break this definition down further into the component elements: calling impulse or desire to follow particular job: a strong urge to follow a particular career or do a particular type of work specialized devote time to particular activity: to devote time exclusively to a particular interest, skill, or field of study knowledge information in mind: general awareness or possession of information, facts, ideas, truths, or principles all that can be known: all the information, facts, truths, and principles learned throughout time long going on for lengthy period: lasting for an extended period of time intensive concentrated: involving concentrated effort, usually in order to achieve something in a comparatively short time preparation preparatory measure: something done in advance in order to be ready for a future event skills ability to do something well: the ability to do something well, usually gained through experience and training something requiring training to do well: something such as an art or trade that requires training and experience to do well; methods way of doing something: a way of doing something or carrying something out, especially according to a plan body of scientific techniques: the body of systematic techniques used by a particular discipline, especially a scientific one principles basic assumption: an important underlying law or assumption required in a system of thought ethical standard: a standard of moral or ethical decision-making organization group: a group of people identified by shared interests or purpose, for example a not-for-profit professional organization

25 Chapter 1: Introduction

concerted opinion achieved or performed together: an opinion reached by two or more people working together or with the same aim high standards (of achievement and/or conduct) above average: greater than the normal or average, for example in quantity, number, quality, intensity, or cost, or well above a smaller or lower level or amount continuing education specialist courses to update professionals: regular courses or training designed to bring professionals up to date with the latest developments in their particular field lifelong adult education: adult education, usually in the form of short or part-time courses, continuing throughout an individual’s life providing a public service provision of essential services: the business or activity of providing the public with essential goods or services such as electric power service benefiting the general public: a service that is run for the benefit of the general public, for example the utilities, the emergency services, transport, and broadcasting

Restating the definition of profession using the clarifications above, and put in the context of project management (or any other occupational specialty) the definition of profession can be more completely rephrased as: A strong drive or desire to do the kind of work normally associated with [project management] [Or Insert your chosen Occupation], by devoting considerable time and effort, over a long period of time, to master as much information, facts, truths, and principles as possible, by making a deliberate and concerted effort to develop, through training, experience, and by taking specialized adult education courses, over a life time spent as a [project manager], those systematic techniques to be able to execute the work of a [project manager] by adhering to above average standards of ethical, moral and legal care, as defined by a group of people with shared aims, the primary one being serving the interests of the general public.

This definition of project management has been elaborated in whole or in some part or another through lawyers, economists, sociologists and practitioners, but no single source has been found which brings all aspects of a profession together in a single document. It is believed this research is the first of its kind to combine the legal, social, economic and semantic definitions and apply them to the practice of project management. Searching the literature, conversations with colleagues around the world and listening to clients indicated that considerable confusion exists between the noun profession and the adjective, professional. Due to this widespread confusion, it became imperative before proceeding further with the research to make the distinction

26 Chapter 1: Introduction

between the two terms clear and as unambiguous as possible. Having just defined profession, for the purposes of this paper, the definition of the adjective, professional, follows: professional [prə fésh’nəl] adj very competent: showing a high degree of skill or competence following an occupation as paid job: engaged in an occupation as a paid job rather than as a hobby

To make the distinction between the noun, profession, vs. the adjective, professional, as clear as possible, consider this anecdote in the context of the above definitions: Tiger Woods is unquestionably a talented golfer. One would be very hard-put to dispute the obvious, which is that he is very competent at what he does, perhaps amongst one of the best ever. Therefore he meets the first test of being a professional – skill and competence. In fact, he is sufficiently competent that he makes a very handsome living performing for pay what most of us consider as a hobby; hence, applying the second criteria, he meets the ‘earnings test’ to be considered a professional. Therefore, having met both tests (highly competent and earning a living at what most do for a hobby) qualifies him as a professional golfer. However, just because Tiger Woods (and many other people around the world to a lesser degree) meet the criteria to be appropriately called professional golfers, does that necessarily qualify golf itself as a profession?

The answer is, obviously not, but many in the community of project management practitioners seem to conflate the two terms, concluding that if they are, in fact, professional at what they do, then what they do must, by extension, be considered a profession. This is false logic and a trap easily fallen into. This analogy can be expanded to include not only golf and other sports, but to many occupational specialties including acting, singing and other talent based performances. Taken to an extreme, consider applying the above test to the world’s oldest profession – prostitution. As will be shown through analysis of the data collected, failure to make this simple distinction between the noun and the adjective is a root cause of confusion among project management practitioners, one which must be addressed by all occupations trying to ‘professionalize’ what they do. There are two remaining terms which must be defined prior to proceeding further with the exploration of this topic. They are project and management. project n [prójjekt] (plural projects) unit of work: an organized unit of work

27 Chapter 1: Introduction

proposed or planned undertaking: to achieve a particular aim or objective; a scheme for which there seems hope of success management [mánnijmənt] n administration of business: the organizing and controlling of the affairs of a business or a particular sector of a business handling of something successfully: the act of handling or controlling something successfully skill in handling or using something: the skilful handling or use of something such as resources responsible for results: the executive function of planning, organizing, coordinating, directing, controlling, and supervising any industrial or business project or activity with responsibility for results

To restate the definition of project management according to Webster’s third international dictionary (Merriam-Webster, 2002): project + management: the administration supervision or executive function to plan, organize, coordinate, direct and control a proposed or planned undertaking to achieve a particular aim or objective within a specific time frame, with some reasonable expectation for success, through the skillful handling or use of limited or constrained resources and the successful organization, administration and controlling these affairs in a business-like manner.

To avoid getting trapped, constrained or risk misinterpretation and disputes over definitions, as this was a global study, definitions were garnered primarily from Webster’s third new international dictionary (Merriam-Webster, 2002), (including restatements) cross referencing the Concise Oxford English dictionary (OUP, 2004) whenever necessary or appropriate. The objective was to base the research as much as possible on standardized, globally accepted and recognized English language definitions rather than any terms ‘jargonized’ or ‘idiomized’ by the practitioners of project management (or any other occupation referencing this research publication.) As an anecdotal illustration of the importance standardization of the lexicon has to the development of a profession, consider the potentially disastrous impact of a miscommunication in the field of commercial airline piloting. Not only is this occupation universally recognized as being a profession, but it is one of only a handful of occupations which requires a license to practice which is recognized across all borders (mutual global reciprocity). One of the early conventions agreed to by governmental airline regulators, airlines and pilot associations was to adopt English as the official language, not only for the

28 Chapter 1: Introduction

pilots, but also for the key supporting roles, including air traffic controllers, ground crews and aircraft maintenance professionals. Furthermore, world governments agreed (more or less) on the terminology used by pilots and related support, often involving the use of arcane acronyms. Without this agreement on the lexicon, ‘rules of the road’ and an ‘official’ language, the growth of commercial air travel would have been severely limited, not to mention the inherent possibilities of miscommunications leading to disasters both in the air and on the ground. The rather enviable safety record of air travel owes a lot to this standardization of language, terms and operational rules (Ask Captain Lim, n.d., online; FAA, n.d., online). This is an important lesson which should not be overlooked by the project management community as practitioners search for ways to professionalize the practice of project management. Another example which illustrates dramatically the importance of speaking a single, common language is the attempted separation of the Iranian twins, Ladan and Laleh Bijani, who were conjoined at the head. The operation, which is expected to last at least two days, consisted of a team of eight international experts, 18 local doctors and more than 100 support staff, performing the operation, led by Dr. Keith Goh, from Singapore. (Singapore Straits Times, 2004)

Had the team not spoken a common language (English) using a common vernacular, this operation would not have been possible. In terms of project management, not only have practitioners yet to agree on a common language and working lexicon (vernacular), practitioners cannot even agree on defining what it is practitioners do. Even the simplest sounding agreement, such as defining what a ‘successful’ project looks like has eluded practitioners for more than 40 years. If practitioners cannot even agree on a definition of project, project manager or successful project, how can they pretend to call what they do a profession?

29 Chapter 1: Introduction

1.10 Organization of the thesis In Chapter 1, the key questions being researched are introduced. The five primary objectives of the research were to: 1) answer the question Is project management a profession? 2) determine what project management is, if not a profession 3) ascertain where project management stands in relation to other emerging occupations claiming to be professions 4) determine what can be done, if anything, to enhance the image of the occupation 5) decide what, if anything, can be done to improve the delivery of successful projects. The reasons, aims and objectives of the research are outlined and it is explained that project management is caught in a paradox. On one hand, project management has become the ‘flavour of the month’ for senior managers, while at the same time, organizations are expecting miracles from project managers. Professional organizations which make money representing professional practitioners are proclaiming project management to be a profession yet credible research indicates project management is failing to deliver on time, within budget and substantially conforming to the technical requirements, much less trying to satisfy any need for which the project was undertaken in the first place. The key stakeholders involved in project management – those who stand to gain or lose if project management were to be accepted as a profession – were introduced. These stakeholders include: „

those who do not believe project management to be a profession

„

those who do believe project management to be a profession

„

those who believe it really doesn’t matter.

This introduction also included a brief reason for selecting two rather disparate case studies – construction project management and commercial aircraft piloting around which to develop the research.

30 Chapter 1: Introduction

Key terms have been highlighted and the flow of the research outlined with a brief explanation to serve as a roadmap. Chapter 1 closes by setting the stage for Chapter 2, where we will start the literature review to determine by what standards a profession can or should be judged and start by testing these attributes against the case studies. In Chapter 2, we scour sociological, legal, economic, semantic writings to determine what attributes can be used to create a numerical model to test for a degree of professionalism. We come up with a total of 22 attributes, representing both traditional and non-traditional evolution of occupations currently accepted as professions. In Chapter 3A, we test those attributes against construction project management, an occupation which has been recognized for well over 50 years as being a discipline of both architecture and engineering, but has never been able to establish itself as a stand alone profession. As general applications of project management are largely derived from construction project management, the evolution of construction project management should provide some indication of what will likely happen to other manifestations or incarnations. In Chapter 3B, we take the same 22 attributes and compare commercial airline piloting to see where it ranks on the scale. commercial airline piloting was chosen because despite the occupation not being recognized legally as a profession, in the minds of the consuming public, commercial aircraft piloting is considered to be a prestigious job, with recognition as a profession. Chapter 4 offers a discussion of the development of the research questions and a survey instrument (Appendix A) designed to answer the research questions. Two major questions are the focus of Chapter 4: 1 Is project management a profession and if not, what is it? 2 Where does project management rank on a continuum between the trades and the learned professions of law and medicine. Survey questions were created to see how both project managers and non-project managers ranked the occupation against other occupations and to see what both practitioners and those outside of the practice defined project management. 31 Chapter 1: Introduction

Unfortunately, do to a lack of response by non-project managers, the full comparison could not be completed. However, with close to 400 respondents, the survey instrument did provide a lot of valuable insight into perceptions. In Chapter 5, we start to work with the results from the survey, to see what makes sense and what we can use and what we can’t use. Here, we start to answer the research questions and test the various hypotheses. We learn that practitioners do not consider project management to be a profession, that it is a process or methodology; We learn that project management is less of a profession than being an MBA, but more of a profession than being an electrician. We are also able to start to quantify the 22 attributes, to see which ones are more important and how practitioners rate project management against each of those 22 attributes. Chapters 6 and 7 summarize both the qualitative and quantitative results of the survey and draw conclusions, resulting in the creation of a Professionalization Index that provides a relative weight adjusted value for each of the 22 attributes. Unfortunately, as there were an insufficient number of non-project managers who responded to the survey, it proved impossible to compare project management against other occupations. However, as the professionalization index provides for a true zero, follow on research will be able to calculate a true ratio scale to see if project management becomes more or less of a profession, and how much more or less compared to other occupations. Use of a professionalization index is also applicable to any other occupation wishing to measure where they are vis a vis other occupations.

32 Chapter 1: Introduction

CHAPTER 2

Literature review and preliminary research 2.1

Introduction: Considering the nature of professions

In Chapter 1, the overall strategy and objectives of the research were outlined, recognizing that like any project plan, change was likely to occur. Key stakeholders were identified and a clear idea of their needs, wants and expectations was established. Consistent with the philosophy of Steven Covey (1991), I began this journey with the ‘end in mind’ (p. 42). Consistent with a positivist approach, in Chapter 2, I take the next logical step, which is to conduct a literature review to determine what research in the area has been done before, and whether it is possible to craft a study that will create some new and better understanding of project management as an occupational specialty. In Chapter 2, the vision defined in Chapter 1 begins to take on substance and reality. For historical background on the evolution of professions, reliance was on (in rough order of importance) Bruce Kimball’s (1995) The true professional ideal in America – A history and the work of C.L. Gibb (1966) in Hidden hierarchies: professionalism and government, along with Howard Mills and Donald Mills (1966), editors of Professionalization. To gain an appreciation for the dynamics of evolving professionalism, the work of David Maister (1993), including the research underlying True professionalism – The courage to care about your people, your clients and your career and Managing the professional services firm played an influential role; as did the work of Harold Wilensky (1954) in ‘The professionalization of everyone’, published in the American Journal of Sociology. To understand the structure or substance of the professions, John Cullen’s (1979) research on The structure of professionalism – A quantitative examination gives insight, as does Slayton and Trebilcock’s (1978) Professionals and clients – A constitutional struggle. To understand the issues of licensing and regulation, the initial resource was the work of Shimberg, Esser and Kruger (1973) in Occupational licensing: Practices and policies, and Regulating the professions by Roger Blair and 33 Chapter 2: Literature review and preliminary research

Stephen Rubin (1980). Given the trend towards more conservative views, heavy reliance was placed on the work of Carolyn Cox and Susan Foster (1990) entitled The costs and benefits of occupational regulation, a report prepared for the Bureau of Economics, Federal Trade Commission. As the importance of licensing to the development of a profession is clear, research by David S. Young (1987), The rule of experts: Occupational licensing in America was included, along with a study commissioned by the Pew Research Centre and conducted by Finochio, Dower, Blick and Gragnola of the Task Force on Health Care Workforce Regulation (1998) titled Strengthening consumer protection: Priorities for health care workforce regulation. Lastly, Licensing and the UK engineering profession – A review, funded by the Engineering and Technology Board (ETB) and researched by Eras, Ltd. (2003) provided an international perspective reflecting current thinking and political trends. As this research also looked at the development of the professions from a legal perspective, the work of Michael J. Pollele (1999), ‘Who’s on first and what’s a professional?’ played a key role, as did Norman Spaulding’s (2003) ‘Reinterpreting professional identity’ and Christopher Gawley’s (2002) ‘Protecting professionals from competition: The necessity of a limited antitrust exemption for professionals’. Also figuring heavily in defining a profession from a legal perspective was the decision by Chief Justice Breitel in re: Matter of Freeman, 40 AD 2d 397. A legal perspective wouldn’t be complete without referencing Ronald Rotunda, and his 1997 Arkansas Law Review article ’Professionalism, legal advertising and free speech in the wake of Florida v. Went For It, Inc and his professional responsibility, 6th edition were cited frequently. Consultation with Dr. Lynn Crawford, UTS & ESC Lille, an authority on global project management standards, suggested the real need was to look at the more nontraditional perspectives of how professions develop or evolve. Emphasis was then placed on the research of W.J Haga (1974), particularly ‘Perils of professionalism’ published in Management Quarterly; and W.J. Goode (1969), with his ‘Theoretical constraints of professionalism’ published in The semi-professions and their organizations. Another strong recommendation from Dr. Crawford was Sociologist by Andrew Abbott (1988). In ‘The system of professions: An essay on the division of

34 Chapter 2: Literature review and preliminary research

expert labor’ Abbott presents an analytical model that looks at professions in terms of their turf, their control and the interactions, interdependencies and conflicts (‘turf wars’) they experience. Abbot is also an advocate of the principle of estoppel: that by claiming what you do is a profession in fact makes it so. Professor J. Rodney Turner, also from ESC Lille, has recommended that the views of Plato also be included, especially Plato’s perspective that learning must occur as a prerequisite to being a professional.

2.2

The concept of a profession: An historical and semantic perspective

To grasp the contemporary meaning of professions and professionals in the context of the 21st century, we start by looking to Aristotle, Socrates, Plato and of course, Hippocrates. Of specific relevance to this research is Aristotle’s ‘What we learn to do, we learn by doing’. Fundamental to all professions is the principal that you cannot learn what you need to know for any profession by reading books. Hence, we refer to the ‘practice’ of medicine, implying that the acquisition of knowledge and skills is never ending. Likewise of importance is Socrates views on deception- that ‘self deception is the worst of all deceptions’. As we explore the professionalization of project management, we are going to have to question whether, as practitioners we are deceiving ourselves in making claims that project management is a profession. Several of Plato’s philosophies also have relevance to this research. ‘Knowledge obtained under compulsion obtains no hold on the mind’ becomes important when we consider project management being known as the ‘accidental profession’. Plato also admonishes physicians to ‘put the good of their patients above making money’, which to this day forms one of the core ‘attributes’ of a profession, at least in the traditional sense. Another quotation by Plato includes: ‘the object of knowledge is what exists and its function to know about reality’. But perhaps the most important contribution by Plato was summarized by Christopher McGinnis, in his capstone thesis ‘The social status of physicians in Greece and Rome’, where he explained: In Greece around the 5th century B.C. there was a rise in a new class of healer. These physicians were not priests with divine gifts, but simply craftsmen practicing an art. They were often approached with mistrust and caution because there were no regulations on who could practice medicine, and the only way to know if a doctor was competent was from his results.

35 Chapter 2: Literature review and preliminary research

These physicians in Greece were of two types: The educated Hippocratic physicians who studied medical theory at a medical school such as those at Cos or Cnidus, and the slave-doctor who learned his craft through apprenticeship and observation. The Hippocratic physicians came from the upper echelon of Greek society, as is seen in the example of Eryximachus in Plato’s Symposium, while the slave-doctor or doctor’s assistant would often be a slave or freedman.

This points out two important considerations which need to be incorporated into the research: „

The key differentiator between the professional practitioner and the priest lies in the source of the underlying knowledge. While the priest relies on a belief in faith, the true professional relies on knowledge substantiated by fact and observations.

„

Ultimately, the only way to prove a doctor was competent at the time was based on his results. Isn’t this exactly the state we find project management in today?

Lastly, there is Hippocrates, who said, ‘I will apply dietetic measures for the benefit of the sick. I will keep them from harm and injustice’, upon which the Hippocratic Oath, still taken today by all medical doctors, was based (Rothman, ‘The crisis in the ethics and profession of medicine: A historical perspective’). While Aristotle, Socrates, Plato and Hippocrates established the philosophical basis, to examine the true beginning of the professions as we know them today, we must start by understanding the European guilds of the Middle Ages and Renaissance periods (Kimball, 1995; Polelle, 1999). These guilds began as groups of workers talented in performing certain skills or crafts. As they became more organized and realized that not everyone in the community was capable of performing these essential services, they became more powerful, eventually challenging the king and other Medieval hierarchy by demanding the right to be self-policing. Once granted this authority, the guilds controlled two primary elements (Goode, 1969): „

who and how many apprentices were accepted and trained (control over the secret methods, formulae and skills)

„

who could own the tools, workshops and other means of production.

By the 17th century, the guilds by nature of having created essentially a monopoly, had grown corrupt, collusive and nepotistic (Kimball, 1995; Goode, 1969). This eventually led their downfall. The only guild to survive was the Scholars Guild,

36 Chapter 2: Literature review and preliminary research

which to this day continues to exhibit several of the traits common to the original guilds – lifetime tenure and faculty governance – in most universities around the world. From the Scholars Guild came the birth of specialties from within the scholarly community, namely, divinity, law, medicine and university teaching. The ethical aspects of professions came about as a result of the 16th century requirement that divinity students make a public declaration of their religious vows. However, by the 17th century, these vows had become a commitment to adhering to and maintaining the ‘secrets’ of the profession, and remain the reason why medicine and law continue to this day to use Latin or Greek terminology. This commitment to protect or keep secret the ‘body of knowledge’ necessary to execute the duties of the profession formed the basis for what we know today as the code of ethics common to essentially all professions, whether new or old. Despite the demise of the trade or craft guilds, the scholarly guilds, which evolved into what we call today the ‘learned professions’, have managed to keep many of the attributes associated with the original guilds, such as privileged social standing, lifetime tenure and the prevention of outside interference or intervention in the administration or management of their affairs (Kimball, 1995). 2.2.1 The concept of professional project managers in the professions The work of Shimberg, Esser and Kruger (1973) in the mid 1960s showed how persuasive the idea of being a professional had become in the 20th century, as did later seminal work by Kimball (1995) in The true professional ideal in America – A history, and Willenski (1954) in The professionalization of everybody. The trend towards people wanting their occupation to be perceived as a profession, thereby reaping the respect, prestige and benefits associated with that descriptor, is easy to understand. More specifically, the Project Management Institute (PMI), the world’s largest organization representing those practicing project management, has clearly stated in their policy governance manual that, ‘Project management is a profession’. In the most recent version of the PMI Code of Professional Ethics, released in early January, 2007, reference to project management as a ‘profession’ occurs 11 times and ‘professional’ 20 times.

37 Chapter 2: Literature review and preliminary research

But does merely repeating the same mantra often enough result in it becoming fact? As the parable about the emperor’s new clothes tells us, human beings are prone to believe not what they see, but what they have heard, and are heavily influenced by peer pressure to conform to group norms. Unfortunately, this may well be the case with project management, for instead of being a stand alone profession, there is growing evidence that each specialty is justified in ‘claiming’ project management as it applies within that sector; Thus, like the civil engineering schools of 40 years ago, laying claim to construction project management as a discipline falling under the auspices of civil engineering, there is evidence that the discipline of information technology is claiming project management for computer science majors; while aeronautical engineers are establishing the academic basis for claiming project management related to aerospace. Meanwhile telecommunications engineering schools are creating project management courses designed specifically to address the nuances of project management as it relates to telecommunications projects. Further evidence of this trend can be found by looking at the membership profiles of organizations competing against PMI, such as the Association for the Advancement of Cost Engineering International, (AACE)1; Construction Management Association of America, (CMAA)2; American Institute of Constructors (AIC)3; and the American Society of Civil Engineers (ASCE)4. One of the reasons PMI has such a small percentage of their membership coming from the construction sector (~4%) may be because the specialty organizations (e.g. CMAA, AACE or ACI) satisfy the needs of the mature user more effectively than does PMI. While at the periphery of this research, it is intended that through the application of a combination of modern and post modern thinking, sufficient evidence will evolve to support this fledgling hypothesis. These examples of organizations wanting to establish their members’ professional standing represent a developing ‘turf war’ not unlike those that plagued the construction sector during the early 1960s, when each trade was fighting with the 1 2 3 4

www.aacei.org www.cmaa.org www.aicnet.org www.asce.org

38 Chapter 2: Literature review and preliminary research

other to ‘claim’ ownership of each new method or material being introduced. This observation is consistent with Abbot’s (1988) theory of competition. The ‘infighting’ and ‘turf wars’ were one of the leading causes of the demise of unionism in the construction sector in the late 1970s and early 1980s, and the ascendancy of open shop operations. The possibility of this sort of conflict happening in the area of project management is not an insignificant matter, and one that, while not directly a topic of this research, should not be ignored. Using the construction sector as a model, not only do we see the owner’s project manager on most projects, but also project managers from the architects, engineers and consultants, as well as the general contractor and each of his/her subcontractors and vendors. There is a trend, therefore, towards the specialization of project managers around each sector. But who will manage the project managers? Given that project management is truly a legitimate delivery method and not just another management fad, and given the competition in the education sector is becoming increasingly competitive, how likely are the business schools to stand idly by waiting for the engineering schools to lay claim to project management? Thus can be seen the development of portfolio and program management by the business schools, recognizing that someone will eventually need to watch over and coordinate the plethora of project managers assigned to each project. These inevitable turf wars have long been evidenced within the medical profession. It wasn’t all that many years ago that chiropractic was not formally ‘recognized’ by the medical profession, nor was traditional (Chinese) or alternative medicine. Yet because they were accepted by the consuming public, the medical profession was forced to accept them on equal terms. Another fact which illustrates the importance of this topic is the representation of project managers by no less than three professional organizations, which do $100 million USD plus global business, with each claiming to represent the global project management practitioner, with no shortage of local or regional entities claiming the same. Looking historically at either the construction or the medical professions in particular, can only lead to the conclusion that battles for the hearts, minds, loyalties, but more importantly, the pocketbooks of those practicing the art and science of project management have been considerable and are ongoing. 39 Chapter 2: Literature review and preliminary research

2.2.2 Evolution of the concept of professionalism The evolution of professions has been studied for over 100 years, resulting in at least four schools of thought. The more traditional extrinsic school looks to those attributes or traits which manifest themselves through observable characteristics or signs, indicating the evolutionary progress of an occupation to achieving professional status. The second, less common approach, is to look at professionalization through the actions of the consuming public. This perspective is known as the intrinsic or semantic approach, and relies more on the belief, trust and acceptance of the practitioners. The intrinsic school can be evidenced by the relatively recent acceptance of such occupations as traditional Chinese herbal medicine, acupuncture, and chiropractic as ‘professions’, at least in the Western world. The evolution of these ‘professional practices’ was driven from the bottom up, by the consuming public, often despite the ridicule and debasement by the established medical professions. The difference between these two schools of thought is centered on how each interprets the concept of professionalism. One is willing to rely on principles of trust and real or perceived results, with little or no regard to the extrinsic attributes normally associated with the established professions; while the other bases its interpretation on measurable attributes: education, accreditation, organization into widely recognized and representative bodies. This certainly supports the argument of the importance and relevance of the intrinsic (semantic) over extrinsic (trait) attributes as being the better determinants of how to define a profession. Other theories exist, including control, feminist and conflict, all of which were well covered by Zwerman et al. (2004). Control theories explore the relationship between professionals and bureaucracy, deducing that the two are incompatible, and are exemplified by the tensions between power and control (Zwerman et al., 2004, pp. 29-30). The control theories apply to occupations which evolved as part of the bureaucracy (Hugman, 1991, p. 201, cited in Zwerman et al., 2004, p. 29) Zwerman et al. go on to explain the feminist concept of professionalism as one which contains predominantly masculine values and constructs, as espoused by ‘Adams, Lugsden, Chae, Arber and Bond’ (Zwerman, 2004, p. 30) Lastly, the conflict approach explores the ways that class and racial distinctions are maintained through the professionalization process (Zwerman et al., 2004, p. 30). 40 Chapter 2: Literature review and preliminary research

While these later theories may represent a more sophisticated approach, when all is said and done, it is the perception of the consuming public which determines what is and what is not considered to be a profession, indicating that while perhaps not as much in favour as before, the traits are important indicators. (e.g Covey, 1991, p. 21; Spencer & Spencer, 1993, p. 12). Since prior research has addressed extrinsic and intrinsic attitudes towards professionalism, the current study focussed on a holistic approach which included not only the semantic perspective, but also legal, economic, sociologic and dictionary definitions. The semantic approach or philosophy derives from the work of Cogan (1955), who in ‘The problem of defining a profession’ identified three strategies or approaches used in defining a profession: 1 Persuasive definitions designed to argue in favor of an occupation being accepted or recognized as a profession; these are the arguments based around safety or health issues, and are amongst the most common rationales for professionalizing an occupation This model applies to engineering sector, medical practitioners and commercial airline piloting. 2 Operational definitions used to facilitate decisions about the organization and practices of an occupation becoming a profession These are the processes or procedures used by engineers, physicians and pilots. 3 Logistic definitions descriptive attempts to establish boundaries or ‘turf’ that an occupation claims as its domain (that which separates a brain surgeon from a heart surgeon, or a chemical engineer from a civil engineer, or the pilot of a 747 from one who flies a commuter airline). It is within the context of the logistic definition that the majority of this thesis is focussed. After Cogan in 1955 several other notable researchers (Goode, 1969; Luthans, 1976) argue that the traditional attributes normally used to identify or classify a profession were no longer valid, and that ‘intangible’ attributes were beginning to change the traditional methods of defining a profession. But the researcher who seemed to

41 Chapter 2: Literature review and preliminary research

capture the essence of this trend was M. L. Haga. Haga (1974) identifies three primary, compelling or intrinsic aspects of an occupation which enable it to move beyond being merely an occupation, ultimately evolving into a profession. These three intrinsic attributes are: 1 cruciality 2 mystique 3 denouement Example

To illustrate this concept, consider the occupation of commercial airline pilot. Here is a service that some, but not all, deem ‘crucial’. In some cases, even though many don’t like flying, and some actually fear flying, work, social, education or family demand it be done. Flying a plane involves skills, knowledge and competencies that are well beyond the average person’s interests or desires. Given the cost of a flight ticket, why would anyone want to bother investing the time and effort in becoming a pilot when it makes more sense to ‘outsource’ it to a specialist? But once the decision is made to fly, in doing so, you turn over full and complete control to this specialist, the pilot. You are told to sit in a specific seat, and for all intents and purposes, don’t leave that seat until you reach your destination. In between, your life is literally in the hands of a total stranger. The importance of this act of trust cannot be overstated. This is the practical definition we will be using while evaluating the nature of project management. 2.2.3 Researching words and their meanings Profession and professional. In examining prior research into the concept of professionalism, the noun profession was considered from legal, economic, sociological, semantic, and standard English language dictionary perspectives. Summarized, the research paints a comprehensive picture of those attributes commonly identified with all occupations considered to be professions to some degree or another. (Refer to Exhibits and Tables throughout this chapter.). While not all attributes have equal weighting, as acceptance as a profession is more about perceptions than it is about quantifiable criteria, the whole is greater than the sum of the individual components. In terms of perception, the most valid measure is not the extrinsic variables but the intrinsic variables.

42 Chapter 2: Literature review and preliminary research

At the same time as the noun was being investigated, the adjective professional was also examined. Anecdotal evidence indicates that there is considerable confusion between the noun profession and the adjective professional. While one can certainly be professional at what one does for a living, simply being able to earn a living does not necessarily mean that the occupation one has chosen automatically qualifies as a profession. Construction management and project management. Research on the development of construction management as a ‘case study’ for this research proved to be somewhat frustrating. That very little has been written about the history and philosophy surrounding the evolution of construction management as a precursor to the world of project management known today is disappointing. The death in early December of Dr. Ed Haltenhoff, who was the primary source of information about the history of CM, has negatively impacted the research in this area. Other research that proved interesting and relevant was the work of Russia’s Nicolai Kondratieff (1929, cited in Modelski, n.d.), Joseph Schumpeter (1942) and more recently, MIT’s Jay Forrester (1998). Their research on business cycles has relevance to project management. The fact that nothing significant has been developed in the field of project management since the creation of the critical path method (CPM) in the mid 1960s, and made popular by Primavera and more recently, Microsoft Project, indicates some sort of stagnation in the growth cycle, at least for the project management body of knowledge. Yet outsourcing and project based work is becoming more and not less prevalent. This situation exposes an important dichotomy. While the knowledge base of project managers seems to have largely stagnated, the demand for project management as a delivery approach is growing exponentially, at least judging by the membership in the world’s largest organization representing the professional practitioner – the PMI. So if the knowledge base has essentially stagnated, what is the driver behind the popularity of project management as a delivery method? And more importantly, what implication does this have for the future of project management, not so much as a delivery method, but in terms of the perceived level of professional respect accorded it as an occupation?

43 Chapter 2: Literature review and preliminary research

In the 1800s, the French economist J.B. Say (1803) coined the term entrepreneur and defined it as one who ‘shifts economic resources out of an area of lower and into areas of higher productivity and greater yield’ (Younkins, online). Isn’t that essentially what project managers are doing? Say further defined an entrepreneur as ‘one who upsets and disorganizes’ (Younkins, online); or, stated more succinctly by Joseph Schumpeter (1942), the entrepreneur’s role is ‘creative destruction’ (pp. 8285). Is a project manager an entrepreneur according to these definitions? On the other hand, if a company of 10,000 people is downsized to a fraction of its former self, and the work performed at one time by employees outsourced to subcontractors who specialize in doing that particular series of tasks, who coordinates and supervises the work of the subcontractors? Where does project management end and general management begin? Furthermore, when one works for an organization, isn’t the organization in effect ‘lending’ the project manager the name and credibility of the organization, the human and physical resources, interim funding, all in the expectation that the project manager will generate a positive return on investment or achieve some other objective favorable to the organization? The link between entrepreneurship and successful project managers is one that needs to be explored, as does the link between project management and general management. Such an investigation could lead to a new concept of intrapreneurship. 2.2.4 General and dictionary definitions George Bernard Shaw, in his play The doctor’s dilemma (1906), cynically defines professions as ‘conspiracies against the laity’ (Oxford quotations, 2005). Talcott Parsons, (1939) opines that professions attracted people who had a psychological predisposition to seek ‘social recognition for their work over wealth’ (p. 3). Michael Davis (1987) explains that ‘professions differ from self-interest groups such as labor unions and for-profit corporations in that they altruistically aim to ensure that the services they provide are beneficial to those who receive them’ (p. 27). Counterpoint these definitions with Eliot Freidson (1970), who argues that the ‘professions differ from trade unions only in their sanctimoniousness’ (p. 360). More recently, Bowie & Jamal (1995) and Linda Emanuel (1997) note that the public’s 44 Chapter 2: Literature review and preliminary research

expectations of those in the professions is ‘higher than that of those in business’ (Jamal & Bowie, 1995, p. 33) and that the professions apply ‘expert action on behalf of people in vulnerable states’ (Emanuel, 1997, p. 54). Moving away from the altruistic perspective of Emanual (1997), Jamal and Bowie (1995), Davis et al., another group of economists and sociologists, have taken a different approach, that of the knowledge and skill sets required to be a professional. The desired knowledge includes the idea from noted economist Milton Friedman (1970) who believes the ‘social responsibility of those working for corporations was to increase profits’ (p. 8). In between these two extremes, other sociologists, jurists and economists argue from a position that seeks to provide a balance between the altruistic and pure business models. (i.e. capital asset pricing model) taught in most business schools is more appropriate to the concept of the professions. Chief Justice Breitel argues in the Matter of Freeman that ‘a profession is not a business’ and that a profession has higher order standards than a business’ (Matter of Freeman, 34 NY2d 1, 9-10 [1974]). The development of Donaldson and Preston’s (1995) stakeholder model; Dunfee’s (1997) social contract model; or Woods (1991) corporate social performance model seems to indicate a growing awareness that a profession (and by extension, those practitioners who work in the profession) creates obligations which go beyond delivering projects ‘on time and within budget while fulfilling technical specifications’, but also include a softer, more human touch as espoused or implied by Juran’s (1988) ‘fitness for use’ principle and Deming’s (1986) ‘total customer satisfaction’ (p. 3). Given the plethora of project management terms being used and in some cases, intentionally or otherwise, distorted by industry specific interpretations, the current research was based on a generally accepted and respected English language dictionary, rather than one coming from any particular sector or industry. Therefore, unless otherwise stated, Webster’s third new international dictionary (MerriamWebster, 2002) was used as the primary source and basis of the fundamental definition. While this approach is consistent with the methods employed by some jurists (courts), some economists, some sociologists, and nearly all semanticists, the weakness lies in

45 Chapter 2: Literature review and preliminary research

the ambiguity and vagueness contained in classic dictionary definitions. Thus the journey started with the dictionary definition but included an examination of restatements, interpretations or deviations from the standard definition as required, in order to conclude with a definition for profession and professional which worked in the context of project management. As the research was global in scope, the definitions were cross checked against the Concise Oxford English dictionary (OUP, 2004) and if there were any discrepancies, those were noted. For comparisons between different sectors and professional organizations, Wideman’s (2004) Comparative glossary of project management terms (version 4.1) was used. The need for a comparison was important to the research because project management has become a veritable Tower of Babel of terms, definitions and interpretations. While the project management Institute has done work to standardize the terminology, their success has been limited by a lack of consensus globally. Profession: The noun. Webster’s third new international dictionary (MerriamWebster, 2002) defines profession thus: profession: a calling requiring specialized knowledge and often long and intensive preparation including instruction in skills and methods as well as in the scientific, historical, or scholarly principles underlying such skills and methods, maintaining by force of organization or concerted opinion high standards of achievement and conduct, and committing its members to continued study and to a kind of work which has as its prime purpose the rendering of a public service. learned professions: theology, law and medicine

This definition and any derivatives, enhancements or clarifications will form the basis of this research. In order to develop a full and complete understanding of these terms, let’s break this definition down further into the component elements (Exhibit 2.1).

46 Chapter 2: Literature review and preliminary research

Exhibit 2.1 Examination of words used to define the concept of profession Word

Definition

calling

impulse or desire to follow particular job: a strong urge to follow a particular career or do a particular type of work

specialized

devote time to particular activity: to devote time exclusively to a particular interest, skill, or field of study

knowledge

information in mind: general awareness or possession of information, facts, ideas, truths, or principles; all that can be known: all the information, facts, truths, and principles learned throughout time

long

going on for lengthy period: lasting for an extended period of time

intensive

concentrated: involving concentrated effort, usually in order to achieve something in a comparatively short time

preparation

preparatory measure: something done in advance in order to be ready for a future event

skills

ability to do something well: the ability to do something well, usually gained through experience and training; something requiring training to do well: something such as an art or trade that requires training and experience to do well

methods

way of doing something: a way of doing something or carrying something out, especially according to a plan; body of scientific techniques: the body of systematic techniques used by a particular discipline, especially a scientific one

principles

basic assumption: an important underlying law or assumption required in a system of thought; ethical standard: a standard of moral or ethical decisionmaking

organization

group: a group of people identified by shared interests or purpose, for example a not-for-profit professional organization

concerted opinion

achieved or performed together: an opinion reached by two or more people working together or with the same aim

high standards (of achievement and/or conduct)

above average: greater than the normal or average, for example in quantity, number, quality, intensity, or cost, or well above a smaller or lower level or amount

continuing education

specialist courses to update professionals: regular courses or training designed to bring professionals up to date with the latest developments in their particular field; lifelong adult education: adult education, usually in the form of short or part-time courses, continuing throughout an individual’s life

providing a public service

provision of essential services: the business or activity of providing the public with essential goods or services such as electric power; service benefiting the general public: a service that is run for the benefit of the general public, for example the utilities, the emergency services, transport, and broadcasting

Restating the definition of profession using the definitions above, and put in the context of project management (or any other occupational specialty) the definition of profession can be more completely rephrased as: A strong drive or desire to do the kind of work normally associated with (project management; or insert your chosen occupation), by devoting considerable time and effort, over a long period of time, to master as much information, facts, truths, and principles as possible, by making a deliberate and concerted effort to develop, through training, experience, and by taking specialized adult education courses, over a life time spent as a (project manager), those systematic techniques to be able to execute the work of a 47 Chapter 2: Literature review and preliminary research

(project manager) in a sound, business-like manner, by adhering to above average standards of ethical, moral and legal care, as defined by an organized group of practitioners with shared aims, the primary one being serving the interests of the general public.

This definition of project management has been elaborated in whole or in some part or another through lawyers, economists, sociologists and practitioners, but no single source has been found which brings all aspects of a profession together in a single document. It is believed the current research is the first to combine the legal, social, economic, semantic and dictionary definitions and apply them to the practice of project management. Professional: Noun, adjective and confusion. It became clear during the literature search, conversations with colleagues around the world and listening to clients, that there is considerable ambiguity and nuance associated with the noun profession and the noun professional, along with the adjective, professional. It was imperative before proceeding further with the research, therefore, to make the distinction between the terms as clear and as unambiguous as possible. For the noun, profession, this proved more difficult than expected due to common usage that has seen the word’s meaning expand to include contexts that are not encompassed by the standard definition offered at the beginning of this section. Use of the adjective and the noun professional in a wide variety of contexts has resulted in profession expanding its meaning and becoming more generalized in everyday speech and writing. The word professional in both nominal and adjectival construction, varies with context. The dictionary definition of the adjective, professional is: professional (prə fésh’nəl) adj very competent: showing a high degree of skill or competence following an occupation as a paid job: engaged in an occupation as a paid job rather than as a hobby

The dictionary definition of the noun, professional is: professional (prə fésh’nəl) n a professional person, not an amateur a person having impressive competence in a particular activity

48 Chapter 2: Literature review and preliminary research

To illustrate how broad the definition of professional is, and how this has affected the original meaning of profession, let’s revisit and reconsider the anecdote about Tiger Woods in the context of the above definitions: Tiger Woods is unquestionably a talented golfer. One would be very hard-put to dispute the obvious, which that he is very competent at what he does, perhaps one of the best ever. Therefore he meets the first test of being a professional (n) – skill and competence. In fact, he is sufficiently competent that he makes a very handsome living performing for pay what most of us consider a hobby; hence, applying the second criterion, he meets the ‘earnings test’ to be considered a professional (n). He is not an amateur. Having met both tests (highly competent and earning a living at what most do for a hobby) entitles him to be termed a professional (adj.) golfer. However, just because Tiger Woods meets the criteria to be called both a professional (n) and a professional (adj) golfer, golf does not qualify as a profession, although Woods might call it his profession (his paid job).

It is no wonder that many in the community of practice of project management confuse what is means to belong to a profession. There is the tendency to make the connection that if they are in fact professional (extremely competent) in the way they work, then what they do must, by association, be considered a profession. This is false logic and a semantic trap easily fallen into. The analogy can be expanded to include not only golf and other sports, but many occupational specialties including acting, singing and other talent-based activities. Taken to an extreme, consider applying the above test to the world’s oldest ‘profession’ – prostitution. As will be shown through analysis of the data collected, failure to make the requisite distinction between the noun profession and the noun and adjective professional is a semantic lapse which must be addressed and resolved by all occupations trying to ‘professionalize’ what they do. Project and management. There are two remaining terms which must be defined prior to proceeding further with the exploration of this topic. They are project and management. project (prójjekt) n (plural projects) unit of work: an organized unit of work proposed or planned undertaking: to achieve a particular aim or objective; a scheme for which there seems hope of success;

49 Chapter 2: Literature review and preliminary research

management (mánnijmənt) n administration of business: the organizing and controlling of the affairs of a business or a particular sector of a business handling of something successfully: the act of handling or controlling something successfully skill in handling or using something: the skilful handling or use of something such as resources

Project management, according to Webster’s third international dictionary (MerriamWebster 2002) is: A proposed or planned undertaking to achieve a particular aim or objective, with some reasonable expectation for success, through the skillful handling or use of resources and the successful organization, administration and controlling these affairs in a business-like and ethical manner.

Life skill. A final definition important to establish as the journey continues is life skill. Project management, using the dictionary definition, is an essential life skill which everyone must master to some degree or another in order to survive as human beings. How then can project management ever be considered as a profession unto itself? To reinforce this point, let us use the analogy of accounting. At the most basic level, each and every one of us has to learn some basic accounting skills; otherwise we would be unable to function in the modern world. Balancing our check book, purchasing automobiles, houses and other assets, managing our credit cards, planning for retirement, all require we understand basic accounting functions and principles. Obviously, some individuals have great difficulty balancing their check books each month, while other ‘amateurs’ are sophisticated investors. So it is with project management. Some individuals manage their lives with much greater aplomb than others. And, unlike golfing, basic accounting skills are life skills, without which we risk being unable to function adequately. None of the preferred dictionaries used for this study – the Concise Oxford English dictionary (OUP, 2004), the Encarta dictionary online (Microsoft Corporation, n.d.) or Webster’s third new international dictionary (Merriam-Webster, 2002) – contain a definition of life skill, and lacking any specific academic research containing a citable definition, for the purposes of this research, the definition provided by Wikipedia (n.d.) was used:

50 Chapter 2: Literature review and preliminary research

Life skills are the skills necessary for successful living, whether that is with a family, with a career, or institutional. These include setting a worthwhile and realistic goal, and then developing a plan to accomplish that goal that details the steps, courses, money, time, and reasons for achieving the goal. Then, step by step, one moves towards the goal. (n.d., online)

To summarize, Webster’s third new international dictionary (Merriam-Webster, 2002) definitions of profession, professional, and project management were used as the basis for this research, including the restatements as appropriate. However, the nuances between how a profession is perceived from legal, socio-economic and semantic stances must have considerable bearing on the acceptance of any occupation as a profession.

2.3 The concept of a profession: Legal definitions Author’s note: This research is not intended to constitute or be relied upon as legal advice, but a description of applicable principles in the context of an academic research on project management. Please consult with your attorney for specific legal advice on issues or questions raised or contained in this research.

2.3.1 Legal definitions Legal definitions of what constitutes a profession have a profound influence on any occupation gaining acceptance as a profession. Of all the perspectives pertaining to defining a profession, in the end, it is the legal definition which will most likely prevail over all others. Nevertheless, in the US, both state and federal legislators have been reluctant to define the term, fearing repercussions from voters if their occupation happened to be excluded (Polelle, 1999; Gawley, 2002). Likewise, the courts have often provided ‘varying, inconsistent and frequently conflicting methods of analyzing which occupations merit the protections conferred by the title of profession’ (Polelle, 1999,). About the closest legislative definition of a professional derives from the United States Code (Labor Management Relations Act 29 USC152(12) 1995) which states: The term ‘professional employee’ means A. any employee engaged in work which is: predominantly intellectual and varied in character as opposed to routine mental, manual, mechanical, or physical work; involving the consistent exercise of discretion and judgment in its performance; of such a character that the output produced or the result accomplished cannot be standardized in relation to a given period of time; 51 Chapter 2: Literature review and preliminary research

requiring knowledge of an advanced type in a field of science or learning customarily acquired by a prolonged course of specialized intellectual instruction and study in an institution of higher learning or a hospital, as distinguished from a general academic education or from an apprenticeship or from training in the performance of routine mental, manual, or physical processes; OR B. any employee, who (i) has completed the courses of specialized intellectual instruction and study described in clause (iv) of paragraph (a), and (ii) is performing related work under the supervision of a professional person to qualify himself to become a professional employee as defined in paragraph (a).

Compared against the current Merriam-Webster’s (2002) definition, the two remain close, but with important differences. What is it that separates ordinary business people from professionals? What provides professionals with special status in the eyes of the law? For most people, the law applies a standard of ‘ordinary negligence’ as defined or perceived by the ‘common’ or ‘average man’. However, those accorded the status of professional receive preferential treatment in five areas: „

peer reviews – standard of practice established by ones professional peers

„

medicine school rule – standard determined by discipline or sector

„

locality rule – standard determined on geographical location

„

error of judgment rule – all things being equal, benefit of the doubt goes to the practitioner

„

limited or reduced statute of limitations – reduced time in which consumers can initiate legal actions for malpractice/professional negligence.

Peer review. Professionals, because of the complex nature of their work, are afforded special protection because, it is argued, the ‘ordinary man’ does not understand the complex nature of a professional’s work. The only people capable of judging whether a professional acted appropriately under a given set of circumstances would be fellow professionals. So in the context of proving negligence or professional liability, the burden of proof is not ‘ordinary negligence’ as defined by the ‘common man’, but by a standard set by the professional practitioner’s own peers. Restated, professionals are judged by the standard of ‘what is normal and customary in the profession’, not ‘best practice’ but merely an ‘average’ level of care. In exchange for this apparent conflict of interest and marginal standards, the practitioner must demonstrate that they adhere to a code of ethics/code of conduct

52 Chapter 2: Literature review and preliminary research

which clearly puts the interests of the consuming public before that of the commercial interests of the practitioner. Effectively, in exchange for the right to be self governing, a professional needs to put service to the public above and before any commercial interests. However, there are even more special legal privileges accorded those occupations deemed to be professions and professionals subject to peer review. Ordinary care concept. While the standard for non-professionals is ‘ordinary negligence’, the standards the peer review boards or experts must establish are not ‘best practice’ within any occupation, but merely what a ‘reasonably careful practitioner in the same field would or would not do if they practiced in the same locality under the malpractice defendant’ (Polelle, 1999; Gawley, 2002). Thus the level of care is not the ‘best’, but merely an average, and that average is not a global or even a national average, but a local one. This legal principal has important relevance in establishing, much less enforcing, global standards of competency. School of medicine rule. Another important part of the peer review concept is the school of medicine rule, which states that in setting the standard of care, the peer experts setting that standard of care must be from the same school or discipline. (Polelle, 1999). Applied to project management, it would mean that only IT project managers could set the standards against which IT project managers could be judged; construction project managers would be the only ones qualified to set the standards for fellow construction project managers. The same applies to other disciplines within project management. The impact of this on the professionalization of project management is critical, as it contradicts the principle being advocated by several organizations that there is a single methodology which will work for all cases. It also raises some interesting topics for discussion about setting global competency standards. Locality rule. Further restricting the definition of a profession is what is known as the locality rule, which states that not only must the peer review committee/experts come from the same school or discipline, but ‘interpreted strictly, those setting the standards must also be from the same geographic region’ (Polelle, 1999,). Thus a practitioner living and working in a remote site could only be evaluated by others who live and work at remote sites; or an Australian practitioner could only be measured against the standards of fellow Australians, not Americans, not Europeans. This raises another important issue for those attempting to establish global project 53 Chapter 2: Literature review and preliminary research

management standards, for only airline piloting has come close to establishing truly global standards of practice, and even within that application, not all standards are the same in all countries. Honest error of judgment rule. The honest error of judgement rule is another very special rule which applies only to professional practitioners. This rule states that ‘if a practitioner must choose between two or more courses of action, all of which may be appropriate under similar circumstances, then it is not malpractice to choose one over the other’ (Polelle, 1999,), even if, in hindsight, one would have been preferable over another. Statute of limitations. Lastly, a crucial difference between ordinary negligence and professional negligence lies in the statute of limitations, or the amount of time between an event occurring and when an aggrieved party may file suit against a practitioner. In almost all jurisdictions, the statute of limitations is shorter for professional negligence than for ordinary negligence (Polelle, 1999, p. 3). As can be appreciated, given the special treatment accorded the professions, determining whether an occupation is or is not a profession has profound impacts in terms of legal liabilities. The entire legal concept of peer review should be taken seriously as we move from skills and knowledge-based credentials to competency-based credentials. Especially important for any occupation working to establish global competency-based credentials is the need to address the average vs. the best practices issue, the school of medicine issue and the locality issue. For inevitably, challenges will occur around these issues, and better to address them up front than after the credential has been put in place. 2.3.2 Fiduciary responsibility defined and explained Polelle (1999), Schein (1972) and Cox (1990) offer a good grounding in the concept of fiduciary responsibility, and the issue of fiduciary responsibility to the consuming public remains a common thread throughout most research related to project management. The concept is not always a well understood, however, or accepted (Simon & Reish, 2006), especially in the developing nations (Grace, 2002; Harden, 1968). Rather than raise questions about fiduciary responsibility with the respondents, therefore, the research tool was a key word analysis of documents produced by those 54 Chapter 2: Literature review and preliminary research

organizations most often representing project managers, compared against the benchmark of the Employee Retirement Income Security Act of 1974 (ERISA). This US federal law sets minimum standards for most voluntarily established pension and health plans in private industry to provide protection for individuals in these plans. Further research indicated that the most closely related application of fiduciary responsibility to project portfolio, program and project management could be found by looking at the modern portfolio theory found in the Uniform Prudent Investor Act and the Restatement 3rd of Trusts (Prudent Investor Rule) (Simon, 2005) and the ERISA 404(c) rules (USDoL, 2006). To understand fiduciary responsibility in the context of project management, it is important to recognize the fact that project management involves often complex relationships between the project manager, subcontractors and the project sponsors. Aspects of these relationships have led to the development of principal – agent or agency theory which recognizes the inevitable tensions that will arise between an agent (the project manager) and a principal (the project sponsor) when the principal must transfer some of the decision making authority to the agent: If the aim of both parties is to maximize their economic position, then there is good reason to believe that the agent will not always act in the best interests of the principal. Delegation of decision making authority from principal to agent is therefore problematic…. (Muller & Turner, 2005, p. 399)

It is common during a project for the project manager to understand all issues, great and small, more fully than the sponsor; and to be making decisions without the sponsor fully understanding why or whether they are the right decisions from their point of view. Furthermore, since the project manager has their own interests to consider, the interests of the sponsor will be best served only when the two interests are aligned. Such an alignment can be managed by entering into appropriate contracts, but, even with these instruments in place and legally enforceable, the success of the project will hinge largely on the ethical behaviour and willingness to communicate shown by all parties involved. Explained in more detail: think of fiduciary responsibility as a sacred trust between you and your project stakeholders – backed up by federal law. You want to do the right thing for your stakeholders, and they are depending on you to make decisions in their best interests. What if you make a mistake? What if you depend on subcontract or outsourced service providers who don't have your stakeholders' best interests at heart? Can you plead ignorance? Can you say you got bad advice, your advisor or consultant didn't give you the whole story?

55 Chapter 2: Literature review and preliminary research

Sorry, but excuses may not keep you from being sued, and they won't protect you from liability. According to the Employee Retirement Income Security Act of 1974 (ERISA), as amended, the federal law that governs proper conduct regarding retirement plans, ignorance is no excuse. (Gnabasik, 2002, p. 29)

To put apply the principles contained in ERISA to project management: most fiduciary breaches are the result of a lack of prudence. Many plan sponsors are not aware that they are a fiduciary and don’t understand what responsibilities and potential liabilities such a designation entails. Following these steps should prevent most of the violations that lead to lawsuits: Determine who is a fiduciary Briefly, a fiduciary is anyone who (1) Exercises discretionary control over the management of a plan; (2) Has any discretionary authority or responsibility regarding plan administration or execution; (3) Offers investment advice regarding plan assets and derives compensation from it. Although determining who exactly is a fiduciary depends on the facts and circumstances of each case, fiduciaries usually include the plan administrator designated in the plan document and which may be an administrative, or investment committee, the investment manager and the broker/consultant, if the plan relies on her advice. (Gnabasik, 2005)

As can be appreciated by restating this criterion in the context of project management, applying ERISA legal principles certainly would appear to create fiduciary responsibilities not only in project managers, but also in portfolio and program managers as well. Review basic fiduciary responsibilities The next step is to remind everyone of a fiduciary’s basic responsibilities. First, fiduciaries must act in the best interest of participants and their beneficiaries (stakeholders). Although this is usually only a problem involving cases of ‘selfdealing’ or conflicts of interest (the classic example involves selling goods or services at an inflated price compared to the planned value), it also means that sponsors have a duty to keep costs reasonable. Finally, to quote ERISA, plan decisions must be made ‘with care, skill, prudence and diligence under the circumstances then prevailing that a prudent man acting in a like capacity and familiar with such matters in the conduct of an enterprise of a like character and with like aims.’ This means that a fiduciary must seek outside expertise when lacking the requisite education, experience or skill. (Gnabasik, 2002, p. 32)

Evidence of this is both clear and emphasized through the civil engineers Obligation of an Engineer, but is largely ignored in the leading organizations representing project management (PMI, AIPM, IPMA Code of Conduct/Code of Ethics). Perform a self audit ERISA stipulates that each plan (project) must have a plan document and that the plan is operated (executed) according to those plan documents. An annual spot check of basic plan operations (e.g., rollovers, loans, hardship withdrawals, nondiscrimination testing, etc.) and a review of all required plan documentation will go a long way in preventing any potential problems. (Gnabasik, 2002, p.33)

56 Chapter 2: Literature review and preliminary research

Implicit in the context of portfolio, program or project management would be the requirement for portfolio, program and project managers to take more responsibility in creating a project plan and then ensuring it is followed. While this is done by the more sophisticated applications of project management (for example, construction project management and government contracting) under the auspices of claims prevention/documentation, applying the ERISA standards would make it mandatory for all practitioners to apply the same level of diligence. Ensure a sound process is in place for investment (project) selection and monitoring Plan sponsors need a process in place for selecting and monitoring investments.(projects) Many advisors work with their clients by drafting a customized investment policy statement (project plans) and helping their clients to monitor the investment (project performance against the plan) on a regular basis. The important point here is that a plan sponsor has a bona fide process in place that they can refer to should anyone question a plan’s investment decision-making process. (Gnabasik, 2002, p.33)

The implications of this ‘test’ should be clearly evident, especially to those who are at the program or portfolio management level. Applying the ERISA laws to project management, it would appear to mean that an organization MUST have in place a process or methodology, or risk being held accountable for breaches of their fiduciary obligations. Consider buying fiduciary liability insurance Advisors need to apprise their clients of the potential liabilities associated with fiduciary status. Fiduciaries can be held personally liable for breaches. In addition to the ERISA-required fidelity bond, advisors may want to raise the idea of buying additional fiduciary liability insurance for the plan. Large plans routinely buy such extra protection as it is relatively inexpensive and offers broad coverage. (Gnabasik, 2002, p. 34)

It would be hard to argue against the analogies between portfolio, program and project management in the context of ERISA laws. Whether the practitioners of project management and the organizations which represent them are willing to accept this liability in return for having project management recognized as a profession remains to be seen, but having made this comparison, it would appear as though Polelle (1999) is correct in stating that: judges should take judicial notice that a particular occupation qualifies for professional malpractice status when it has put in place and actively enforces an express and fiduciary system of ethical regulation of its members, FOR THE BENEFIT OF THE PUBLIC SECTOR rather than serve the anticompetitive instincts of the regulated members. (n.d., online)

57 Chapter 2: Literature review and preliminary research

2.3.3 ‘Legal no man’s land’ in defining a profession While we know ‘legislators are reluctant to address the issue of defining a profession for fear of alienating potential voters’ (Gawley, 2002,), the courts have not been much better. Generally speaking, the courts have taken three approaches to defining a profession (Polelle, 1999): „

Limit the special protections accorded under malpractice law to those professions recognized by early common law (doctors, lawyers, educators and clergy).

„

Recognize occupations with a license to practice issued by a government entity.

„

Choose some ‘middle course’ between the two extremes.

As can be appreciated, under the first scenario, architects, engineers, airline pilots and others who play crucial roles in the health, safety and welfare of the consuming public would not be recognized as being professions. Taken at the other extreme, if the ‘licensing’ test were applied, that would include not only doctors, lawyers, teachers, architects, engineers and airline pilots, but also encompass barbers, beauticians, school bus drivers, truck drivers, electricians, plumbers and a host of other less common occupations. Gales Professional and occupational licensing directory (1996) contains over 1,300 occupations in the US which require a license as a precondition upon which to work at an occupation. The list includes the occupations of abstractor through wrestler, and some pretty bizarre and unusual occupations in between. Given the alternatives between two such extremes, most courts, at least in the US, tend to favor some middle of the road approach (Polelle, 1999). Amongst the middle of the road approaches has been the use of standard dictionary definitions, restated in the context of the case. However, as can readily be seen from the standard dictionary definition used for profession, a great deal of the meaning can be subjective. What constitutes ‘specialized knowledge’ or ‘long and intensive preparation including instruction in skills and methods as well as in the scientific, historical, or scholarly principles underlying such skills and methods’? Furthermore, as evidenced by Wideman’s (2004) Comparative dictionary of project management terms, having multiple meanings for even such basic terms as project or project manager only confuses the problem further. Finding that dictionary definitions were not precise enough for legal purposes, two US states, Florida and North Dakota, out of exasperation that the legislators were unwilling to address this issue (Gawley, 2002), attempted to develop their own standards.

58 Chapter 2: Literature review and preliminary research

In Pierce v. AALL Insurance Inc. [513 So.2d 160, 161 (Fla. 5th DCA)], the Florida Supreme Court held that a common denominator in all occupations generally considered to be professions was holding a four year degree or better. In Pierce, the courts held that an insurance salesman, despite needing a license to practice, and requiring special knowledge was not a profession, as holding a four year degree was not required. This was supported by the findings of the North Dakota Supreme Court in Jilek v. Berger Electric (441 N.W. 2d 660 [ND, 1989]) in finding that being an electrician was not a profession, for while it did require a license and it did require a long period of training and specialized knowledge, it did not require, at minimum, a four year college degree (Polelle, 1999). The impact of this on project management is profound, for none of the most commonly recognized certifications in project management require a four year degree. It is worth considering how other occupations have been treated by the courts in terms of their status as professions. EDUCATORS

Worth noting in the context of project management is the fact that in the field of education the (US) courts have overwhelmingly refused to allow a tort action (malpractice suits) for a number of reasons, including: „

lack of satisfactory standards against which to evaluate the academic performance of educators

„

inherent uncertainties about damages

„

excessive involvement of the judiciary in overseeing higher education

„

potential for a flood of litigation (Polelle, 1999).

This decision has significance for project management in so far as not only have project managers not been able to agree on terminology or definitions (Wideman, 2004), but they have as yet to establish standards of competence. While these are in the process of being developed (Crawford, 2005), they are far from completed, much less widely accepted and adopted. There is also the question of ascertaining damages. As project managers generally do not control the scope of the project, the time for performance or the initial budgeting, should they be held financially liable or accountable for a project that fails to deliver what the client wanted? What of the project manager who completes a project on

59 Chapter 2: Literature review and preliminary research

time, within budget and in substantial conformance with the technical specifications, but the project as originally conceived was flawed? Should the project manager assume financial liability for direct damages? Consequential damages? While these are rhetorical questions for the purpose of this research, they are serious questions which at some point will have to be answered. Like education, it would seem ridiculous to expect the courts to become involved as the arbiters of what is ‘good’ or ‘bad’ project management any more than they should get involved in defining what is ‘good’ or ‘bad’ education. That should derive from and amongst those who provide project management services and those who consume them. Noted legal scholar Roscoe Pound (1953) defined profession as ‘a group of men (sic) pursuing a learned art as a common calling, in the spirit of public service, its work no less a public service because it is also a means of livelihood’ and in this context, the role of the professional organization in negotiating with the end consumer becomes of critical importance. Laws such as the Sarbanes Oxley Act and BASIL II5, however, may have opened the door ever so slightly for project managers to be held accountable for the consequences of their activities in the foreseeable future, opening floodgates of litigation for malpractice. If and when that happens, it will be interesting to see if the courts will determine project management to be a profession, subject to the special treatment accorded professionals or not. Worth noting at this point is UK courts have established an early precedent, holding that project management is not a profession (Pride Valley Foods Ltd v. Hall& Partners, 2000; EWHC Technology 106, 2000, Line 140). CLERGY

As with teachers, lacking any clear standards of care against which to baseline, the US courts have been reluctant to entertain malpractice against clergy. Except in such egregious instances such as Catholic priests molesting children, other considerations

5

The Basil II accord was reached in May, 2004 by a committee of the Bank for International Settlements. It calls for a new method of reporting solvency ratios and risk management to be adopted by the end of 2006. All European banks are required to follow that accord. In addition, in June, 2002, the European Union adopted a regulation requiring all ‘listed companies, including banks and insurance companies, to prepare their consolidated accounts in accordance with IAS [now IFRS] from 2005 onwards’, with some provision for deferrals until 2007. http://www.xbrl.ca/newsletters/01/Hyperlink%208.htm accessed 18 Feb 07

60 Chapter 2: Literature review and preliminary research

(not the least of which in the US is the 1st Amendment rights separating Church from State) have over ridden the courts’ willingness to entertain malpractice suits (Polelle, 1999; Gawley, 2002; Rotunda, 2006). ARCHITECTS AND ENGINEERS

Almost without question, architecture and engineering have been held to be professions. However, the tests applied appear to be somewhat arbitrary, and as both professional architecture and engineering organizations have been functioning in the USA from the mid 1800s, they have had more time than most organizations in which to establish their claims on the ‘tasks’ they do (Abbot, 1988). Much of the basis for architects and engineers claiming to be a profession is based on the legal principle of estoppel. Under estoppel, if you claim specialized knowledge or skills in doing something, and you fail at doing it correctly, then you are stopped or prevented from using as a defense that you didn’t actually have that knowledge or skill. Similar to the rule of locality and the same school rules, the second restatement of torts concludes that ‘one who undertakes to practice a profession or a trade, will be held to the skill and knowledge normally and customarily held by members of that trade or profession in good standing in similar communities’ (Polelle, 1999). As the services rendered by both architects and engineers have a clear and direct bearing on the public safety, in many countries, these occupations require a license to practice, specifically to protect the interests of public safety. However, one very important case has relevance to the current state of project management. In Garden v Frier [602 So.2d 1273 (FL 1992)] the Florida Supreme Court held that land surveying was not a profession, because the land surveying license requires a four year degree or additional experience in lieu of a degree. Although the defendant in the case had a four year degree, it was not universally required, so the practice of land surveying was not considered to be a profession. This has a profound implication for all of the most popular or recognized certifications in project management, as none (at least as of this writing) requires a four year degree. BURDEN OF PROOF

The definition of the concept of burden of proof was essential to this research. As both qualitative and quantitative analysis were utilized in this study, and the definition of any occupation is subjective at best, different burdens of proof were applied

61 Chapter 2: Literature review and preliminary research

throughout the research. Three relevant burdens of proof were taken from the legal system, and a fourth, semantic definition – tipping point –was applied as appropriate and necessary. Generally speaking courts tend to apply three burdens of proof (Simon, 1970): Preponderance of the evidence. This is also known as the ‘balance of the probabilities’. It is a standard or ‘test’ applicable in civil (non-criminal) cases. The standard has been met if the allegations of fact are more likely to be true than false. Essentially, this standard requires 51% probability or more that the allegations are true (Nolo, 2006). Clear and convincing evidence. An intermediate or stronger burden of proof also applied in some US civil cases is the ‘clear and convincing evidence’ standard. Applying this test requires that the judge or jury be convinced that the facts are ‘substantially more likely than not to be true’. For the purposes of this study, clear and convincing evidence was defined to be about 80%, but less than 95%, which would meet the test of ‘beyond a reasonable doubt’. The 80% value is derived from the findings of Judge Jack B. Weinstein in United States v. Copeland (228 F.Supp.2d 267) (EDNY 2002) (Tillers, 2006). Beyond a reasonable doubt. This standard or test applies primarily to criminal cases (Nolo, 2006). Applying this standard, the party responsible for proving the facts of a case must do so to the extent that there exists no ‘reasonable doubt’ in the mind of the judge or jury in determining the facts of a case. Based on Rita Simon’s research, ‘the quantifiable definition of ‘beyond a reasonable doubt’ would be around 95%’ (Saunders, 2005, p. 3). Tipping point. There is yet another ‘burden of proof’ coming from semantics. Malcolm Gladwell, in his 2002 best selling book The tipping point – How little things can make a big difference, defines a tipping point as ‘that magic moment when an idea, trend or social behavior crosses a threshold, tips and spreads like wildfire’ (p. 12). John Sterman (2000), in Business dynamics: Systems thinking and modeling for a complex world, defines tipping point as: A threshold condition that, when crossed, shifts the dominance of the feedback loops that control a process. (p.305 )

62 Chapter 2: Literature review and preliminary research

Systems tend to remain stable as long as conditions remain ‘below’ the tipping point and controlling feedback is dominant. (p. 306). But when conditions cross the tipping point behavior can become (temporarily) unstable and, in the case of projects, lead to failure. (p.314)

Despite the popularity of tipping point, and subsequent qualitative validation of the concept, no research could be identified which came up with an actual, quantifiable value. However, consistent with the findings of Barabasi (2003) in Linked, and given that the work of Vilfredo Pareto (1896-97) figures so prominently in project management (PMBOK, 2004), for the purposes of this study, a tipping point of 80% was assumed, pending further research. As the concept of a tipping point may well describe the global popularity of project management, it would have been remiss not to incorporate it as a consideration in the current study. Legal definitions summarized. While this research is not intended for legal use, the following terms and their associated values were used, noting that the quantifications for the burden of proof have not been established within the legal community. These terms and their associated values were used for the purposes of consistency only (Table 2.1). Ultimately, the primary legal hurdles for any occupation being recognized by the judiciary come down to five major considerations: „

Does the occupation have in place and actively enforce an express and fiduciary system of ethical regulation of its members? (Polelle, 1999)

„

Is the system of ethical regulation focused on the benefit of the consuming public and not to serve the anti-competitive instincts of the regulated members? (Polelle, 1999)

„

Does the occupation require a license to practice? (Morgan & Rotunda, 1995)

„

Does the occupation require a four year degree or better? (Morgan & Rotunda, 1995)

„

Does the practitioner have a demonstrated commitment to public service? (Gawley, 2002)

63 Chapter 2: Literature review and preliminary research

Table 2.1 – Summary of legal definitions of a profession Occupational specialty

Profession yes/no

Underlying philosophy, rational or legal argument

Selected citations

no

lack of standards; limitations in establishing damages; potential flood of litigation; excessive involvement of the judiciary in religion; first amendment (separation of church and state)

Barnes v Outlaw, 937 p.2d 323, 327, [AZ, 1996]; Nally v Grace Community Church, 763 P.2d 948, [CA 1988]

no

lack of standards; limitations in establishing damages; potential flood of litigation; excessive involvement of the judiciary in education;

Ross v Creighton Univ, 957 F.2d 410,413-415, [7th Circuit, 1992]; Andre v Pace University, 655 N.Y.S. 2d 777 [NY App Term 1996]

social workers

sometimes

only those who are licensed and providing counseling similar to psychologists are considered professionals.

Horak v Biris, 474 N.E.2d 13, 17, [IL Appelate Court, 1982]; Crain v Martinez, 93-942, Civ-ORL-22, 1994

physicians, dentists, surgeons and podiatrists

yes

standards exist; damages can be established; professional organizations exist;

Mathews v Walker, 296 N.E.2d 569570, Ohio Ct Appl, 1973; Bryant v Bauguss, 1993

no

lack of independent medical discretion in making decisions

Richardson v Doe, 199 N.E.2d 878 Ohio, 1984; Kambas v St Joseph's Mercy Hospital, 205 N.W.2d 431,435, [MI, 1973]

yes

yes if licensed, no if unlicensed, but generally, yes.

Hospital Computer Systems v Staten Island Hospital, 788 Supp. 1351, 1992; Javier v Immigration and Naturalization Service, 335 F Supp. 1391, 1971; Owyhee County v Rife, 593 P.2d 995 [ID 1979]

no

lack of advanced education and training

Advincula v United Blood Services 678 N.E.2d 1009, 1018 [IL 1996]; Owyhee County v Rife, 593 P.2d 995 [ID 1979]

no

lack of advanced education and training

Advincula v United Blood Services 678 N.E.2d 1009, 1018 [IL 1996]; Owyhee County v Rife, 593 P.2d 995 [ID 1979]

no

lack of advanced education and training

Advincula v United Blood Services 678 N.E.2d 1009, 1018 [IL 1996]; Owyhee County v Rife, 593 P.2d 995 [ID 1979]

barbers & cosmetologists

no

lack of advanced education and training

Advincula v United Blood Services 678 N.E.2d 1009, 1018 [IL 1996]; Owyhee County v Rife, 593 P.2d 995 [ID 1979]

pharmacists

yes

advanced education and training required

Harrell v Lusk, 439 S.E. 2d 896, 898, [GA 1989]

computer programmers

maybe

no – professional service was merged into the final product; yes – holding themselves out to be professionals. (estoppel)

Hospital Computer Systems v Staten Island Hospital, 788 Supp. 1351, 1992; Data Processing Services v LH Smith Oil Co. 492, N.E.2d [ID Ct.Appl. 1986]

real estate brokers

no

license required, but lack of advanced education and training (4 year degree not required)

Tyle v Zoucha, 412 N.W.2d 438 [NB, 1987]

clergy

teachers/educators

nurses

accountants

insurance salesmen,

embalmers

pest control

64 Chapter 2: Literature review and preliminary research

Occupational specialty

Profession yes/no

Underlying philosophy, rational or legal argument

Selected citations Pierce v. AALL Insurance Inc. [513 So.2d 160, 161 (Fla. 5th DCA)], Garden v Frier, 602 So.2d 1273 [FL 1992]

land surveyor

no

same as project management. 4 year degree is not required. it can be substituted for more experience in the field.

electrician

no

no college degree required

Jilek v Berger Electric, 441 N.W. 2d 660 [ND, 1989]

no

for aircraft piloting, questions of tort liability arising out of the operation of aircraft, ordinary rules of negligence have been applied.

Southern Helicopter v Jones, 379 S.W.2d 10 [AR, 1964]; T-Craft Aero Club v Blough, 642 P.2d 70 [ID 1982]; Mackey v Miller, 273 S.E.2d 550 [VA, 1981]

there is no chartered or professional institution of project managers nor a recognizable profession of project managers No 4 year degree required

Pride Valley Food v Hall & Partners, England and Wales High Courts, EWHC Technology, 106, 2000, line 140 Pierce v. AALL Insurance Inc. [513 So.2d 160, 161 (Fla. 5th DCA)], Garden v Frier, 602 So.2d 1273 [FL 1992]

airline pilot

project managers

no

This summary provides an overview of the rather limited or strict legal interpretations of what occupations are considered professions from a judicial perspective. But to quote Polelle from his 1999 law review Whose on first and what’s a professional? , society will not tolerate any contradiction between professional enthuses which seek to reap the benefits of an uninhibited marketplace mentality while at the same time demanding insulation from the competitive costs of that same marketplace.

This same philosophy is reiterated by Freidson who identifies three ideal or imagined models of work (Freidson, 2001): Free Market Model (based on Adam Smith) where we are all free to buy and sell anything we choose, competing against one another to buy at the lowest possible price and sell at the highest, with no regulation Ration-Legal-Bureaucratic Model (based on Max Weber/Karl Marx) where production and distribution of goods and services is centrally controlled and is governed by a strict set of rules (standard operating procedures) and establish the qualifications of who and do what duties and descriptions which carefully define those duties (job descriptions) Professional Market Model (defined by holders of specialized knowledge, who have the power to organize and control their own work). Legally, only they can do the work or offer the services they provide and only they are allowed to determine whether the work was done correctly or to supervise the work. In return for these privileges, they do not abuse their rights, as they are doing the work not for the money, but for the pure pleasure and enjoyment of helping others, and not primarily for the monetary or pecuniary gains to be had. (pp. 1-3)

65 Chapter 2: Literature review and preliminary research

What we need to understand is despite Abbott’s (1988) claim that ‘because the professions are so well studied and well defined, it has proven easiest to analyze the division of expert labor’ (p. 317) most others do not see it that way.

2.4

The concept of a profession: Sociological definitions

2.4.1 Sociological definitions Harvard Business School professor, David Maister (1997) writes in his book, True professionalism: Once upon a time, clients trusted professionals. It was automatic, based on the learned professionals exclusive knowledge and respected calling. A sound character was assumed and business was confidentially conducted on a handshake. Great firms and institutions were born out of the natural expectation of trust. But those days are over. (pp. 68-69)

The effects of Enron, Global Crossings, Tyco, Parmalat and Worldcom, the increasing costs of healthcare due to damages for pain and suffering, not to mention the settlements by the Catholic Church for sexual abuse by priests, have been an erosion of the public trust in professions. Be they lawyers, health care providers, clergy, teachers or accountants, we are seeing an ever increasing erosion in the trust of professionals by the consuming public. Thus the relevance of sociological perspectives is perhaps more important than ever. Sociologists seem to tend towards a liberal perspective which disfavors the granting of the special privileges and status accorded professionals, and most appear to believe that ‘few humans, when faced with mutually exclusive decisions between financial and ethical interests, can find the moral latitude necessary to maintain professional integrity’ (Donaldson, 2000). An examination of the literature related to sociological perspectives on what constitutes a profession or a professional indicates that the sociological interpretation of the term is broader and less clear than the legal or dictionary definitions. Sociologists fall into two major schools of thought with regard to the concept of a profession. Talcott Parsons (1977), Michael Davis (2001), Linda Emanuel (2002), Jamal (1995) and Bowie et al. (1995) tend towards a ‘moralist’ or ‘altruistic’ school of thought. These scholars define the true professional as psychologically predisposed 66 Chapter 2: Literature review and preliminary research

to put doing good for mankind ahead of doing things for money, fame or power. In this regard, the moralist school resembles the legal perspective. The second major school of thought takes a more pedestrian or pragmatic approach, which emphasizes the importance of a body of knowledge, (usually transferred through advanced degrees), supported by research funded/initiated by professional organizations, dedicated to achieving goals established by those organizations representing the professional practitioner. Researchers in this category include A.I. Goldman (1999), Shimberg (1973, 1982), Esser (1973), Kruger (1973) and Abbot (1988). Research for this present study indicated that a middle of the road approach to defining a profession and a professional practitioner currently dominates. Researchers such as Donaldson and Preston (1995), with their stakeholder model; Dunfee (1999) with his social contract model and Wood (1991) with her corporate social program all focus on a combination of moralist and pragmatic perspectives. Other recent researchers, Eliot Freidson (2001, 1994, 1986) and Christopher McKenna (2006), have updated the work of Abbot (1988) and Larson (1979) in particular to reflect the significant evolutionary changes in the past 25 years, not the least of which have been the perfidious activities of Enron, Global Crossings, Worldcom and Parmalat to name but a few. 2.4.2 Sociological models The stakeholder model is already well represented in project management. As noted previously, the term ‘stakeholder’ appears over 150 times in the ~400 page long PMBOK guide. However, at least based on the actions of some organizations purporting to represent those practicing project management, the social contract or corporate social program are not as evident. For example, PMI, reasonably claiming to be the world’s largest representative of practitioners, despite having in excess of USD$80 million in the bank, unlike other comparable professional organizations, has yet to link its dues structure to purchasing power parity. While the stakeholder model is a core element of PMI’s philosophy, little is written about the social contract or corporate social program (CSP) approaches, at least in the context of project management. However, as can be seen from the legal perspective, emphasis on the importance of the social contract model remains quite strong.

67 Chapter 2: Literature review and preliminary research

An important sociologist is Andrew Abbot (1994, 2001), who is noted for creating ‘a rich and imaginative analysis of the institutions connected with occupational jurisdictions’ (Freidson, 2001, p. 6). He is also faulted for ‘not creating a clearly structured analytic procedure nor explicit connections to other bodies of data and theories’ (Freidson, 2001, 6). Abbot (2001) also subscribes to the philosophy contained in the legal principle of estoppel. He claims that an unfilled need is sufficient for an organization to simply claim ownership of the void, or if there is no void, it can fight other organizations for their ‘turf’. While the principal of estoppel has a solid basis in law, and in fact, architecture and engineering in particular owe their current status as professions largely to estoppel (Pollele, 1999), the work of Abbott (2001) most closely describes the situation between the architects, engineers and construction management (Abbott, 1988, p. 86). In Andrew Abbott’s (1988) book, The system of professions: An essay on the division of expert labor. he represents a somewhat unclear analytical model that looks at professions in terms of: „

their jurisdictions; (their ‘turf’, domains or claims on a specific body of knowledge)

„

the tasks they do; (their methodology or processes)

„

the expert skills and knowledge needed for those tasks

„

how competitive forces internally and externally work to change both the jurisdictions and the tasks.

Abbott (1988) attempts to show that professions are interdependent systems, containing internal structures. He accomplishes this task by means of analyzing the emergence of modern professions and their relationships with each other cooperatively and competitively. Of particular interest to this research is Abbot’s (1988) view that: sometimes problems of classification arise. For some problems are constantly shifting classifications, and fall under more than one classification, due to their defining traits. This may lead to intervention or competition by other professions who want to assimilate the unclear problem into their own professional repertoire. (p. 4)

The relevance of this becomes apparent in the long standing and often acrimonious ‘turf wars’ between architects and engineers over the management of construction

68 Chapter 2: Literature review and preliminary research

projects. This lack of clarification is one of the leading reasons behind construction project management being a ‘bastard child’ of both architects and engineers, with neither of them willing to give ground. Thus like in many divorce cases, construction project management has become the proxy battleground between two feuding parents – architects and engineers. Another important sociological perspective comes from Eliot Freidson (1994). In Professionalism reborn: Theory, prophecy and policy, he states: In general there are three ways to organize a market. First, there is the free market in which workers compete freely to be chosen and paid by employers or clients. Secondly, there is a bureaucratic market which is hierarchically organized and controlled. And thirdly, there is a professional market which is organized and controlled by the specialized occupations themselves. In the first the consumer is in command; in the second the manager or executive is in charge; and in the third, control is vested in the specialist workforce. In free markets the common interest of all participants is in monetary price and gain. Workers typically have little interest in the work they do or in the way they do it. They do what is necessary to maximize income. Price and profit are the measures of success, with efficiency defined by the minimization of price in the production of a particular good or service. Workers generally have little job security. Actions are more constrained in a bureaucratic labor market. The incentives and values are different. The emphasis of those in charge is on reliability and predictability in the production of specified goods and services. ‘Quality’ is defined by formal rules and standards which guide performance appraisal. Prices are specified for goods and services of standardized quality. Workers compete for jobs on the basis of qualifications and relevant experience, and can gain a measure of job security. In professional labor markets the choice of workers by both consumers and employers is limited by occupational regulation. While there may be some competition between members of a given occupation, within their sheltered position in the labor market, the emphasis is on collegiality and collective action. In the past, most health professionals have had secure employment. In the free market model the prime incentive is material gain, and value is measured in money. In the bureaucratic model the prime motive is security, and value is measured by reliable conformity to established standards. In the professional model the prime incentive is the respect and approval of colleagues, and value is rooted in the quality of the work. (pp. 32-45)

Contrast Freidson’s (1994) ‘collegiality – respect and approval of colleagues’ against Abbot’s (1988) ‘intervention or competition by other professions who want to assimilate the unclear problem into their own professional repertoire’ and you can begin to appreciate the intra and extra organizational tensions impacting the professions today.

69 Chapter 2: Literature review and preliminary research

To further cloud the picture, in Thomas Friedman’s (2005) Flat world and Tom Osenton’s (2004) Death of demand both authors describe a world in which the developing nations (China, India, Indonesia, for example) are not satisfied with lower rungs of the supply chain, but aspire to very quickly move up the supply chain by providing professional services. Some of the best examples of this would be the outsourcing of call centers to India or the recent purchase of IBM’s retail computer business by Lenova. Other important evidence of this trend is the proliferation of ‘medical tourism’ in such places as Singapore, Thailand and China, where people can obtain world class medical care at a fraction of the cost of similar care in North America or Europe. Abbot (1988) addresses this issue by predicting that ‘organizational (rather than occupational) dominance will emerge’ (p. 39). Freidson doesn’t address it directly, but notes that research by Casey (1995), and presented in Work, self and society in post industrialism, predicts ‘a post occupational society, in which all workers [are] part time, temporary workers’ (p. 11). Piore and Sabel (1984) two other researchers who has predicted many of the trends popularized more recently by Peters, (2003) Friedman (2005) and Osenton (2004). In 1995, Sabel predicted ‘in a post-modern economy, jobs will be composed of flexible skills rather than tasks, and occupational rather than firm identity will once again become the norm in the workforce’ (p. 12). Put in the context of Abbot (1988) and Freidson (2001), one can appreciate the potential impacts the ‘outsourcing’ of professional services is going to have in terms of ‘collegiality’ (intra-organizational competition), as well as inter-organizational clashes which the globalization of professional services is likely to engender. These ideas point to the possibility that professional services (and the concept of a profession) will move from a fraternal model towards a more free-market model. Should this become fact rather than theory or conjecture, the concept of many occupations professions will likely diminish while the importance of the individual as a professional will increase. This supports Ronald Rotunda’s (1997) assertion that ‘the only fruitful use of the term profession today relates only to individuals, not groups’ (p. 2). This is born out by a study conducted by the American Bar

70 Chapter 2: Literature review and preliminary research

Association’s Working Group on Civil justice: Blueprint for improving the civil justice system 12 (ABA, 1992) which concludes that ‘professionalism lies not so much in following certain rules as in the development of personal attitudes and a manner of deportment that leads to appropriate professional relationships and courtesy in manner and creed’. This relatively simple observation has profound implications for the future, not only for project management, but for all occupations which involve the provision of professional services. In summary, the various sociological models offer the least consistent definitions of either profession or professional. While most models are founded upon academically sound research, no one model completely, consistently or accurately defines the attributes of a profession in sufficient detail to enable quantification. Stated another way, as Polelle (1999) discusses, lacking legislative definition, and despite the use of semantic and other guidelines, from a legal perspective, there is no ‘bright line’ rule to help in defining a profession. A review of the sociological attributes does nothing to clear up the picture. However, the following outline captures the essence of the various sociological views and perspectives in a sequential order: Caplow, 1954

a structured, functional, sequential model;

Wilensky, mid 1960s

a logical sequence of identifiable steps

Millerson, mid 1960s

there is no one model (each case is unique)

Larson, mid 1970s

monopolistic stages

Bledstein, mid 1970s

cultural attributes

Arney, mid 1980s

cultural traits

Freidson, mid 1990s

ideological system

Abbot, late 1980s

competitive control of jurisdictions (knowledge)

Freidson, early 2000s

logical, systematic framework

While the sociologists have written the most, their models leave as much confusion as they do solutions for determining whether any single occupation is or is not a profession. This is especially obvious when compared to the legal scholars, who have attempted (and been somewhat successful) in determining ‘bright-line’ tests.

71 Chapter 2: Literature review and preliminary research

2.5

The concept of a profession: Economic definitions

While often economic and sociological thinking are lumped together under the guise of socio-economic perspectives, they were kept separate in this research because, while both address many of the same issues, economists tend to look mostly towards the economic impact imposed by the legal wrangling over licensing and the implied or real impact that artificially limiting the number of practitioners has on the cost of services to the consuming public and the relative quality of those services. Some of the major issues identified by economists are: „

anticompetitive impacts of restrictions on advertising (Muris & Cheney, 1978; Bond, 1980; Feldman & Begun, 1978 1980 & 1985; Cady, 1976; FTC, 1984; Benham, 1972; Bond, 1980; Kwoka, 1984)

„

benefit: cost ratio of licensing (Young, 1986; Healey, 1973; Paul, 1984)

„

reciprocity (practitioners coming into an area from out of the state or out of the country) (Martin, 1982; Shepard, 1978)

„

barriers to entry (residency requirements, certifications, quotas, education requirements, use of interns) (Hass-Wilson, 1986; Kwoka, 1984; Bond, 1980; Liang & Ogur, 1987; Conrad & Sheldon, 1982; Martin, 1982; Feldman & Begun, 1985; Carol & Gaston, 1981; Holen, 1978)

Several other reasons for separating sociologists from economists were that: „

Economists are the most diametrically opposed to the legalist perspective in terms of providing a balanced approach to defining a profession.

„

Economists are perhaps the best researched in terms of quantitative vs. qualitative analysis (consistent with modernistic thinking).

To summarize, Tables 2.2 and 2.3 offer a compilation of research on the economic impacts of various restrictions on the quality and cost of professional services. As can be appreciated from Tables 2.2 and 2.3, while the impact of various restrictions on the quality of services is mixed, the costs of services when restricted clearly go up significantly. While this data was taken from a 1990 report prepared by Carolyn Cox and Susan Foster for the US Bureau of Economics, Federal Trade

72 Chapter 2: Literature review and preliminary research

Commission (Cox & Foster, 1990), subsequent research by the highly regarded Pew Research Group (Finocchio, Dower, Blick & Gragnola, 1998) presented in Strengthening consumer health care protection through health care workforce regulation confirmed the findings of Cox and Foster (1990), reiterating that amongst other recommendations, ideally, ‘voluntary credentialing (in the case of the medical profession means Board Certification) was preferable to licensing’ (Finocchio et al., p. 103; Cox & Foster, 1990, p. 59) but with the proviso that ‘peer review boards should consist of 51% (or more) of consumers and 49% (or less) of practitioners’ (Finnochio et al., 1999, p. 103). Table 2.2

Impacts of restrictions on QUALITY of services

Occupation

Type of restriction

Impact on quality

Researcher

accountants

licensing required

neutral

Young, 1986

dentistry

exam requirements and reciprocity

positive

Holen, 1978

electricians, plumbers, optometrists, real estate agents and veterinarians

oral exams and prior occupational experience

negative

Carroll & Gaston, 1981

legal

advertising restrictions

negative

Muris & McChesney, 1978

optometry

advertising & commercial practice restrictions

neutral

Bond, 1980

optometry

advertising

negative

Kwoka, 1984

optometry

commercial practice, advertising restrictions and continuing education

positive (note: study was considered flawed in that it was impossible to separate out the impacts of each element)

Feldman & Begun, 1985

laboratory personnel

licensing required

neutral

Healey, 1973

pharmacy

reciprocal licensing

positive

Martin, 1982

pharmacy

citizenship

positive

Martin, 1982

pharmacy

non-reciprocal licensing

neutral

Martin, 1982

pharmacy

advertising

neutral

Cady, 1976

physicians

licensing required

neutral

Paul, 1984

73 Chapter 2: Literature review and preliminary research

Table 2.3

Impacts of restrictions on COST of services

Occupation

Type of Restriction

Impact on Cost

Researcher

dentistry

reciprocity

15%

Shepard, 1978

dentistry

commercial practices

4%

Conrad & Shelton, 1982

dentistry

use of hygienists

4%

Conrad & Shelton, 1983

dentistry

use of hygienists

11%

Liang & Ogur, 1987

legal

advertising

5-11%

US Federal Trade Commission, Cleveland Office

optometry

advertising

25%

Benham, 1972

optometry

advertising

33%

Bond, 1980

optometry

commercial practices

33%

Bond, 1981

optometry

advertising

20%

Feldman & Begun, 1978 & 1980

optometry

commercial practices

5-13%

Haas-Wilson, 1986

optometry

advertising

20%

Kwoka, 1984

optometry

commercial practices

20%

Kwoka, 1984

pharmacy

advertising

5%

Cady, 1976

Another important recommendation from both the Pew study (Finocchio et al., 1999) and Cox and Foster (1990) was that, ‘National competency standards were preferable to state or local ones’ (Finocchio et al., 1998, p. 115; Cox& Foster, 1990, p. 36). While not coming out totally against licensing and other restrictions, the Pew researchers felt that the best interests of the consuming public were not being served, and were calling on significant reforms, at least in the healthcare sector. The implications of this for project management are reasonably clear, emphasizing that the trend for voluntary credentialing, as opposed to government required licenses remains strong, and that national standards are preferable to local or state standards. Extrapolating the findings of Cox and Foster (1990) and Finocchio et al. (1999) in the context of project management, there is strong support for the Global Alliance for Project Performance Standards (GAPPS) (www.globalpmstandards.org).

2.6

The concept of a profession: Semantic definitions

2.6.1 Semantic definitions While the semanticists can and do come from philosophic, legal, sociology or economic backgrounds, in order to explore whether or not the whole is more than the parts or there might be meaning in the social context through usage (post modernist view), the semantic perspective was of sufficient importance to warrant a separate treatment from the purely legal, economic and sociological point of views in this study. In many cases (Polelle, 1999; Abbot, 1988) they appear in both their respective fields (legal and sociology) as well as in the semanticist category.

74 Chapter 2: Literature review and preliminary research

The leading researchers in terms of the semantics of the concept of a profession include M. L. Cogan (1955), M. J. Haga (1979), Michael Polelle (1999) and Ronald Rotunda (1997); but as noted previously the distinction is often blurred, depending on whether the researcher is looking at the definition from the perspective of their original field or linguistically as a semasiologist (linguist). One of the earlier semantic researchers was M. L. Cogan (1955). In Problems of defining a profession, Cogan (1955, p. 107) proposed three types of definitions associated with professions: „

persuasive definitions designed to convince others that what they did was a profession

„

operational definitions designed to convince others that the nature of the work they performed established the claim that what they did for work was a profession

„

logical definitions designed to convince others that their work was a profession by dint of the historical or customary application of the term.

The Semantic School derived from this basis (Maurutello, 1981). Haga (1974) in particular, although a rather obscure researcher, published a pithy and hard hitting description of the difference between true professions and ‘would be’ professions in the Management Quarterly Review. In this article he noted: The utility of distinguishing between the true professions and merely ‘would be’ professions can be appreciated when one looks at why so many occupations pursue the elusive status of profession. Here are the chief things that occupational groups seek when they undertake ‘professionalization’: – Above all else, they want AUTONOMY. That is, they want freedom from supervision in carrying out their jobs; – They want RECOGNITION based not upon the name of their employer, but upon their identity with their occupation; – They want the POWER to determine who is ‘in’ their occupation from those who are ‘out’; – They want to establish a MONOPOLY over a certain line of work, freeing it from influence of ‘outsiders’ (mostly employers, but also clients and the general public) who do not share or necessarily understand the ideology; – They want the POWER to discipline wayward colleagues who deviate from their work ideology. (p. 4)

75 Chapter 2: Literature review and preliminary research

Haga (1974) goes further to explain that: there is no denying that once autonomy is achieved, greater success in negotiating for monetary rewards will likely follow. While money cannot be totally dismissed as an important professionalizing force, establishing autonomy becomes the means to that end. (p. 4)

But even more relevant to project management, Haga (1974) states: the chief means of maintaining autonomy is INTIMIDATION. The essence of truly professional behavior lies in intimidation – of clients; of employers; of whatever audience stands to threaten a profession’s autonomy. (p. 6)

This observation is interesting having seen first hand evidence of exactly these behaviors by more than one of those organizations purporting to represent the ‘professionalization’ of project management. Evidence of this intimidation included a major global organization suing several long time members and threatening legal actions against a host of others, and other threats of legal actions in several of the other larger, well known global organizations representing project managers. Assuming Haga’s (1974) perspective is still valid, taken in the context of today’s world as defined by Friedman (2005) or Osenton (2004) as being one of overcapacity and brutal global competition at all levels, whether or not anything but the ‘traditional’ or ‘fee for services’ practitioner should qualify as being a profession becomes highly questionable and certainly fodder for further research. In Professionalization of project management: Mapping the past to explore the future, Zwerman et al. (2004, p. 24) identify three categories of profession: Set A – traditional ‘fee for services’ model Set B – employer-employee model Set C – new knowledge worker model Assuming that Friedman (2005), Osenton (2004), Peters (2003) and other futurists are correct and that we are headed for a world where nearly everyone becomes a part time, outsourced worker, employed on yearly or project-based contracts, wouldn’t it seem likely that Set A and Set C will eventually become indistinguishable, and that those people remaining in Set B will become a relatively small minority of the working world? Freidson (2001) touches on this in Professionalism, the third logic,

76 Chapter 2: Literature review and preliminary research

by quoting Krause (1996), Abbott (1988), Casey (1995) and Sabel (1995) as noted previously, supporting the more recent work of Friedman (2005), Osenton (2004), Peters (2003) and Maloney (2004). Haga (1974) also established three semantic criteria against which to measure a profession: 1

Criticality – An occupation possesses criticality when some significant group of others determines that what the occupation does or provides is crucially necessary to their life, (now or hereafter) health, purse or freedom; Criticality means that the occupation has an almost life-ordeath relationship to it’s clientele, employers or the consuming public. ‘Significant Group’ means not everyone considers it a near life or death concern, but that a relatively large enough population considers it so to enable the practitioner to earn a living off the fear of whatever threat to life, health, purse or freedom is facing the population.

2

Mystique – This second differential of a profession is a direct derivative of the first. A person with a crucial problem dare not seek out just anyone to solve this problem. The person who has the ability to solve the problem must be perceived as having much more knowledge/skill or understanding than the lay person experiencing the problem or those of his/her peers. In today’s world, this often translates into those who hold advanced degrees but could equally apply to your auto mechanic, a plumber, electrician or anyone who can do something you perceive to be beyond your comprehension. (i.e. setting up and maintaining your computer)

3

Dénouement – This is the ‘third leg’ required to support this ‘three legged semantic stool’ definition of profession. Like Mystique, Dénouement is a critical ingredient that, when mixed with criticality, defines a profession. Dénouement the bringing together of Criticality, Mystique by an individual perceived by the recipient of the services as having special powers, well above and beyond those of the recipient. In this professional, you are willing to put your full faith and trust, with no concept of what the person is doing, in the service provider to render relatively quick and seemingly effortless results. (PAGE)

Examples of this very real and very common phenomena described by Haga (1974) abound in our everyday lives. For example, you are a women driving alone down the highway at night. Your car breaks down. You phone 911 on your cellphone and the police send out an auto repair person. You haven’t a clue what the person is doing to your automobile, but you have to trust in this person because you have no other choice. Within 15 minutes, the auto repair person has replaced some filter or other faulty mechanical part, and you are on your way. Do you complain about the bill?

77 Chapter 2: Literature review and preliminary research

Not very likely. Do you really care if the person was a licensed mechanic? You are simply grateful you were able to get help in the first place and thankful that the problem was fixable without having to be towed or spend a night in a hotel. Other examples abound in our day to day world: Your computer crashes, and you have a major report due first thing tomorrow morning….How do you perceive the computer repair person who is willing to come to your home and repair your computer? Your toilet backs up on a Friday night when you have house guests arriving on a Saturday morning to stay the weekend….How do you perceive your local plumber who arrives at your home at 22:30 on Friday night to unclog your drain? Do you first ask for his/her plumbers license? Do you ask in advance what he will charge? Do you negotiate hard with him to drop his rate? Your find a hard lump in your breast and you are unable to get to a doctors appointment for a week….What do you say to the first doctor who will accept you immediately? You’ve been arrested and jailed for a crime you did not commit….What is the first call you make? Do you really care who shows up to bail you out? You are a religious person and have just been in a bad traffic accident and are dying, and you want to receive final absolution….Do you really care whether the first religious person to arrive on the scene is not from your religion? Do you tell him or her to go away and try to find someone else?

All are examples where we are put in such stressful situations that we grasp at the first person who makes any promise to help, and are extremely happy to put our problem in their hands for a speedy resolution. While this may well be the mark of a professional person, or it could just as easily be the traits of a charlatan. Evidence of the dangers in relying solely on this definition abound, with the proliferation of faith healers, white magicians and others who prey on the fears and desperation of the afflicted members of our societies. So while this definition may very well be one of the components of a truly compassionate professional, as we have seen with legal, sociological and economic criteria, these traits alone do not make a profession. This is especially true in the field of project management. Given that risk management, especially in the proactive identification and management of risk before risk events happen, plays such a prominent and crucial role in the practice of our craft (PMBOK Guide, 2004), one could certainly question how, as professional practitioners, we ever could let ourselves get into such a situation requiring the kind of solution where criticality/mystique and dénouement would be appropriate.

78 Chapter 2: Literature review and preliminary research

2.6.2 Equitable estoppel as a special case definition To a somewhat lesser degree, both the legal scholars (Polelle, 1999; Rotunda, 1997; Gawley, 2002) and the sociological scholars (Abbot, 1998; Freidson, 2002; McDonald, 2000) recognize the semantic principles through the legal principle of estoppel as applied to defining a profession. Essentially, estoppel is a legal principle that prevents a person from asserting or denying something before a court of law that contradicts what has already been established as the truth, either by something the defending person wrote or through the actions of that person. This is also known as estoppel in pais (Black’s Law dictionary, 2006). While there are several types of estoppels, equitable estoppel is a type of estoppel that bars a person from adopting a position in court that contradicts his or her past statements or actions when that contradictory stance would be unfair to another person who relied on the original position. For example, if you hold yourself out to be a professional project manager, and then get sued for failing to deliver the project on time or within budget, under the principle of equitable estoppel, you would be ‘estopped’ or prevented from claiming after the fact that you did not represent yourself to be a professional project manager. Whether you acted in a professional manner will be a matter of fact determined by the court of law. The important concept to remember here is if your occupation does NOT meet other definitions of being a profession, then despite you having claimed you were one, you will be judged by the rules of ordinary negligence as determined by the average man and not professional negligence, to be judged by the standard of your peers, from the same discipline and from the same region; nor you will not enjoy the privilege of a reduced statute of limitations. Those who believe that simply saying project management is a profession in the belief this will automatically make it one, are setting themselves up for a potentially nasty surprise, at least from the legal perspective.

2.7

Definitions summarized

Where does this leave us? We know from the dictionary that the definition of profession and professional is vague and subject to interpretation (Merriam-Webster, 2006). We know from an analysis of a broad review of relevant case law that while some ‘bright line’ definitions exist for profession and professional (Polelle, 1999),

79 Chapter 2: Literature review and preliminary research

there remains a general lack of consensus. We also know from the legal analysis that legislatures are unwilling to risk offending voters by legislating what is and what is not a profession (Rotunda, 2002). We know from an analysis of the sociological literature that sociologists are the least consistent in their definitions. Coming full circle, the semanticists also have produced some definitions, and while each has justification under a given set of circumstances, there does not exist any single definition – dictionary, legal, socio-economic or semantic – which consistently answers the question What is the definition of a profession and/or a professional? Lacking any single ‘bright line’ standard for defining a profession, consistent with the approach taken by the judiciary and sociologists, some sort of middle of the road approach seems to be the only appropriate and logical choice. One of the legal terms commonly found is burden of proof. For serious cases, such as murder, the burden of proof has to be beyond a reasonable doubt (Black’s Law dictionary, 2005). While this doesn’t mean ‘100%’ sure, it certainly should mean 90% or better, especially if you are going to send someone to the gas chamber or to prison for life’. However, for most civil cases, the burden of proof is ‘the preponderance of the evidence’. In most cases, it means as low as 51%. Given that no one single perspective fully or adequately defines a profession, then I have proposed through this current research that all the key attributes from all sources attempting to define the concept of profession be considered to create a scoring model which will provide an all encompassing approach yielding a score against which the relative professionalism of any given occupation can be scored or judged. In summary, the literature research has yielded 22 attributes generally ascribed to professions. Of these 22, the only ones which appear in all of them are: 1) a ‘body of knowledge’ which is somehow ‘unique, esoteric, secret, complicated or abstruse’ 2) a ‘long period of training’ which normally requires at minimum a four year university degree

80 Chapter 2: Literature review and preliminary research

3) a commitment to serving or protecting the public (normally includes performing work pro bono, although there are other responsibilities included in this general heading) 4) one or more organizations which purports to represent the practitioner of the occupation (responsibilities of this organization to include the establishment of proven methodologies, setting of standards, creating and enforcing codes of ethics). It is worth noting that implicit in the writing and enforcement of the code of ethics is the idea that they should be focused on protecting the consuming public and not focused primarily on policing the actions and behaviors of the members vis a vis each other. As can be seen in Table 2.4, the semantic definitions (Abbott, 1988; Haga, 1974; Cogan et al., 1955) provided the highest ranking, with a score of 67% out of a possible 100%. However, the fact that the highest score is only 67% certainly indicates that even the semantic definition is missing many key aspects. The semantic rankings are followed by legal perspectives with a score of 58%, sociologic, with a close 56%, economic with 47% and lastly by the dictionary definition with a score of only 32%. This summary helps explain why there is so much confusion over the definition of a profession (Freidson, 2001, p. 197). The 22 attributes distilled from the data in Table 2.4 are listed in Table 2.5.

81 Chapter 2: Literature review and preliminary research

Table 2.4

Attributes of a profession ranked by source and relative scores

80

3

Clearly Defined

2

Clearly Implied

1

Marginally Implied

0

Not Cited

EXTRINSIC

Semantic

Legal

Sociologic

Economic

Dictionary

Total

Total

%

Score

Possible

Score

body of knowledge

3

3

3

3

3

15

15

100%

2

unique body of knowledge

3

3

3

3

service to the public

3

3

3

3

3

15

15

100%

3

3

15

15

100%

4

professional association

3

3

3

3

3

15

15

100%

5

performance standards

3

6

long period of training

3

3

3

3

3

15

15

100%

3

3

3

3

15

15

100%

7

establishes standards

8

practice limited by government license

3

3

3

3

3

15

15

100%

3

3

3

3

0

12

15

80%

9

establishes/enforces code of ethics

3

3

3

1

2

12

15

80%

10

subscribe to a code of ethics

3

3

3

0

2

11

15

73%

11

procedural standards /methodologies

2

2

3

1

3

11

15

73%

12

autonomy in decision making

3

3

3

1

0

10

15

67%

13

lifetime commitment/calling

0

3

3

0

3

9

15

60%

14

fiduciary obligation to public

0

3

3

3

0

9

15

60%

15

advertising not permitted or restricted

3

3

0

3

0

9

15

60%

Color/Ranking Key

#

1

Attribute Name

16

professional association defines ‘best practices’

2

0

0

3

3

8

15

53%

17

pro bono work

0

3

3

2

0

8

15

53%

18

identify with occupation, not employer

3

1

3

1

0

8

15

53%

19

held in high esteem by the community

1

1

3

1

2

8

15

53%

20

requires professional liability insurance

0

3

1

3

0

7

15

47%

82 Chapter 2: Literature review and preliminary research

Clearly Defined

2

Clearly Implied

1

Marginally Implied

0

Not Cited

2

0

2

3

0

7

15

47%

0

3

2

2

0

7

15

47%

symbolic costumes/uniforms

3

0

3

0

0

6

15

40%

24

publishing in learned journals

3

1

1

0

0

5

15

33%

25

use of title restricted by law

0

3

0

1

0

4

15

27%

26

apprenticeship/ internship/residency

0

2

0

0

2

4

15

27%

27

mystique

3

0

0

0

0

3

15

20%

28

importance of need

3

0

0

0

0

3

15

20%

29

immediacy of need

3

0

0

0

0

3

15

20%

30

highly ritualistic procedures

3

0

0

0

0

3

15

20%

31

dénouement

3

0

0

0

0

3

15

20%

32

cruciality

3

0

0

0

0

3

15

20%

33

access to knowledge limited/restricted

3

0

0

0

0

3

15

20%

34

abstruse/secret/ complicated body of knowledge

3

0

0

0

0

3

15

20%

Color/Ranking Key

3

21

earn higher than average compensation

22

college degree

23

Total Score

73

58

57

46

35

total possible score

102

102

102

102

102

rating (total/possible)

72%

57%

56%

45%

34%

83 Chapter 2: Literature review and preliminary research

Table 2.5 #

22 attributes commonly associated with a profession

Attribute Extrinsic attributes

1

Professional association- defines methodology/defines standards/enforces CoE

2

Autonomy in decision making

3

Lifetime commitment/calling

4

Earn higher than average compensation

5

Publishing in learned journals

6

Long period of training including higher education

7

Subscribe to a code of ethics

8

Fiduciary obligation to public

9

Professional association defines ‘best practices’

10

Identify with occupation, not employer

11

Held in high esteem by the community

12

Apprenticeship/ internship/residency

13

Service to the public- including pro bono work

14

Practice limited by government license

15

Advertising not permitted or restricted

16

Requires professional liability insurance

17

Symbolic costumes/uniforms

18

Use of title restricted by law

19

Body of knowledge

unique/esoteric/secret (both intrinsic and extrinsic)

Intrinsic attributes 20

Cruciality- need is important/need is immediate

21

Dénouement- practitioner is trusted to produce positive results relatively quickly.

22

Mystique- highly ritualistic/access to knowledge is restricted

While the scoring methodology is admittedly simplistic (it doesn’t take into account the relative order of magnitude differences between any two scores), it does support the judicial perspective (Polelle, 1999) that using a dictionary definition is insufficient as a ‘bright line test’ for a profession, while at the same time being indicative that neither the semantic, legal, sociological, economic or dictionary definitions is complete or sufficient, which is consistent with Schein (1988). This leads to the inevitable conclusion that only by taking a holistic approach can we possibly define a profession, and that there does exist some tipping point at which an occupation becomes perceived to be a profession, and that the perception is not static, but ever changing. This too has relevance for those who truly do believe project management to be a profession, as this ranking will serve to identify areas of weakness which can be improved upon, resulting in an improved perception or professionalization of the practice of project management. 84 Chapter 2: Literature review and preliminary research

CHAPTER 3, PART A

Case study 1: Construction project management Introduction to the case studies In Chapter 2, after extensive literature reviews from legal, socio-economic and semantic perspectives, a total of 22 attributes associated with professions were clearly identifiable. While some of them were common to all perspectives, there was enough diversity to provide a reasonable expectation that the combination of attributes would yield a method or formula to quantify the relative professional standing of project management, benchmarked against other occupations. Chapter 3 explains how the 22 attributes were tested against two familiar occupational specialties, with the objective of creating a standardized instrument or methodology against which to evaluate project management (or any other occupational specialty) as a profession. Construction project management and commercial airline piloting were chosen as case studies for three reasons: 1 I have first hand experience with both, having worked for much of my life as a construction project manager and because I am a licensed aircraft pilot. Although my license is not commercial, I have come to know many commercial pilots and am closely familiar with the processes and procedures that are integral to their occupation. 2 Construction project management, while it has been around for well over 50 years as a stand alone occupation, has never been able to realize equality with the two established professions with which it most often interfaces – architecture and engineering. 3 Aircraft piloting represents an interesting comparison because, despite being only 75 years old as a recognized occupation, it is generally perceived as a prestigious, highly desirable and well paid profession, despite the fact that to be a licensed commercial pilot does not even require a high school diploma. The choice of two ‘outlier’ occupations – construction project management, which appears on the surface to have all the ‘traditional’ requirements associated with a profession but has not been successful in being recognized as one and commercial

85 Chapter 3A: Case Study 1, Construction Project Management

airline piloting, which does not meet the primary requirement of requiring a four year or more university degree – was intended to help in the quest to separate those attributes which are essential to defining a profession and quantifying them from those which are less important or not important at all.

3A.1 Case Study 1: Construction project management 3A.1.1 The concept of construction project management (CM) Construction project management is a global occupation, and one that, despite having been recognized since the mid 1950s, has largely stagnated in terms of evolving as a stand alone practice, much less a profession. While relatively large and complex construction projects have been going on since well before the Great Wall of China or the pyramids, construction project management as a discipline separate from architecture or engineering is a relatively new term or concept, having evolved from the mid 1800s, and only being formally recognized since the end of WWII. While relatively new in terms of recognition in historic times, construction project management is the oldest and most mature of the sub-disciplines currently adopting project management as a delivery system. With CM being the oldest of a new discipline, it offered an ideal chance to look at the evolution of a new occupational category and see if there were lessons which a practitioner could learn or guide posts which a practitioner could use to help discover what is likely to happen as other users of project management reach the same level of maturity as construction project management. The Construction Management Association of America (2007) defines construction project and program management: CM is a professional service that applies effective management techniques to the planning, design, and construction of a project from inception to completion for the purpose of controlling time, cost and quality. CM is a discipline and management system specifically created to promote the successful execution of capital projects for owners. These projects can be highly complex. Few owners maintain the staff resources necessary to pay close, continuing attention to every detail – yet these details can ‘make or break’ a project.

86 Chapter 3A: Case Study 1, Construction Project Management

The American Institute of Constructors (AIC) (2007) mission statement explains: The purpose of The American Institute of Constructors (AIC) is to promote individual professionalism and excellence throughout the related fields of construction. (online)

Notice that both organizations are consistent in the wording of their statements. While each organization is dedicated to helping its practitioners perform their duties professionally they specifically refrain from making any claims that what they represent is a stand alone profession. CMAA is more specific in stating that what construction managers do is a discipline and a management system. The picture is further clouded by the fact that the American Society of Civil Engineering (ASCE) and the American Institute of Architects (AIA) also ‘claim’ jurisdiction over construction management professional services. This competition is consistent with observations by Andrew Abbott (1988) in his book, The system of professions: An essay on the division of expert labor. Abbott (1988) identifies three types of conflict which emerge intra- (within a single organization) and interprofessionally (different professional organizations or groups competing for jurisdiction). The first case, known as excess jurisdiction occurs, when an incumbent profession cannot grow to meet demand, or increase output, and thus faces invasion by outsiders. The second kind of conflict arises when a professional group’s potential output exceeds its current jurisdiction. The third type of conflict occurs when groups who provide equivalent services at lower prices seek to invade into a settled jurisdiction. (p. 27)

Up until recently, the problem facing most construction project managers was of the second type, where tasks normally performed as part of the traditional role of either architects or engineers became specialized enough that a group of practitioners focused solely on those types of professional services. What started out as a niche market soon grew into an emerging specialty or discipline within the ranks of civil engineering or architecture. However, with countries like China and India producing highly qualified, experienced and competent construction project managers, and outsourcing becoming more prevalent, transition from the second type of conflict identified by Abbot (1988) into the third type is likely to occur more and more frequently.

87 Chapter 3A: Case Study 1, Construction Project Management

What makes construction project management even more interesting as a case study is the fact that substantially the same body of knowledge which forms the basis for construction project management forms the basis for all other incarnations or manifestations of project management. This observation leads to the conjecture that given construction project management has had the longest experience in using project management as a delivery system; and given that the underlying body of knowledge is substantially the same for all project management; and given that the applications or manifestations of the body of knowledge, processes, methodologies and procedures are substantially the same, then construction project management provides a very reliable indicator of what is likely to happen to project management as it applies to information technology, telecommunications and other sectors. Stated another way, unless some major discoveries are made, the stifled professionalization of construction project management is likely to occur in other applications as well. The chronology of development in CM supports the statement by Prof. Peter Morris (n.d., online) on the home page of the Rethinking Project Management Forum, that ‘project management is a discipline caught in a 1960s time warp’ (n.d., online). Rethinking Project Management represents: a multi-disciplinary network of academics and practitioners; academics from UK universities, the US, Canada and Europe, with representation from the professional associations (APM, PMI & IPMA). Industry collaborators include Roll-Royce and Human Systems Ltd and Funded by the UK’s Engineering & Physical Sciences Research Council. (Rethinking Project Management, n.d., online).

To quote from the Rethinking Project Management Overview (n.d., online): as the world of project management practice continues to develop across different industries and sectors, the subject of project management is now attracting major criticisms, and the gap between conventional project management theory and the developing practice is widening. There are also increasing calls for the identification of new research perspectives and new research topics from other related disciplines. It was against this background that UMIST and UCL submitted a research proposal to the EPSRC proposing a new network to help rethink this emerging discipline. The principal argument of the research proposal was not that conventional project management theory should now be abandoned, but rather there is a need to extend, enrich, reshape and develop this field beyond its current intellectual foundations. (Overview)

88 Chapter 3A: Case Study 1, Construction Project Management

3A.1.2

History of construction project management

Table 3A.1 Time line of the history of construction management Date

Description

Where

5000 BC

Egyptians construct huge pyramids which have lasted for thousands of years. Unfortunately, no record exists of how they managed the projects.

Egypt

770-470 BC

Great Wall of China constructed. As no records of how it was managed have survived, we have no way of knowing if it was completed on time and within budget

China

1842

London Provident Association of Builder Foremen and Clerks of the Works.

London, UK

1852

American Society of Civil engineers Founded (ASCE)

1856-1915

Frederick Taylor, Father of Scientific Management

Philadelphia, USA

1857

American Institute of architects Founded (AIA)

New York City, USA

1861-1919

Henry Gantt, Creator of the Gantt Chart

1870s

Railroad Expansion in the USA and UK created a huge demand for skilled supervisors and trades people

USA, UK

1887

Royal Melbourne Institute of Technology offers 3 year diploma in carpentry, joinery, brick laying and masonry

Melbourne, Australia

1917

Henry Gantt uses visual charts to substantially reduce the time to build ships for WWI

1920

National Federation of Builder Foremen and Clerks of the Works established

London, Birmingham, Sheffield, UK

1945

First US University to offer CM Degree- University of Florida- Gainsville

Gainsville, FL, USA

1946

Johns-Manville funds the start up of CM programs in 20 Universities across the USA, anticipating the pent up demand for housing after WWII

USA

1949

National Federation of Builder Foremen and Clerks of the Works Spreads to all parts of UK

UK

Cold War begins. Military spending on major projects (Nuclear Weapons Stockpiles) 1950

Europe/USA Canada Bechtel uses project management Organization structure to construct pipelines

89 Chapter 3A: Case Study 1, Construction Project Management

Date

Description

Where

1950-60

PERT, WBS, Critical Path Method, Single Point of Contact

USA/Europe

Association for the Advancement of Cost engineering (AACE) founded. 1956

USA Navy and Remington Rand Univac create first computerized CM schedules.

1957

Sputnik Launched by USSR. NASA is created to ‘put a man on the moon’

USA

1960

Institute of Building Site Management founded

London, UK

1963

Earned Value and Project Life Cycle adopted by USAF, British required to use CM Scheduling and Earned Value on Polaris Submarine

USA

1968

Royal Melbourne Institute of Technology offers two year Diploma in Building construction

Melbourne, Australia

1969

project management Institute (PMI) founded

USA

1972

Royal Chartered Institute of CM Conceived http://www.ciob.org.uk/ciob/

London, UK

1974

American Council on construction Education (ACCE) formed http://www.acce-hq.org/

Texas, USA

1978

Royal Chartered Institute of CM Inaugurated

London, UK

1980

American Council for construction Education (ACCE) accredits Universities offering degrees in CM. IBM launches PC

USA

1983

Harvard Graphics released; Primavera Project Planner released

USA

While it is not clear exactly when construction project management made it to America, it would appear that while the concept evolved in London during the mid 1800s, it made the move to North America or developed there simultaneously and fairly quickly, as evidenced by the formation of the American Society of Civil Engineers in 1852 followed in 1857 with the formation of the American Institute of Architects.

90 Chapter 3A: Case Study 1, Construction Project Management

A telling statement taken from the AIA website (AIA, n.d., online) provides insight as to what the driver behind the formation of these groups might have been: The group sought to create an architecture organization that would ‘promote the scientific and practical perfection of its members’ and ‘elevate the standing of the profession. Until this point, anyone who wished to call him-or herself an architect could do so. This included masons, carpenters, bricklayers, and other members of the building trades. No schools of architecture or architectural licensing laws existed to shape the calling. (History of the American Institute of Architects, n.d., online)

It would appear from the wording that the implied purpose was some sort of restraint of trade. This is an interesting point, for as we enter a world of overcapacity, it will prove interesting to see if the same kinds of thinking will apply in today’s markets as did in the mid to late 1800s. Also worth noting is the use of the term ‘calling’ to describe the practice of architecture. But the real impetus driving CM, especially in the US, was the return of servicemen from Europe and Asia after WWII. Their demand for housing and the infrastructure required to support those houses and their baby boomer families (schools, interstate highway system, shopping malls) lasted well into the 1960s. These circumstances match quite closely with Abbot’s (1988) definition of ‘excess jurisdiction’, which occurs, when ‘an incumbent profession cannot grow to meet demand, or increase output, and thus faces invasion by outsiders’ (p. 33). Considering what the history of CM can tell us (Table 3A.1), it is worth noting that the timeline stops at 1983. Since the invention of the PC, which enabled the creation of the Harvard Graphics and Primavera Critical Path Method (CPM) Scheduling software to move to the desktop, there has been not a single major innovation of any significance to occur. Yes, there have been refinements, and all manner of combinations, permutations and distortions, but the basic concepts discovered in the 1960s and 1970s remain largely unchanged. This fact stands as one of the major impediments to project management becoming more professionalized than it is today. When one compares the strides made in

91 Chapter 3A: Case Study 1, Construction Project Management

medicine between 1983 and 2006, and then compares the gains made in project management during the same period, it can been argued that project management lacks the culture, structures and aspirations that drive a profession like medicine. Widespread adoption of specialized CM programs at junior colleges and full four year degree universities began about 40 years ago, and CM practitioners have been aggressively attempting to raise themselves to be equal to the status and authority accorded professional engineers and architects since the programs’ inception. The fact they have met with little success stands as an early warning to others. One of the reasons for this frustrated effort lies in the fact that the architects and professional engineering societies are themselves fighting over who owns the right to oversee, supervise, approve, modify or accept work. Given these recognised professionals are fighting over project management, why should they allow a third set of claimants enter the field? The two leading organizations representing construction project managers are the CM Association of America (CMAA)1 and the American Institute of Constructors (AIC)2. Consistent with Abbot’s (1988) Theory of Competition, it would not be surprising to see IEEE eventually try to claim project management in the IT and Telecom sectors from PMI, IPMA and AIPM. AACE and ASCE have also been focusing more on the technical aspects of project management as it applies to their market niche. The activities of these organizations are all about big business. With PMI sitting on $USD80 million+ in liquid assets, and bringing in fresh money at the rate of $USD2 million per month (PMI, 2005), it is only a matter of time before the other professional organizations begin manoeuvring to strengthen their positions in the marketplace. Since the IT and telecommunications sectors make up the majority of PMI membership, what would the consequences be if they were to decide to realign themselves with IEEE? If Abbot (1988) is correct, we should start to see evidence of this competition beginning to evolve.

1 2

www.cmaanet.org www.aic.org

92 Chapter 3A: Case Study 1, Construction Project Management

3A.1.3

Construction project management: Is it a profession?

Having been offering construction project management curricula since the early to mid 1960s, the construction project management sector is recognized as being a leader in the development of tools for project management education. A broad range of degrees from Associates Degree (two year diploma program in the UK and Commonwealth Nations) to Bachelors (four year); Masters and even PhDs with a specialty or focus in construction project management are available from reputable and highly regarded institutions of higher learning. Yet, despite the availability of degrees, there is no specific requirement from any of the major professional organizations representing construction project management that a practitioner actually has a four year degree or better. As we know from Gardner v Fries (Polelle, 1999), one of the ‘bright line’ tests established by the Florida Supreme Court was that a ‘professional’ would be expected to have a four year college degree. A typical four year Bachelors degree in construction project management is described as: The bachelor’s degree curriculum in construction management prepares graduates for employment in a variety of positions in the construction industry. Graduates are employed as construction planners, site supervisors, schedulers, and contractors. Courses in the construction management program are a blend of practical courses with emphasis on hands-on experiences and classroom based courses which emphasize management principles, estimating, and scheduling. Elective courses or transfer courses provide for experiences in related areas including HVAC, architectural design, and electrical wiring. (Northern Michigan University,2006, online)

Having established the basic criteria to become construction project manager, let’s look at the evolution of CM as a profession against the 20 primary traits and 15 qualifying or sub-traits of a profession. Primary traits include those outlined in Exhibit 3A.1.

93 Chapter 3A: Case Study 1, Construction Project Management

Exhibit 3A.1 Primary traits of a profession body of knowledge

require professional liability insurance

long period of training

autonomy in decision making

lifetime commitment

identify with profession

adhering to a code of ethics

not employer

fiduciary obligation to the public

held in high esteem by their home community

other obligations to the public

earn higher than average compensation

professional associations

exhibit mystique

publishing in learned journals

body of knowledge is abstruse, esoteric or secret

advertising not permitted or restricted

highly ritualistic procedures

use of the title is restricted by law

access to knowledge is limited

symbolic costumes or uniforms practice limited by government license

BODY OF KNOWLEDGE

The body of knowledge for construction project management is virtually identical to the body of knowledge for all other disciplines, with the exception that safety, health and the environment play a major role in CM, much more so than is found in methodologies such as PRINCE2 (PRINCE2, online) or PMBOK guide (2004). But regardless, even with the addition of those sub-processes associated with safety, health and the environment, the body of knowledge will not meet the criteria of being in any way unique, esoteric, abstruse or secret. While architects and engineers each have a robust and clearly identifiable body of knowledge which meets these criteria, construction project management does not, and this will continue to constrain CM from being recognized as a stand alone profession, at least in the context of the traditional or extrinsic benchmarks. Of interest is the fact that the trades do have a body of knowledge which meets the test of being somewhat unique (the terms a carpenter uses are often not the same as those an electrician would use) which reflects a vestigial link with the trade guilds of the 15th-17th centuries. So while the ‘parents’ of CM – architecture and engineering – have a body of knowledge which is unique, esoteric, abstruse, complicated or secret, as do the trades which the construction project manager oversees, CM per se lies in that grey area, someplace between architects and engineers and the trades. About the only aspects of construction project management which could be considered ‘unique’ might be the application of CM scheduling and/or earned value management.

94 Chapter 3A: Case Study 1, Construction Project Management

LONG PERIOD OF TRAINING

As evidenced by the Purdue History, up until the late 1960s and even well into the 1970s, construction project managers traditionally came from the trades, in most cases, carpentry. Prior to having construction project managers, most construction projects were run by a general superintendent. Invariably, this individual was in his 60s (very few were women), and had worked his way up through the trades, starting usually around 30 years old by becoming a trade foreman, assistant or area superintendent, trade superintendent and eventually general superintendent. While the career path wasn’t formalized, everyone knew what it was and how to go about moving up the ladder3. Because the focus was not on degrees or knowledge, but on demonstrated competency, very rarely did anyone become a foreman before the age of 30, assistant or area superintendent by 40, and general superintendent before age 50. Most superintendents on large projects were well into their 60s before they had earned a reputation as a competent professional. This model started changing during the 1970s as the experienced superintendents managers started retiring; they were replaced not with people who had come up through the trades, but with ‘shake and bake’4 construction managers with engineering or architectural degrees. By the turn 21st century what used to take 20 years of work experience to develop had been compressed into a four year degree and perhaps four or five years of experience. Below is a typical CM degree program. Most four year degrees require approximately 130 semester hours of course work, with approximate 800 hours of field experience (Purdue University, 2006, online). Presented here is a sample Plan of Study for students pursuing a Building CM degree at Purdue University. This document presents an overall idea of the order in which courses might be taken in a four-year plan during a student’s college career.

3

4

The researcher is writing from first hand experience, having come up through the ranks as a union carpenter, while going to school at night to earn an undergraduate degree in CPM. In essence, following the exact career path being described. Authors Note: The term ‘shake and bake’ refers to commercially available breading mixes complete with all spices, allowing people to dip raw chicken into the pre-mixed packet of materials and throw it in the oven. It has come to mean any ‘instant’ product, with little or no mess or labor involved.

95 Chapter 3A: Case Study 1, Construction Project Management

Please Note: The Plan of Study on this page is presented for general information only – for exact personal plans of study, each student must see his or her counselor for an accurate record of the Plan of Study that he or she received when entering the BCM program. Curriculum plans of study may change often, while the plan of study received upon entering remains the same for that entering student. The Building Construction Management program is accredited by the American Council for Construction Education (ACCE). Credit Hours Required for Bachelor’s Degree: 129 Table 3A.2 Typical CM Curriculum Semester 1 Course Number

Course Name

Credit Hours

BCM 100

Introduction to Construction

2

CPT 135

Introduction to Computer Technology and Applications

2

MA 159

Pre-Calculus

5

ENGL 106

English First Year Composition

4

Elective

Human Relations Elective

3

TOTAL

16

Course Number

Course Name

Credit Hours

BCM 175

Construction Materials & Methods

4

CGT 164

Graphics for Civil engineering & construction

2

MA 221

Calculus for Technology I

3

OLS 274

Applied Leadership

3

COM 114

Fundamentals of Speech Communication

3

TOTAL

15

Course Number

Course Name

Credit Hours

BCM 112

Construction Surveying Fundamentals

3

BCM 215

Mechanical construction

3

PHYS 218

General Physics

5

BCM 275

Construction Plans and Measurements

3

Elective

Communication Selective

3

TOTAL

17

Course Number

Course Name

Credit Hours

BCM 212

Construction Layout

3

BCM 216

Electrical construction

3

Semester 2

Semester 3

Semester 4

ECON 210

Principles of Economics

3

BCM 285

construction Mechanics

4

Elective

Science

4

TOTAL

17

Semester 5 Course Number

Course Name

Credit Hours

BCM 385

Soils and Foundations

3

BCM 375

Estimating

3

BCM 350

Construction Site Planning

3

BCM 380

Concrete construction

3

Accounting Principles

3

TOTAL

15

MGMT 190B

96 Chapter 3A: Case Study 1, Construction Project Management

Semester 6 Course Number

Course Name

Credit Hours

BCM 301

Construction Accounting and Financial Management

3

BCM 345

Scheduling

3

MGMT 455

Legal Background for Business I

3

STAT 301T (or IT 342)

Statistical Methods (or Introduction to Statistical Quality)

3

Elective

General Education Elective

3

Elective

Technical Elective

2

TOTAL

17

Semester 7 Course Number

Course Name

Credit Hours

BCM 355

Construction Supervision

3

BCM 450

Construction Contracts & Administration

3

BCM 475

Construction Costs

3

ENGL 420 ( or ENGL 421)

Business Writing (or Technical Writing: engineering & Science Applications)

3

Elective

Technical Elective

3

TOTAL

15

Course Name

Credit Hours

Semester 8 Course Number BCM 455

Construction Company Management

4

BCM 457

Construction Safety

3

Elective

BCM Elective

3

Elective

Technical Elective

Elective

Free Elective

3

TOTAL

16

In addition to Course Work, also required Construction Work Experience A minimum of 800 hours of construction work experience is required before graduation. Summer construction jobs, BCM internships, or BCM Co-op programs may be used to satisfy this requirement.

An hypothesis has begun to evolve that explains the apparent stagnation of the professionalization of construction project management as a result of the focus on academic study with reduced emphasis on practical skills. Given that government licensing is clearly less desirable in today’s professional world than voluntary Board Certifications, the competency model being advocated by GAPPS, when combined with a four year degree seems be the ideal combination, and is consistent with the concept of ‘life long learning’. The study of the professionalization of airline transport pilots confirms that. Once a construction project manager has earned his or her degree, certification is required by a professional organization in order to work in the industry. The educational requirements necessary to become a Certified Construction Manager

97 Chapter 3A: Case Study 1, Construction Project Management

(CCM) through CMAA or a Certified Professional Constructor (CPC) through AIC are not stringent. CMAA (CMAA, n.d., online) requirements are outlined in Exhibit 3A.2: Exhibit 3A.2 CMAA requirements for certification 1.

(a) 48 months experience as a CM in the qualifying areas as defined by the Qualifications Matrix

AND 2.

ONE OF THE FOLLOWING:

a.

An undergraduate (4-year BA/BS level) or graduate degree in construction management, architecture, engineering or construction science.

b.

A 2-year undergraduate degree (AA/AS level) or certificate in construction management, architecture, engineering or construction science AND 4 years experience in general design/construction. (This experience is in addition to the 48 month CM requirement.)

c.

No degree/certificate in construction management, architecture, engineering or construction science AND 8 years experience in general design/construction. (This experience is in addition to the 48-month CM requirement.)

While the civil engineer’s PE process and the architect’s licensing process both focus on demonstrated competency (as evidenced by having to create and present a portfolio of project experience before one’s peers) rather than academic achievement, the CM credentials, while requiring varying years of on site field experience, do not have anyway of qualifying whether or not this experience was good or bad. This has considerable relevance to project management in general, for like flying, sex or even riding a bicycle, you can read all the books, take all the courses and pass all the knowledge-based exams, but until you actually fly, have sex, ride a bicycle or manage projects, and can demonstrate competency by consistently doing them well, everything else is irrelevant. LIFETIME COMMITMENT

When project management began to be recognized as a separate activity on a building site, a tradesman knew that if he was good at the organizational aspects of his job, he had the opportunity to take on more responsibility and get increasingly larger and more complex projects to work on. In construction, there is a tremendous sense of accomplishment, as the efforts of your labor are readily visible and will last for your entire life. Under those circumstances, it was easy to make a long term commitment from the days you first started in the trade.

98 Chapter 3A: Case Study 1, Construction Project Management

While few CMs start out in the trades any longer, construction project management remains an identifiable career path. Being a construction project manager is not something most people do as part of moving on to something else, but an end objective in and of itself. This is evidenced by reference in the American Institute of Architects web to the ‘calling’ of architecture (AIA, n.d., online). For construction project managers, as with architects and engineers, ‘project management’ has the potential to be the top of a career path in an organization, unless they work for a very large firm or become private practitioners. Novice construction project managers, however, have no process of ‘building hours’ in a competency-based program to sort out those who are committed from those who are not. Project management is often called the ‘accidental profession’ (Wideman, n.d., online); and while calling it a profession remains a dubious practice, the fact remains that many do enter into general project management by accident rather than choice. With the proliferation of degree programs in project management, and project management being recognized as a career path option in many companies, this may be changing to follow more along the lines of construction project management. ADHERING TO A CODE OF ETHICS

Architects, civil engineers, construction project managers, cost engineers, quantity surveyors, the Royal Chartered Institute of Building and the American Institute of Constructors all have codes of ethics. Both the American Institute of Constructors and Construction Management Association of America have rather general codes of ethics, especially when compared to the more established American Society of Civil Engineers (ASCE) and the American Institute of Architects (AIA). All of these representative bodies publish their codes on their websites and in their literature. The CMAA code of ethics reads (CMAA, n.d., online): As a professional engaged in the business of providing construction and program management services, and as a member of CMAA, I agree to conduct myself and my business in accordance with the following: 1. Client Service. I will serve my clients with honesty, integrity, candor, and objectivity. I will provide my services with competence, using reasonable care, skill and diligence consistent with the interests of my client and the applicable standard of care.

99 Chapter 3A: Case Study 1, Construction Project Management

2. Representation of Qualifications and Availability. I will only accept assignments for which I am qualified by my education, training, professional experience and technical competence, and I will assign staff to projects in accordance with their qualifications and commensurate with the services to be provided, and I will only make representations concerning my qualifications and availability which are truthful and accurate. 3. Standards of Practice. I will furnish my services in a manner consistent with the established and accepted standards of the profession and with the laws and regulations which govern its practice. 4. Fair Competition. I will represent my project experience accurately to my prospective clients and offer services and staff that I am capable of delivering. I will develop my professional reputation on the basis of my direct experience and service provided, and I will only engage in fair competition for assignments. 5. Conflicts of Interest. I will endeavor to avoid conflicts of interest; and will disclose conflicts which in my opinion may impair my objectivity or integrity. 6. Fair Compensation. I will negotiate fairly and openly with my clients in establishing a basis for compensation, and I will charge fees and expenses that are reasonable and commensurate with the services to be provided and the responsibilities and risks to be assumed. 7. Release of Information. I will only make statements that are truthful, and I will keep information and records confidential when appropriate and protect the proprietary interests of my clients and professional colleagues. 8. Public Welfare. I will not discriminate in the performance of my Services on the basis of race, religion, national origin, age, disability, or sexual orientation. I will not knowingly violate any law, statute, or regulation in the performance of my professional services. 9. Professional Development. I will continue to develop my professional knowledge and competency as construction Manager, and I will contribute to the advancement of the construction and program management practice as a profession by fostering research and education and through the encouragement of fellow practitioners. 10. Integrity of the Profession. I will avoid actions which promote my own self-interest at the expense of the profession, and I will uphold the standards of the construction management profession with honor and dignity.

The American Institute of Constructors (AIC) also has a rather basic code of ethics: I.

A member shall have full regard to the public interest in fulfilling his or her responsibilities to the employer or client.

II.

A member shall not engage in any deceptive practice, or in any practice which creates an unfair advantage for the member or another.

III.

A member shall not maliciously or recklessly injure or attempt to injure, whether directly or indirectly, the professional reputation of others.

IV.

A member shall ensure that when providing a service which includes advice, such advice shall be fair and unbiased.

100 Chapter 3A: Case Study 1, Construction Project Management

V.

A member shall not divulge to any person, firm, or company, information of a confidential nature acquired during the course of professional activities.

VI.

A member shall carry out responsibilities in accordance with current professional practice, so far as it lies within his or her power.

VII. A member shall keep informed of new thought and development in the construction process appropriate to the type and level of his or her responsibilities and shall support research and the educational processes associated with the construction profession

Critical analysis of construction project management codes of ethics. A brief critical analysis on both CMAA’s code of ethics as well as that of AIC in the context of prior literature reviews follows. Construction Management Association of America „

professional engaged in the business As we know from previous literature research, a profession is more than just a business (Polelle, 1998).

„

reasonable care We also know from literature reviews that the standard of care by which nonprofessionals are judged is that of the ‘reasonable man’, while professional standards of negligence are those determined by ones peers coming from the same discipline and the same region. This would imply that construction project management is not a profession (Polelle, 1998; Rotunda, 2002).

„

technical competence Competence is defined as ‘the quality or state of being functionally adequate, characterized by marked or sufficient aptitude + attitude + skills + strength + knowledge’ (Merriam-Webster, 2006). It would appear as though the CCM credential does not meet any identifiable competency criteria.

„

established and accepted standards of the profession This definition of standards stands in direct opposition to the ‘reasonable care’ standard identified above. Is the standard of care the ‘reasonable man’ or that of a profession?

„

construction and program management practice as a profession Based on the precedent set by Pierce v. AALL Insurance Inc. Gardner v Frier, we know (at least in Florida and North Dakota) that one of the ‘bright line’ rules is that a profession requires at minimum a four year degree. As we can see from the certification application process that as in Gardner v Frier, a four year degree is not required.

101 Chapter 3A: Case Study 1, Construction Project Management

„

standards of the CM profession As with previous comments, there seems to be confusion as to whether or not construction project management is or is not a profession (reference CMAA mission statement) and whether the standards of care are those of the ‘reasonable man’ or those of one’s peers.

American Institute of Construction Managers „

current professional practice As with CMAA, AIC seems to be confused as to whether project management is or is not a profession. Their certification requires a two year degree (Associates Degree) but also fails to meet any ‘bright line’ tests at least thus far.

As far as professional ethics are concerned, it would appear that professional organizations representing construction project managers are not as aggressive or proactive about ethical issues as are architectural or engineering societies. The 150 year old American Society of Civil Engineers in particular is sensitive about the image of its members and the responsibility of ‘policing its own’, particularly in regard to malpractice. Of special interest is the ASCE’s recently adopted zero tolerance policy on bribery. Guidelines and Policies Encouraging Zero Tolerance for Bribery ASCE, in cooperation with others, has begun a journey that will lead over time to the development of principles of professional conduct that will help reduce corruption in all its forms, such as fraud, bribery and other unethical and illegal practices in the engineering and construction industry. Only with openness and transparency in the procurement and delivery of engineering services can resources be efficiently allocated for their intended purpose. As a direct result, with the additional financial resources available, sustainable development and the welfare of the world’s population can improve and the quality of life for people everywhere be achieved. The American Society of Civil engineers is joining the consulting engineering profession, the construction industry, major lenders, the legal profession, and organizations committed to the battle against worldwide corruption. To meet this challenge, the Society pledges the development of principles and over time adoption of guidelines and policies for ethical professional practice by members of engineering societies in the U.S. and worldwide. These guidelines and policies will include zero tolerance for bribery and promote effective programs for reporting unethical behavior. The Society will consider the best practices adopted by the engineering and construction industry in developing or benchmarking anti-bribery programs. ASCE, supported by the Task Committee on Global Principles for Professional Conduct, will provide – through its media outreach and forums of leaders, and other avenues – a global communications platform for the issue. Our work will be performed in an open and transparent manner to encourage good faith discussion and negotiations leading to principles and policies that other societies could adopt. (ASCE, n.d., online)

102 Chapter 3A: Case Study 1, Construction Project Management

Likewise, architects, sensitive about the possibility of anti-trust violations (AIA, n.d., online), have dedicated a special series of papers outlining their policies towards noncompetitive practices. The fact that ASCE has a robust policy on bribery and the AIA has several position papers pertaining to anti-trust actions, indicates the greater maturity and sophistication of two 150 year old professional organizations compared to two 35 year old organizations which represent emergent or developing professions. A robust code of ethics and the sense of fiduciary obligation (see below) are tacit acknowledgement of the tensions inherent in the position of construction manager who will be working in a situation fraught with possibilities for malfeasance or simple negligence, both on the part of the construction manager and the sponsor. The relationship of the PM and the sponsor is likely to be mediated by a contract, but moral responsibilities remain that can only be honoured by the exercise of goodwill and good communication between the parties, as suggested by agency theory (Muller & Turner). FIDUCIARY OBLIGATION TO THE PUBLIC

Fiduciary obligation defined. A fiduciary relationship is: the relation by law existing between certain classes of persons (as confidential advisor and the one advised; executors or administrators and legatees or heirs; conservators and wards, trustees, or beneficiaries; partners, joint adventurers, corporate directors or officers and stockholders; majority and minority stockholders; factors, agents, or brokers and principals; attorneys and clients; promoters and stock subscribers; mutual savings banks or investment corporations and their depositors or investors; receivers, trustees in bankruptcy, or assignees in insolvency and creditors). (Merriam-Webster, 2002)

In the context of construction project management professionals, there are three legal obligations which carry with them implied fiduciary obligations: „

jobsite health and safety

„

environmental protection

„

conformance to building codes

These obligations of a professional construction project manager in terms of jobsite safety (which includes health) apply to both the workers on the project and the general public. As construction projects are known legally as ‘attractive nuisances’, the construction project manager has a legal and ethical obligation to ascertain that not only are the workers on the jobsite protected against harm by conformance to

103 Chapter 3A: Case Study 1, Construction Project Management

OSHA laws and regulations, but also that the project is protected against trespassers, particularly children, from being injured or killed during hours when the project is shut down. The construction project manager also has a legal obligation to ensure that all building codes are met in both the execution of the project and in delivering the finished project, and that no environmental laws are broken. The fiduciary relationship implies that a professional construction project manager has the obligation not only to meet the letter of the law, but also to make reasonable efforts to understand and comply with the intent of the law. The importance of owing the first loyalty to the health, safety and welfare of the general public is common to most professions. The medical profession has their Hippocratic Oath which states in part a doctor ‘should do no harm’ (Wikipedia, n.d., online). This theme of doing no harm is common to the ASCE, AACE and IEEE codes of ethics as well. Construction project management appears to be weak on this aspect of professionalism. The AIC code of ethics addresses the fiduciary responsibility with the statement ‘full regard to the public interest’ while the CMAA code of ethics talks about public welfare, but then describes in more detail that the certified construction manager is not allowed to discriminate and will obey all laws and regulations. Not quite the same as having a fiduciary obligation to the consuming public, which is implicit in true professions (Maister, 1996). OTHER OBLIGATIONS TO THE PUBLIC

Construction project management as represented by its societies doesn’t appear to be overly focused on service to the public, save for the responsibility to safety, health and the environment, which more appropriately falls under other headings. However, community service projects, such as constructing playgrounds in poor communities or volunteering to work for such organizations as Habitat for Humanity are not at all unusual. So from that perspective, construction project managers are well positioned to offer pro bono services, the primary difference being that they often provide these pro bono services as individuals rather than as organizations. This differs somewhat from law and medicine, where the organization formally supports the provision of pro bono services.

104 Chapter 3A: Case Study 1, Construction Project Management

PROFESSIONAL ASSOCIATIONS

Professional associations are considered legally, socio-economically and semantically to serve four major functions: „

to create and enforce the code of ethics

„

to establish acceptable standards

„

to establish, monitor and maintain performance standards

„

to develop, monitor and maintain procedural standards (methodology)

The primary organizations representing the global practice of CM are those subscribe to by architects, civil engineers, CM societies, cost engineers, Royal Chartered Surveyors and the Royal Chartered Institute of Building and the American Institute of Constructors, the Project Management Institute, and the International Project Management Association. There are two major US organizations serving to represent construction project managers: „

the American Institute of Constructors (AIC) www.aicnet.org

„

Construction Management Association of America (CMAA) www.cmaanet.org

In other countries, notably South Africa and Ukraine, construction project management has evolved to the point where the practice of construction project management is licensed. Whether that will or even should happen in the USA remains to be seen. However, despite both the CMAA and AIC being 35 years old, construction project management has not been able to establish equal professional footing with either architects or engineers on a project and, given the inevitable conflicts, invariably the architects and/or engineers prevail over construction project managers, at least prior to dispute resolution processes or litigation. Professional organizations establishing acceptable standards. While all CM organizations have created ‘standards’, those closest to the architectural and structural engineering societies have the most credibility. Consistent with the observations of Andrew Abbot (1988) is the fact that the creation of standards has led to vitriolic turf

105 Chapter 3A: Case Study 1, Construction Project Management

wars among architects, engineers and construction project managers. These battles have resulted in some interesting unintended consequences, again which reinforce Abbot’s (1988) theory of competition. At the time that CM was in its infancy, battles were being fought between architects, primarily represented by the American Institute of Architects (AIA) and the National Societies of Professional Engineers (NSPE) over who had the right to create specifications. Out of this dispute, a compromise was reached, which in effect created a sub-specialty profession –specification writing. Thus was born the Construction Specifications Institute (CSI) which consists of architects, engineers and lawyers, who specialize in the writing of technical specifications and integrating them into standardized contract documents used by architects, engineers, construction managers and anyone else involved in the built environment. (See http://www.csinet.org/s_csi/index.asp.) An indication of the turf wars between the architects, engineers and the building contractors can best be evidenced by the fact each of them issue their own sets of contract documents (AIA, online). The Construction Specifications Institute (CSI) has become the defacto middle ground, however, as all actors on the CM stage have agreed to use the CSI specifications. This has significance to other emergent or nascent users of project management as a delivery method, in so far as the Construction Specifications Institute’s Master Format (CSI, online) and Uniformat coding structure have enabled the creation of a WBS template used by everyone in the built environment. While not very well communicated, the effect the CSI has had on cost estimating, scheduling and contract administration by creating a standardized coding structure has been instrumental in the integration of architectural and engineering design documents to the cost estimating and CM scheduling software. Thus it is possible for architects and engineers to create the structure using 3D computer assisted drafting while at the same time generating real time cost estimates and construction schedules. This level of sophistication has yet to reach the IT and telecommunications sectors. This section on standards would not be complete without including some discussion of the Construction Industry Institute (CII):

106 Chapter 3A: Case Study 1, Construction Project Management

CII is a consortium of leading owners, engineering and construction contractors, and suppliers who have a singular mission: to improve the cost effectiveness of the capital facility project life cycle, from pre-project planning through completion and commissioning. By collaborating on important industry issues and by providing guidance on best practices discovered through research, the CII members are collectively an industry forum for the engineer-procure-construct process. CII, through its research, implementation, education, and other initiatives, is a learning organization with a wealth of knowledge and information. The CII funded research program, with more than 30 leading U.S. universities involved, is unique in the engineering and construction industry. The research results lead to best practices for the entire industry to share and implement to improve project success. (Construction Industry Institute, n.d., online)

As seen from the limited sampling above, CM is rich with professional organizations willing to set standards. While this can and has resulted in conflicts and disputes over areas of responsibility, the organizations have managed to reach compromises through joint alliances, partnering or collaboration. The fact that CM remains in many cases embedded in either schools of civil engineering or schools of architecture, serves as a deterrent or constraint to the construction manager being perceived as being on an equal standing as a separate and distinct discipline with a valuable role to play in the built environment. PUBLISHING IN LEARNED JOURNALS

Each of the organizations representing construction project managers publishes journals, but few have the prestige and ranking of the American Society of Civil Engineers Journal of Construction Engineering and Management. http://www.pubs.asce.org/journals/co.html While each of the professional organizations directly or indirectly related to construction project management publish journals, all of which are available from the relevant websites, several are worth noting. The Construction Industry Institute (CII) is noted for its strong research efforts (CII, n.d., online). From the standpoint of construction project management, there are volumes of highly regarded publications, not only coming from ASCE and AIA, but also from CMAA, CSI, AACE and RICS.

107 Chapter 3A: Case Study 1, Construction Project Management

ADVERTISING NOT PERMITTED OR RESTRICTED

Advertising for CM practitioners is generally covered through the codes of ethics. But essentially, there are no limits on advertising comparable to those imposed on the legal, medical or social working communities of practice. USE OF THE TITLE IS RESTRICTED BY LAW

The best example of this rather archaic practice comes from a civil engineers policy adopted 14 May, 2004. As the trend is clearly to move away from restricting titles by law, it is incumbent upon those organizations representing construction project managers to establish a strong brand image. Use of the term ‘engineer’ Approved by the Committee on Professional Practice on January 17, 2004 Approved by the Board Policy Committee on March 12, 2004 Adopted by the Board of Direction on May 14, 2004 Policy The American Society of Civil engineers (ASCE) believes that the following standards are the only basis on which any title or designation should include the term ‘engineer’. –

Graduation from an accredited engineering program with a degree in engineering;



Registration as a professional engineer or engineer-in-training under a state engineering registration law; or,



An official ruling designating an individual or a group in an engineering capacity as meeting the definition of ‘Professional engineer’ under the Taft-Hartley Act or the Fair Labor Standards Act.

Only persons in one of these categories should be designated by the title ‘engineer’ or ‘professional engineer’. This policy shall not be construed to prohibit using the word ‘engineering’ as a modifier in titles such as ‘engineering assistant’, ‘engineering aide’ and ‘engineering technologist’ where the title clearly implies that the duties of the position are not those of professional engineer. ASCE further encourages registered professionals to always use their P.E. title on all professional correspondence and communication. Issue Improper use of the term ‘engineer’ is sometimes confusing or misleading to the public. Employers and employees misuse the term in titles and resumes. This misuse of the title by groups and people who are usually knowledgeable tends to diminish the value of the title which should be applied to people qualified professionally by accepted standards of education, law and/or engineering practice.

108 Chapter 3A: Case Study 1, Construction Project Management

Rationale There is a need within ASCE as well as within government and other organizations with practicing professional engineers to provide employee titles and/or classifications that properly identify the individual’s level of responsibility or expertise within that organization. A title such as ‘designer’ is not proper for a graduate engineer with several years of experience; ‘associate engineer’ or similar title as used by ASCE in designating professional grades is more appropriate and strongly encouraged. (ASCE 2004)

Perhaps a more modern approach has been adopted by the National Association of Realtors (NAR), who, instead of attempting to get legislatures to restrict the title by law, registered the term ‘Realtor’ as a trademark. Only members of the National Association of Realtors are permitted to use the registered term, along with several others registered by the association, and indicated by the symbol ®5. PMI seems to have adopted the same strategy, registering all its trade names, including the Project Management Professional (PMP). While PMI has done an outstanding job of marketing and establishing brand awareness, none of the organizations focused specifically on construction project management have been anywhere near as successful. As noted previously, much of that had to do with the fact that most construction project managers were coming from either a civil engineering background, and tended to identify more with being an engineer than being a construction manager, or from a background in architecture, where again, the loyalty or professional identity was assigned to architecture rather than CM. SYMBOLIC COSTUMES OR UNIFORMS

In their 1969 song Streets of Laredo, the Smothers Brothers parodied a traditional western US folk song The Cowboy’s Lament. The original message was intended as a dig at the consumer culture of the 1960s, but in the context of this research, it stands as a simple yet powerful example of the importance of a uniform in establishing a brand image. Original. As I walked out in the streets of Laredo, as I walked out in Laredo one day. I spied a young cowboy dressed in white linen. Dressed in white linen and cold as the clay.

5

See http://www.realtor.org/letterlw.nsf/pages/mmmPartOne#OneI for further information about the NAR’s trademarks.

109 Chapter 3A: Case Study 1, Construction Project Management

Smothers’ rejoinder. I can see by your outfit that you are a cowboy. You can see by my outfit I’m a cowboy, too. You can see by our outfits that we are both cowboys. If you get an outfit you can be a cowboy, too! (Smothers, D. & Smothers, T., n.d., online)

While intended for amusement, there is a message in the song about the importance played by uniforms in identifying members of the same group, and establishing an immediate bond. The power of a uniform on the wearer and those who come into contact with them cannot be emphasized highly enough. While somewhat peripheral to this research, several serious academic studies exist which emphasize the importance of uniforms in the context of professional image. Several of the most widely cited are Diana Crane’s (2000) Fashion and its social agendas: Class, gender and identity in clothing. Her study examines the social significance of fashion and clothing choices in France, the US and England from the middle of the 19th century to the present day. Most relevant to this research, Professor Crane dealt with the importance of uniforms and dress codes as a form of social control (Crane, 2000). Unlike commercial airline piloting, CM has no uniforms or costumes. However, civil engineers in North America have a secret ceremony where a ring of iron (Canada) or stainless steel (USA) is worn on the fifth finger of the working hand, awarded during a ceremony in which the engineer takes the following pledge: Obligation of the engineer I am an engineer, In my profession I take deep pride. To it I owe solemn obligations. Since the stone age, Human progress has been spurred By the engineering genius. Engineers have made usable, Nature’s vast resources of material and energy For humanity’s benefit. Engineers have vitalized And turned to practical use The principles of science And the means of technology. Were it not for this heritage of accumulated experience, My efforts would be feeble. As an engineer, I pledge to practice integrity and fair dealing, Tolerance, and respect And to uphold devotion To the standards and the dignity of my profession,

110 Chapter 3A: Case Study 1, Construction Project Management

Conscious always That my skill carries with it The obligation to serve humanity By making the best use of earth’s precious wealth. As an engineer, I shall participate in none but honest enterprises. When needed, My skill and knowledge Shall be given without reservation For the public good. In the performance of duty And in fidelity to my profession, I shall give the utmost.

While perceived by some as being corny and archaic, the ceremony does instil a sense of obligation and responsibility. Given civil engineering has long qualified as a profession, and construction project management has not yet made it, establishing some sort of ritual, initiation process or ceremony may have something to do with the image of the practitioner. PRACTICE LIMITED BY GOVERNMENT LICENSE

For architects and engineers, to practice requires a license. The license is almost always granted at the state or provincial level and, while there is reciprocity, it is given sparingly. In the case of structural engineers, it makes sense that one license does not fit all cases. In Florida and the Gulf coast states, hurricanes are the most common cause of structural damage, while in the northeast and mountains states, snow loads are the leading cause of structural failures. In the West and Alaska, earthquakes pose the greatest threat. As an engineer is not likely to have expertise in all these areas, it makes sense that a structural engineer from Boston probably won’t have the skills necessary to design work in San Francisco. So unless he or she has evidence of taking specialized courses or working under a person licensed in that area, reciprocity would be unsafe. This is not a problem if the construction manager is coming from a background in civil engineering or architecture, as those professions are recognized and do require licenses to practice. The challenge today is, as CM is starting to claim a niche of its own (as evidenced by degrees in construction project management alone, such as the Purdue example) demand for licensure will probably not occur. However, in all but two states (Arkansas and Mississippi), building contractors are required to be licensed. The licensure of builders in all other states is based on the knowledge of the

111 Chapter 3A: Case Study 1, Construction Project Management

relevant building codes. As the distinction between builders and construction project managers is slight, arguments could be made that licensing of construction project managers already exists. REQUIRE PROFESSIONAL LIABILITY INSURANCE

Architects and engineers are clearly required to have and maintain professional liability insurance [also known as Errors and Omission (E&O) Insurance], and professional liability insurance is not uncommon for project managers. An example from the University of California (University of California, n.d., online) seems typical of what is demanded of architects, engineers and others operating in the built environment. 1. General Liability: Comprehensive or Commercial Form (MINIMUM LIMITS) (1) Each Occurrence $1,000,000 (2) Products/Completed Operations Aggregate $1,000,000 (3) Personal and Advertising Injury $1,000,000 (4) General Aggregate* $2,000,000 * (not applicable to comprehensive form) If the above insurance is written on a claims made form, it shall continue for three years following termination of the agreement. The insurance shall provide for a retroactive date of placement prior to or coinciding with the effective date of the agreement. 2. Business Automobile Liability: (MINIMUM LIMITS) for Owned, Scheduled, Non-Owned, or Hired Automobiles with a combined single limit of not less than $1,000,000 per occurrence. 3. Workers’ Compensation: as required under California State Law. 4. Professional Liability Insurance: (MINIMUM LIMITS) (1) Each occurrence $1,000,000 (2) Project Aggregate $2,000,000 If the above insurance is written on a claims made form, it shall continue for three years following termination of the agreement. The insurance shall provide for a retroactive date of placement prior to or coinciding with the effective date of the agreement.

112 Chapter 3A: Case Study 1, Construction Project Management

At least for construction project management, liability insurance is required in some instances, and is commercially available6. Given construction project management is one of the more mature users of project management as a delivery system, this is interpreted to be a sign of the future. AUTONOMY IN DECISION MAKING

A construction project manager generally has full profit and loss statement responsibility for the project. He or she can hire or fire temporary employees, sign checks, issue purchase and work orders and give bonuses. Within the constraints of the technical specifications and the contract terms and conditions, a construction project manager has near total authority for all decisions. He or she determines who does what, when they do it and how they do it. Along with this authority comes ultimate responsibility. When compared to the authority of project managers in the IT and telecommunications sectors, there is generally far less flexibility, although the accountability may we the same. However, construction project managers generally do not have the same clear cut responsibility or authority as a pilot in command of an aircraft. According to ENR, 20%-30% of construction projects were late, over budget or failed to substantially satisfy the needs, wants and expectations of their clients. While considerably better than the 30%-60% failure rates reported in the IT sector by Gartner, Standish META or FMI groups, for a sector that is one of the most mature users of project management to still be experiencing 20%-30% failure rates after 50 years or more of ‘professionalization’ of construction management is not acceptable. Using the rate of litigation as a measure of a ‘failed’ project clearly indicates the trend is continuing upwards. While difficult to prove, using the airline transport pilot as an example, it looks as though the traditional model of knowledge and management skill acquisition where construction managers worked their way up through the trades, with the focus on experience and competency rather than theoretical knowledge, had a lot to offer.

6

See Professional Design Insurance Management Corporation (n.d.). Project construction management insurance. Retrieved 7 Jun 06 from http://www.project-management-insurance.com/

113 Chapter 3A: Case Study 1, Construction Project Management

IDENTIFY WITH PROFESSION, NOT EMPLOYER

The opening statement from the ‘Obligation of the engineer’ pretty much sums up what this attribute or trait is all about: Obligation of the engineer I am an engineer, In my profession I take deep pride.

As most construction project managers are employees rather than independent contractors, it is unlikely that they would identify with the profession first, unless they have come from a technical discipline with which they can identify, or construction project management is able to extricate itself from its position in the shadow of architects and engineers and stand on its own. Evidence this may be occurring exists. ‘The most common degree title is Construction Management among 51 of the 88 members of the Associated Schools of Construction that responded to an Engineering News-Record (ENR) survey’ (Rosenbaum & Rubin, 2001). However, the relative stagnation of CM’s efforts at professionalization, raises serious questions in terms of the intrinsic attributes. There are three categories of people who are likely to belong to professional organizations representing project management practitioners. There are the private practitioners, primarily trainers and consultants, who work on a fee for service basis. These practitioners more likely perceive the ‘organization of their dreams’ as being much along the lines of the Bar or Medical Associations. The second category of members consists of employees of large companies. These are typified by members of other organizations, such as those representing airline pilots, nurses, school teachers and even some physicians, if they are working for an HMO on the basis of a salary and not fee for service agreement. This category of person is probably seeking an organization much along the same lines as airline pilots, where the professional organization functions not only to meet the professional development needs of the practitioner but also to function as a collective bargaining agent. Probably the two most active models are the teachers unions7, and the pilots union, ALPA8. 7

See National Education Association, http://www.nea.org/index.html; United Federation of Teachers, http://www.uft.org/ 8 See Air Line Pilots Association, International, http://www.alpa.org/

114 Chapter 3A: Case Study 1, Construction Project Management

The third category of practitioners, identified by Zwerman et al. (2004) is the knowledge worker. This category of worker is not as well defined, but assuming Tom Osenton in his 2002 book Death of demand is correct that global demand is leveling off; and that as the developing nations increase their skill levels, they are quickly moving up the supply chain from agriculture to low skilled manufacturing, to high skilled manufacturing, to knowledge work, it would not be surprising to see more outsourcing. Tom Peters (2003), in his book, Re-Imagine, is painting a picture of a world in which most of us will be temporary workers. Assuming that model to be correct, then the ‘traditional’ model of an organization will ultimately prevail. If this is the case, then AACE was and remains an ‘appropriate’ model. HELD IN HIGH ESTEEM BY THEIR HOME COMMUNITY

The test for this is fairly simple. How does your mother introduce you to her friends? Does she say, ‘my son/daughter works for Fluor Daniel, as a project manager’, or does she say ‘my son/daughter, the project manager, currently working for Fluor Daniel’? This simple test will tell as much as needed about the professional standing of any occupation.

Figure 3A.1

Esteem question results from the survey by - project managers

As can be seen from the survey results, project managers have mixed emotions about the real or perceived status of project management. With an average score of 3.52 out of a possible score of 5, most practitioners perceive project management as a strong moderate in terms of their perceived esteem in the community. Given that construction project managers comprised approximately 12% of the respondents, and the results were statistically consistent with the full population results. 115 Chapter 3A: Case Study 1, Construction Project Management

Having noted this, coming from a background in construction, to hold the title of ‘construction project manager’ carries with it an aura of prestige and respect. Normally, a person does not become a construction project manager on a project of any size or significance until well into his or her 40s. But, ultimately, the esteem one is accorded in a large part, is a direct reflection of the confidence and self esteem that one exudes. The survey results indicated that there was little statistical difference in how people responded to this question based on age, sex, geographical region or other data. Those identifying themselves as construction project managers produced results statistically similar to those of the entire population. This is not consistent with the results of the AACE survey results shown in Table 10, and my own personal experience from the USA. The apparent conflicting results are sufficiently surprising to warrant further study. EARN HIGHER THAN AVERAGE COMPENSATION

Figure 3A.2

Salary survey by sector

116 Chapter 3A: Case Study 1, Construction Project Management

As can be seen from the AACE Salary Survey, project managers coming from within the ranks of construction project management clearly earn ‘above average’ salaries.

Figure 3A.3

Job satisfaction by salary

But salaries alone are not or should not be the only measure. An overwhelming 75% of construction project managers are ‘somewhat satisfied’ or ‘very satisfied’ with their jobs. While less than 7% is very dissatisfied. And those who are the most dissatisfied are making the most money. This trait or attribute is directly linked to the lifetime employment trait. One does get into construction project management by accident. When one had to come up through the ranks, it was clearly a career choice. And since switching over from the competency centric mode followed by the airlines and the 1940s, 1950s and 1960s model of CM, to the more knowledge centric model of the 1970s and beyond, favoring degrees over field experience, the trend of construction managers to remain with the profession for life remains very strong. THE MYSTIQUE OF CONSTRUCTION PROJECT MANAGEMENT

There is no research indicating construction project management could be considered mystical in any way. Nor is there anything indicating it is not mystical. BODY OF KNOWLEDGE IS ABSTRUSE, ESOTERIC OR SECRET

As noted previously, the body of knowledge for construction managers, even with the additional knowledge areas of Safety, health and the environment is hardly any different than for project management as defined by PRINCE2, the PMBOK Guide or AACE’s Total Cost Management Methodology or their 11R-88 Skills and Knowledge of a cost engineer. Thus the body of knowledge remains substantially the same.

117 Chapter 3A: Case Study 1, Construction Project Management

HIGHLY RITUALISTIC PROCEDURES

Construction project management is full of tradition involving ritualistic procedures, such as ground breaking ceremonies or topping off a building with an evergreen tree when the structure is complete. The Order of the Engineer ceremony previously described, or, if one is considering the trades, the swearing in ceremony and serving an apprenticeship, all stand as examples of ritualistic procedures which give a sense of belonging to something larger than oneself. This provides a link taken from the past which helps light a path for the future- assuring continuity. Until the 1970s and the beginning of affirmative action programs, membership if the trades was available to you if your father, uncle or other close relative was willing to sponsor you. People were not taken in off the street. Now, this has both good and bad sides, but it tended to build a lifetime of loyalty not to any company, but to the trade or profession. This trait is often still seen in firefighters or police, where you will find three generations working in the same sector. While undoubtedly some rituals are performed based on company culture or practices, project management has yet to develop widespread rituals which pertain uniquely to the practice of project management. ACCESS TO KNOWLEDGE IS LIMITED

As noted above, when the trades dominated CM, access to knowledge was limited to those who, for whatever reason, were able to get into the unions. However, since the move from experience-based construction managers to knowledge-based, access to the knowledge has been opened up to just about anyone who is interested. However, comparing the stagnation in the professionalization of CM relative to airline transport pilots indicates that the knowledge itself is of relatively little importance. The single most important aspect of being a construction project manager or any other kind of project manager is the demonstrated ability to safely and conservatively deliver your project, under varying degrees of difficulty and conditions. This focus, not on knowledge but on competency, is an important issue which is just beginning to come to the fore among project managers. CRUCIALITY

Immediacy of need. The immediacy of need for the services of a construction project manager is an intrinsic characteristic helping define the occupation; that is, the need is sited in the consumer and by demanding certain actions gives meaning to the project

118 Chapter 3A: Case Study 1, Construction Project Management

manager’s tasks. The immediacy of the need for the construction project manager to deliver the project on time may be important, but it does not compare with the immediacy of need for an attorney if one faces arrest or a doctor if there is a medical emergency. Comparing the sense of urgency felt by the customers of construction project management against the customers of project management in general, every client wants things done ‘fast and good and cheap’. So in this instance, one could speculate that here is an attribute that is more important to project managers. Importance of need. Along with the sense of urgency goes the actual importance of what needs to be done. The fact that most customers want their project yesterday, when the contract hasn’t even been signed, is an indication that at least in some general context, construction project management meets the tests for both urgency and importance as far as the consumer is concerned. And, although the nature of project management ‘emergencies’ is highly unlikely to ever match the types of emergencies facing medical or law professionals, customers clearly need and need as quickly as possible, management services in the construction industry. A common sense test of cruciality can be found in the 2004 Tsunami which affected Indonesia, Thailand and Sri Lanka. Or in the 2005 Katrina hurricane which ravaged New Orleans. These examples point out both the strengths of the intrinsic or semantic attributes of measuring a profession and the weaknesses. The need for construction project management was paramount, but a high standard of ethics guiding project management activities, it is very easy for unscrupulous practitioners to take advantage of people who, for whatever reason, are at a disadvantage. DÉNOUEMENT

Here is where the difference between the aviation profession and project management differs substantially. When your plane is delayed, you may grumble, curse the pilot and the airline, but even pre-9/11 wouldn’t even think of entering the cockpit and offering your advice, suggestions or ‘assistance’. Yet, with project management, the client or customer is continually sticking their noses into what you are doing, making changes and otherwise preventing you from doing what you know how to do. While proposed somewhat tongue in cheek, the fact remains that when you are using the

119 Chapter 3A: Case Study 1, Construction Project Management

services of a professional pilot, you trust his or her judgment and abide by those decisions, no matter how important it is you get to where you need to go or how urgent it is that you get there. Applying the Dénouement principle to the example of the Tsunami or Katrina relief efforts, the people did trust the government; the people did trust the US Corp of engineers. And while the emergency response to the Tsunami Crisis was generally perceived to have been done professionally, the response to Katrina has not been perceived to have gone well, thus violating the trust of the people in the government and social agencies responsible.

3A.2 Summary of case study 1: Construction project management Table 3A.3 Summary of CM attributes Color/Ranking Key

3

Clear Evidence

#

Attribute

1

Professional association- defines methodology/defines standards/enforces CoE

2

Partial Evidence

Construction project manager 3

1

Slight Evidence

0

No Evidence

Comments Many professional organizations purporting to represent CM practitioners

3

CM generally perceived as allowing extensive autonomy in making decisions.

3

CM generally perceived as an end ‘career path objective’. What I want to be when I grow up.

2

Autonomy in decision making

3

Lifetime commitment/calling

4

Earn higher than average compensation

3

5

Publishing in learned journals

3

Robust professional publications

6

Long period of training including higher education

2

Long period of training but higher education not generally required. Ref Garden v Frier

7

Subscribe to a code of ethics

2

Codes of ethics exist, but not overly robust or focused on fiduciary obligations.

8

Fiduciary obligation to public

9

Professional association defines ‘best practices’

Yes

2

Weak connections

2

Many professional organizations purporting to represent practitioners. But ‘best practices’ generally not defined. Usually only minimum standards.

10

Identify with occupation, not employer

2

11

Held in high esteem by the community

2

12

Apprenticeship/ internship/residency

13

Cruciality- need is important/need is immediate

2

14

Dénouement- practitioner is trusted to produce positive results relatively quickly.

2

2

Mixed Favorably perceived if not held in high esteem Nothing formal. All Certifications require experience, but not documented or validated. Circumstantially based Circumstantially based

120 Chapter 3A: Case Study 1, Construction Project Management

#

Attribute

Construction project manager

Comments

15

Service to the public- including pro bono work

1

Informal or Individual pro bono work, but not usually organizational

16

Practice limited by government license

1

CM Licensing only in Ukraine and South Africa. US voluntary certification only

17

Body of knowledgeunique/esoteric/secret

0

BoK is neither unique, esoteric, abstruse or secret

18

Advertising not permitted or restricted

0

19

Requires professional liability insurance

0

No restrictions Imposed Rarely

20

Symbolic costumes/uniforms

0

No symbolic costume/uniform

21

Use of title restricted by law

0

No

22

Mystique- highly ritualistic/access to knowledge is restricted

0

Total score

35

Total possible score

66

Rating (total/possible)

No

53%

As can be observed from applying the results on CM extracted from relevant literature reviews against the attributes or definitions of a profession we can see that in terms of gross numbers, CM, which we know is neither perceived as a profession nor qualifies as a profession under any ‘bright line’ rules, only scores 53% out of a possible 100%. As noted previously, the flaw in the analysis at this point remains that while it can be intuitively recognized that these attributes are not equally weighted, there is no way of establishing the relative rankings. In other words, having a symbolic costume should not be equal in weight to having a body of knowledge, but at this point in the research, relative rankings have yet to be established. The purpose of this research was that it would serve not only as an academic exercise, but offer assistance to occupations trying to improve their professional image, which could use this research as the basis for analyzing their strengths and weaknesses, with the objective of improving their professional ranking or score. A score of 53% indicates that construction project management barely meets the ‘preponderance of the evidence’ test (51%) much less approaches the standard of ‘beyond a reasonable doubt’ (~>90%). So, while it would appear that construction project management is a profession when the ‘preponderance of the evidence’ test is applied, many opportunities remain for improvement.

121 Chapter 3A: Case Study 1, Construction Project Management

3A.3 Recommendations to improve the standing For any professional group or organizations wishing to enhance the professional image of CM below are some considerations, based on this research: „

Long period of training including higher education Organizations offering credentials in construction project management should require a four year degree as a minimum requirement to qualify for any certifications, particularly those aimed at the top levels.

„

Subscribe to a code of ethics While codes of ethics exist for all the professional organizations, they seem to miss the point that a profession has a moral obligation to the consuming public. For an organization to increase its standing in the eyes of the consuming public, it must focus on delivering results which are measurable. Codes of conduct should be revisited and modeled more along the lines of the various ‘bills of rights’ published by hospitals and airlines.

„

Fiduciary obligation to public As with the recommendation regarding the code of ethics, the focus of the practitioners and those organizations which purport to represent them must be towards the obligations of practitioners to the consuming public. This is being legislated through laws such as Sarbanes-Oxley, but rather than wait for legislation, it would be preferable for those professional organizations representing practitioners to take the lead in this area. The American Society of Civil Engineers has set a good example through their ‘zero tolerance’ policy for corruption; and given that construction project management is generally recognized as being one of the more prominent sources of corruption (at least in the developing nations) the lead of ASCE in this are should be noted and emulated.

„

Professional association defines best practice Consistent with the code of ethics and fiduciary responsibility issues, practitioners and the organizations which deem to represent them should move beyond average practice and strive for best practices. Stated another way, instead of setting minimum standards of performance, those organizations wishing to professionalize the occupations they represent should be striving for something better than the minimum.

122 Chapter 3A: Case Study 1, Construction Project Management

„

Identify with occupation, not employer Assuming Tom Peters (Re-Imagine, 2003), Tom Osenton, (Death of demand, 2004), Tom Malone (Future of work, 2005), and Tom Friedman (Flat world, 2006) are close to being correct, within 10-15 years most of us will become part time, ‘free agent’ contractors, working on a ‘per project’ basis. If this is true, it will mean a whole different role for the professional organization to play. More than likely, the professional organization may well start to look more like the pilots union or the teachers unions, providing health and retirement services to their constituencies.

„

Held in high esteem by the community This is something which is earned and not demanded. No academic research could be found which specifically addressed the issue of high social esteem in the community of construction project managers, but from first hand knowledge, and applying Eliot Freidson’s (2001) common sense approach, it is average.

„

Apprenticeship/internship/residency While CM has historically relied on field experience as the basis for the credentials of its practioners, one way to increase the real or perceived professionalization of this manifestation of project management would be to take a lesson from commercial aircraft pilots or PADI scuba divers and require that a logbook of experience be compiled. This is consistent with the research of Lyle and Signe Spencer as outlined in their 1993 book, Competence at work Models for superior performance and Kenneth Cooper’s (2000) more recent Effective competency modeling and reporting, both of which indicate the importance using a range of assessment tools, including:

„

„

testing ⎯

attitudinal and behavioral



skills and knowledge

360° evaluations, done within a relatively short period after the execution of the services; effective for soft skills

„

individual assessments, conducted by skilled assessors

As with any assessment process, care needs to be taken to ensure the results are both valid and reliable.

123 Chapter 3A: Case Study 1, Construction Project Management

„

Cruciality: Need is important/need is immediate There is not much that can be done to enhance this attribute, for cruciality is defined by the needs of the person seeking the services. If your toilet is clogged on a Friday evening and you will be entertaining your boss and his wife for supper, the services of a plumber will most surely meet this criteria.

„

Dénouement: Practitioner is trusted to produce positive results relatively quickly Like cruciality, this trait or attribute is defined by the consumer of the professional services and not the provider. One of the reasons emergency rooms are so popular even for relatively trivial or non-life threatening events is, despite the anonymity of having to accept whatever doctor happens to be available, most emergency rooms produce results relatively quickly.

„

Service to the public, including pro bono work As should be appreciated, a profession (or a professional) is not merely another businessman or woman. The true professional honestly cares about other people and the tasks that comprise their work. This passion transcends money and reflects the importance of doing the right things as much if not more than doing things right. While all professional organizations give lip service to public service, the mark of the true professional lies more in individual behaviours.

„

Practice limited by government license Unless a credible case that the health, safety or welfare of the public can clearly be demonstrated and proven, licensing of construction project managers is unlikely to be accepted, at least not in the developed nations. The trend is clearly towards the adoption of voluntary credentialing, allowing the the marketplace to determine which credentials have value and which do not.

„

Body of knowledge, unique/esoteric/secret The body of knowledge relating to construction project management (and project management in general) could hardly be classified as unique, esoteric, secret or abstruse. Because the body of knowledge is a very important element in defining a profession (Haga, 1974; Abbot, 1988; Freidson, 2000) since it helps delineate the scope of the professional tasks and responsibilities, the lack of a clearly defined project management-specific body of knowledge makes it all the more difficult to claim the ‘turf’ which belongs to the occupation of project management. Assuming, therefore that the body of knowledge cannot be defined, then the only other way to ensure recognition of PM as a profession would be for project managers to establish jurisdiction over the actual tasks, or series of tasks which comprise what they do. This would translate into substituting the methodology or process for the body of knowledge. This has never been

124 Chapter 3A: Case Study 1, Construction Project Management

successful in the past, as there are many methodologies and new ones are evolving so quickly, that to found a profession on a methodology would be akin to building a house on a foundation of sand. „

Advertising not permitted or restricted As long as the majority of workers in construction project management fall under Zwerman’s (2004) ‘Set B’ category (employer/employee relationship), the question of advertising being restricted is largely irrelevant. However, assuming the predictions of Peters (2003), Osenton (2004), Malone (2005) and Friedman (2006) are close to correct and we find ourselves part time, contingency or contract workers, then advertising restrictions may well become an important issue in the coming years.

„

Requires professional liability insurance Liability insurance requirements make sense if we are all independent professional service providers, working out of our own offices as entrepreneurs. However, as most construction managers are employees, our employer is expected to insure against professional liability. Again, as in the advertising issue, this may very well change in the coming few years.

„

Symbolic costumes/uniforms This is a difficult attribute to come to terms with. Most people simply smile when discussing it, and tend to write it off as being an anachronism, a throwback to a bygone era. The fact remains, however, that powdered wigs, black robes, military style uniforms with ribbons and regalia, white jackets and ‘scrubs’ all become trademarks that enable someone to quickly tell the doctors from the nurses or the lawyers from the judges or the cockpit crew from the cabin crew. So while we may laugh at what we think to be the absurdity of it, the importance cannot be underestimated. About the closest construction managers come to a uniform is the wearing of white or gold hard hats, but even this is far from universal.

„

Use of title restricted by law This attribute in today’s market is being done through the registration of trademarks and branding. Realtors were perhaps the first organization to appreciate that while it may be difficult to get legislators to pass laws restricting the use of a title, another way existed by using copyright and trade marking. The most aggressive application of this principle in the field of project management is PMI and their trade marking of anything to do with the organization or the credentials produced by the organization. The question becomes whether the use of trademarks alone is sufficient to protect the name and image of any organization.

125 Chapter 3A: Case Study 1, Construction Project Management

„

Mystique: Highly ritualistic/access to knowledge is restricted As with cruciality and dénouement, the other two legs of the three legged stool of semantics, we know that perception is in the eyes of the recipient of the services. To my 83 year old mother, her computer consultant (a 20 something computer geek) has mystique. She neither knows nor cares what he is doing or how he is doing it, as long as within 15-20 minutes he has her computer fixed and she is back on line. In today’s world of Google, where access to knowledge is becoming less restricted, it may very well be possible for a lay person to be as expert if not more expert than the ‘professionals’ in any given field. The classic example of this is Lorenzo’s oil, a true story about Augusto, Michaela and Lorenzo Odones (Odones, n.d., online), who did their own research and experimentation to develop an oil to help their son, Lorenzo, recover from adrenoleukodystrophy (ALD) disease. As knowledge proliferates, it reduces the mystique of all but the most advanced and esoteric research.

3A.4 Summary In this chapter, we have explored construction management, which, despite being recognized as a career path objective, despite having degree programs at the Bachelor, Masters and PhD levels and despite filling a well defined niche, has yet to achieve or realize equal professional standing with the other professionals in the built environment. The architects and engineers, who work on these projects and who are often supervised by the construction manager. In the next chapter, we will explore another occupation- commercial aircraft piloting, which is an occupation which IS respected as a profession, despite the fact that to become a pilot does not even require a high school diploma. These extreme comparisons are expected to shed some light on project management in general.

126 Chapter 3A: Case Study 1, Construction Project Management

CHAPTER 3, PART B

Case study 2: Commercial airline piloting 3B.1 Commercial airline pilot 1: A project manager? Commercial airline pilots are an interesting case study of the development of a profession despite the fact it does not meet many of the traditional criteria or attributes of other occupations considered to be a profession. Commercial aircraft piloting is a global occupation that has evolved rather quickly over the past 75 years into being generally perceived as a profession, despite the fact that, at least under US case law, the courts have not held it to be one (Polelle, 1999). Unlike law or medicine, which have been around for hundreds of years, commercial air piloting is relatively new, thereby offering the chance to look at the evolution of a new profession and see if there are lessons from which project managers can learn or guide posts which they can use to help discover what stage of professional evolution project management has reached and what direction it needs to be going in. In order to make the best use of a comparison of airline piloting with project management, it is important to first establish the relationship between the two activities – if there is any. That is, what similarities does airline piloting share with project management? According to Webster’s third new international dictionary (Merriam-Webster, 2002), a project is: A proposed or planned undertaking to achieve a particular aim or objective, with some reasonable expectation for success, through the skillful handling or use of resources and the successful organization, administration and controlling these affairs in a business-like manner.

Considering this definition in terms of airline piloting, it can be said that each flight is certainly a ‘planned undertaking’. In accordance with FAA regulations, a flight plan must be filed for any flight carrying passengers for hire. Not only must the flight plan be filed, but it must also be closed out at the completion of the flight, lest it trigger an

1

For the purposes of this research the terms airline transport pilot and commercial airline pilot are synonymous and are what this researcher means when talking about a professional pilot. 127 Chapter 3B: Case Study 2, Commercial Airline Piloting

emergency search and rescue for an overdue or missing plane. And most clearly, each flight has a particular aim and certain objectives, not the least being to arrive safely and on time from the perspective of the customer as stakeholder, and to fly profitably, from the perspective of the investor stakeholders in the airline. Given the rather enviable safety record, air travel provides a very reasonable expectation that one will arrive safely, and, depending on the circumstances, on time. The ‘skilful handing or use of resources’ is one of the primary reasons commercial airline pilots have earned respect as professionals. Flying a plane is not easy (although with advanced electronics, it is certainly easier than it once was). Moreover, it is often the case that the plane’s owners are operating within tight profit margins, and face an ever increasing price for fuel. In this situation, the pilot is a key element in optimizing the consumption of fuel while getting the plane, passengers and crew to their destination safely and on time. So the pilot has full responsibility and control in optimizing the triple constraints of time, cost and quality trade-offs. In Wideman’s (2004) Comparative glossary of project management terms (version 4.1) a project is defined as: A novel undertaking or systematic process to create a new product or service the delivery of which signals completion. Projects involve risk and are typically constrained by limited resources.

In accordance with this definition, each flight undertaken is clearly novel, in the sense that, although each flight is part of a program of flights travelling on regular routes and at consistent times, each one is also a brand new undertaking every time a plane leaves the ground. Flying a plane from one point to another is without question, however, a systematic process. And delivering the passengers to the gate, closing out the flight plan and finishing the paperwork all signify completion of the project. Any attempt to defy the laws of gravity is inherently risky, and, given today’s highly competitive market, rising fuel and labor costs, and the security demands imposed post 911, pilots are having to contend with limited resources. As a final confirmation that commercial aircraft piloting incorporates project management as a key element of the delivery system, it is worth noting what Dr. Martin Barnes had to say at the 16th IPMA Conference, Berlin, Germany, June, 2002:

128 Chapter 3B: Case Study 2, Commercial Airline Piloting

Another definition of a project which I like and use often is that it is getting from State A to State B. State A is some aspect of the state we are in now. State B is that aspect changed. State B is always seen as better than State A by the people who initiate the project.

Well, why would you get on a plane in the first place if you didn’t feel your destination (State B) was somehow ‘better’ than where you were? (State A) It seems from these observations that airline transport piloting does incorporate project management. It is integral to the successful execution of each flight. What, then, allows the pilot to engage in the project? What requirements must he or she meet in order to fly? Basic eligibility requirements to become an airline transport pilot include: § 61.153 Eligibility requirements: General. To be eligible for an airline transport pilot certificate, a person must: (a) Be at least 23 years of age; (b) Be able to read, speak, write, and understand the English language. If the applicant is unable to meet one of these requirements due to medical reasons, then the Administrator may place such operating limitations on that applicant’s pilot certificate as are necessary for the safe operation of the aircraft; (c) Be of good moral character; (d) Meet at least one of the following requirements: (1) Hold at least a airline transport pilot certificate and an instrument rating; (2) Meet the military experience requirements under §61.73 of this part to qualify for a commercial pilot certificate, and an instrument rating if the person is a rated military pilot or former rated military pilot of an Armed Force of the United States; or (3) Hold either a foreign airline transport pilot or foreign airline transport pilot license and an instrument rating, without limitations, issued by a contracting State to the Convention on International Civil Aviation. (e) Meet the aeronautical experience requirements of this subpart that apply to the aircraft category and class rating sought before applying for the practical test; (f) Pass a knowledge test on the aeronautical knowledge areas of §61.155(c) of this part that apply to the aircraft category and class rating sought; (g) Pass the practical test on the areas of operation listed in §61.157(e) of this part that apply to the aircraft category and class rating sought; and (h) Comply with the sections of this part that apply to the aircraft category and class rating sought. (Doc. No. 25910, 62 FR 16298, Apr. 4, 1997; Amdt. 61–103, 62 FR 40905, July 30, 1997)

129 Chapter 3B: Case Study 2, Commercial Airline Piloting

There are, furthermore, an airline transport pilot certification and an airline transport pilot certification. An airline transport pilot may work for hire, but may not act as pilot for a commercial airline. Possible airline transport pilot careers include air tour and air taxi, pipeline patrol, traffic reporting, and more.i Having established the basic criteria to become an airline pilot, let’s look at the evolution of piloting as a profession against the 20 primary traits and 15 qualifying or sub-traits of a profession. Primary traits include: body of knowledge; long period of training; lifetime commitment; adhering to a code of ethics; fiduciary obligation to the public; other obligations to the public; professional associations; publishing in learned journals; advertising not permitted or restricted; use of the title is restricted by law; symbolic costumes or uniforms; practice limited by government license; require professional liability insurance; autonomy in decision making; identify with profession; not employer; held in high esteem by their home community; earn higher than average compensation; exhibit mystique; body of knowledge is abstruse, esoteric or secret; highly ritualistic procedures; access to knowledge is limited; cruciality: BODY OF KNOWLEDGE WHICH IS UNIQUE, ESOTERIC/COMPLICATED/SECRET

The body of knowledge associated with airline transport piloting is unique in that it began evolving from the military use of planes in World War I, and has gradually been codified by government bodies since that time. In the case of the USA, the body of knowledge is defined by the Federal Aviation Administration (FAA)2, and consists primarily of rules, regulations and policies established by the US Federal Government. Canadian, Australian and all other governments have similar rules, regulations and policies, administered through the Convention on International Civil Aviation. Anyone looking at the US Federal Aviation Regulations (FAR)3 would be hard pressed to describe them as being anything but complicated in ways that only governmental and legal jargon can create, especially when combined with the esoteric vocabulary associated with aircraft and flying. The complexity of so many arcane rules and regulations makes them understandable only by those who are trained and 2 3

http://www.faa.gov/regulations_policies/ http://ecfr.gpoaccess.gov/cgi/t/text/text-idx?&c=ecfr&tpl=/ecfrbrowse/Title14/14tab_02.tpl 130 Chapter 3B: Case Study 2, Commercial Airline Piloting

have been initiated into piloting. The body of knowledge pertaining to piloting is demonstrably esoteric, complicated and unique; and can be secret in some circumstances, for example military flying. LONG PERIOD OF TRAINING

There is no formal education requirement to commence flying lessons; however you must be able to read, write and speak English. The minimum flying time for the issue of a Private Pilots License (PPL) is 20 hours with a minimum of five hours solo. The average time taken to reach the required standard is about 25 hours. Note: the 20 hours required for the issue of a PPL is in addition to the 20 hours required for the General Flying Progress Test, ie a total of 40 hours. (Moorabin Flying Services, n.d., online) Total Time: 250 hours of flight time as a pilot that consists of at least: A. 100 hours in powered aircraft, of which 50 hours must be in airplanes. B.

100 hours of pilot-in-command flight time, that includes at least-

1. 50 hours in airplanes; and 2. 50 hours in cross-country flying of which at least 10 hours must be in airplanes. C.

Dual: 20 hours of flight training on the Airline transport pilot areas of operation that includes at least

1. 10 hours of instrument training of which at least 5 hours must be in a multiengine airplane; (See Note 4) 2. 10 hours of training in a complex multiengine airplane or turbine powered multiengine airplane; 3. 1 cross-country of 2 hours in a multiengine airplane in day VFR conditions of a total straight line distance of more than 100 nm. from the departure point; 4. 1 cross-country of 2 hours in a multiengine airplane in night VFR conditions of a total straight line distance of more than 100 nm. from the departure point; 5. 3 hours of flight training in a multiengine airplane within the preceding 60 days prior to the practical test. D. Solo or Performing PIC: 10 hours of solo flying or performing the duties as PIC with an instructor in a multiengine airplane on the Airline transport pilot areas of operation, that includes at least – (see Notes 3 and 7)

1. One cross-country flight of not less than 300 nm. with landings with a min of 3 points, one of which is a straight line distance of more than 250 nm.; and 2. 5 hours in night VFR conditions with 10 takeoffs and 10 landings at a controlled airport. (Government Printing Office, n.d., online)

131 Chapter 3B: Case Study 2, Commercial Airline Piloting

Generally, to obtain a Airline transport pilot’s license you need to have 250 hours of flight time. The specific requirements can be found in Section 14 of the Code of Federal Regulations, Part 61.121 - 61.133. However, don’t confuse a Airline transport pilot’s license with a airline transport pilot’s job. Right now the US major airlines are not hiring and only the regional airlines are. An example is American Eagle which, right now, requires 1500 total time and 300 multi-engine. ASA, ComAir and ACA are other examples of regional airlines that all have web pages with their requirements. (Phillip Greenspun, n.d., online)

The education requirements necessary to become a pilot are not stringent at all. However, the occupation requires an extensive internship, starting with a private pilot’s license (~40 hours) and moving up by ‘building hours’ to the point where a practitioner would qualify to get a job flying passengers for money. Assuming an average pilot works 250 to 300 hours per month, and is able to log 75-95 actual flying hours per month (D’Agostino, 2000, online), to log 1500 hours of flying time would normally take between 17 to 20 months. When compared with the medical, legal and engineering professions, this would hardly be considered ‘long’, but when compared against the ‘accidental’ project manager, it certainly would be. The focus with airline transport piloting is on demonstrated competency rather than academic achievement. This has considerable relevancy to project management, for like flying, you can read all the books and take all the knowledge based exams, but until you demonstrate you can manage projects consistently and well, everything else is irrelevant. LIFE TIME COMMITMENT

In terms of commitment, the process of building hours normally weeds out aspirants who don’t truly love flying. So unless the practitioner develops medical problems, pilots tend to remain practitioners throughout their working careers. The Federal Aviation Agency (USA) requires that airline transport pilots retire at age 60. Lobbying to raise that age to 65 is occurring. (USA Today, 2007, online). As the youngest age a person can qualify under FAA regulations is 23 years old, that provides a career potential of 37 years, barring health or eyesight limitations. The relevancy of this to project management lies in the fact that practitioners have no process of building hours (competency based program) to sort out those who are committed from those who are not. Project management is often called the accidental profession (Curling, n.d., online) because, profession or not, many who enter into project management do it by accident rather than choice. However, with the proliferation of degree programs in project management, and project management being recognized as a career path option in many companies, this may be changing. 132 Chapter 3B: Case Study 2, Commercial Airline Piloting

ADHERING TO A CODE OF ETHICS

There are several professional organizations which represent pilots, but the one most often associated with airline transport pilots is the Airline Pilots Association International, (ALPA)4 representing North American (US and Canadian) pilots, and the International Federation of Airline Pilots Associations (IFALPA)5. ALPA has a code of ethics6 which focuses on (in order of priority) the passenger, the employer, professional image, (two canons) and relationship with the professional organization representing professional pilots (Exhibit 3B.1 below) As is shown in Appendix B, the ALPA code of ethics and canons are as not quite as robust as the codes of ethics (CoE) associated with the representative bodies of the engineering professions (e.g. IEEE, ASCE, AACE). Notice in reading them, there is considerable focus on the implied duties owed to the ALPA by the members. Consistent with the philosophy of Haga (1974), this clearly reflects the fact that the ALPA has a duel function. It is not only as a professional organization, but is also a union, representing and negotiating for and on behalf of pilots. This role of the professional body will be explored in more detail under the heading of Professional Organizations, and will play a major role in the synthesis, but suffice to say, the code of ethics appears to be secondary to the rules, regulations and policies promulgated by the Federal aviation agencies responsible for air safety. The ALPA code of ethics seems very much to have been written as a union document focussed on prescribing the activities of the members rather than protecting the consuming public, whose primary responsibility is the FAA, and the airlines themselves due to insurance pressures.

http://www.alpa.org/ http://www.ifalpa.org/ 6 http://www.alpa.org/?tabid=270 4 5

133 Chapter 3B: Case Study 2, Commercial Airline Piloting

Exhibit 3B.1 The Airline Pilot An Air Line Pilot will keep uppermost in his mind that the safety, comfort, and well-being of the passengers who entrust their lives to him are his first and greatest responsibility. −

He will never permit external pressures or personal desires to influence his judgment, nor will he knowingly do anything that could jeopardize flight safety.



He will remember that an act of omission can be as hazardous as a deliberate act of commission, and he will not neglect any detail that contributes to the safety of his flight, or perform any operation in a negligent or careless manner.



Consistent with flight safety, he will at all times operate his aircraft in a manner that will contribute to the comfort, peace of mind, and well-being of his passengers, instilling in them trust in him and the airline he represents.



Once he has discharged his primary responsibility for the safety and comfort of his passengers, he will remember that they depend upon him to do all possible to deliver them to their destination at the scheduled time.



If disaster should strike, he will take whatever action he deems necessary to protect the lives of his passengers and crew.

An Air Line Pilot will faithfully discharge the duty he owes the airline that employs him and whose salary makes possible his way of life. −

He will do all within his powers to operate his aircraft efficiently and on schedule in a manner that will not cause damage or unnecessary maintenance.



He will respect the officers, directors, and supervisors of his airline, remembering that respect does not entail subservience.



He will faithfully obey all lawful directives given by his supervisors, but will insist and, if necessary, refuse to obey any directives that, in his considered judgment, are not lawful or will adversely affect flight safety. He will remember that in the final analysis the responsibility for safe completion of the flight rests upon his shoulders.



He will not knowingly falsify any log or record, nor will he condone such action by other crew members.



He will remember that a full month’s salary demands a full and fair month’s work. On his days off, he will not engage in any occupation or activity that will diminish his efficiency or bring discredit to his profession.



He will realize that he represents the airline to all who meet him and will at all times keep his personal appearance and conduct above reproach.



He will give his airline, its officers, directors, and supervisors the full loyalty that is their due, and will refrain from speaking ill of them. If he feels it necessary to reveal and correct conditions that are not conducive to safe operations and harmonious relations, he will direct his criticism to the proper authorities within ALPA.



He will hold his airline’s business secrets in confidence, and will take care that they are not improperly revealed.

An Air Line Pilot will accept the responsibilities as well as the rewards of command and will at all times so conduct himself both on duty and off as to instill and merit the confidence and respect of his crew, his fellow employees, and his associates within the profession. −

He will know and understand the duties of each member of his crew. If in command, he will be firm but fair, explicit yet tolerant of deviations that do not affect the safe and orderly completion of the flight. He will be efficient yet relaxed, so that the duties of the crew may be carried out in a harmonious manner.



If in command, he will expect efficient performance of each crew member’s duties, yet he will overlook small discrepancies and refrain from unnecessary and destructive criticism, so that the crew member will retain his self-respect and cooperative attitude. A frank discussion of minor matters of technique and performance after the flight will create goodwill and a desire to be helpful, whereas sharp criticism and peremptory orders at the moment will result only in the breakdown of morale and an inefficient, halting performance of future duties.

134 Chapter 3B: Case Study 2, Commercial Airline Piloting



An Air Line Pilot will remember that his is a profession heavily dependent on training during regular operations and, if in command, will afford his flight crew members every reasonable opportunity, consistent with safety and efficiency, to learn and practice. He will endeavor to instill in his crew a sense of pride and responsibility. In making reports on the work and conduct of his crew members, he will avoid personal prejudices, make his reports factual and his criticisms constructive so that actions taken as a result of his reports will improve the knowledge and skill of his crew members, rather than bring discredit, endanger their livelihood, and threaten their standing in the profession.



While in command, the Air Line Pilot will be mindful of the welfare of his crew. He will see to it that his crew are properly lodged and cared for, particularly during unusual operating conditions. When cancellations result in deadheading, he will ensure that proper arrangements are made for the transportation of his crew before he takes care of himself.

An Air Line Pilot will conduct his affairs with other members of the profession and with ALPA in such a manner as to bring credit to the profession and ALPA as well as to himself. −

He will not falsely or maliciously injure the professional reputation, prospects, or job security of another pilot, yet if he knows of professional incompetence or conduct detrimental to the profession or to ALPA, he will not shrink from revealing this to the proper authorities within ALPA, so that the weak member may be brought up to the standards demanded, or ALPA and the profession alike may be rid of one unworthy to share its rewards.



He will conduct his affairs with ALPA and its members in accordance with the rules laid down in the Constitution and By-Laws of ALPA and with the policies and interpretations promulgated there from. Whenever possible, he will attend all meetings of ALPA open to him and will take an active part in its activities and in meetings of other groups calculated to improve air safety and the standing of the profession.



An Air Line Pilot shall refrain from any action whereby, for his personal benefit or gain, he take advantage of the confidence reposed in him by his fellow members. If he is called upon to represent ALPA in any dispute, he will do so to the best of his ability, fairly and fearlessly, relying on the influence and power of ALPA to protect him.



He will regard himself as a debtor to his profession and ALPA, and will dedicate himself to their advancement. He will cooperate in the upholding of the profession by exchanging information and experience with his fellow pilots and by actively contributing to the work of professional groups and the technical press.

An Air Line Pilot the honor of his profession is dear, and he will remember that his own character and conduct reflect honor or dishonor upon the profession. −

He will be a good citizen of his country, state, and community, taking an active part in their affairs, especially those dealing with the improvement of aviation facilities and the enhancement of air safety.



He will conduct all his affairs in a manner that reflects credit on himself and his profession.



He will remember that to his neighbors, friends, and acquaintances he represents both the profession and ALPA, and that his actions represent to them the conduct and character of all members of the profession and ALPA.



He will realize that nothing more certainly fosters prejudices against and deprives the profession of its high public esteem and confidence than do breaches in the use of alcohol.



He will not publish articles, give interviews, or permit his name to be used in any manner likely to bring discredit to another pilot, the airline industry, the profession, or ALPA.



He will continue to keep abreast of aviation developments so that his skill and judgment, which heavily depend on such knowledge, may be of the highest order.

Having endeavored to his utmost to faithfully fulfill the obligations of the ALPA Code of Ethics and Canons for the Guidance of Air Line Pilots, a pilot may consider himself worthy to be called…an AIRLINE PILOT.

135 Chapter 3B: Case Study 2, Commercial Airline Piloting

During the period of this current research, PMI was in the process of rewriting its code of ethics, and it will be interesting to see where that organization’s focus lies (on controlling the member/owners of the organization or in protecting the best interests of the consuming public); and exactly what will be expected of project managers in terms of ethical behavior. Worth emphasizing, the American Society of Civil Engineers (ASCE) has adopted a zero tolerance policy on bribery and corruption (American Society of Civil Engineers, n.d., online). The promotion of the code of ethics and the sense of fiduciary obligation (see below) are tacit acknowledgement of the tensions inherent in the position of the airline pilot, who may have a legal contract with an employer, as well as one with passengers. Beyond legal obligations, a moral contract binds these individuals to one another. Employer and employee, in particular, have responsibilities to one another reminiscent of those recognised by agency theory (Muller & Turner). It is important that the pilot is able to communicate with and trust the employing airline in the same way that the airline must treat fairly with the pilot. Every flight is in a sense a ‘project’ requiring management of the highest order, not only on the part of the pilot, but on the part of the ultimate sponsors of the trip – the airline company’s owners – who must guarantee the ground support that ensures the safety of the project. FIDUCIARY OBLIGATION TO THE PUBLIC

To be clear, let us start by defining what a fiduciary relationship is. the relation by law existing between certain classes of persons (as confidential advisor and the one advised; executors or administrators and legatees or heirs; conservators and wards, trustees, or beneficiaries; partners, joint adventurers, corporate directors or officers and stockholders; majority and minority stockholders; factors, agents, or brokers and principals; attorneys and clients; promoters and stock subscribers; mutual savings banks or investment corporations and their depositors or investors; receivers, trustees in bankruptcy, or assignees in insolvency and creditors). (Merriam-Webster, 2002)

Recognizing they have a fiduciary responsibility to put the safety of their passengers above all else, airline pilots have placed their responsibility to the consuming public ahead of anything else. As retired airline pilot Captain Alex Paterson’s web says: In order to develop an insight into the Profession of Airline Pilot, it is important to realize that a professional pilot’s primary task is to fly the aircraft under his command from the departure airport to destination safely. This fact cannot be over emphasized. After this primary objective has been addressed, the myriad of other important considerations such as operating the aircraft economically, on time, smoothly, quickly, efficiently etc can then be tackled. But unless the aircraft is operated safely it ultimately cannot be any of the latter. The fact is, airlines that do not operate their aircraft as safely as possible eventually ‘lose’ aircraft and airlines that lose aircraft do not usually survive in the market place and as such are not viable. (Paterson, 1999, online) 136 Chapter 3B: Case Study 2, Commercial Airline Piloting

The importance of owing the first loyalty to the health, safety and welfare of the general public is common to most professions. The medical profession has the Hippocratic Oath which states a doctor ‘should do no harm’ (Wikipedia, n.d., online). The theme of doing no harm is common to the ASCE, AACE and IEEE codes of ethics as well. This aspect of professionalism is not emphasised in the field of project management. As of this writing, the PMI code of ethics does not address the issue of fiduciary responsibility by a practitioner. SERVICE TO THE PUBLIC

Being a pilot is not an occupation with any public service focus on the whole. Although pilots do participate, often bravely, in rescue missions to evacuate their countrymen and women from disaster sites overseas and fly for public services, such as Australia’s Flying Doctors, as well as defending their nations as part of national air forces, flying is not associated with altruism, and these are paid missions. This is in contrast to lawyers, doctors or teachers, whose occupations are perceived on the one hand to be lucrative, but on the other to contain an element, even the opportunity for, altruism and sacrifice for the public good. It must be acknowledged, however, that unlike doctors or lawyers who can carry his or her tools with him, the ‘tool’ of an airline pilot is a plane. So unless the pilot actually owns the plane, he or she is in no position to provide pro bono services. When pilots own their own planes, they do often offer their services for free in order to participate in search and rescue operations. The ALPA web indicates, however, that most of pilots’ volunteer work pertains only to the organization itself, and not to the general public. It is unclear whether it is their time that is offered or whether pilots also pay for their own fuel and maintenance as part of a volunteer effort. Volunteering a plane as a free service is not just income forgone; it may also involve actively spending their own funds. It is unlikely pilots could afford to do this even if they wanted to. The relevance this has to project management is that, as with ALPA, a good portion of -project managers’ pro bono work is done for and on behalf of the professional organization. To compare, top legal firms dedicate ~5% of their billable hours as being pro bono, while the response to question 29 pertaining to how much pro bono work project managers do is less than 2%.

137 Chapter 3B: Case Study 2, Commercial Airline Piloting

PROFESSIONAL ASSOCIATIONS

Professional associations are supposed serve four major functions: „

Create and enforce the code of ethics

„

Establish acceptable standards a) Performance standards b) Procedural standards (methodology)

Professional associations create and enforce the code of ethics. The largest professional association for airline pilots is the Airline Pilots Association International (ALPA), as discussed. It represents North American. While ALPA has a code of ethics, emails to the organization asking about how many ethical violations they dealt with were never returned. However, the primary enforcer of rules for pilots is not their professional organization, which functions as a union, but the various state and Federal aviation agencies or ministries. Pilots are held accountable for their failures in judgment and failure to meet standards imposed by these government agencies through the license renewal and suspension process; and it can be reasonably inferred that professional organizations representing commercial pilots are not expected to be responsible for pilot adherence to the rules established for pilots by government agencies. Relating this to project management, a poll of IPMA, PMI, AACE and AIPM regarding code of ethics activity over the past five years indicates that these professional organizations are generally not aggressive in enforcing codes of ethics or codes of conduct, especially for violations which impact the consumer of their services. Nor are they as proactive about ethical issues as are the engineering societies. In the case of airline pilots, the National Transportation Safety Board (NTSB) is responsible for investigating all accidents. Between the FAA and the NTSB, most issues pertaining to the behavior of pilots are addressed, not through any professional body, but through the administrative processes adopted by government agencies. One of the primary roles of the professional organizations is to defend pilots or help them defend themselves should they come into conflict with these agencies.

138 Chapter 3B: Case Study 2, Commercial Airline Piloting

The model of the professional organization as union along the lines of the aircraft pilots or teachers may well be something PMI, AIPM and IPMA might consider, given the nature of the project management industry. Like commercial airline pilots, most construction project managers are in the employ of a firm, be it an IT, telecommunications or building company. It is logical for their professional organizations to consider the option of acting as a union. Ironically, their members will be increasingly controlled by the very organization designed to represent them. This would represent a trade off between gaining some autonomy over their clients and employers, but at the price of giving up autonomy to the professional organization. Professional organizations establishing acceptable standards. Aviation is regulated by the Federal government, with the standards of practice and procedures determined by the relevant agencies. However, as the ALPA functions as a union as much as a professional organization, they do play an active role in advocating for and on behalf of the best interests of their members, especially providing commentary on proposed changes to any standards, policies or procedures. Evidence of this can be seen in their posting of issues for comment7. Another organization which represents both commercial and private pilots is the US based Aircraft Owners and Pilots Association (AOPA). This group is quite unlike ALPA, in that it is primarily a subscription (member based) organization which is not a union, but more of a lobbying organization. AOPA is unique in that it does not publish a code of ethics. Another interesting aspect of AOPA is that while it helps people get trained as pilots, it does not offer training to become a pilot8. PUBLISHING IN LEARNED JOURNALS

While articles are written about safety and airline pilots play active roles in their respective professional organizations, no evidence could be found for professional journals relating specifically to commercial aircraft piloting. Aerospace in general is associated with several professional journals, such as Aerospace Engineering, 7 8

See Air Line Pilots Association, Int’l at http://www.alpa.org/ See Aircraft Owners and Pilots Association online at http://www.aopa.org/advocacy/ 139 Chapter 3B: Case Study 2, Commercial Airline Piloting

published by the Society of Automotive Engineers9 or the Air & Space Power Journal published by the US Air Force’s College of Aerospace Doctrine, Research and Education (CADRE) at Maxwell Air Force Base, Alabama10. The majority of publications relating to airline piloting appear to be magazines, however, and are not peer reviewed journals11. Comparing airline pilots to project managers, it would appear that the two occupations are similar in this respect. There is no shortage of magazines, but only two journals are available to project managers: PMI’s Project Management Journal and IPMA’s International Project Management Journal. Although efforts are underway to upgrade both journals, neither is considered to be top tier (Association for Information Systems, n.d., online). ADVERTISING NOT PERMITTED OR RESTRICTED

As airline pilots are generally employees, and do not provide services for a fee, advertising is not applicable in most cases. However, there are similarities worth noting. Both in project management and in airline transport piloting, a healthy industry has evolved providing training and instruction in preparation for certification (in the case of project management) or licensing (in the case of pilots). The fact that both sectors require that the people who provide training be certified at or above the level of training they are providing, lends support to the idea that those providing training are a sub-set of a profession. Training for project management practitioners, however, is strikingly different to training for aircraft pilots. Neither ALPA nor AOPA provide training, nor do the Federal agencies that set the standards, procedures and policies. In each case, there is a clear distinction. The professional organizations do not get into the training business, and the creation and setting of standards is left to a totally separate and independent entity.

See http://www.sae.org/aeromag/index.htm 10 See http://www.airpower.maxwell.af.mil/apjinternational/aci/aci.html 11 A list of aviation journals and magazines can be found at http://www.aeroinfo.org.in/aviation/journals.html 9

140 Chapter 3B: Case Study 2, Commercial Airline Piloting

In reviewing the literature, and making comparisons between project management and aircraft piloting, in the publications for the airline sector there is no advertising suggesting that a trainee could ‘pass [his or her] FAA written exam in 3 days or [get their] money back’. Nor do you see books published with titles like Earn your pilot’s license for dummies. It appears that the organizations representing pilots are content to stay out of the training business and focus on representing the best interests of their constituencies, which in most cases, includes those members of the practicing community who provide training. USE OF TITLE IS RESTRICTED BY LAW

As airline transport piloting requires a rigorous licensing process, there is no need for the title pilot to be restricted by law. Normally, the title for a pilot relates more to rank, such as Captain or First Officer or Engineering Officer. Pilot is the job description. Captain would be the rank. Pilot-in-command is one special term in the airline industry, however, that does not get used outside the industry. This is not regulated by any legislation, however, but by the reality of flying. The entire process of piloting revolves around ‘building hours’. These hours are captured and documented in a log book. Pilots are allowed to log hours either as dual or as pilot-in-command (PiC) whenever flying the aircraft. However, the only hours which really count as those logged as pilot-in-command. So while a co-pilot will log his or her hours as dual, the only time she can log them as being a PiC is when the control of the aircraft has been formally handed over to her by the Captain. Project management has no equivalent system of significant nomenclature. The trend towards title restrictions is getting less, not more, and governments are less interested in getting into the licensing process unless some clear issue pertaining to the health, safety or welfare of the voting public can be identified. The case of allowing unlicensed pilots to be flying around stands as an excellent example in favor of licensing, as does the prospect of unlicensed civil engineers designing bridges or buildings.

141 Chapter 3B: Case Study 2, Commercial Airline Piloting

To make the argument that a failed project in the context of an IT or telecommunications activity endangers the health, safety or welfare of the general public requires a leap of faith. However, with Sarbanes Oxley the day may come when project managers are held accountable for the financial health, safety and welfare of their clients. This is an interesting development and one which those organizations representing project management practitioners may need to consider. In this context, it is worth noting that the Association for the Advancement of Cost Engineering International (AACEi)12 has a rigorous certification process, more technically demanding than the PMP, but still a knowledge based exam. Passing this exam results in the successful examinee earning the designation Certified Cost Engineer (CCE) if they are a graduate of an ABET accredited four year engineering curriculum. If they are not graduates of an ABET accredited engineering curriculum, even though they sat exactly the same exam, they are prohibited from using the CCE designation. Instead, they are awarded the Certified Cost Consultant (CCC) credential. This has caused considerable dispute over the years as people question whether the two separate designations are ‘equal’. At this writing, with the demise of restricting the use of the title, AACE is only adhering to this restriction in the USA. Outside the USA, the designation of Certified Cost Engineer is allowed regardless of the institution from which the cost engineer graduated. SYMBOLIC COSTUMES OR UNIFORMS

Perhaps reflecting the military roots of commercial aircraft piloting, the use of uniforms and assorted military style paraphernalia is the norm in the world of airline transport piloting. Hats with gold braid (scrambled eggs), uniforms with epaulets, stripes and bars, all go to indicate clearly who is who in the cockpit. This same philosophy is also evident in the cabin, with cabin crew members also having different uniforms indicating rank or authority. On almost any commercial flight, the in-flight magazine contains pictures indicating the various uniforms, what they mean and how to tell senior crew from junior. As the same is true in hospitals, not only with the doctors, but also in the ranks of the nursing staff, and even all the way down to the ‘candy stripers’, rank and authority are clearly communicated through uniform. 12

See www.aacei.org 142 Chapter 3B: Case Study 2, Commercial Airline Piloting

While the data from the current study indicated that the respondents felt that uniforms were the least important attribute of a profession, in the already established professions (law, medicine, commercial aircraft pilots) the importance of the uniform cannot be underestimated. PRACTICE LIMITED BY GOVERNMENT ISSUED LICENSE

For the aircraft industry, this is the single most important factor determining the status of professional. Few professions are more regulated than airline transport piloting. Of particular interest from a global context is the adoption of English as the only official language of piloting. Part of the licensing process includes a demonstrated proficiency in both written and spoken English, even for pilots coming from other countries. In the event you cannot read, speak, understand and write English, and it is determined it may impact your ability to pilot a plane safely, you will have restrictions placed upon your licenseii. Without a valid pilot’s license, and a recent medical check up and a logbook capturing your total hours as pilot-in-command, you cannot find work as an airline transport pilot. A valid pilot’s license is assigned according to the type of aircraft the individual has built up hours flying. The first pilot’s license, the Private Pilot’s License (PPL), is normally for flying a single engine plane, not pressurized, with a fixed pitch propeller and with fixed landing gear. The pilot is also limited to flying under Visual Flight Rules (VFR). Without an endorsement on his or her license for ‘complex’ aircraft (aircraft with retractable landing gear, variable pitch propellers); for instrument rating to fly in bad weather; for float planes; for multi engines or helicopters; a pilot is unable to fly another type of aircraft as pilot-in-command when the craft is of a more complex or a different type from that which he or she has been licensed to operate. [Hours can be built towards operating another type of aircraft by flying as a co-pilot under the watch and responsibility of a current pilot-in-command; by flying in flight simulators, and/or by taking formal lessons from a Certified Flight Instructor (CFI)]. While licensing is a very significant part of maintaining standards in the airline industry, the trend in project management lies with voluntary certification instead of licensing (Cox & Foster 1990; Finnochio, Dower, Blick & Gragnola, 1998). PMI’s

143 Chapter 3B: Case Study 2, Commercial Airline Piloting

movement toward Certificates of Advanced Qualification (CAQs), therefore, seem at to be a step in the right direction. AACE has followed suit and now has, in addition to the Certified Cost Engineer (CCE) designation, the Planning and Scheduling Professional (PSP) and Earned Value Professional (EVP) credential. The organization is currently in the process of coming out with a similar credential for cost estimating as well. The key difference between project management and the airline sector is the airline transport pilot requires demonstrated competency, as well as success on a knowledge based exam. So if the intent is to professionalize project management, the next step must be to move from a knowledge based to a demonstrated competency based credential. Lacking the ‘authority’ of a license, the credential must be set up in such a way that it results in better products or services, thereby earning the trust and respect of the consuming public. REQUIRES PROFESSIONAL LIABILITY INSURANCE

As pilots almost exclusively work for airlines, and almost never work as independent contractors on a fee for services basis, professional liability insurance (also known as errors and omissions insurance) is almost never required. This was confirmed by an email dated 20 September, 2006 from ALPA HQ stating that ‘as airline pilots are employees, they are not required to carry professional liability insurance’. Pilots usually carry insurance, however, to cover their own salaries in the event they lose their license for some reason, often only a minor infraction of the FAA rules or because of ill health. In the field of project management, clients can, and do, require architects, engineers, consultants and construction and construction project managers to carry professional liability insurance. The University of California’s online Business and Finance Bulletin BUS-63 seems typical for architects, engineers and other built environment professionals (University of California, 2006). At least for construction project management, liability insurance is required in some instances, and is commercially available (Professional Design Insurance Management Corporation, n.d., online). Given construction project management is one of the more mature users of project management as a delivery system, this can be interpreted to be a sign of the future.

144 Chapter 3B: Case Study 2, Commercial Airline Piloting

AUTONOMY IN DECISION MAKING

This is a major factor in the acceptance of airline piloting as a profession. The very essence of piloting revolves around the concept of the ‘pilot-in-command’ (PiC) To quote once again from Captain Alex Paterson: factors are crucial to a safe airline operation because the pilot-in-command of an aircraft is in a unique position. Not only is he the only person aware of all the factors and operational constraints pertaining to his particular flight, but ultimately he is the only person on location qualified to deal in a safe manner with the myriad of problems that invariably arise throughout the course of a flight. To summarize, an aircraft captain is the only person capable of managing his particular flight and as such his primary role in an airline is as the manager of his particular flight. (Paterson, 1999, online) To quote an address to pilots from the former President of the Australian Federation of Air Pilots, Captain Dick Holt (now retired), who said of the position of airline pilot: Through his seat at the front of the aircraft flow the efforts of thousands of people who provide the means by which he carries out his task. However, it is an undeniable fact that: His is the final responsibility. His is the ultimate decision in any course of action. He can never be complacent. He must be humble; the elements keep him so. He must prove himself to his peers over and over again throughout his career, or seek another job. He must exude a quiet but magnetic confidence in his own ability and his aircraft. He must create an aura of efficiency and capability such that the passengers stream on and off the aircraft without even a thought about what is occurring at the front of the aircraft. Finally, he must be ready during every second of his working life to defeat the ultimate emergency he may encounter at any time. These responsibilities are recognized in the definition of his title of ‘Captain’, which means ‘in command’ and as such legally the final responsibility for the safety of the aircraft rests solely with the pilot-in-command. It is these professional responsibilities that not only make pilots ‘Sui Generis’, but also worth every cent of their pay and conditions. (Holt, in Paterson, 1999, online) In a sense, pilots are the stewards of an airline’s three most valuable assets; it’s passengers, it’s aircraft (worth up to $200+ million each) and the public’s confidence in the corporate identity of the airline. As such, unpalatable though it might be to some airline managers and civic leaders who think ‘pilots are just glorified bus drivers’, the most important people in any successful airline are its pilots, for on their backs rides the very survival of the airline. (Paterson, 1999, online)

145 Chapter 3B: Case Study 2, Commercial Airline Piloting

The principles outlined here have some profound implications if project management is ever to be recognized as a profession. Compare the criteria Dick Holt outlined for airline pilots and apply those to construction project managers. Do PMs have the ‘final responsibility’ for projects? Are they the ‘ultimate decision makers’? Assume that Gartner, Standish Group, META and other researchers are correct in stating that 20%-60% of projects fail. Would the public be willing to get on an airplane knowing that the pilot had between a 20% to 60% chance of being late, that they would be charged more for their tickets than they had been advised, or that some major component in the aircraft will fail to perform as intended, or in the worst case, the craft will crash? Based on these criteria alone, project management is very unlikely to ever reach the same acceptance as a profession as that enjoyed by professional airline pilots. The missing element is the level of accountability and not just the responsibility, but the authority for executing their projects that airline pilots have. IDENTITY NOT WITH EMPLOYER BUT PROFESSION

Despite most airline pilots being employees of an airline rather than independent contractors, the fact most of them earned their pilots license on their own before being able to apply for a job makes it unlikely they would be inclined to identify with the airline first. The rather tense relations between the unions and airlines over contract issues lends further support to the argument that the pilot’s first loyalty is likely to lie not with the employers, but with their occupation first and then to the professional organization (union) which represents them. Few people fly unless they truly love it (D’Agostino, 2000). D’Agostino (2000) compares the working conditions of younger pilots working for newer commuter airlines against those working for national or international carriers. Her findings offer a picture of what the future may well hold when professional services become outsourced, and more individuals begin working on a fee for service basis. In her article, D’Agostino (2000) portrays a pilot who is paid on a unit price basis (~$46/hour of flight time) which requires two hours of unpaid prep for each hour of flight time, which works out to be about $15.00 per hour on average.

146 Chapter 3B: Case Study 2, Commercial Airline Piloting

In terms of project management, PMI membership profiles indicate that there are three categories of people who are likely to belong to professional organizations representing project management practitioners. There are the private practitioners – primarily trainers and consultants – who work on a fee for service basis. These practitioners are more likely to perceive their professional organizations as being much along the lines of the Bar or Medical Associations. The second category of members consists of employees of large companies. These are typified by airline pilots, nurses, school teachers and even some physicians, if they are working for an HMO on the basis of a salary and not fee for service agreement. This category of person is probably seeking an organization much like the ALPA model, where the professional organization functions not only to meet the professional needs of the practitioner but also t as a collective bargaining agent. Probably the two most active models are the teachers unions13, and the pilots union, ALPA, already considered. The third category of practitioners identified by Zwerman et al. (2004, p. 24) is the knowledge worker. This category of worker is not as well defined, but assuming Tom Osenton (2005) is correct that global demand is leveling off, and that as the developing nations increase their skill levels, they are quickly moving up the supply chain from agriculture, to low skilled manufacturing, to high skilled manufacturing to knowledge work, it would not be surprising to see more outsourcing. Tom Peters (2003), in his book, Re-imagine, is painting a picture of a world in which most of us will be temporary workers. This theme is reiterated by Friedman (2006) in Flat world and Osenton (2005) in Death of demand. Assuming that model to be correct, then the group of workers identified by Zwerman et al. (2000) as private practitioners is likely to increase, while the number of individuals permanently employed for wages decreases and knowledge workers increasingly move to the private practitioner sector, selling their expertise on contract. These individuals would be much more likely to associate themselves with their professions than with their employers. 13

See http://www.nea.org/index.html and http://www.uft.org/ for examples of teachers’ organizations 147 Chapter 3B: Case Study 2, Commercial Airline Piloting

HELD IN HIGH ESTEEM BY THE COMMUNITY

Despite technological advances which make it very nearly possible for planes to take off and land by themselves (Klesius, 2003, online), most people feel more comfortable knowing a human being is in charge, if doing nothing more than watching the computers to see whether they crashed, and not the plane. Moreover, because how a plane flies and the skills and knowledge to fly it, although not airplane travel itself, still seem so miraculous and mysterious, pilots continue to be highly regarded in their communities. While they may well eventually end up as nothing more than highly paid drivers of exceptional technology, at the moment, commercial airline pilots pursue a prestigious occupation, despite conflicts between their unions and the major international and national airlines and regional carriers. It is unclear, on the other hand, what level of status should be accorded to project management. As can be seen from the survey results, project managers have mixed emotions about the real or perceived position of project management as an occupation. With an average score of 3.52 out of a possible score of 5, most practitioners perceive project management as a strong moderate. Personal experience in the construction industry indicates that to hold the title of manager does carry with it an aura of prestige and respect. Normally, a person does not become a construction project manager on a project of any size or significance until he or she is in the latter part of their career; and their experience appears to be highly regarded in the industry and the wider community. Whether this equates to the construction project manager being a professional is an interesting question and an aspect of this current research. EARN HIGHER THAN AVERAGE COMPENSATION

For airline pilots, especially those who work for the major airlines (NW, Delta, and Continental), the pay is clearly on the high end of the scale. Salaries for Captains of $100,000+ are possible. But many airlines in financial difficulty are operating on a dual pay scale system, with those pilots hired during the 1970s and 1980s earning the high salaries, while those hired in the 1990s and later on a much lower scale. Given the Census Bureau released income based on the CPS for calendar year 2001 on September 24, 2002. Median household income for all households is $42,228; iii

148 Chapter 3B: Case Study 2, Commercial Airline Piloting

MYSTIQUE (WHAT YOU DO FOR WORK IS BEYOND MY COMPREHENSION OR ABILITIES TO UNDERSTAND)

Body of knowledge is esoteric/abstruse/secret. The body of knowledge associated with flying is an intrinsic attribute that is likely to rank very high on the scale as to why piloting a commercial aircraft is considered a profession. As noted previously, due to the fact that commercial aviation evolved from the military, along with the fact that the rules, regulations and procedures are formulated, written and maintained by governments, the body of knowledge concerning flying is esoteric and abstruse and secret (at least in the context of military flight). Highly ritualistic procedures. Due in part to the military influence on aviation, the entire process of flying is extremely ritualistic. Starting with the mandatory visual inspection of the plane by the pilot-in-command prior to departure, to the use of extensive checklists for virtually every cockpit procedure, to the elaborate and necessary procedures to depart from and enter into a landing pattern at each airport, flying a plane is shrouded in ritual. Not all the rituals pertain to operating the plane. Some, like the practice of cutting off the shirt-tail of a student pilot upon successful completion of his or her first solo flight is a ritual of initiation into a select group. It helps forge the bond that is characteristic of pilots. When these sorts of rituals lead to the abuse of newcomers, perpetrators must be punished14. Yet, the existence of any ritual reinforces the air of secrecy and selectivity of the occupation. While undoubtedly some rituals are performed based on company culture or practices, project management has yet to develop widespread rituals which pertain uniquely to the practice of project management. Access to knowledge is limited. Unlike medicine and law, where access to knowledge has been, until recently, guarded and protected to make it as difficult as possible for the uninitiated to be able to research and learn on their own, access to the knowledge required to become a licensed airline transport professional is available to 14

For an account of the abusiveness of some initiation rituals, see http://usmilitary.about.com/od/womeninthemilitary/Women_in_the_United_States _Military.htm, which considers the question of sexual harassment and unethical abuse, particularly in the ‘tail hook’ incident of 1991. 149 Chapter 3B: Case Study 2, Commercial Airline Piloting

anyone with the money to invest in pilot training. Ready access to this information was one of the enabling factors in the events leading up to 9/11. From the perspective of project management, the relevance of this is the fact that knowledge about flying is in itself of almost no importance. This is one reason why there is no requirement to have even a two year college degree in order to become a pilot. The single most important aspect of being a commercial airline pilot is the demonstrated ability to safely and conservatively fly a plane under varying degrees of difficulty and conditions. This focus not on knowledge but on competency is an important point when considering the professionalization of project management. CRUCIALITY (WHAT YOU DO FOR WORK IS SOMETHING I NEED NOW, NOT LATER)

Immediacy and importance of need. On the whole, departure, landing and flying resemble most closely catching and riding a bus or train. There is rarely any sense of emergency, even though there is considerable aggravation and anxiety at the perception of time wasted when planes fail to take off or land on schedule. Delays, however, are not usually the fault of the professional pilot. Once the plane is in motion, an experienced professional at the controls is of paramount importance, however. his or her presence is absolutely crucial to your safe and successful travel; and, if you are taxiing down a runway, lifting off and landing, you will need a pilot’s expertise with an almost unparalleled immediacy. While most people who can drive a car can control a bus, and rails, on the whole, control the train, the cockpit of an airplane remains as much the mysterious domain of the professional pilot as the operating room of a surgeon remains his or her secret realm15. There is no apparent life and death urgency to be found in project management. In fact, there is little life and death urgency to be found in most professions. This is not to say, however, that there isn’t an immediacy and an unrelenting pressure on project managers to complete the job in good time, with good quality and at a good cost. And, since project managers ostensibly have under their control the means by which 15

Note that this is not to compare the pilot’s task to that of the surgeon. Their training and knowledge are vastly different. However, the context within which each operates is highly technologically specialized and absolutely foreign to the ordinary citizen, in spite of medical dramas on television and films like Airport and Top Gun. 150 Chapter 3B: Case Study 2, Commercial Airline Piloting

to achieve these outcomes, unlike pilots and surgeons whose tasks are obviously influenced by many circumstances beyond their control, the man or the woman managing a project carries a heavy burden in terms of its failure or success. DENOUEMENT (I TRUST THAT YOU CAN APPLY WHAT YOU KNOW TO SOLVE MY PROBLEM QUICKLY)

Application of ‘mysterious principles’ to effect quick solutions to problem. The ordinary citizen would not, on the whole, contemplate entering a cockpit to advise a pilot how to do his job. Firstly, the ordinary person knows so little about how to fly a plane that no meaningful advice for the pilot would occur to them. Slow down, speed up, turn right or turn left, and put the wheels down or the brakes on, don’t really work as commands unless you know how to use the controls and understand their idiosyncrasies, possibilities and limitations. In terms of project management, however, customers freely offer advice, make changes and otherwise impede or prevent PMs from doing what they are confident they know how to do, in spite of the fact that the client rarely appreciates the resource constraints, personnel issues or technical issues facing the construction project manager. Whereas passengers trust their pilot’s judgment and abide by his or her decisions (there is no other course of action, given the circumstances, of course), construction project managers face a plethora of advice and pressure that impinge on their activities. This is exactly what Haga (1974) meant when he commented about professionals that: Above all else, they want AUTONOMY. That is, they want freedom from supervision in carrying out their jobs; They want RECOGNITION based not upon the name of their employer, but upon their identity with their occupation; They want the POWER to determine who is ‘in’ their occupation from those who are ‘out’; They want to establish a MONOPOLY over a certain line of work, freeing it from influence of ‘outsiders’ (mostly employers, but also clients and the general public) who do not share or necessarily understand the ideology; They want the POWER to discipline wayward colleagues who deviate from their work ideology.

151 Chapter 3B: Case Study 2, Commercial Airline Piloting

From the perspective of commercial airline piloting, all five of Haga’s (1974) criteria have been fully met. Pilots have autonomy (pilot-in-command). They have recognition (it is clear what a pilot does and that it is important). They have the power to determine who is one of them (licensing). They have a monopoly over their work (no license, no job). They have the means to discipline wayward colleagues (FAA and other regulations). See Exhibit 3B.2 with regard to the role, the autonomy and the accountability of the pilot. Exhibit 3B.2 The Aloha Airlines incident, 1988 To drive home the point of the near total autonomy and accountability consistent with a profession, consider a pilot in the event his/her plane crashes from mechanical failure or some cause other than ‘pilot error’. Despite doing his/her best to land the plane safely, people do get killed or injured. What is the image of the Aloha Airlines pilot in the true incident below? 1988, The Aloha Incident The structural failure on April 28, 1988 of a 19 year old Boeing 737, operated by Aloha Airlines, was a defining event in creating awareness of aging aircraft in both the public domain and in the aviation community. This aircraft lost a major portion of the upper fuselage in full flight at 24,000 feet, near the front of the plane. Miraculously, the pilot managed to land the plane on the island of Maui, Hawaii. One flight attendant was swept to her death.

Is the pilot who safely landed this plane with the loss of only one life, the kind of project manager you’d wish to have on your projects? Was this individual in command of his/her resources? Is this the kind of project manager to make you proud to be one?

As with construction management CPM, the results of mapping the literature review against the attributes have been tabulated, and notes have been included to try to explain how the scoring model was derived. This is somewhat subjective, but a more rigorous investigation of this aspect has been included in follow on research.

152 Chapter 3B: Case Study 2, Commercial Airline Piloting

Table 3B.1 Summary of commercial airline pilot ranking mapped against attributes of a profession Color/Ranking Key #

1

2 3 4 5 6 7 8 9 10 11 12 13 14

15

16

17 18 19 20 21 22

3

Clear Evidence

attribute name Professional associationdefines methodology/defines standards/enforces CoE Autonomy in decision making Lifetime commitment/calling Earn higher than average compensation Fiduciary obligation to public Professional association defines ‘best practices’ Identify with occupation, not employer Held in high esteem by the community Apprenticeship/ internship/residency Practice limited by government license Body of knowledgeunique/esoteric/secret Requires professional liability insurance Symbolic costumes/uniforms Mystique highly ritualistic/access to knowledge is restricted Cruciality Need is important/need is immediate Dénouement Practitioner is trusted to produce positive results relatively quickly. Long period of training including higher education Subscribe to a code of ethics Service to the publicincluding pro bono work Publishing in learned journals Advertising not permitted or restricted Use of title restricted by law Total score Total possible score Rating (total/possible)

2

commercial airline pilot

Partial Evidence

1

Slight Evidence

0

No Evidence

comments

3

Professional organization functions as a union, representing the pilots best interests

3

‘Pilot-in-command’ concept

3

Commercial pilots generally perceived as an ‘career path objective’. What I want to be when I grow up.

3

Yes

3 3 3

Very strong obligation to protect the public safety and other interests Pilots’ professional organizations function as unions, advocating for and on behalf of the interests of pilots in partnership with the FAA and airline representation. Almost all become qualified pilots before they can even obtain a job with any airline.

3

Very prestigious and high esteem job

3

Very extensive ‘internship’ which requires they not only ‘build hours’ but pass several written and ‘in-flight’ exams as they progress from private pilot to commercial pilot.

3

Cannot legally fly any plane without a license.

3 3

BoK is unique, esoteric and abstruse and in the case of military, secret. Whether individually or as an organization, aircraft piloting requires extensive liability insurance

3

Near military use of uniforms, rank and insignia

3

Very mystical occupation. Rituals abound (i.e. pre flight check)

2

Circumstantially based

2

Circumstantially based

1

Long period of training but higher education not generally required. does require demonstrated competency

1

Code of ethics is replaced by extensive FAA regulations.

1

Informal or individual pro bono work, but not usually organizational

0

Rarely

0

No restrictions imposed

0

No

49 66 74%

153 Chapter 3B: Case Study 2, Commercial Airline Piloting

A score of 74% indicates that aircraft piloting certainly goes well beyond meeting the ‘preponderance of the evidence’ test (51%) but has yet to meet the standard of ‘clear and compelling evidence’ (~80%). So while it would appear that commercial aircraft piloting is a profession in most senses of the definition, there remain some question marks. Of particular concern are the three legal actions in which aircraft piloting was not deemed to be a profession, at least in the context of tort actions [Southern Helicopter v Jones, 379 S.W.2d 10 (AR, 1964); T-Craft Aero Club v Blough, 642 P.2d 70 (ID 1982); Mackey v Miller, 273 S.E.2d 550 (VA, 1981)]. Assuming that the objective was to raise the professional image of commercial airline pilots, those attributes scoring at the bottom of the list would have to be considered, not the least of which would be to require at minimum, a 4 year degree to become a commercial pilot.

3B.2 Comparison and preliminary evaluation: Commercial aircraft pilot 3B.2.1

Cruciality: Need is important/need is immediate

There is not much a pilot or the organization which represent them can do about this, as need is driven by the consumer. However, aircraft of any sort can’t fly without pilots. They are needed and if the plane is ready to fly, they are needed immediately. 3B.2.2

Dénouement: Practitioner is trusted to produce positive results relatively quickly

A funny story is circulating about piloting: Given the increasing automation of aircraft, using sophisticated GPS and other technical equipment, the plane of the future will be piloted by a human and a dog. The human is there to monitor the systems to ensure they are working properly and the dog is there to bite the human if she attempts to touch anything.

The moral of this story in the context of this research is that with increasing automation, the human factor, while never going away, will be diminished. This is happening in the field of engineering and in medicine, as computers play an increasingly prominent role in doing work once capable of being performed only by humans. As ‘mechanization’ (enabled by computer technology) increases, the likelihood of ‘deprofessionalization’ in fields such as engineering and medicine may well become more prevalent.

154 Chapter 3B: Case Study 2, Commercial Airline Piloting

3B.2.3

Long period of training including higher education

If commercial airline pilots want to increase their level of professionalism, requiring a four year degree is one of the most logical places to start. Practically speaking, most commercial airlines now require pilots to have a four year degree and degrees are beginning to be offered by a wide variety of universities in aviation related fields. Embry-Riddle Aeronautic University in Florida, USA16, specializes in anything to do with aviation or space, for example. 3B.2.4

Subscribe to a code of ethics

While the professional organizations representing pilots do have codes of ethics, they are largely symbolic, as FAA regulations largely control many of issues normally addressed by a code of ethics. As the professional organizations representing pilots also functions as their union, one would suspect that the true purpose of codes of ethics in this instance would be to control the actions of the members internally rather than externally. This is a major problem with any codes of ethics: Is the intent of the code truly to protect the interests of the consuming public or is it being used to control outspoken or heretical practitioners? Haga (1974) is particularly straightforward (almost brutal) is his statement that: The chief means of maintaining autonomy is, to put it bluntly, intimidation. The essence of truly professional behavior is intimidation – of clients, of employers, or whatever audience stands to threaten a profession’s autonomy. (p. 6 )

3B.2.5

Service to the public, including pro bono work

While pilots undoubtedly play an active role in the community, to increase the perception of the occupation as a profession would require that those organizations representing them make a more concerted effort to build the brand image through public relations efforts focused around what pilots do in terms of volunteer efforts. 3B.2.6

Publishing in learned journals

While pilots and the organizations which represent them play an active role in advising FAA and advocating for and on behalf of regulation changes favorable to the pilots, there are no reputable academic journals specific to the occupation of aircraft piloting. However, with the growth of Bachelor of Science Degrees in Aeronautical

16

See http://www.erau.edu/ for more information about Embry-Riddle Aeronautic University. 155 Chapter 3B: Case Study 2, Commercial Airline Piloting

Science with majors in Airline and Commercial piloting (Embry Riddle, 2005) it will only be a matter of time before academic journals proliferate. 3B.2.7

Advertising not permitted or restricted

Given that most pilots are employees, the question of aggressive advertising is probably moot. 3B.2.8

Use of title restricted by law

As piloting requires a license and a minimum number of hours flying time in order to even get a job, there is little or no sense in restricting the use of the title. Another consideration is there are many people who are not commercial pilots, who fly only as a hobby or for private business purposes. To conclude, this research is being undertaken not only as an academic exercise, but also to serve a useful purpose. One of the purposes would be for those occupations trying to improve their professional image to use this research as the basis for analyzing their strengths and weaknesses, with the objective of improving their professional ranking or score.

3B.3 Comparison and preliminary evaluation: Pilot and construction project manager comparison summarized Having completed the evaluation of construction project management and commercial airline piloting, let me summarize them together as a table, and then compare them (Table 3B.2). This comparison provides the insight into why commercial aircraft piloting is considered to be a profession, while construction management is not. Simply observing the number of green cells in aircraft piloting (14) vs the number of green cells in construction management (5) serves as an early indication that aircraft piloting meets more criteria than does construction project management. Irrespective of the weighting of each specific attribute, commercial aircraft pilots score a 74% in terms of the number of attributes the occupation exhibits. Consistent with modern or post-positivist thinking, pilots have done a better job of ‘defining’ their occupation in the context of the ‘big picture’ (holistic perspective). Whether this

156 Chapter 3B: Case Study 2, Commercial Airline Piloting

was intentional or not is irrelevant to this study at this point, but is something to consider for those who believe that project management is a profession and hope to make changes to enhance that perception. Table 3B.2 Ranking of CM and commercial aircraft piloting compared Color/Ranking Key

3

Clear Evidence

#

Attribute

Construction project manager

1

Professional association- defines methodology/defines standards/enforces CoE

3

2

Autonomy in decision making

3

Lifetime commitment/calling

4

Earn higher than average compensation

5

Publishing in learned journals

6

Long period of training including higher education

7

Partial Evidence

2

1

Slight Evidence

0

No Evidence

Attribute

Commercial airline pilot

1

Professional association- defines methodology/defines standards/enforces CoE

3

3

2

Autonomy in decision making

3

3

3

Lifetime commitment/calling

3

3

4

Earn higher than average compensation

3

3

5

Fiduciary obligation to public

3

2

6

Professional association defines ‘best practices’

3

Subscribe to a code of ethics

2

7

Identify with occupation, not employer

3

8

Fiduciary obligation to public

2

8

Held in high esteem by the community

3

9

Professional association defines ‘best practices’

2

9

Apprenticeship/ internship/residency

3

10

Identify with occupation, not employer

2

10

Practice limited by government license

3

11

Held in high esteem by the community

2

11

Body of knowledgeunique/esoteric/secret

3

12

Apprenticeship/ internship/residency

2

12

Requires professional liability insurance

3

13

Cruciality- need is important/need is immediate

2

13

Symbolic costumes/uniforms

3

14

Dénouement- practitioner is trusted to produce positive results relatively quickly.

2

14

Mystique- highly ritualistic/access to knowledge is restricted

3

15

Service to the public- including pro bono work

1

15

Cruciality- need is important/need is immediate

2

16

Practice limited by government license

1

16

Dénouement- practitioner is trusted to produce positive results relatively quickly.

2

17

Body of knowledgeunique/esoteric/secret

0

17

Long period of training including higher education

1

18

Advertising not permitted or restricted

0

18

Subscribe to a code of ethics

1

19

Requires professional liability insurance

0

19

Service to the public- including pro bono work

1

20

Symbolic costumes/uniforms

0

20

Publishing in learned journals

0 0 0

21

Use of title restricted by law

0

21

Advertising not permitted or restricted

22

Mystique- highly ritualistic/access to knowledge is restricted

0

22

Use of title restricted by law

Total score

35

Total score

49

Total possible score

66

Total possible score

66

Rating (total/possible)

53%

Rating (total/possible)

74%

157 Chapter 3B: Case Study 2, Commercial Airline Piloting

Looking at what the two case studies have in common, we find that the following are top ranked in both occupations: „

professional association that sets standards and enforces the code of conduct

„

autonomy in decision making

„

lifetime commitment

„

higher than average salaries are common to both.

Given construction project management is not considered to be a profession while commercial aircraft piloting is, provides at least a preliminary indication those items in common can probably be eliminated. Next I looked at those attributes pilots scored 3 in but CMs did not as being likely differentiators. These are: „

fiduciary responsibility to the public

„

professional association which defines best practices

„

identify with the occupation, not the employer

„

held in high esteem by community

„

serve a formal apprenticeship/internship/residency

„

requires a license to practice

„

has a body of knowledge which is complicated, secret, abstruse, esoteric or unique

„

requires liability insurance to practice

„

uses symbolic costumes or uniforms

„

requires highly ritualistic processes.

If we are willing to accept the modern or post positivist thinking, the differentiator lies with one or more of these attributes. However, if the post modern or post positivist thinking more closely represents truth, then it will not be any one, but the entirety of them all being more than just the sum of the parts. Furthermore, not all of these attributes are reasons for an occupation being considered a profession. Many of them, especially the extrinsic ones, are most likely the effects of the occupation being a profession.

158 Chapter 3B: Case Study 2, Commercial Airline Piloting

3B.3.1

Fiduciary responsibility to the public

While this is what Polelle (1998) suggests as one of the ‘bright line’ tests of a profession, the fiduciary responsibility of both pilots and construction managers is defined more by the FAA regulations in the case of pilots and to a lesser degree by OSHA, the Environmental Protection Agency (EPA) or relevant building codes in the case of construction managers. Additionally, as both pilots and construction managers tend to be employees rather than fee-for-service providers, fiduciary responsibility is more than likely not a key differentiator (Polelle, 1998). It is interesting to note that in the architects’ code of ethics, it is specifically stated that architects do not accept responsibility for the safety, health and the environment until such time as the project has been completed. If, in the course of their work on a project, the Members become aware of a decision taken by their employer or client which violates any law or regulation and which will, in the Members’ judgment, materially affect adversely the safety to the public of the finished project, the Members shall… Commentary: This rule extends only to violations of the building laws that threaten the public safety. The obligation under this rule applies only to the safety of the finished project, an obligation coextensive with the usual undertaking of an architect.

This leaves it quite clear that ‘someone else’ is responsible for the health, safety and the environment while the project is being constructed or executed. Strange to find that neither ACI nor CMAA has picked up on this gap and exploited it. Yet compare the AIA code of ethics wording against the very powerful wording contained in the pilots’ code: Through his seat at the front of the aircraft flow the efforts of thousands of people who provide the means by which he carries out his task. However, it is an undeniable fact that: His is the final responsibility. His is the ultimate decision in any course of action. He can never be complacent. He must be humble; the elements keep him so. He must prove himself to his peers over and over again throughout his career, or seek another job.

159 Chapter 3B: Case Study 2, Commercial Airline Piloting

He must exude a quiet but magnetic confidence in his own ability and his aircraft. He must create an aura of efficiency and capability such that the passengers stream on and off the aircraft without even a thought about what is occurring at the front of the aircraft. Finally, he must be ready during every second of his working life to defeat the ultimate emergency he may encounter at any time.’ These responsibilities are recognized in the definition of his title of ‘Captain’, which means ‘in command’ and as such legally the final responsibility for the safety of the aircraft rests solely with the pilot-in-command. (Patterson, 1999, on-line

For those who desire to turn project management into a profession, this is the kind of accountability practitioners are going to have to be willing to subscribe to in order to win the respect of the consuming public and earn the right to be called professionals. Whether this is done on an individual basis or through an organization will have to be decided, but as John Rotunda (1997) concluded writing in the Arkansas Law Review Journal: ‘The only fruitful use of the term ‘profession’ today relates to individuals, not groups’ (Rotunda, 1997, p. 21). 3B.3.2

Professional association defines best practices

This may well be another significant differentiating factor between an occupation and a profession. The Airline Pilots Association plays a unique dual role as it functions as not only the professional organization but also serves as a bargaining agent (union) for and on behalf of member pilots. In this role, it is by nature in conflict with both the Aircraft Owners Association, IATA and the Federal Aviation Agency (FAA). In this case, we have a triumvirate responsible for negotiating the regulations that both pilots and the airlines must abide by. This tension between the three parties, each with different tasks to perform and conflicting and in some cases competing agendas, make Abbot’s (1988) competition theory applicable, at least in part. 3B.3.3

Identify with the occupation and not the employer

Identifying with an occupation is important from a self esteem perspective but there is no support from the literature which would indicate this is a major contributing factor to the professionalization of piloting. Airline pilots are, on the whole, employees and not free agents as per Zwerman et al. (2004); and their peak organization, the Airline Pilots Association functions more as a union than as an organization maintaining

160 Chapter 3B: Case Study 2, Commercial Airline Piloting

standards and defining the parameters of the occupation. Commercial airline pilots, while identifying themselves as being pilots, also identify themselves with their airlines. Whether this identification has been weakened by the current system of split wage scales would be interesting to investigate, but is beyond the scope of the current research. 3B.3.4

Held in high esteem by the community

Esteem is an effect of professionalization, not a cause. As Haga (1974) points out, ‘autonomy leads to greater success in negotiating for monetary rewards’; and while he doesn’t say it, implicit with earning more money, and having what is seen to be a ‘glamorous job’ (D’agostino, 2000) inevitably means greater esteem in the community. 3B.3.5

Serving a formal apprenticeship or internship

While this too is an important and relevant contributing factor to an occupation being recognized as a profession, it alone does not establish a ‘bright line’ test for any other profession. Yet ‘building hours’, not only total time but time in each type of plane, is essential to obtaining jobs as a pilot; and the importance of continuous training and continuously proving one’s competency by flying safely and successfully cannot be overemphasized. This is yet another area that project management needs to investigate more closely. 3B.3.6

License required to fly a plane

The need for a license can define a profession, given the precedent established by Owyhee County v Rife stating that ‘accountants who were licensed are professionals, but accountants who are not do not qualify as professionals’ (Polelle, 1998). However, school bus drivers are also licensed as are long haul truck drivers, and although driving a school bus or a long haul truck may not be as difficult as flying a plane, is the requirement for a license sufficient alone to define a profession? In the world of construction management, no licenses are generally required. Research indicates that the only two countries requiring the licensing of construction project managers are South Africa and the Ukraine. At this point, no research was available attesting to the efficacy of either licensing program.

161 Chapter 3B: Case Study 2, Commercial Airline Piloting

3B.3.7

Has a body of knowledge which is complicated, secret, abstruse, esoteric or unique

Without question, the body of knowledge required to earn and maintain a pilot’s license is an excellent example of knowledge being abstruse (difficult to understand or comprehend) and esoteric (able to be understood only by those initiated into the occupation). While medicine and law rely on Latin or Greek, the language of aircraft piloting is unique only to aircraft piloting and consists of arcane terminology and acronyms compiled over the years by military and government technocrats. 3B.3.8

Requires liability insurance to practice

While this criteria also forms the basis of what Polelle (1998) believes to be a valid ‘bright line’ test of a profession from the legal perspective, this requirement works best for people who are free agents in terms of their employment. Most pilots and most construction managers are employees. So this test for a profession falls short in and of itself. 3B.3.9

Uses symbolic costumes or uniforms

The use of distinctive clothing is a very interesting extrinsic attribute which seems on the surface to be almost laughingly foolish, yet in many professions, is something quite serious. Doctors wearing white coats; judges wearing black robes and powdered wigs; barristers wearing powdered wigs; accountants wearing green eye shades; engineers wearing rings made out of steel; pilots wearing uniforms and military style chevrons, braids and hats; no matter how foolish it may seem, there is something about a uniform or costume which lends an aura to some occupations. For project management, a lot of energy has gone into the design of the various pins denoting PMP, CCE and other similar credentials. 3B.3.10

Requires highly ritualistic processes

Ritual in an occupation is a seemingly archaic practice, but one which has been maintained for very good reasons. In the operating theater, each instrument is carefully accounted for and tracked, both as it is being used and after it has been used. Thus when you hear the doctor calling for a scalpel from the nurse, it is because she is responsible for tracking that instrument prior to use, while being used and after it has been used.

162 Chapter 3B: Case Study 2, Commercial Airline Piloting

Aircraft piloting is full of the ritualistic processes. One of the most obvious is what is known as the ‘pre-flight checklist’ and it consists of an entire walk around the outside of the plane, checking not only the fuel for water and the oil level in the engine, but signs of damage on the skin. The same applies to the inside of the plane. A written checklist is referred to before any pilot can take off. Undoubtedly important and no matter how boring, rituals have a purpose, and few would argue that following some ritual tends to indicate the occupation is a profession. So where does this leave us? We know that aircraft piloting is regarded as a profession and that construction project management is not. We know what the two occupations have in common. We also know those attributes which are not important to either of them. This leaves only those which are featured in piloting but not in construction management. Yet, in an analysis of the ten attributes of a profession assigned to piloting, not a single one meets the legal, sociological, economic, semantic and dictionary definition of a profession beyond a reasonable doubt. This leads to the preliminary conclusion, according to a post modern or post positivist model, that the idea of piloting as a profession is based as much on perception as on fact; and that the whole is more than the sum of the parts. Acceptance of piloting as a profession derives from meaning and interpretation through a social context. This may well support Zwerman et al.’s (2004) position that the newer alternate theoretical frameworks (control, feminist and conflict theories) (pp. 29 & 30) are in fact more valid than using the attribute models when evaluating project management.

163 Chapter 3B: Case Study 2, Commercial Airline Piloting

CHAPTER 4

Survey development and analysis, Part 1 4.1

Purpose and design of the survey

Consistent with the plan established in Chapter 1, following the review of the literature (Chapter 2), from which 22 attributes of a profession were extrapolated, two occupations – construction project management and airline piloting – were analysed against the attributes with the objective of determining if these attributes could form the basis of a test of the degree to which an occupation could be considered a profession: a professionalization index. That is, could the degree to which an occupation had become a profession be measured? If so, would a continuum result which would not only establish the rank order of any occupation, but would provide a ratio of similitude between any two or more occupations. Chapter 4 describes how, based on the observations from the research in Chapter 3, a survey instrument was developed, the results collated and analysed. It was intended that a survey of individuals in a variety of occupations, including project management, would generate data that could be used to address the research questions. This would include using the survey data to develop a test instrument to test for the degree to which project management (as distinct from construction project management) was a profession. The aim was to develop the test and apply it to the occupation of project management to gain insight into both project management as a profession and the instrument as a test of the degree to which any occupation is a profession. It should be noted, that prior to the distribution of the survey, it was already clear that there was no consensus that project management was a profession. The results of stakeholder meetings demonstrated very clearly that not only was there no agreement between and amongst practitioners, but that the topic was sensitive and highly polarized. Moreover, it was also clear that the perception of an occupation as a profession was neither a black or white, yes or no issue; nor was the perception static. It was anticipated that if the survey were sufficiently sensitive that these tensions would be revealed in the results and a test for determining the level of professionalization of an occupation could be extrapolated..

164 Chapter 4: Survey development and analysis, Part 1

The survey that was subsequently administered consisted of questions numbered up to 47, based on the 22 attributes of a profession, with a selection of questions inserted randomly to collect data related to the demographics of the responding population. The survey also included four (4) foundation questions, which appeared randomly. The answers to these four questions would be essential for the creation of a test of the degree to which an occupation could be regarded as a profession – a professionalization index.

4.2

The participants: A demographic survey

The participants in the survey came from mass mailings of the invitation to participate in the survey, to: the Project Management Institute, (PMI) Construction Management Association of America, (CMAA) American Society of Civil Engineers, (ASCE) Aircraft Owners and Pilots Association, (AOPA) Association for the Advancement of Cost Engineering International, (AACE) International School System (Teachers and Administrators) (ISS)

International Project Management

Association, (IPMA) Australian Institute of Project Managers (AIPM), The George Washington University Alumni List (Lawyers, Doctors and Non- Projects people). All told, it is estimated conservatively that the total population reached between 100,000 to 200,000 potential respondents, from which around 400 responded. As this was designed to be a global investigation, the participants had to be selected with their geographic location in mind, as well as their occupations. Questions appearing randomly in the survey instrument recorded the location, age, gender, education, licenses/certifications held, industry experience, salary, industry sector and job title of survey respondents. 4.2.1 Geographic area of origin The first consideration was to explore whether or not the there were any regional differences. Recognizing the trade-off’s inherent between detail and size of the survey, the decision was made to analyze regions on a continental or sub-continental level, the underlying rational being that other factors such as age or experience would be more relevant differentiators (Figure 4.1).

165 Chapter 4: Survey development and analysis, Part 1

Figure 4.1

Results from survey question 14

From the standpoint of responses, it was pleasing to see such a good response from virtually all areas to which the survey was sent, with the exception of Russia. Of even more significance was the response from Asia-Pacific region. Given that this region encompasses the world’s most populous nations (China, ranked #1, with 1.3 billion people; India, #2, with 1.1 billion people; Indonesia, ranked #4, with 220 million people) with the world’s largest Islamic population, the participation of so many respondents from the region makes this research unique. With China projected to become the world’s largest economy by 2010, and India projected to move into second place by 2015, and Japan currently ranked as the world’s second largest, the opinions of this group represent a key indicator as to what the future of the ‘professionalization’ of project management will look like. 4.2.2 Age As can be seen from the survey results, representation was obtained across all age groups, with ~65% of the respondents being under 42 years of age. These individuals would have a minimum of 25 years to a maximum of 50 years of work left ahead of them; and the opinions expressed by this age mix are therefore likely to represent a reliable indicator of the future trends in the professionalization of project management for a considerable period of time (Figure 4.2).

166 Chapter 4: Survey development and analysis, Part 1

Figure 4.2

Results of survey question 6

4.2.3 Gender The researcher felt gender might influence responses, and while there was a good sample from both men and women, there were surprisingly similar opinions expressed by both genders (Figure 4.3).

Figure 4.3

Results from survey question 7

4.2.4 Education As with other demographics, the sample size of ~400 provided a very representative cross section of education levels (Figure 4.4).

167 Chapter 4: Survey development and analysis, Part 1

Figure 4.4

Results from survey question 8

As this study was designed specifically to capture the views of practitioners of project management, the analysis of current holders of various certificates relating to project management formed a core element of the research. While those holding PMI’s PMP designation dominated the respondents, it was interesting to note that fully 27% held more than one credential. 4.2.5 Licenses/certifications held The fact that over 25% of the respondents felt it necessary or desirable to hold more than one certification indicates that ultimately there will be a partitioning of project management into various specialties, just as engineering has been partitioned into areas of practice (i.e. civil, mechanical, electrical) (Figure 4.5). Of even further interest, this research will explore the phenomenon that even within disciplines such as civil engineering, sub-specialties have emerged, including environmental, structural, fire safety, hydrogeological and waste treatment engineering.

Figure 4.5

Results from survey question 9

168 Chapter 4: Survey development and analysis, Part 1

But most importantly to this research, construction project management, which, at close to 60 years old is perhaps one of the most mature applications of project management, is analyzed as the basis for predicting future trends likely for IT, telecommunications and other less mature users of project management. 4.2.6 Experience It was felt that years of experience would provide some interesting comparisons. Given some ~70% of the respondents have less than 15 years experience, and assuming a typical career lasts for 30 years, it can be reasonably assumed that the projections derived from this population represent another 15 years or more of thinking, subject to changes wrought by economic or social upheavals or technological advances which have the potential to alter the face or understanding of project management as currently practiced (Figure 4.6).

Figure 4.6

Results from survey question 10

4.2.7 Salary One indicator which originally was anticipated as being significant to the analysis of survey data was salary (Figure 4.7). However, as this was a global survey, incorporating many respondents from the developing nations, the salary information proved to be largely irrelevant, unless adjusted for purchasing power parity. Furthermore, the question of origin applied to region/country of birth and did not necessarily indicate where the respondent was currently based. This rendered salary data unusable for the purposes of the research project.

169 Chapter 4: Survey development and analysis, Part 1

Figure 4.7

Results from survey question 11

In terms of future research, when a global response is required, and if salary is important to the research, not only the region of origin for each respondent, but also the primary area of the world in which they are working and the nature of the company for whom they work needs to be identified. The question of salary also has to be delicately phrased to accommodate the respondents’ need for privacy. Despite not being able to use the salary data which was collected, further research identified similar data which has been collected for many years by the Association for the Advancement of Cost Engineering International (AACEI) (www.aacei.org). Using the data from Figure 4.8, which clearly indicates a correlation between experience and salary, experience can be reasonably substituted for salary in most analyses.

170 Chapter 4: Survey development and analysis, Part 1

Figure 4.8

Graph showing salary as a function of work experience (AACE, 2005)

4.2.8 Industry sector The next important demographic consideration was industry sector. This was potentially one of the more important aspects of the survey, as very clearly there is a difference in maturity between the early adopters of project management (construction, aerospace, entertainment) and later adopters, such as IT, telecommunications and financial services. Considerable attention was given, therefore, to comparing the differences between the perceptions from each of the sectors (Figure 4.9).

Figure 4.9

Results from survey question 12

171 Chapter 4: Survey development and analysis, Part 1

4.2.9 Job title Lastly, the final demographic of interest was whether job title had any bearing on how project management was perceived. Again, considerable focus has been devoted to looking at the data generated by this question in order to see if any trends could be identified which would impact the future direction of project management (Figure 4.10). From the perspective of grounded theory, this also offers a rich area for consideration.

Figure 4.10

Results from survey question 13

As 22% of the respondents listed ‘other’ in terms of job title, particular interest was devoted to this category. However, there was no statistically significant difference between this category of respondents and others, aside from minor differences in ranking internally.

172 Chapter 4: Survey development and analysis, Part 1

Figure 4.11

Normalized salaries by job function (ACCE, 2005)

Future researchers looking at project management might want to include these job titles in their demographic surveys. ‘Other’ leads to possible trends in where project management might be headed, for why would a person respond to a survey on project management, if they had no interest in this subject?

4.3

Other survey questions: Towards a professionalization index

4.3.1 The foundation questions Please note that the foundation questions appeared at random in the survey, and that due to constraints imposed by the software used for analysis of the answers, single questions had to be divided into separate numbers. Hence questions 30/31 are essentially one question, as are questions 39-/42 and are referred to in that way.

The four foundation questions included in the survey are outlined in Table 4.1 and distinguished by colour. Details of the questions, such as list of occupations (question 30/31) and the occupational pairings (question 39-/42) are available from Appendix A).

173 Chapter 4: Survey development and analysis, Part 1

Foundation Question 1

Table 4.1

Foundation questions in the survey

Survey Question 28 Below are 8 definitions. Please rank the definition that MOST represents your definition, understanding or interpretation of PROJECT MANAGEMENT as it is practiced by you or within your organization. Mark 1 being the ‘best’ definition, 2 being the second best and so on, with 8 representing the least appropriate definition in your opinion. NOTE. Please give a different rank for each option. Do not use the same number twice. ‘a subject, field or duties executed within my existing occupation or profession’ ‘a way of doing something or carrying something out, according to a plan’ ‘to produce something in a particular or standardized way’ ‘the job by which a person earns a living’ ‘an occupation that requires long and specialized education or training’ ‘activity(ies) that somebody does regularly for pay ‘a method or set of procedures for achieving or producing something’ ‘a series of actions directed towards achieving a specific aim or objective’

Survey Question 30/31 Below is a list of 21 occupations generally considered to be Professions based upon the requirement of a license to practice. Compare each occupation against the baseline of a fresh graduate of a 4 year university, with no experience and no license, marking how much more you perceive the RELATIVE VALUE of the occupation compared against the fresh university graduate in terms of PROFESSIONAL IMAGE or PRESTIGE.

Foundation Question 2

EXAMPLE: Comparing a fresh graduate of a 4 year University against a Candlestick Maker, by marking the ‘40%’ box, you are saying that in your opinion, the Candlestick Maker is perceived by you to be forty percent more of a profession than the fresh graduate (out of a possible maximum of 100%).

appears as Survey Question 31, a continuation of question 30 Below is a list of 21 occupations generally considered to be ‘Professions’ based upon the requirement of a license to practice. Compare each occupation against the baseline of a fresh graduate of a 4 year university, with no experience and no license, marking how much more you perceive the RELATIVE VALUE of the occupation compared against the fresh university graduate in terms of PROFESSIONAL IMAGE or PRESTIGE. EXAMPLE: Comparing a fresh graduate of a 4 year University against a Candlestick Maker, by marking the ‘40%’ box, you are saying that in your opinion, the Candlestick Maker is perceived by you to be forty percent more of a profession than the fresh graduate (out of a possible maximum of 100%).

174 Chapter 4: Survey development and analysis, Part 1

Survey Question 39-/42 Listed below are pairs of occupations (see Appendix A). From each pair, mark the circle that best represents your ideal IMAGE or IMPRESSION of how much of a PROFESSION each occupation is relative to the other. If you consider them EQUAL, mark the 0%.

Foundation Question 3

EXAMPLE- Based on the selection in the sample answer below, you are saying that in your opinion, a Baker is perceived by you to be 80% more of a profession than that of a Candlestick Maker

appears as Survey Question 40, a continuation of question 39 Listed below are pairs of occupations. From each pair, mark the circle that best represents your ideal IMAGE or IMPRESSION of how much of a PROFESSION each occupation is relative to the other. If you consider them EQUAL, mark the 0%. EXAMPLE- Based on the selection in the sample answer below, you are saying that in your opinion, a Baker is perceived by you to be 80% more of a profession than that of a Candlestick Maker

appears as Survey Question 41, a continuation of question 39 Listed below are pairs of occupations. From each pair, mark the circle that best represents your ideal IMAGE or IMPRESSION of how much of a PROFESSION each occupation is relative to the other. If you consider them EQUAL, mark the 0%. EXAMPLE- Based on the selection in the sample answer below, you are saying that in your opinion, a Baker is perceived by you to be 80% more of a profession than that of a Candlestick Maker

appears as Survey Question 42, a continuation of question 39 Listed below are pairs of occupations. From each pair, mark the circle that best represents your ideal IMAGE or IMPRESSION of how much of a PROFESSION each occupation is relative to the other. If you consider them EQUAL, mark the 0%. EXAMPLE- Based on the selection in the sample answer below, you are saying that in your opinion, a Baker is perceived by you to be 80% more of a profession than that of a Candlestick Maker

175 Chapter 4: Survey development and analysis, Part 1

Foundation Question 4

Survey Question 47 Rank the following list of professions in the order you perceive them to represent your professional ideal. (1 being most professional, 21 being least professional) NOTE. Please give a different rank for each question. Do not use the same number twice. Aircraft Pilots (Commercial Jet) Accountant (Certified Public) Commercial Tractor Trailer Truck Driver Barber/Cosmetologists MBAs/Business Consultants Librarians Civil Engineers/Land Surveyors Dentists Lawyers Physicians (Medical Doctors) High School Principal Construction Managers Project Managers School Bus Drivers Computer/Software Engineers Electricians Plumbers Registered Nurses Real Estate Brokers/Appraisers Insurance Agents Electrical/Telecommunication Engineers

Survey question 28 was designed to establish whether project management was considered a profession at all by the respondents. It was expected that survey questions 30/31, 39-/42 and 47 would address that part of the research questions that asked whether a continuum of professionalization existed. At the same time, it was realized that the initial indication of a continuum would only provide a rank ordering, and not provide a true ratio scale. It would still be impossible to determine the degree of separation of the professions. That is, would a doctor be considered 10, 20 or 300 times the professional that a project manager was? 4.3.2 General survey questions In order to develop a ratio scale, other survey questions relating to the 22 attributes of a profession were inserted into the survey. Consistent with the concerns expressed by Zwerman et al. (2004, pp. 20-30), in the development of the survey, not only were the extrinsic or traditional attributes included, but also those attributes derived from the non-traditional perspectives, particularly Haga (1974). This resulted in four intrinsic attributes being considered as well (Table 4.2).

176 Chapter 4: Survey development and analysis, Part 1

Table 4.2

#

22 attributes commonly associated with a profession (from the review of the literature)

Attribute Extrinsic attributes

1

Professional association- defines methodology/defines standards/enforces CoE

2

Autonomy in decision making

3

Lifetime commitment/calling

4

Earn higher than average compensation

5

Publishing in learned journals

6

Long period of training including higher education

7

Subscribe to a code of ethics

8

Fiduciary obligation to public

9

Professional association defines ‘best practices’

10

Identify with occupation, not employer

11

Held in high esteem by the community

12

Apprenticeship/ internship/residency

13

Service to the public- including pro bono work

14

Practice limited by government license

15

Advertising not permitted or restricted

16

Requires professional liability insurance

17

Symbolic costumes/uniforms

18

Use of title restricted by law

19

Body of knowledge

unique/esoteric/secret (both intrinsic and extrinsic)

Intrinsic attributes 20

Cruciality- need is important/need is immediate

21

Dénouement- practitioner is trusted to produce positive results relatively quickly.

22

Mystique- highly ritualistic/access to knowledge is restricted

From the literature review, it had not been difficult to identify those attributes which appeared to be more important (refer Chapter 3), understanding that, like the seven blind men trying to describe an elephant, the relative importance of any single attribute varied, depending on the culture and traditions of each occupation. Some occupations the wearing of a uniform is very much a part of being a profession, (e.g. doctors, airline pilots) while for others, the wearing of a uniform has little or no relevance) In terms of the survey, it was felt that appropriate analysis and use of the answers to the questions would provide move beyond a mere rank ordering, by creating a true ratio scale that could be used to refine the concept of a professionalization

continuum

and

assist

with

the

development

of

a

professionalization index (test for degree of professionalization). Therefore, survey question 45/46 was included to provide a weighting factor to indicate the relative importance of the attributes (Table 4.3).

177 Chapter 4: Survey development and analysis, Part 1

The answers to question 45/46 and the other questions would indicate where on the continuum project management could be considered to lie if it were deemed not to be a full-fledged profession. Table 4.3

Survey question 45/46

Survey question 45/46 Below is a list of 11 attributes normally associated with an occupation being recognized or accepted as being a ‘profession’. From the selection below, please indicate the relative weighting or importance you give to each of these attributes. You can mark any value between 0 (No importance to evaluating or determining a profession), to a maximum of 100 points (Essential attribute necessary to be considered a profession) (Note: 11 different attributes are to be found in each section of question 45/46.)

4.3.3 Summary: The purpose and development of the survey questions Table 4.4 outlines in detail how all the survey questions (other than those related to demographics) were developed, the attributes upon which they were based or the questions they were intended to answer and the hypotheses derived from the investigation of the literature. Thus, each question on the survey was generated via a key question mediated by an hypothesis based on the key findings (Figure 4.1).

key findings

V

questions generated by the key findings

Figure 4.12

V

hypotheses based on the key findings

survey questions developed in response to the key questions

Generation of a survey question (excluding demographics)

178 Chapter 4: Survey development and analysis, Part 1

KQ2 Is there a continuum and where on that continuum of professionalization; if so, where does project management lie on the continuum?

All occupations exist on a continuum

Relative value or weighting of each attribute

KQ1 Is project management a profession?

Multiple, conflicting and confusing definitions of project management and profession abound.

Defining project management as a profession is not a yes or no answer; as an occupational specialty, project management lies on a continuum someplace between the trades and the learned professions.

Project management is not a profession. It is a discipline within all other technical occupations.

Hypotheses based on the key findings, to be tested as part of addressing KQs

SQ 47 Rank the following list of professions in the order you perceive them to represent your professional ideal. (1 being most professional, 21 being least professional)

SQs 39, 40, 41 and 42 Listed below are pairs of occupations. From each pair, mark the circle that best represents your ideal image or impression of how much of a profession each occupation is relative to the other. if you consider them equal, mark the 0%.

SQs 30 and 31 Below is a list of 21 occupations generally considered to be ‘professions’ based upon the requirement of a license to practice? Compare each occupation against the baseline of a fresh graduate of a 4 year university, with no experience and no license, marking how much more you perceive the relative value of the occupation compared against the fresh university graduate in terms of professional image or prestige.

SQ28 Below are 8 definitions. Please rank the definition that most represents your definition, understanding or interpretation of project management as it is practiced by you or within your organization. Mark 1 being the ‘best’ definition, 2 being the second best and so on, with 8 representing the least appropriate definition in your opinion.

Survey questions

Each attribute has a numeric value which can be calculated and that the sum total of the attributes will produce a ‘professional Index’

SQs45/46 Below is a list of 22 attributes normally associated with an occupation being recognized or accepted as being a ‘profession’. From the selection below, please indicate the relative weighting or importance you give to each of these attributes. You can mark any value between 0 (No importance to evaluating or determining a profession), to a maximum of 100 points (Essential attribute necessary to be considered a profession)

179 Chapter 4: Survey development and analysis, Part 1

KQ3 Intuitively, not all attributes are of equal value. What is the actual numeric value for each attribute by respondents?

Question refining the survey data: Finding the relative value or weighting of each attribute

Questions generated by the key findings (KQ) – to be addressed by a variety of test questions

Development of the survey questions, designed to generate data for answering the research questions (demographics excluded)

Key findings (from literature research)

Table 4.4

Life time commitment

b. apprenticeship/internship/ residency

Hypotheses based on the key findings, to be tested as part of addressing KQs

Survey questions

The term ‘accidental profession’ commonly applied to project management indicates that project management is not a calling,.

A profession requires at least a 4 year degree AND 4 years of practical experience.

The project management body of knowledge is NOT unique.

Attributes of a profession

SQ24 When I introduce myself to people, I would be more likely to do so by stating: ‘I am a project manager who works for XYZ company’ OR ‘I work for XYZ company as a project manager’

SQ20 I relate more to being a project manager than I do to working for any particular employer

SQ16 I consider project management to be my life’s work

SQ36 How many years of apprenticeship, internship or supervised work experience do you associate with the term ‘profession’?

SQ35 How many years of work experience does it take to produce a professional project management practitioner?

SQ43 How many years of university level education do you associate with the term ‘professional’?

SQ34 How many years of education beyond high school does it take to produce a professional practiticing project manager?

SQ15 Please evaluate the following terms or phrases by how unique you consider each as they are used in project management compared to general usage

180 Chapter 4: Survey development and analysis, Part 1

KQ6 Is project management a calling?

KQ5 How much experience/apprenticeship/education is associated with the term profession?

Long period of training

a. higher education

KQ4 Is the project management body of knowledge unique?

Body of knowledge a. unique b. esoteric/ complicated/ secret

Extrinsic

Questions generated by the key findings (KQ) – to be addressed by a variety of test questions

Questions based on the 22 attributes identified from the literature – Used to mediate the analysis of the answers to Foundation Questions

Key findings (from literature research)

Publishing in learned journals

b. establishes acceptable standards i. performance standards ii. procedural standards (methodology)

If project management is a profession, then project managers publish in peer reviewed journals

SQ32 During the previous 3 years, I have written and published the following number of articles in professional journals/publications.

N/A

Not a test question; investigated through key word analysis of professional society codes of ethics

Although responses were limited, it appears as though most of the professional organizations representing project managers do NOT aggressively enforce their codes of ethics, at least not in the context of protecting the consuming public. First hand experience and limited responses indicate they use the CoE primarily to control their membership.

This was not on the regular test instrument This was answered through emails sent to the executive directors/CEOs of major professional organizations representing project managers.

SQ29 During the past year, I have donated the following number of hours performing voluntary, uncompensated community service requiring the use of my project management skills

SQ27 People who use my services as a project manager are not concerned about how I execute the project as long as the project is successful (using their definition of success).

Survey questions

181 Chapter 4: Survey development and analysis, Part 1

KQ10 Do project managers publish papers in peer reviewed journals?

N/A

There is a difference between the codes of ethics/codes of conduct of organizations which are recognized as professions from those which are not

This attribute was ignored for the purposes of this research, as all the organizations involved in the study purport to establish performance standards and/or a methodology.

KQ9 (original) How do organizations representing practitioners compare against other organizations in terms of ethics?

Professional association

Project managers meet or exceed the total number of person hours donated by established professions.

A code of ethics adopted for project managers which clearly focuses on the health, safety and welfare of the consuming public will be a clear indication that project management is a profession.

KQ8 How do project managers compare against other professionals in terms of pro bono work?

Service to the public (pro bono work)

Project managers have professional level ethics.

Hypotheses based on the key findings, to be tested as part of addressing KQs

KQ9 (substitute question) If a robust code of ethics plays a key role in determining a profession, how do various CoEs applicable to project management compare against accepted professions?

KQ7 How do project managers perceive the practice of individual ethics?

Adhering to a code of ethics

a. enforces code of ethics

Questions generated by the key findings (KQ) – to be addressed by a variety of test questions

Key findings (from literature research)

KQ11 Does advertising by project managers conform to other professional norms?

KQ12 Is (or should) use of the title ‘project manager’ be restricted by law?

KQ13 Are symbolic costumes, uniforms or other identifying insignia necessary to define a profession?

KQ14 Should project managers be licensed?

KQ15 Should project managers be required to carry professional liability insurance?

Advertising not permitted or restricted

Use of title is restricted by law

Symbolic costumes/uniforms

Practice limited by govt. license

Requires professional liability insurance

Project managers should carry professional liability insurance only if they are held responsible AND financially accountable.

Licensing project managers will protect the consuming public.

Costumes and symbolic uniforms or paraphernalia are necessary to define or identify a professional project manager.

As the term ‘project manager’ is so ubiquitous, there is no way at this point it could be restricted.

As few project managers work on a ‘fee for services basis’ this question was deemed irrelevant to this research.

Hypotheses based on the key findings, to be tested as part of addressing KQs

SQ26 As a project manager, I feel I have ________________ responsibility for the decisions I make related to the execution of the project for which I am in charge.

SQ18 I believe that project managers should be held financially accountable for mistakes they/their teams are responsible for when managing a project.

SQ44 I believe occupational licensing of any type results in a monopoly and a form of restraint of trade.

SQ38 I believe that licensing of project managers will lead to projects being completed on time, within budget and substantially fulfilling all technical requirements.

SQ37 I believe the primary purpose of occupational licensing of any profession is to protect the public from quacks, charlatans or incompetence.

SQ17 When in a work environment, I am able to identify other project managers in my organization by the clothes they wear or some other identifiable part of their costume. (i.e. special tie, rings, headgear or other unique part of their dress or appearance)

Key word analysis of the term ‘project manager’

N/A

Survey questions

182 Chapter 4: Survey development and analysis, Part 1

Questions generated by the key findings (KQ) – to be addressed by a variety of test questions

Key findings (from literature research)

KQ16 Given autonomy in decision making is a key attribute of a profession, do project managers have autonomy in making decisions?

KQ17 Given Identifying with the occupation and not with an employer is a key attribute of a profession, who do project managers identify with?

KQ18 Those in existing professions enjoy high community esteem. What is the community esteem of project managers?

Autonomy in decision making

Identity not with employer but profession

Held in high esteem by the community

Project managers are not held in the same high esteem as existing professionals.

Project managers do not identify with their employer, but with the occupation of project management.

Autonomy in making decisions is a required element of a profession.

Hypotheses based on the key findings, to be tested as part of addressing KQs

SQ21 As a project manager, I consider myself to be held in ______ esteem in my home (nonworking) community because of the work I do as a project manager.

SQ24 When I introduce myself to people, I would be more likely to do so by stating ‘I am a project manager who works for XYZ company’ OR ‘I work for XYZ company as a project manager’

SQ20 I relate more to being a project manager than I do to working for any particular employer

SQ16 I consider project management to be my life’s work

SQ27 People who use my services as a Project Manager are not concerned about how I execute the project as long as the project is successful (using their definition of success)

SQ25 In the fulfilment of my duties in the role of a project manager, I feel I am able to consistently exercise discretionary judgment in how the project gets executed

SQ19 As a project manager, I feel I have ________________ autonomy in making decisions related to the execution of the Project I am responsible for.

Survey questions

183 Chapter 4: Survey development and analysis, Part 1

Questions generated by the key findings (KQ) – to be addressed by a variety of test questions

Key findings (from literature research)

Project management does not have dénouement.

SQ19 As a project manager, I feel I have ________________ autonomy in making decisions related to the execution of the Project I am responsible for. SQ25 In the fulfilment of my duties in the role of a project manager, I feel I am able to consistently exercise discretionary judgment in how the project gets executed. SQ26 As a project manager, I feel I have ________________ responsibility for the decisions I make related to the execution of the project for which I am in charge.

SQ33 People who use the services of a project manager generally consider those services critical to the success of their project.

N/A

SQ23 People not familiar with project management consider what practitioners do to be mystical and/or highly ritualistic.

This important attribute was confirmed /reinforced by comparing the results of AACE's annual salary survey.

SQ22 Compared to other people of my age and education level, in my position as a project manager, I feel I am compensated ___________ for my services.

Survey questions

184 Chapter 4: Survey development and analysis, Part 1

KQ22 A leading intrinsic attribute for a profession is the dénouement or the ability to produce positive results very quickly and without limited involvement of the consumer of the services. Does project management have dénouement?

Denouement quick solutions to problems

Project management does have cruciality/criticality.

N/A

This attribute was so patently obvious that it was given a value of zero for the purposes of the research. Not only is the body of knowledge an agglomeration coming from a multitude of sources, as most of the BoK derives from general business teachings, access is far from limited.

KQ21 A leading intrinsic attribute for a profession is the cruciality (criticality) of the services. Does project management have criticality?

Project management does not have mystique.

Cruciality a. immediacy of need b. Importance of need

Mystique (average of 15, 23 and 0) (highly ritualistic procedures)

Project managers earn higher than average salaries.

Hypotheses based on the key findings, to be tested as part of addressing KQs

KQ20 A leading intrinsic attribute for a profession is the mystique which surrounds it. Does project management have mystique?

KQ19 Those in occupations recognized as professions earn higher than average salaries. What is the perception of project managers in terms of the salaries they earn?

Earn higher than average compensation

Intrinsic

Questions generated by the key findings (KQ) – to be addressed by a variety of test questions

Key findings (from literature research)

KQ23 How does the fiduciary responsibility of project managers compare against that of established professions?

Fiduciary obligation to the public

Project managers do not exhibit a level of fiduciary responsibility comparable to that of established professions.

Hypotheses based on the key findings, to be tested as part of addressing KQs

SQ26 As a project manager, I feel I have ________________ responsibility for the decisions I make related to the execution of the Project for which I am in charge.

SQ25 In the fulfilment of my duties in the role of a project manager, I feel I am able to consistently exercise discretionary judgment in how the project gets executed.

SQ19 As a project manager, I feel I have ________________ autonomy in making decisions related to the execution of the Project I am responsible for.

SQ18 I believe that project managers should be held financially accountable for mistakes they/their teams are responsible for when managing a project.

Survey questions

185 Chapter 4: Survey development and analysis, Part 1

Questions generated by the key findings (KQ) – to be addressed by a variety of test questions

Key findings (from literature research)

4.4

Summary of Chapter 4

In Chapter 4, part 1 of the survey development and analysis, we explored how the 22 attributes generated a series of key questions. These key questions, in turn, led to hypotheses which led to questions on the survey instrument to prove or disprove the hypotheses. the development of the was discussed. The questionnaire was prepared using the 22 attributes distilled from Chapter 2, and pre-tested in Chapters 3a and 3b. Chapter 4 also indicates how the results were collected, collated and analysed. As noted previously, it was intended that a survey of individuals in a variety of occupations, including project management, would generate data that could be used to address the research questions. This would include using the survey data to develop a test instrument to test for the degree to which project management (as distinct from construction project management) was a profession. The aim was to develop the test and apply it to the occupation of project management to gain insight into both project management as a profession and the instrument as a test of the degree to which any occupation is a profession. While the failure to obtain sufficient responses meant that a comparison between project management and other occupations was not realized, the fundamental principles remain valid and the results have proven appropriate in answering some of the initial research questions. This will be explored in part two of the survey development and analysis, Chapter 5.

186 Chapter 4: Survey development and analysis, Part 1

CHAPTER 5

Survey development and analysis, Part 2 5.1

Analysing answers to the key questions

While Chapter 4 (part 1 of the survey development and analysis) focused on the development of the survey, including the demographics, in Chapter 5 (part 2 of the survey development and analysis) the data obtained from the replies of the 400 respondents is considered. In Chapter 5, preliminary answers to the research questions began to be formed. Please note that for all statistical analysis, ranking and other quantitative analysis, unless otherwise stated, the mean values, calculated from the survey responses was used. 5.1.1 Key question 1 Is project management a profession? Initially, the current research was triggered by this question, which was then refined to Can a valid test be developed that indicates to what degree an occupation (such as project management) can be considered a profession? AND If a valid test were applied to project management, would project management be a profession? If is it not a profession, what is it? The research initiating question, however, became a key question in the development of the survey instrument: Key question 1: Is project management a profession? From the key question, mediated by a hypothesis, the survey question was developed, as indicated in Exhibit 5.1. Exhibit 5.1

Generation of survey question 28

Key finding

Multiple, conflicting and confusing definitions of project management and profession abound.

Question generated by the key finding

KQ1 Is project management a profession?

Hypotheses

Project management is not a profession. It is a discipline within all other technical occupations.

Survey question

SQ28 Below are 8 definitions. Please rank the definition that most represents your definition, understanding or interpretation of project management as it is practiced by you or within your organization. Mark 1 being the ‘best’ definition, 2 being the second best and so on, with 8 representing the least appropriate definition in your opinion.

187 Chapter 5: Survey development and analysis, Part 2

As can be seen from the literature review, the question of what is and what is not a profession remains unresolved. Not only did the initial stakeholder analysis turn up three clearly distinct and mutually exclusive perspectives, but from the literature reviews, particularly from the legal and semantic perspectives, we have seen Polelle (1999), Rotunda (1997), Gawley, Breitel (2002), Abbot (1988), Freidson (1994), Haga (1974) and Cogan (1955) all postulate their interpretations. Polelle (1999) in particular laments the lack of a ‘bright line’ definition for the courts to use and he has formulated a set of criteria which he believes to be appropriate (Polelle, 1999). A review of US labor laws to gain insight into how the US Department of Labor defines a profession was undertaken, as was an examination of the definitions of profession and professional from not only legal, but also from sociological, economic and semantic perspectives. Consistent with the practices of some jurists, I looked at the dictionary definition of profession and professional and restated those definitions to see if they provided any further enlightenment. In order to minimize the probability of the answers to one question biasing the answers to other questions, the sequencing of each question in the test instrument was intentionally randomized. Hence no significance can be imputed to the sequential numbering of the question. Note, therefore, that test question numbers do not match key question numbers. Figure 5.1 presents the responses to survey question 28. Because this is a ranking question, the choices selected first, second or third will have the shortest bars, while those ranked sixth, seventh or eighth will have the longer bars. This question proved to be one of the most important and interesting of the research. Through this forced ranking, a very clear consensus favored process, method or system (ranked 1-3 respectively) vs. vocation (trade) occupation or profession, (ranked 8th, 7th or 6th, respectively). Discipline, which was an anticipated answer, fell in between the two groupings. But more importantly, profession was ranked in the middle grouping. This can be seen more clearly using a linear scale, 1-8, incremented by divisions of .1. Note that for test question 28, as respondents were being asked to rank from Most preferred definition (1) to Least preferred (8) the lower numbers represent the highest ranked definitions while the higher numbers indicate the lowest preference. 188 Chapter 5: Survey development and analysis, Part 2

Figure 5.1

Results from survey question 28

Very clearly and by a wide margin, project management practitioners chose the definition of process (1.99 out of a possible 8.00) to best describe what it is they do, followed by method at 2.95, and system at 3.04. This formed Cluster 1 in Figure 5.2. The second cluster consists of procedure, (4.3) discipline, (4.6) and profession, (5.2) followed by a gap of to the third cluster, consisting of occupation (6.8) and vocation (6.9). Given the semantics, although ‘vocation’ was intended to mean the trades, the word ‘calling’ is synonymous and in that context, is often linked with being a profession, especially in terms of the learned professions, theology, medicine and law. While certainly an interesting finding, it remained premature to draw any conclusions or read anything meaningful into these results until they could be taken in the context of the whole. However, at least on the surface, it appears as though respondents may well be saying that all three interpretations have meaning or relevance? That in fact ‘systems engineering’ as INCOSE defines it may be correct? That ‘project management’ (which is hardly distinguishable from systems engineering) is in fact a ‘process, method or system’ that is ‘multi-disciplinary’ and cross functional? (INCOSE, online, n.d.). 189 Chapter 5: Survey development and analysis, Part 2

Another interesting observation is the respondents selected discipline over profession, which tends to support Drucker’s consistent references to all incarnations and applications of management as being a discipline (Drucker, 1954, 1973, 1999, 2001, 2003). While useful and meaningful for the intended purposes of this research, the full significance and meaning of this clustering remains a ripe area for future exploration.

Figure 5.2

Survey question 28, responses plotted on an interval scale

5.1.2 Key question 2 Is there a continuum and where on that continuum of professionalization; if so, where does project management lie on the continuum? Key question 2 asked if the degree to which an occupation could be considered a profession dwelt on a continuum, and if it were possible to discern where one profession stood in relation to others. The continuum includes the licensed trades through the learned professions of law and medicine, and incorporates many of the occupations considered to be ‘emerging’ or ‘evolving’ professions. Key question 2 directly addressed the original research question.

190 Chapter 5: Survey development and analysis, Part 2

The objective was, if possible, to develop a ratio scale, with a fixed and known zero point. While the findings from the survey questions were interesting and consistent within reason, they were unable to conclusively identify where on the continuum between the trades and the learned professions project management lies. While Zwerman et al. alluded to project management as a profession not being a black or white, yes or no answer, (Zwerman, 2004, p. 174), at least three other researchers had already expressed this point of view. Leading was Bruce Kimball (1995), Professor of Educational Leadership, University of Rochester. In his book, A history of the true professional ideal (Kimball, 1995) he posited the idea that professions are subject to a normal life span or life cycle, with a conception, period of development, exponential growth, leveling off and eventually declining phase (Kimball, 1995). To the concept that all professions were subject to the ebb and flow of life spans, I added thoughts from MIT’s social psychologist, Edgar H. Schein (1972),who theorized in his Professional education: Some new directions (Schein, 1972) that ‘sociologists have agreed on the necessity to use a multi-hypothesis approach in defining a profession’ (Schein, 1972, p 12). Another influential researcher who contributed to the development of key question 1 was Malcolm Gladwell (2002), in his best-selling book Tipping point. The essence of Tipping point is that some ‘magical moment’ exists when an ‘idea, trend or social behavior crosses some threshold, tips, and spreads like wildfire’ (Gladwell, 2002, back cover). Combining the ideas of Kimball (1995), Schein (1972) and Gladwell (2002), I theorized that, given the normal life span applies to occupational popularity (Kimball, 1995), that using a multi-hypothesis approach (Schein, 1972) it would be possible to create an instrument that could measure and track where an occupation was vis-a-vis other occupations, and in doing so, could identify a ‘tipping point’ where an occupation moves from merely being an occupation to actually becoming recognized as a profession. Applied to project management specifically, the objective was to measure where project management lay on a continuum between the trades (which is where the concept of the professions originated) to the learned professions of law, medicine, theology and education.

191 Chapter 5: Survey development and analysis, Part 2

Given the importance of this concept of a continuum to the overall research, groups of separate questions were conceived which it was hoped would reinforce or support the findings and also to eliminate bias (Exhibit 5.2). Exhibit 5.2

Generation of survey questions 30/31, 39-/42, and 47. (Details of questions, such as lists and pairs, and combined results available from Appendix A.) Key finding

All occupations exist on a continuum

Question generated by the key finding

KQ2 Is there a continuum and where on that continuum of professionalization; if so, where does project management lie on the continuum?

Hypothesis

Defining project management as a profession is not a yes or no answer; as an occupational specialty, project management lies on a continuum someplace between the trades and the learned professions.

Survey questions

SQ 30/31 Below is a list of 21 occupations generally considered to be ‘professions’ based upon the requirement of a license to practice? Compare each occupation against the baseline of a fresh graduate of a 4 year university, with no experience and no license, marking how much more you perceive the relative value of the occupation compared against the fresh university graduate in terms of professional image or prestige. SQ 39-/42 Listed below are pairs of occupations. From each pair, mark the circle that best represents your ideal image or impression of how much of a profession each occupation is relative to the other. if you consider them equal, mark the 0%. SQ 47 Rank the following list of professions in the order you perceive them to represent your professional ideal. (1 being most professional, 21 being least professional)

RESULTS AND ANALYSIS FROM SURVEY QUESTION 30/31

The results from this question are summarized by the Excel graph shown in Figure 5.3. In this graph, the average (mean) score for each occupation is shown by the pink diamond, with the lines showing the mean plus and minus one standard deviation. The possible range was from 0, meaning the subject occupation was equal to a recent four year graduate with no experience and no license to practice, to 11, meaning the occupation was 10 times greater than that of a recent (fresh) graduate. As can be seen in Figure 5.3, the average and average plus/minus one sigma is interesting, but fails to clearly show the relative distance between any two occupations. To address this, an Excel spreadsheet was created with identically sized cells, the cells were numbered from 0 to 11 in .1 increments, and the average results from the spreadsheet were plotted. Figure 5.4 indicates the relative ranking of each occupation against the baseline of a recent (fresh) university graduate.

192 Chapter 5: Survey development and analysis, Part 2

Now, is it fair or justifiable to say that a high school principal, at 6.93, is roughly twice as professional as a school bus driver at 3.37? Or that a doctor at 9.25 is 2.75 times as professional as a school bus driver, at 3.37? (9.25/3.37 = 2.75). While the objective was worthy, using a recent university graduate didn’t really generate a true zero, thus it failed to create a true ratio scale, where a practitioner can say with any degree of certainty that any given occupation is X times more or less of a profession than any other. The initial test instrument also contained the flaw of introducing a bias favorable to the respondents’ own occupation. As the sample size contained such a large percentage of project managers, this phenomenon could not help but introduce a bias favorable to project managers. 11.00 10.00 9.00

Ranking (1-11)

8.00 7.00 6.00 5.00 4.00 3.00 2.00 1.00

PM De nt is La t wy er CP A Do ct or Pi lo t

El

R ec N tE ng M B HS A Pr in c CM Ci vi lE ng

0.00 G ra du a Bu t e sD rv Tr r kD r Ba vr In rb e su r ra nc Pl e um RE be Br r o Li ker br El aria ec tri n Co cian m pE ng

Mean +1 Sigma Mean -1 Sigma Mean

Occupational Specialties

Figure 5.3

Results of survey question 30/31; relative ranking of occupations compared against recent college graduate responses plotted showing mean, and range, +/1 sigma

193 Chapter 5: Survey development and analysis, Part 2

Figure 5.4

Professionalization continuum developed from data in response to survey question 30/31; interval scale showing relative relationships between licensed occupations and project managers

CONSTRUCTION AND ANALYSIS OF SURVEY QUESTION 39-/42

Question 39-/42 was set up using combinations of occupations commonly licensed by the top 1/3 of Organization for Economic Cooperation and Development (OECD) countries. [Note: The reason for selecting only the top 1/3 of OECD countries was based on the fact that many developing countries use licensing as a form of ‘legalized extortion’ or ‘economic rents’. (Transparency International, 2005)]. However, if the full 21 occupations used in survey questions 30/31 and then 47 were to be used for pair-wise comparisons in question 39-/42, the survey instrument would have been unwieldy, resulting in a 21 X 21 matrix, or 441 data points. To cut this down, the most obvious occupations at the low end of the scale (barber, truck driver) were eliminated, and the various engineering occupations were simply aggregated into a single occupational listing of ‘engineer’. Another category, ‘building contractor’ was also added, to address a point raised by a stakeholder during the Anchorage presentation, that while construction managers are not always licensed, in most instances, building contractors, who certainly play the same role as construction managers, almost always are. While the number of occupations was scaled down, the primary objective of using pair-wise comparisons and applying the statistical analytical approach devised and 194 Chapter 5: Survey development and analysis, Part 2

made famous by Thomas Saaty (1980) for the analytical hierarchy process (AHP) was to eliminate the bias of respondents naturally favoring their own occupations and ranking them higher. As the bias only applied to a total of 10 sets of analysis out of the 45 sets, the impact of any bias was ‘washed out’ by the other 35 pairs of data. Recognizing that structuring of a survey question asking people to rank their own occupation against that of others would inevitably result in bias, an approach had to be applied which would compensate or enable the bias to be removed or accounted for. The tool chosen for this was pair-wise comparison, then applying Thurstone’s Law of Comparative Judgment (1927; see Appendix 3) to factor out bias. Thurstone’s Law of Comparative Judgment Thurstone presents a simple theory of the discriminal process and how its nature allows the construction of an interval scale based on comparisons of pairs of stimuli. This Law of Comparative Judgments is often used as a psychophysical method to derive interval scales of perceptual qualities. The resulting rank order rated by each respondent can be combined in one scale using Thurstone’s Law of Comparative Judgment to translate rank orders to a single group interval scale for the group as a whole. Thurstone’s Law of Comparative Judgment (1927) is based on the following five propositions: 1.

Each stimulus gives rise to a discriminal process, which has some value on the psychological continuum of interest.

2.

Due to momentary fluctuations (which can be considered as internal fluctuations occurring within or between observers), the value of a stimulus may be higher or lower on repeated presentations. The distribution of this fluctuation can be characterized by a postulated normal distribution (or some other known distribution).

3.

The mean and standard deviation of the distribution associated with a stimulus are its internal scale values and discriminal dispersion, respectively.

4.

Therefore the distribution of the difference between two stimuli is also normally distributed and it is a function of the proportion that one stimulus is chosen as greater than the other.

5.

The difference in scale values, R, between two stimuli are the standard deviations of the respective discriminal dispersions, rij is the correlation between the two discriminal processes, and zij is the normal deviate (the z-score) corresponding to the proportion of times stimulus j is judged greater along the psychological continuum than stimulus i.

Certain assumptions can be made that result in a simplification of this equation that also leads to a very simple procedure for analysis of paired comparison data. These assumptions, Thurstone’s Case V, are:

195 Chapter 5: Survey development and analysis, Part 2

1.

The evaluation of one stimulus along the continuum does not influence the evaluation of the other in the paired comparison.

2.

The dispersions are equal for all stimuli. (Thurstone, 1927, pp. 273-286)

For this research, Case V was applied. Montag (2006) further explains that, while simple to use and understand, Thurstone’s Law has the following disadvantage: Practitioner’s have had some problems in implementing Thurstone’s Law because the ability to compute confidence intervals is missing in the formulation. (online)

Despite the complex sounding definition and explanation, the actual process of applying Thurstone’s Law is fairly easy and straightforward. The first operation being to collect pair-wise data (1983), which was done via survey question 39-/42, which gave respondents the opportunity to rank nine occupations against one another on a pair-wise basis. Applying Thurstone’s Law of Comparative Judgment (1927) to the data collected from the survey produced a very interesting map measuring the relative degree of professionalization exhibited by project management when compared against licensed occupations generally considered to be professional in nature. What do practitioners do with this information? As noted previously, one advantage of using pair-wise comparison and applying Thurstone’s Law is a way to compensate for or eliminate bias by respondents towards their own occupation. What does that look like? (See Figure 5.5.) Figure 5.6 then shows that, based on the pair-wise comparisons, project management is less of a profession than medical doctor but more than an electrician. More importantly, project management can be said to be roughly half way between an electrician and a medical doctor in terms of ‘professional evolution’ (6.1 vs. 6.3).

196 Chapter 5: Survey development and analysis, Part 2

Figure 5.5

Question 39-/42, professionalization continuum: Interval scale showing relative relationships between licensed professions and project managers, by applying Thurstone’s Law of Comparative Judgement

Figure 5.6

Applying the bias shift from SQ39-/42 against the results of SQ47, illustrates the interim step to eliminate bias from the results of both SQ 30/31 and SQ 47.

RESULTS AND ANALYSIS OF SURVEY QUESTION 47

The last and most simple of the three questions designed to address key question 2 was a rank ordering of 21 professions (see details, Appendix A). It was hoped that the results of this question would provide sufficiently common rankings to form a pattern that would indicate agreement as to where occupations dwelt on a professions

197 Chapter 5: Survey development and analysis, Part 2

continuum. The disadvantage of using rank ordering lay in the large number of items to be ranked. While ranking is fairly easy to do with 10 or less items, 21 items are difficult to rank and results can be arbitrary or misleading. The top four or five items are easy enough to rank, likewise, the bottom four or five. Those items between the poles are often difficult to categorize, however. Furthermore, because of human nature, one tends to introduce a bias towards one’s own occupation. Given most of the respondents were project managers, it was expected that the rankings in response to survey question 47 would contain some bias (Figure 5.7). Figure 5.7 shows the same treatment accorded survey question 47 that was given to question 39-/42, which also contained inherent bias, which we have no way of quantifying using a simple forced ranking approach. 21.00 19.00

Response Value (11-1)

17.00 15.00 13.00 11.00 9.00 7.00 5.00 3.00

CP A La wy er s De nt i st Ai s rc ra ft M Pi ed lo ic t al D oc to rs

M Ci BA vi lE ng Pr i ne oj ec er tM s an ag er s

Sc ho ol Bu Ba s rb Dr er iv /C Co er os m s m m et er ol cia og lT is t ru ck In su Dr ra ive nc r e Br ok er s Re al to rs Pl um be rs Li br ar ia ns Hi El gh ec tri Sc ci ho an Co ol s ns Pr tru in cip ct io al n Co M Re an m g pu ag ist te er er Te r/S ed le o Nu co ftw m rs ar m es e un En ic gi at ne io er ns s En gi ne er s

1.00

Occupation

Figure 5.7

Question 47, responses plotted showing mean, and range, +/-1 sigma

The values for each of the average readings were entered into an Excel spreadsheet containing identically sized cells (constant horizontal scale). Each cell was numbered from 21 to 0 in increments of .1. The average values calculated from the database for each response in the test were entered into this spreadsheet, turning the ordinal data 198 Chapter 5: Survey development and analysis, Part 2

into an interval scale. In order to enable comparing apples to apples, the scales were kept the same, and the comparisons were made irrespective of the actual values generated, only their position relative to one another. Realizing that using forced ranking (as in survey question 47) or relative values (as in question 30/31) would produce a bias, the pair-wise analysis, adjusted by applying Thurstone’s Law of Comparative Judgment (1927), was considered to produce the more reliable values, hence was used as the baseline. Results are presented in Figure 5.8.

Figure 5.8

Question 47 illustrating bias adjustment; professionalization continuum: Interval scale showing relative relationships between licensed occupations and project managers, adjusted for bias using Thurstone’s Law. (reading left to right, least professional 21.0 to most professional at 1.0)

COMPARING THE RESPONSES TO SURVEY QUESTIONS 30/31, 39-/42 AND 47

The process followed in analyzing this series of three questions was consistent. The data from the test were first averaged by occupation. The second step was to calculate the standard deviation of each column, and plot it in Excel, showing the mean and the mean plus and minus 1 standard deviation (refer Figure 5.9). While this produced a picture of the relative ranking, it didn’t present a satisfactory picture of the relative perception of one occupation vs. another. To convert the ordinal data into an interval scale, an Excel spreadsheet was created with identically sized cells. The cells were 199 Chapter 5: Survey development and analysis, Part 2

numbered from 21 to 1 in .1 increment’s. The average score for each occupation was positioned over the appropriate cell value. (Note, no two occupations were exactly the same value). Average values were rounded up or down to the nearest .1 increment (Figure 5.9). The advantage of this method was to produce a very easily read and understood interval scale rating, with an accuracy suitable to convey some sense of where project management ‘fits’ compared against occupations that almost always require a license to practice. The disadvantage of this approach was that it failed to eliminate the natural tendency of respondents to rank their occupation higher than it should be. As can be seen in Figures 5.9 and 5.10, a method had to be found which would eliminate the effects of bias in the data. Another disadvantage was that with no true zero the data could only be analyzed by the distance between any two occupations. It could not be said that one occupation was a multiple of any other. That is, it could not be said that a project manager, with an average score of 7.36, was roughly twice as much of a professional as a plumber, with an average score of 14.44. All that could be said was that the difference in perception between a plumber and a project manager is 14.44 – 7.36 = 7.08. There is no way to judge the value of a single increment, unless there is a true zero. When the three sets of data were compared using the same horizontal scale, some extremely exciting information became apparent, however. In Figures 5.9 and 5.10, the green boxes indicate instances where all three questions produced the same results. As suspected in the preliminary analysis, regardless of whether people use pair-wise comparison or rank ordering, those occupations at the extremes pose little or no confusion or disagreement. This is particularly well supported by the results for survey questions 30/31 and 47, where the correspondence of the middle occupations (save for electrician) is only off by one or two places. There are no radical displacements of any occupation in favour of another.

200 Chapter 5: Survey development and analysis, Part 2

Figure 5.9

201 Chapter 5: Survey development and analysis, Part 2

Questions 30/31, 39-/42 and 47 compared BEFORE adjustment for bias

Given that applying Thurstone’s Law of Comparative Judgment (1927) has eliminated bias, when the results from the three questions are compared on a common horizontal scale, there is almost perfect agreement as to the position of project management among other occupations, which are also fairly consistently aligned.. Based on this alignment, I feel confident in claiming that in mid-2005, project management was considered by those practicing it to be less of a profession than an MBA and professionally registered engineers, but more of a profession than software or IT engineering, telecommunications engineering and construction management. The ramifications of these findings are numerous. The data provide early indications that while not yet a profession, project management is perceived to be an occupation possessing moderately high esteem within the global community of practice. Construction management, which has been around for 50 years or more, has not had much success in gaining equal footing with architects and civil engineers. Close, but not there yet.

202 Chapter 5: Survey development and analysis, Part 2

Figure 5.10

203 Chapter 5: Survey development and analysis, Part 2

Questions 30/31, 39-42 and 47, compared AFTER adjustment for bias

Construction management, despite having been one of the ‘first movers’ in developing and adopting ‘management by project’ concepts, methodologies, tools and techniques, may be suffering from an ‘identity crisis’ which has cut it off from the more glamorous and highly visible yet more recent adopters of project management as a delivery system, such as IT and telecommunication. This is consistent with Drucker’s (1973) focus on the growing importance of the knowledge worker. The newly emerging engineering practices such as telecommunications engineering, software engineering or computer science, have yet to achieve equivalent status with civil engineers. Project management is close to, but still considered ‘inferior’ to the MBA. Consistent with the research of Christophe McKenna (2006), the MBA (management consultants) may well be poised to become the next profession. Drucker alludes to this possibility in the preface of Management: Tasks, responsibilities and practices (Drucker, 1973, p. xi) but his subsequent works never really follows up on it any further. Given Drucker’s prolific writings, it would seem as though he would have developed this theory more, however, even as late as 2003, despite the title On the profession of management, he does not make ANY case supporting the claim that management is or should be a profession. These early interpretations of the data corresponded to Christopher McKenna’s (2006) new book The world’s newest profession: Management consulting in the 20th century,. Interesting to note that despite the title of McKenna’s book, he concludes ‘by the end of the 20th Century, no credible observer could claim management consulting still constituted a young professional field’ and that to be recognized, ‘the world’s newest profession would have to wait until the 21st century (Mckenna, 2006, p 251). So, even at the start of the 21st century, the MBA is not considered a profession. McKenna’s reasons for management consulting not being recognized as a profession lie in the real or perceived lack of professional ethics within the ranks of management consultants (McKenna, 2006, p. 247).

204 Chapter 5: Survey development and analysis, Part 2

5.1.3 Key question 3 What is the actual numeric value for each attribute by respondents? The design of the survey question to answer key question 3 was based primarily on the ‘multi-criterion’ perspective espoused by Schein (1972). The concept was fairly simple: assuming that Schein is correct in his assertion that multiple criteria will result in a definition for ‘profession’, that if the criteria could be identified and a valid survey instrument could be developed to capture the perspectives from both project managers and occupations considered to be a profession, that a comparison could be drawn between the results. Unfortunately, while the survey produced 400 respondents, there were an insufficient number of respondents from occupations other than project management to be able to make a valid comparison. However, the research did provide a methodology to measure project managers, and the hope is the professional index developed here will be used as the basis to measure other occupations. Exhibit 5.3 illustrates the development of survey question 45/46 to address key question 3: Intuitively, not all attributes are of equal value. What is the actual numeric value for each attribute by respondents? Exhibit 5.3

Generation of survey question 45/46 (Details of question available in Appendix A)

Key finding

Relative value or weighting of each attribute

Question generated by the key finding

KQ3 Intuitively, not all attributes are of equal value. What is the actual numeric value for each attribute by respondents?

Hypotheses

Each attribute has a numeric value which can be calculated and that the sum total of the attributes will produce a ‘professional index’

Survey question

SQ45/46 Below is a list of 22 attributes normally associated with an occupation being recognized or accepted as being a ‘profession’. From the selection below, please indicate the relative weighting or importance you give to each of these attributes. You can mark any value between 0 (No importance to evaluating or determining a profession), to a maximum of 100 points (Essential attribute necessary to be considered a profession)

This question was very important to the research. There are two variables of importance to consider when analyzing the attributes. The first is the average raw score of the response the population provides to each of the survey questions. The second variable is the relative value of each attribute has compared against the others. A search of the literature revealed no research supporting this particular approach to survey data, as sociologically, legally, economically and semantically, all researchers had relied previously on qualitative rather than quantitative approaches. Applying

205 Chapter 5: Survey development and analysis, Part 2

Friedson’s common sense approach (Freidson, 1994), most should agree that not all attributes are of equal value. Furthermore, depending on the occupation in question, some attributes may be more important than others. For instance, the wearing of a uniform, while ranked dead last by project managers, would undoubtedly rank much higher if the survey were to be given to airline pilots or doctors. Worth noting from Figure 5.11, not a single attribute scored above 80% (the test of ‘clear and convincing evidence’ or our assumed ‘tipping point’ based on Pareto’s Law). 100 90 80

60 50 40 30 20 10

B Pr oK of O rg S Fi dO b C ru c Pr of ial O r Pr gB P of O rg Au PE to no m y Id en tif BS y Ed u Bi gS Pr al es tig R es e In te rn C om m it Ti tle R es Li ab t ty Jo Ins ur nl Pu b Li ce ns e Pr oB on Ad o vR es t U ni ifo rm

st Tr u

oE

0

C

Weighting (0-100%)

70

Attributes of a Profession

Figure 5.11

Results for survey question 45/46, attributes rank ordered showing +/1 sigma. Series 1 = Mean + I Sigma; Series 2 = Mean – 1 Sigma; Series 3 = Mean Value (● )

Furthermore, of the top quartile, three of the attributes are most closely associated with the semantic or intrinsic attributes (trust, body of knowledge and cruciality). Also worth noting is the widespread sigma on the body of knowledge and on the fiduciary obligation. Although several correlations were made on the data, there was nothing which indicates any one demographic group had a particularly different perspective than any other group. Here again, further research in this area may well be of more value, for as Freidson (1994) noted ‘scholarship concerned with the

206 Chapter 5: Survey development and analysis, Part 2

professions is in an intellectual shambles today’ (p. 149). Friedson’s (2001) more recent work Professionalism – The third logic reflected on much the same philosophy. While he was attempting to ‘create a model of the logic of professionalism consistent with the free market models or of business-organizational model’ (p. 4), the model he created was impossible to quantify. Given the importance of this data to the remainder of the research, I have also included it in tabular form. For the time being I will leave this data, but will be coming back to it as we draw further conclusions about project management as a profession. In Table 5.1, the 22 attributes of a profession are rank ordered from #1 to #22 based on the average value assigned to them by respondents to the survey. This value is interpreted as being the relative value or weighting that the population of respondents gave to each attribute. Table 5.1

Attributes rank ordered showing actual values (weight) from survey

Rank

Attribute

Weight

1

code of ethics

77.5

2

trust by clients

76.4

3

body of knowledge

70.7

4

prof. org sets standards

69.0

5

fiduciary obligation

68.2

6

crucialilty

67.8

7

proforg establishes best practices

65.5

8

proforg enforces code of ethics

63.6

9

autonomy in decision

61.8

10

identify with occupation

60.3

11

4 year degree or better

58.8

12

higher than ave salary

55.1

13

prestige in community

54.7

14

serve residence/internship

53.9

15

lifetime commitment

49.6

16

title is restricted by law

49.1

17

requires liability insurance

46.8

18

publish in journals

40.8

19

license required

37.8

20

perform pro bono work

35.8

21

adverts restricted

26.1

22

wear a uniiform

14.3

207 Chapter 5: Survey development and analysis, Part 2

5.1.4 Key question 4 Is the project management body of knowledge unique? Key question 4 was derived directly from the literature research indicating that a body of knowledge which is ‘unique, esoteric, secret, complicated or abstruse’ is prima facie requirement underlying any profession. This is one of the few attributes which is common to all perspectives on defining a profession, and as such, one of the first attributes that I wanted to evaluate. Furthermore, based on input from J. Rodney Turner during the defense of this thesis, not only does the BoK need to be ‘unique, esoteric, secret, complicated or abstruse’ but it also must meet three additional tests. It must be: „

theoretically based

„

empirically tested

„

proven to be scientific and predictive

Based on literature review, sufficient evidence exists that the body of knowledge related to project management is NOT ‘unique, esoteric, secret, complicated or abstruse’, (Wideman, 1996; INCOSE, 2006) leading to the null set as being the most likely hypothesis. Exhibit 5.4

Generation of survey question 15 to answer key question 4 (Question details: Appendix A)

Key finding

Body of knowledge a. unique b. esoteric/ c. complicated/secret

Question generated by the key finding

KQ4 Is the project management body of knowledge unique?

Hypotheses

The project management body of knowledge is NOT unique.

Survey question

SQ15 Please evaluate the following terms or phrases by how unique you consider each as they are used in project management compared to general usage

Survey question 15 took the 26 most frequently occurring words in the PMBOK Guide 2000 and asked respondents to evaluate them by how unique they considered each to be in the way it was used in project management compared to general usage (see Appendix A). The design of the question was based on the idea that a broad spectrum of respondents would ‘wash out’ any bias. Unfortunately, a broad spectrum of respondents was unable to be secured, which rendered the intent to compare the results of project managers vs those who were not project managers impossible.

208 Chapter 5: Survey development and analysis, Part 2

However, despite the inability to use the results as designed, the eventual results proved useful, and pointed to the possibility of gaining further insight from a less homogenous group of respondents. To supplement the results of this question, another data analysis, not contained in the original design of the research, was conducted. RESULTS AND ANALYSIS OF SURVEY QUESTION 15

The results of this question indicate an average score of 75.6%, which exceeds the ‘preponderance of the evidence’ test but falls short of the ‘clear and convincing evidence’ test. However, as this question was based upon the 26 words appearing most frequently in the PMBOK Guide, to help evaluate this question further, an analysis of the origins of the concepts most common to project management was also done, indicating that the body of knowledge generally ascribed to project management (PMBoK Guide, 2004; PRINCE2, 2004) does NOT meet the intrinsic requirement of a body of knowledge, that it is ‘esoteric1, abstruse2, complicated or secret’ (Table 5.2). As pointed out by Zwerman, Thomas et al. (2004) ‘professional status ultimately rests on the ability of practitioners to lay claim to a more or less exclusive command of an esoteric body of knowledge’ (p. 179). This is also consistent with Abbot (1988, p. 2). Despite concern summarized by Zwerman, Thomas et al. (2004) that debate exists as to whether a claim to a unique knowledge base is essential (pp. 39-40), as evidenced by the fact that the body of knowledge is the only attribute which appears on both traditional and non-traditional lists of attributes, which stands as evidence that consensus remains that control over a ‘unique, esoteric, secret or abstruse’ body of knowledge is an essential part of the professionalization process. Given that the body of knowledge which project managers have identified comes from within the larger body of knowledge associated with general management or business, there is no rational argument to support a claim that the body of knowledge of project management is or ever will be unique. This question was further elaborated by Morris, Crawford, Hodgson, Shepherd and Thomas in ‘Exploring the role of formal 1

Merriam Webster 3rd International Dictionary defines ‘esoteric’ to mean ‘designed for or understood by the specially initiated alone, that demand special training to be perceived and enjoyed, and its devotees form a cult’

2

Merriam Webster 3rd International Dictionary defines ‘abstruse’ to mean ‘difficult to comprehend or understand’

209 Chapter 5: Survey development and analysis, Part 2

bodies of knowledge in defining a profession – The case of project management’ (Morris et al., 2006) where they conclude that ‘it should not be the professional associations which define the body of knowledge, but researchers testing the concepts theoretically and practically, lest we end up with a self fulfilling prophecy’ (p. 719). Abbot (1988) invokes this principle when he states that ‘control of knowledge and its applications means dominating outsiders who attack that control’ (p. 2). Table 5.2

Source of the major elements of the project management BoK

Project management knowledge area (generic concepts)

Originator/author of the fundamental/ underlying theory

Technical discipline which produced the underlying theory

scientific management

F. Taylor

Industrial Engineering

systems thinking

Weiner/Boulding/Churchman/ Senge/Forrester/Sterman

Operations Research

contingency management

Burns/Stalker/Woodward

General Management

ethics

Weber

Human Resources/General Management

management functions

Fayol

Mining Engineering/ General Management

strategic planning (prevoyance)

Fayol

Mining Engineering/ General Management

planning/controlling/exe cuting

Fayol

Mining Engineering/ General Management

WBS

Fayol

Mining Engineering/ General Management

responsibility assignment matrix

Fayol

Mining Engineering/ General Management

projectized org structure

Fayol

Mining Engineering/ General Management

management by objective

Fayol

Mining Engineering/ General Management

MRP/JIT

Fayol

Mining Engineering/ General Management

communications

Mayo

Human Resources/General Management

motivating

Likert/Magregor/Mazlow/Hertzb erg/Vroom/Locke

Human Resources/Psychology/ General Management

teams/teamwork

Mayo/Mazlow/Likert/McGregor/ Argyris/McClelland

Human Relations/Psychology/ General Management

contracts/procurement

US Govt Procurement Legislation/Regulations

Government Sector FARS

earned value management

F. Taylor/F.& L. Gilbreth/H. Gantt/US Navy/Air Force Procurement

Industrial Engineering/ Government Sector

customer satisfaction

Deming

Statistician/Industrial Engineer

conformance to requirements

Crosby

General Management

fitness for use

Juran

Electrical Engineering/ Industrial Engineering

continuous improvement

Ishikawa

Applied Chemical Engineering

statistical process control charts /plan-docheck-act cycle/six sigma quality

Shewhart

Physicist

210 Chapter 5: Survey development and analysis, Part 2

Based on Table 5.2, given that the body of knowledge is not at all unique to project management, one can only conclude that project management is a subset or specific application of general management techniques. This lends credibility to the results of survey question 28, indicating that project management is a process, methodology or system and not a profession. It also lends credibility to Drucker for his apparent reluctance to make any claims what so ever that management of any type is a profession. However, even though this body of knowledge is clearly not unique to project management, the very fact that the underlying body of knowledge is so pervasive and used by so many different occupations, stands as prima facie evidence that it does meet the three criteria identified by Professor Turner, that: „

the underlying body of knowledge is based in theory;

„

it has been empirically tested;

„

it has been proven to be scientific and predicative.

5.1.5 Key question 5 How much experience/apprenticeship/education is associated with the term profession? Exhibit 5.5

Generation of survey questions 34, 35, 36 and 43 to answer key question 5 (Question details: Appendix A)

Key finding

Long period of training: a. higher education; b. apprenticeship, internship, residency

Question generated by the key finding

KQ5 How much experience/apprenticeship/education is associated with the term profession?

Hypothesis

A profession requires at least a 4 year degree AND 4 years of practical experience.

Survey question

SQ34 How many years of education beyond high school does it take to produce a professional practicing project manager? SQ35 How many years of university level education do you associate with the term ‘professional’? SQ36 How many years of work experience does it take to produce a professional project management practitioner? SQ43 How many years of apprenticeship, internship or supervised work experience do you associate with the term ‘profession’?

This hypothesis that a profession requires at least a four year degree AND four years of practical experience is grounded in the ‘bright line’ legal cases cited from Polelle (1999), Gawley (2002) and Rotunda (1997). In particular, Garden v Frier (1988) and Jilek v Berger Electric (1989) stand out as being indicative that experience alone is

211 Chapter 5: Survey development and analysis, Part 2

not sufficient to qualify a person as a professional. However, as can be seen from the airline pilot case study, demonstrated and proven experience is preferable to a degree. To resolve this apparent contradiction, the hypothesis was framed in the context that neither a degree alone nor experience alone is sufficient. In order to earn recognition as a profession, both elements must be present. A common attribute or trait common to nearly all definitions is the expectation that professions require a ‘long’ period of education and training. Based on the literature research, this attribute is often broken down into two parts: 1) formal education, usually at minimum four years beyond high school, but often longer. Polelle (1999) in particular identified several US State Supreme Court decisions establishing a four year education as one of the ‘bright line’ tests that jurists use to determine whether an occupation is or is not a profession. 2) Some form of supervised, ‘hands on’ training, apprenticeship, internship or experience-based element, designed to build competency. However, as indicated by Garden v Frier (1988), the Florida Supreme Court felt that apprenticeship alone without a four year degree did not qualify as being a profession. (Polelle, 1999). For the purposes of further discussion, restating the Merriam Webster Dictionary definition, competency is defined to mean ‘the quality or state of being functionally adequate, characterized by marked or sufficient aptitude + attitude + skills + strength + knowledge’ (as for a particular duty or in a particular respect). Results and analysis of survey question 34. Respondents to this question indicated somewhat decisively (26.3 + 16.9 + 31.1 = 74.3%) that four or more years of formal education beyond high school should be required to produce a competent professional project management practitioner (Figure 5.12). As with other comparisons, 74% exceeds the ‘preponderance of the evidence’ test (51%) but fails to meet the ‘clear and decisive’ test consistent with Pareto (1896-1897) (80%).

212 Chapter 5: Survey development and analysis, Part 2

Figure 5.12

Results, survey question 34

Results and analysis of survey question 43. Respondents to this question indicate fairly decisively (44.2 + 18.8 + 21 = 84%) that four or more years of formal Education beyond high school should be required to produce professional anything (Figure 5.13). The responses to this question do meet the ‘clear and decisive’ test. Based on these results, a project manager should require a minimum four years of formal education beyond high school.

Figure 5.13

Results, survey question 43

213 Chapter 5: Survey development and analysis, Part 2

Results and analysis of survey question 35. Respondents to this question also indicated nearly as decisively (20.2 + 21.5 + 40.5 = 82.2%) that three or more years of working experience beyond high school should be required to produce a project manager (Figure 5.14). Again, as this data meets the ‘80%’ test, that a project manager should be required or expected to serve some sort of apprenticeship, internship or mentored work experience.

Figure 5.14

Results, survey question 35

Results and analysis of survey question 36. Respondents to this question indicated quite decisively (25.9 + 20.2 + 12.1 + 18.1= 76.3%) that three or more years of formal education beyond high school should be required to produce a competent practitioner (Figure 5.15).

Figure 5.15

Results, survey question 36

214 Chapter 5: Survey development and analysis, Part 2

Discussion of results. The results of the survey indicate a fairly high degree of consensus, and statistical analysis between the various demographic sub-populations showed a substantial consistency across all demographic combinations and permutations. With the exception of minor changes in weighting, no significant differences of opinion were recognized. The survey results for this question indicate quite clearly that nearly 85% of practitioners believe that a four year degree or better is the mark of a ‘profession’, and that more than 40% believe five or more years of university level education is necessary for an occupation to be considered a ‘profession’. Given the current requirements to sit for the PMP is only a four year degree or less, the results of this survey may have profound implications for PMI and similar organizations offering knowledge based credentialing programs. By comparing the two questions above, I found the respondents sending a very clear and consistent message that a four year degree or better is the minimum requirements to have project management recognized as a profession (73%) and 48% saying more than four years is necessary to produce a professional project manager. This further supports the argument that current minimum education thresholds to sit for the PMP exam are insufficient in the eyes of the global practitioner. Yet I see a discrepancy between the experience levels practitioners believe necessary to produce a competent project manager and the basic requirements established by PMI. Currently, PMI requires 7,500 hours of experience spread over not less than five years, nor more than eight years, for anyone holding less than a four year degree, and for those holding a four year degree, the requirement is 4,500 hours spread over not less than three nor more than six years. According to the input from the survey respondents, ~40% believe the minimum should be more than five years and 60% say four years or more. By contrast, the more technically demanding Certified Cost Engineer (CCE) credential offered through the Association for the Advancement of Cost Engineering International www.aacei.org requires eight full years experience for less than a four year technical degree and four full years experience for holders of a four year technical degree. Thus it would appear as though the AACE requirement better match with what a global cross section of practitioners deem appropriate.

215 Chapter 5: Survey development and analysis, Part 2

However, to reiterate, based on Garden v Frier (1988) and to a lesser degree, Jilek v Berger Electric (1989), having the alternate of substituting years of experience in lieu of a degree will certainly jeopardize any claims that project management is a profession. (Polelle, 1999) at least from the important legal perspective. However, recognizing that PMIs PMP, AACE’s CCE, AICs CPM and CMAAs CCP and similar credentials are knowledge based, raises further serious questions about the professionalization of project management. Virtually ALL other professions as well as the more technically demanding trades, (especially those directly related to the health or safety of the consuming public, such as electricians or plumbers) require some form of apprenticeship, internship or residency, conducted under the guise and mentorship of a qualified and reputable ‘master’ practitioner (often, with the master’s license being on the line in the event one of the apprentices, through ordinary negligence, causes harm). The use of some sort of demonstrated competency is perhaps one of the most widespread requirements before an occupation can be deemed a profession. This is consistent with the approach IPMA and AIPM have taken and further supports by the more recent GAPPS initiatives. Beyond the scope of this research but certainly a topic for further research is the nature of what kinds of approach would be better in determining competency. According to Pew Research (Finnocio et al., 2002) and US Federal Trade Commission’s Bureau of Economics, (Cox, 1990) independent boards consisting of at least 50% lay people (consuming public) should sit on these boards. 5.1.6 Key question 6 Is project management a calling? The next attribute or trait common to most professions is a lifetime commitment. While implicitly acknowledged by all researchers, Schein (1972, p. 19) specifically identified a profession as one in which the practitioners ‘have a strong motivation’ and a ‘lifetime commitment’.

216 Chapter 5: Survey development and analysis, Part 2

Exhibit 5.6

Generation of survey questions 16, 20 and 24 to answer key question 6 (Question details: Appendix A)

Key finding

Life time commitment

Question generated by the key finding

KQ6 Is project management a calling?

Hypothesis

The term ‘accidental profession’ commonly applied to project management indicates that project management is not a profession.

Survey questions

SQ16 I consider project management to be my life’s work. SQ20 I relate more to being a project manager than I do to working for any particular employer. SQ24 When I introduce myself to people, I would be more likely to do so by stating: ‘I am a project manager who works for XYZ company’ OR ‘I work for XYZ company as a project manager’.

Likewise the Merriam Webster definition, restated: A strong drive or desire to do the kind of work normally associated with [project management] [Or insert your chosen occupation], by devoting considerable time and effort, over a long period of time, to master as much information, facts, truths, and principles as possible, by making a deliberate and concerted effort to develop, through training, experience, and by taking specialized adult education courses, over a life time spent as a [project manager], those systematic techniques to be able to execute the work of a [project manager] in a sound, business-like manner, by adhering to above average standards of ethical, moral and legal care, as defined by an organized group of practitioners with shared aims, the primary one being serving the interests of the general public. (Merriam-Webster, 2003)

Given the investment in both time and money necessary to prepare oneself to practice in most professions is quite high, the tendency for people to choose carefully before making such an investment results in a relatively high satisfaction level, as well as a willingness to stay with your chosen profession once achieved. This has particular relevance to project management, as it has developed a reputation as being ‘the accidental profession’ (Pinto & Kharbanda, 1995). Results of survey questions 16, 20 and 24. Whether the perception comes about because project management offers such a low barrier to entry, because of a lack of clear understanding by senior managers about the qualifications necessary to effectively manage projects or some other reason is unknown. However as responses questions 16, 20 and 24 show, there are considerable differences of opinion regarding lifetime commitment to the occupation of project management. These questions were not complicated and were designed to eliminate people providing what they believe to be the ‘right’ or ‘best’ answer. The simplest and most

217 Chapter 5: Survey development and analysis, Part 2

forthright approach was to ask the questions in a straightforward way, in the assumption that respondents would be honest in their responses. While the average score for survey question 20 was 77.8%, those scoring > Not Sure averaged only 70.45%. Taken together, this does not send a solid message that project managers consider their work a lifetime commitment, at least not in the context of other occupations. Statistics on most professionals who decide for one reason or another to ‘retire’ from a profession in medicine or law are thin, but two studies give us some indication from which we can draw some comparisons. Alan Mascarenhas (2006), writing in the Sydney Morning Herald on 7 September, quoted a survey conducted by the Australian Financial Review stating that ‘45 per cent of young lawyers surveyed confessed they wanted to quit their job within two years, with 9 per cent planning to quit law altogether’. This fits rather closely with another survey on medicine, conducted by the California Medical Association, which calculated that ‘10.2% of physicians were considering taking early retirement and moving on to other careers’ (Los Angeles Business Journal, 2001, online). As can be seen from the actual responses (Figure 5.16, 5.17 and 5.18), the messages are mixed. On one hand, some 73% consider project management to be their life’s work. And approximately the same percentage say they relate more to being a project manager than they do working for any particular employer. Question 24 may well give a more objective, if somewhat oblique picture of how most project managers perceive themselves. Again, research on this subject is sorely lacking, but based on the engineers’ obligation, opening with ‘I am an engineer’ is strongly indicative of the thinking of professionals. ‘I am a project manager’, therefore, was an answer to survey question 24 that was strongly indicative about the respondents felt about their occupation. When asked a question about how they would introduce themselves, 57% responded in a manner indicating they relate more to a company than to their selected vocation.

218 Chapter 5: Survey development and analysis, Part 2

Figure 5.16

Results, survey question 16

Figure 5.17

Results, survey question 20

Figure 5.18

Results, survey question 24

219 Chapter 5: Survey development and analysis, Part 2

5.1.7 Key question 7 How do project managers perceive the practice of individual ethics? Exhibit 5.7

Generation of survey question 27 to answer key question 7 (Question details: Appendix A)

Key finding

Adhering to a code of ethics

Question generated by the key finding

KQ7 How do project managers perceive the practice of individual ethics?

Hypothesis

Project managers have professional level ethics.

Survey question

SQ27 People who use my services as a project manager are not concerned about how I execute the project as long as the project is successful (using their definition of success).

The question of ethics is a hot one today. Global Crossing, ENRON, Parmalat and a myriad of other relatively recent scandals have rocked the legal and accounting professions, resulting in laws being passed mandating tighter accountability. Laws such as Sarbanes Oxley and BASIL2 have been passed primarily because the responsibility of professionals to police their own in exchange for limited anti-trust protection has been eroded to such a point that the government has had to step in and assume that responsibility. A true profession maintains, as part of their fiduciary obligation to the consuming public, the implied promise that they will put the good of the consuming public over their own interests or those of their company. To test this hypothesis, I tried to phrase question 27 in a way that didn’t encourage an obvious answer. Consistent with the principle of ‘clear and convincing’ evidence, and a ‘tipping point’ of 80%, which is consistent with Pareto (1896-97), anything less than a score close to 80% positive for question 27 would be cause for concern (Figure 5.19).

Figure 5.19

Results, survey question 27

220 Chapter 5: Survey development and analysis, Part 2

56% of the respondents (those scoring > Not Sure or higher) felt that it didn’t matter how they achieved the end result, as long as they satisfied the client or their sponsor’s definition of success, and on average, 65% felt the clients didn’t care. This is a disturbing number, for in today’s post Enron, Millennium Development Goals world, ethics and clean, transparent governance are or should be the rule. This is supported by the proliferation of corporate social responsibility (CSR), which the World Business Council for Sustainable Development in its publication Making good business sense by Lord Holme and Phil Watts (2002), used the following definition: Corporate social responsibility is the continuing commitment by business to behave ethically and contribute to economic development while improving the quality of life of the workforce and their families as well as of the local community and society at large. (p. 9)

The key aspect of codes of ethics is whether they actually are perceived to be focused on protecting the consuming public, or are they being used to keep members in line. As this is a credibility issue, this researcher looked to established professions such as those represented by the American Society of Civil Engineers (ASCE) as being a leader in this area. ASCE’s recent adoption of ‘zero tolerance’ for bribes and corruption stands as a model for other organizations to follow. This is particularly important as engineered projects play such a vital role in achieving the millennium development goals for the developing nations. Therefore, recommendations from this research are that organizations representing project managers adopt the same ‘zero tolerance’ for bribes and corruption that ASCE advocates. Essential to the credibility of the CoE is the perception that the primary focus is on protecting the health, safety and welfare of the consuming public. In the case of airline pilots and construction managers, this translates directly into physical safety. However, to other users of project management as a delivery method, it also means the financial safety, health and welfare by the practitioners towards their consuming public. This is effectively the intent of Sarbanes-Oxley (2002), passed by the US Federal government. Sarbanes-Oxley (2002) has relevance for project managers (Kliem, 2006), as the level of accountability for fraud extends to those in the organization who execute projects. The burden on the project manager to, in the event they discover fraud, bring it to the attention of the appropriate manager, and if the issue is not addressed, to ‘blow the whistle’. The EU and most other nations have passed similar, if not more stringent requirements. 221 Chapter 5: Survey development and analysis, Part 2

5.1.8 Key question 8 How do project managers compare against other professionals in terms of pro bono work? Exhibit 5.8

Generation of survey question 29 to answer key question 8 (Question details: Appendix A)

Key finding

Service to the public (pro bono work)

Question generated by the key finding

KQ8 How do project managers compare against other professionals in terms of pro bono work?

Hypothesis

Project managers meet or exceed the total number of person hours donated by established professions.

Survey question

SQ29 During the past year, I have donated the following number of hours performing voluntary, uncompensated community service requiring the use of my project management skills

In Gawley’s (2002) Protecting professionals from competition he made the point very clearly that one of the differences between ‘a professional and a merchant is the professional does pro bono work’ (Lexis/Nexus, online). This position is supported by American Bar Association (ABA) Model Rule 6.1 which states that: Every lawyer has a professional responsibility to provide legal services to those unable to pay. A lawyer should aspire to render at least (50) hours of pro bono public legal services per year. In fulfilling this responsibility, the lawyer should: provide a substantial majority of the (50) hours of legal services without fee or expectation of fee to persons of limited means or charitable, religious, civic, community, governmental and educational organizations in matters which are designed primarily to address the needs of persons of limited means; and provide any additional services through delivery of legal services at no fee or substantially reduced fee to individuals, groups or organizations seeking to secure or protect civil rights, civil liberties or public rights, or charitable, religious, civic, community, governmental and educational organizations in matters in furtherance of their organizational purposes, where the payment of standard legal fees would significantly deplete the organization’s economic resources or would be otherwise inappropriate; delivery of legal services at a substantially reduced fee to persons of limited means; or participation in activities for improving the law, the legal system or the legal profession. In addition, a lawyer should voluntarily contribute financial support to organizations that provide legal services to persons of limited means. (ABA, online, 2003)

While medicine is not quite as regulated as law, very clearly pro bono work plays an important role in medicine. To test this hypothesis, the survey question was constructed to have respondents identify the number of hours they donate pro bono, and compare that against literature research for other occupations recognized as professions, in this case, law and medicine (Figure 5.20).

222 Chapter 5: Survey development and analysis, Part 2

Figure 5.20

Results, survey question 29

The response to this question from some 400 plus practitioners indicates that 63% of project managers dedicate 20 hours or less of their time to pro bono work. Compare this against a survey done for the American Lawyer. A cursory examination of information included in 1998 firm performance, as reported in the July, 1999 issue of the American Lawyer appears to reaffirm the conclusions of the Galanter/Palay (1992) study. According to that information, lawyers at the nation’s most profitable and largest law firms, on average, donated 40.8 hours of pro bono service during the past year3. As can be seen from the survey data, only 20.2% of project managers donated 41 or more hours, leaving 79.8% who donated less. This is not consistent with either law or medicine4, nor is it consistent with a common attribute of professions. Given that review of the literature consistently identifies commitment to the consuming public as a prerequisite to being considered a professional, and given that only ASCE, being but one of the professional organizations representing project managers specifically address the issue of pro bono work and given only 20.2% of project managers approach the minimum requirements outlined in ABA’s Model Rule 6.1, in the area of pro bono work, project management is sorely deficient in terms of meeting the standard for a profession.

3

Lardent, Esther F., Law Firm Pro Bono Project © 2000 The Pro Bono Institute ‘Making a Business Case for Pro Bono Work’ http://www.probonoinst.org/pdfs/businesscase.pdf

4

Anecdotal information regarding planning Community Service Projects by the AMA http://www.ama-assn.org/ama/pub/category/8650.html

223 Chapter 5: Survey development and analysis, Part 2

5.1.9 Key question 9 How do organizations representing practitioners compare against other organizations in terms of ethics? Exhibit 5.9

Key question 9 (Question details: Appendix A) Professional association: a. enforces code of ethics b. establishes acceptable standards i. performance standards ii. procedural standards (methodology)

Key finding

Question generated by the key finding

KQ9 (original) How do organizations representing practitioners compare against other organizations in terms of ethics?

This attribute was ignored for the purposes of this research, as the question of knowledge based standards (test) vs competency based standards is a study unto itself and is being researched by others.

KQ9 (substitute question) If a robust code of ethics plays a key role in determining a profession, how do various CoEs applicable to project management compare against accepted professions?

Hypotheses

There is a difference between the codes of ethics/codes of conduct of organizations which are recognized as professions from those which are not. A code of ethics adopted for project managers which clearly focuses on the health, safety and welfare of the consuming public will be a clear indication that project management is a profession.

Survey question

Not a test question; investigated through key word analysis of professional society codes of ethics

INVESTIGATING CHAPTER 9: KEY WORD ANALYSIS

The role ethics plays in defining a profession is consistently important, whether the research is a semanticist, economist, legalist, historian or sociologist. Given it plays such a pivotal role in defining a profession, this research analysis was designed to compare codes of ethics from a variety of different organizations including aircraft piloting and construction management, against codes of ethics from those organizations directly or indirectly representing project management. Testing the ‘strength’ of the various codes of conduct used by those organizations most often representing project managers was carried out to see if there was any correlation between the various key words and if so, was it clear, implied or nonexistent. The organizations selected were the International Council on Systems Engineering (INCOSE)5; American Society of Civil Engineers (ASCE)6; Australian Institute of project managers (AIPM)7; Institute of Electrical and Electronics 5

See www.incose.org

6

See www.asce.org

7

See www.aipm.com.au

224 Chapter 5: Survey development and analysis, Part 2

Engineers (IEEE)8; Construction Management Association of America (CMAA)9; Association for the Advancement of Cost Engineering International (AACEi)10; American Institute of Constructors (AIC)11; American Institute of Architects (AIA)12; project management Institute (PMI)13; Airline Pilots Association (ALPA)14. Table 5.3 indicates the key words and their frequency and use in the various codes while Table 5.4 outlines key words as a percentage of the words in the code of ethics. Table 5.3

Frequency of key words as a percentage of total words in the code of ethics

Key words Phrases

AIC

AIA

PMI

ethics or ethical

INCOSE ASCE AIPM IEEE CMAA AACE 8

8

2

2

3

2

0

15

13

ALPA Average Rank 2

5.5

1

safety, health, welfare

2

18

3

3

1

9

0

3

0

10

4.9

2

public interest

2

18

5

0

1

10

1

2

1

1

4.1

3

skill, knowledge

3

6

2

1

1

5

0

11

6

4

3.9

4

honest

2

5

2

2

1

3

0

4

13

0

3.2

5

conflict of interest

3

3

1

1

3

2

0

1

18

0

3.2

6

competence

1

7

0

1

2

2

0

4

1

1

1.9

7

objective or truthful

2

7

1

0

2

2

0

3

2

0

1.9

8

professional reputation

0

3

0

0

1

1

3

2

6

3

1.9

9

integrity

3

4

1

0

3

1

0

5

1

0

1.8

10

responsibility

1

0

2

1

0

1

0

1

7

4

1.7

11

fiduciary, agent, trustee

0

6

2

0

0

5

0

0

0

0

1.3

12

kickback, bribe, gratuity

0

10

0

1

0

0

0

0

2

0

1.3

13

dignified or modest

0

7

1

0

0

5

0

0

0

0

1.3

14

confidential

0

1

0

0

1

4

1

4

2

0

1.3

15

sustainable development

0

9

0

0

0

0

0

0

0

0

0.9

16

corrupt

0

6

0

0

0

0

0

0

1

0

0.7

17

self or personal interest

0

1

0

0

0

0

0

0

2

0

0.3

18

whistle blower

0

0

0

0

0

0

0

0

1

0

0.1

19

governance

0

0

0

0

0

0

0

0

1

0

0.1

20

total occurrences

27

119

22

12

19

52

5

55

77

25

41.3

2199

466

268

571

1661

174

2639

3946

1407

1373.7

total key words in doc

406

key words/total words

6.65%

ranking profession ?

5.41% 4.72% 4.48% 3.33% 3.13% 2.87% 2.08% 1.95% 1.78%

1

2

3

4

5

6

7

8

9

10

Discipline

Yes

?

Yes

No

No

No

Yes

?

Yes

8

See www.ieee.org

9

See www.cmaanet.org

10

See www.aacei.org

11

See www.aicnet.org

12

See www.aia.org

13

See www.pmi.org

14

See www.alpa.org

225 Chapter 5: Survey development and analysis, Part 2

3.64%

Table 5.4

Key words as a percentage of total words in codes of ethics

Key words phrases

AAC E

AIA

AIC

AIPM

ALPA

ASCE

CMA A

IEEE

PMI

INCOSE

Average

safety, health, welfare

9

3

0

3

10

18

1

3

0

2

4.9

fiduciary, agent, trustee

5

0

0

2

0

6

0

0

0

0

1.3

ethics or ethical

2

15

0

2

2

8

3

2

13

8

5.5

honest

3

4

0

2

0

5

1

2

13

2

3.2

whistle blower

0

0

0

0

0

0

0

0

1

0

0.1

corrupt

0

0

0

0

0

6

0

0

1

0

0.7

kickback, bribe, gratuity

0

0

0

0

0

10

0

1

2

0

1.3

dignified or modest

5

0

0

1

0

7

0

0

0

0

1.3

conflict of interest

2

1

0

1

0

3

3

1

18

3

3.2

public interest

10

2

1

5

1

18

1

0

1

2

4.1

governance

0

0

0

0

0

0

0

0

1

0

0.1

integrity

1

5

0

1

0

4

3

0

1

3

1.8

confidential

4

4

1

0

0

1

1

0

2

0

1.3

self or personal interest

0

0

0

0

0

1

0

0

2

0

0.3

competence

2

4

0

0

1

7

2

1

1

1

1.9

objective or truthful

2

3

0

1

0

7

2

0

2

2

1.9

sustainable development

0

0

0

0

0

9

0

0

0

0

0.9

professional reputation

1

2

3

0

3

3

1

0

6

0

1.9

skill, knowledge

5

11

0

2

4

6

1

1

6

3

3.9

total occurrences

51

54

5

20

21

119

19

11

70

26

39.6

total key words in doc

1661

2639

174

466

1407

2199

571

268

3946

406

1373.7

key words/total words

3.07 %

2.05 %

2.87 %

4.29 %

1.49 %

5.41 %

3.33 %

4.10 %

1.77 %

6.40%

3.48%

profession ?

?

Yes

No

?

Yes

Yes

No

Yes

?

No

INVESTIGATING KEY QUESTION 9: LIMITATIONS AND OBSERVATIONS

At the time the survey instrument was being developed, it was assumed that a cross section of respondents from outside of the field of project management would be responding. While ultimately, there was not a good response from people who did not hold the job title of project manager, the broad cross section of respondents from 226 Chapter 5: Survey development and analysis, Part 2

areas outside of project management (specifically, medicine, law, education, commercial aircraft piloting, real estate development and banking) that was being anticipated did not eventuate. Of the ~400 respondents, only ten were clearly from outside the traditional world of project management, and there was nothing statistically significant to deduce anything meaningful from their responses. In relation to key question 9, it was hypothesized that a code of ethics which for project managers clearly focused on the health, safety and welfare of the consuming public would be an indication that project management was a profession. Although the survey responses did not reveal the extent to which a code of ethics influenced professional behaviours, having code of ethics was the top ranked attribute of professionalization. It was anticipated, therefore, that there would at least be certain key words used by professional organizations which could serve as indicators of those organizations representing occupations already considered professions. While this investigation proved promising, and some interesting results came out of it, having neither the appropriate data mining software nor expertise in using such software, this part of the research, which was not central to the project, must be considered interesting and informative, but incomplete and inconclusive. As can be seen from Table 5.3 there is no correlation between the total number of key words contained in the code of ethics and whether or not the occupation is perceived to be a profession. Statistical analysis of the data indicated no strong correlation between any two or more key words and the perceived professionalism of the underlying occupation. Although this was a disappointing result at odds with the semantic theory at the base of this investigation, further research is required to prove or disprove the hypothesis that in the wording and phraseology of an occupation lies a predictor of its perception as a profession. Organizational bodies and standards. Along with a unique, esoteric, complicated, secret or abstruse body of knowledge, perhaps the second most important ‘pillar’ supporting the professionalization of an occupation is the requirement that practitioners be represented by an organization. While these organizations can take several forms, the most common are:

227 Chapter 5: Survey development and analysis, Part 2

„

unions or other collective bargaining agents, which most closely resemble the guilds from which the professions evolved, with most members being self employed or employed temporarily (i.e. carpenters, electricians, plumbers, barbers & beauticians)

„

union or other collective bargaining agents, with most members being full time, ‘permanent’ employees of a single institution or organization (i.e. nurses, teachers, airline pilots)

„

voluntary organizations representing self employed practitioners working on a ‘fee for services’ basis. This group represents the traditional ‘professions’ (i.e. doctors, lawyers, architects, engineers, land surveyors)

„

voluntary organizations representing practitioners who are full time, ‘permanent’ employees. This group is much like that above, but without the organization playing a role as collective bargaining agent. The group includes many of today’s ‘knowledge workers’ [i.e software developers, chief knowledge officers, (CKO); Chief Information Officers, (CIO)].

However, these worlds are beginning to blur, especially between groups latter three groups, as doctors start to join HMOs and work not on a fee for service basis, but as employees. Architects and engineers also are less likely to ‘hang out their shingle’, opting instead to go to work for a large EPC type construction companies (i.e Fluor Daniel, Bechtel, Bouyges, Saipem). These professional organizations serve three primary functions: „

They enforce the code of ethics.

„

They are responsible for setting standards (best practices of the profession vs. normal and customary practices)

„

They are generally responsible to develop the methodologies, processes or procedures or, as in the case of the aviation, provide input to proposed regulations, policies and procedures.

The responsibility of the professional organization to set standards is considerably more complicated. Firstly, there are generally recognized to be two general postures to standards which an organization can assume: best practices of the profession and normal and customary practices of the profession. 228 Chapter 5: Survey development and analysis, Part 2

Why is it important? In legal terms, which standard an organization adopts determines the criteria against which the practitioners will be judged. This is known as the standard of care and is defined by Blacks law dictionary (2005) as: standard of care n. the watchfulness, attention, caution and prudence that a reasonable person in the circumstances would exercise. If a person’s actions do not meet this standard of care, then his/her acts fail to meet the duty of care which all people (supposedly) have toward others. Failure to meet the standard is negligence, and any damages resulting there from may be claimed in a lawsuit by the injured party. The problem is that the ‘standard’ is often a subjective issue upon which reasonable people can differ. (See: negligence, duty of care). (p. 1447)

Setting the standard of care becomes important to the development of a profession, as it determines the legal bar to prove negligence. Obviously, best practices clearly establishes a much higher standard than normal and customary practices. PMI has opted to take the route of normal and customary practices, as the PMBOK Guide states: ‘The primary purpose of the PMBOK Guide is to identify the subset of knowledge that is ‘generally recognized’ as ‘good practice’, with ‘generally recognized’ further defined as ‘knowledge and practices applicable to most projects most of the time and that there is widespread consensus about their value and usefulness’ (PMBOK, 2004). To counter potential criticism that the best practices of today would naturally become the standard practice once institutionalized (personal communication), assuming the objective of any professional organization is continuous improvement, the best practices would not be a static standard, but an ever improving one. The legal implications of this choice of words is significant, for if project management is to be perceived as a profession, are normal and customary standards appropriate for an evolving profession, or would it be better to upgrade the image by adopting best practices and not merely normal and customary ones? This is an important decision, and one which should be clearly explained to the member/owners of all professional organizations in order that they might make an informed decision, recognizing that in adopting best practice they will be held to a higher standard of performance.

229 Chapter 5: Survey development and analysis, Part 2

While the concept of legal care is evident throughout the writings of Polelle (1999), Gawley (2002), Rotunda (1997); and legal definitions exist in Black’s law dictionary and Nolo online legal dictionary, Wikipedia still provides the best summarized definition: In tort law, a duty of care is a legal obligation imposed on an individual requiring that they exercise a reasonable standard of care while performing any acts that could result in foreseeable harm to others. For an action in negligence, there must be an identified duty of care in law. Duty of care may be considered a formalization of the implicit responsibilities held by an individual towards another individual within society. It is not a requirement that a duty of care be defined by law, but it will often evolve through the jurisprudence of common law. Individuals who are considered to be professionals within society are often held to a higher standard of care than those who are not. Engineers and doctors will be held to reasonable standards for members of their profession, (as determined by their peers) rather than those of the general public in cases related to their fields. (Which are judged by the ‘reasonable’ or ‘ordinary’ man standard) Breach of duty of care, if resulting in an injury, may subject an individual to liability in tort. Duty of care is an important prerequisite in the tort of negligence, as the duty of care must exist and must have been breached for the tort to occur. (Wikipedia, online, n.d.)

Lastly, the professional organization is responsible for creating and maintaining the ‘methodologies, processes or procedures’ which make up or constitute the normal and customary or best practices. Methodology, process and/or procedures in project management. While any project management body of knowledge, done correctly, would be hard pressed to qualify as being ‘esoteric, complicated, secret or abstruse’, perhaps the real challenge which will prove to be facing the professionalization of project management lies in the occupation’s methodology, process and/or procedures. Gartner (2002), Standish (2004), FMI (2005) and other credible sources cite project failure rates of anywhere between 20% to 60% or more. To clarify the results of the Chaos Report (Standish, 1994) projects were categorized by how they were concluded:

230 Chapter 5: Survey development and analysis, Part 2

Resolution Type 1, or project success: The project is completed on-time and on-budget, with all features and functions as initially specified. Resolution Type 2, or project challenged: The project is completed and operational but over-budget, over the time estimate, and offers fewer features and functions than originally specified. Resolution Type 3, or project impaired: The project is cancelled at some point during the development cycle.

Overall, the success rate was only 16.2%, while challenged projects accounted for 52.7%, and impaired (cancelled) for 31.1%. (Standish, 1994). Based on this rather ‘easy’ test for project success, there is something radically wrong with the processes being used for project management. Key amongst them being the lack of a competent and consistent project life span methodology (personal communication]. Up until recently, most processes have been based on the scientific method, assuming linearity exists between inputs and outputs, modeling project management very much like an assembly line in an auto factory. Put the right components in the right places at the right times, and finished products will roll off the assembly line at the end, within budget, on time and substantially meeting the technical specifications and any modifications ordered by the ultimate owner. However, this is not what practitioners find happening, especially in the knowledge society. What practitioners have are processes in which the end customer often does not have a complete idea of what it is they want, except in the vaguest terms. In many instances, what the client seeks has never been done before. Under these circumstances, practitioners cannot use the scientific method. What is needed are post methodologies, which are variously known as complex dynamic systems or complex adaptive systems. Stated another way, in the post modernist world we live in, a project becomes a living beast. We can monitor it and we can control it, but because of the feedback loops continually acting to change the behavior, the outcomes and the processes, what appeared to be a sound decision today may cause the project to fail six or 12 months from now. To address this issue, one need only to look more closely at the work of Dr. John Sterman, Jay Forrester, Peter Senge et al. from MIT’s Sloan School of Management, or the work of the Santa Fe Institute15. It appears that the professionalization of

15

www.santafeinstitute.edu

231 Chapter 5: Survey development and analysis, Part 2

project management is being held back not only by a body of knowledge which is fragmented, inconsistent and purloined from so many different sources, but by the lack of a methodology(s) which can consistently deliver projects on time, within budget and substantially meeting stakeholder expectations. Perhaps this is an impossible expectation, given our current state of knowledge. However, the solution more than likely lies in developing a better understanding of complex, adaptive, dynamic systems. Whether this will come from project management organizations or whether ultimately the systems engineering societies will eventually take over is the subject of conjecture, but the issue of the methodology is at the very heart of the professionalization of project management. To put this in perspective, based on the Chaos Report (2004) and FMI’s 2006 Project Management Survey of the Construction Industry16, it would appear as though project management is today where medicine was in the 1700s – An emerging or evolving profession based as much on experimentation, luck and good fortune as on sound methods, procedures and practices (Standish, 1994; FMI, 2006). The Institute of Management (UK) was founded in 1900, and only earned its Royal Charter in 2002. It would seem that it took 100+ years to earn sufficient respect and gravitas to be recognized as a profession. Project management may still have along row to hoe. The problem of bias. In terms of the survey instrument, there seemed to be no way to phrase the ethics question in a manner which would provide meaningful results, as these types of questions, when posed to a population of respondents, will almost surely result in biased answers. And in many cases, the practitioner may not even be familiar with the professional code of ethics of the organization to which he or she belongs. To get around this problem, firstly, the professional organizations were asked to share their ethical violations statistics. While several were very open and cooperative, most did not respond at all. To circumvent this lack of data, key words from a broad variety of professional organizations were analyzed, some of which represented known and accepted professions and others which did not. Professional organizations whose members were likely to be involved in organizations who used project management as a delivery system were favoured.

16

http://www.fminet.com/Research/ResearchMethod/index.jsp

232 Chapter 5: Survey development and analysis, Part 2

To evaluate the enforcement of the codes of ethics, letters (sent via email) were written to each of the four major organizations representing project managers, from which responses were received: PMI; IPMA; AIPM; AACE; and three of the major organizations representing construction project management, from which no response was received: ACI; CMAA; RICS. Each organization was polled to find out: „

„ „

„ „

How many ethics cases are heard each year, going back as long as records have been kept? Is the rate of ethics cases rising faster than or less than the membership growth? How many members have been expelled from the organization due to ethics violations? How many certifications have been rescinded due to ethics violations. How many of the ethics violations were due to infractions of the code of ethics relating to actions within the organization vs. how many were for unethical/unprofessional actions of members coming as a result of complaints from the consuming public?

The results from this direct survey were indicative that the organizations representing project managers do not aggressively enforce the codes of ethics. A second analysis was conducted using key word search technology, to see if a comparison could be made between key words used by those organizations which Are considered or accepted as being professions compared to those which are evolving or claiming to be professions. This approach proved no more fruitful in answering this question than using the survey or direct questioning of the organizations directly. In summary. Key Question 9 addressed the original question, How do organizations representing practitioners compare against other organizations in terms of ethics? with a follow on question, If a robust code of ethics plays a key role in determining a profession, how do various CoEs applicable to project management compare against accepted professions? While all the major organizations representing project management have codes of ethics, it appears as though the enforcement of codes of ethics by these professional

233 Chapter 5: Survey development and analysis, Part 2

organizations is weak at best, and when the codes are enforced, it has more to do with controlling the membership to conform to the organizational standards than anything to do with protecting the consuming public against fraud, charlatans or incompetence. Given that, based on the results illustrated in Figure 5.11, respondents to the survey ranked adhering to a code of ethics as being the highest rated attribute to practitioners of project management, it seems there is a disconnect between what practitioners expect from their professional organizations and what the professional organizations are delivering. Unfortunately, the results of Key Question 9 are a strong indication that Haga’s (1974) posit that the primary role of the professional organization is to intimidate clients, employers, governments is correct 5.1.10 Key question 10 Do project managers publish papers in peer reviewed journals? Exhibit 5.10 Generation of survey question 32 to answer key question 10 (Question details: Appendix A) Key finding

Publishing in learned journals

Question generated by the key finding

KQ10 Do project managers publish papers in peer reviewed journals?

Hypotheses

If project management is a profession, then project managers publish in peer reviewed journals

Survey question

SQ32 During the previous 3 years, I have written and published the following number of articles in professional journals/publications.

An attribute implicitly associated with the need to create a unique body of knowledge is the need to have a reputable peer reviewed journal or technical publication as the vehicle to continually develop and refine that knowledge base. As publishing in peer-reviewed journals is the basis for creating the unique body of knowledge, it certainly is worth exploring the perceptions of today’s practitioners, if for no other reason than to determine if they realize and recognize the importance of this professional attribute to the on-going professionalization of their occupation. This concept is firmly embedded in professional development, particularly in scientific, engineering, medical and legal professions: that a member of a profession has an obligation or responsibility to invest in the profession’s continued development and success. ‘Giving back to the profession’, is an important concept, one that can

234 Chapter 5: Survey development and analysis, Part 2

best be accomplished through formal education and training as well as community education initiatives that attract and mentor the next generation of apprentices or interns in the field. Some societies, such as AACE and ASCE incorporate such activity in the codes of ethics belonging to the profession. For example, the American College of Clinical Engineering describes clinical engineers as follows: Clinical Engineers, by education and training, are members of a proud profession. Through their professional society, the American College of Clinical Engineering (ACCE), they have established a code of ethics, pursued peer recognition and certification programs, and developed a heritage of publishing, teaching and humanitarian programs designed to ‘give back’ to society and to prepare the next generation of practitioners. (Clinical Engineers, 2006)

Likewise, the prestigious Carnegie Foundation’s Initiative on the Doctorate states that ‘

professional stewardship involves educating and preparing those to whom the vigor,

quality, and integrity of the field can be entrusted’ (Carnegie Foundation, 2006, p. 12). Based on the literature research cited above, and in other parts of this paper, it is clear that professionals are expected to publish papers in peer reviewed journals. It seems logical that if project managers published in peer reviewed journals, it would lend prima facie evidence supporting that project management too was a profession. To test this hypothesis, a survey question was developed which enabled the respondent to indicate how many papers he/she published in peer reviewed journals. Responses could be compared to the levels of publications for members of recognized professions. As the numbers show, project managers (72%) don’t publish their technical work. Lack of publication – the building and sharing of a body of knowledge – may prove to be an indicator of the relative maturity of the professionalization process. As noted in previous sections, project management has not offered new insights into the conduct of the activity in more than 20 years. When compared against advances made in existing professions such as law, engineering, science and medicine, it certainly raises some important doubts about project management qualifying as a profession (Figure 5.21).

235 Chapter 5: Survey development and analysis, Part 2

Figure 5.21

Results, survey question 32

5.1.11 Key question 11 Does advertising by project managers conform to other professional norms? Exhibit 5.11

No survey question for KQ11

Key finding

Advertising not permitted or restricted

Question generated by the key finding

KQ11 Does advertising by project managers conform to other professional norms?

Hypotheses

As few project managers work on a ‘fee for services basis’ this question is irrelevant.

Survey question

N/A

The issue of restricted advertising appears in most definitions of a profession, especially those relating to law and medicine. In particular, MIT’s Edgar Schein In most contexts, advertising relates either to ethical practices and/or image of the profession (Schein, 1972) lists ‘limited or not allowed to seek out clients through advertising’ (p. 21) specifically as number 10 of the 10 criteria he uses in defining a profession. And by extenstion, thepractitioner as a professional. The issue of advertising doesn’t appear to be of much concern for project managers, as relatively few practitioners offer services for fees compared with doctors, lawyers, architects, engineers or accountants. The exception to this is the field of construction project management, which is covered in another section of this thesis. However, assuming outsourcing is here to stay (Malone, 2004; Osenton, 2004; Peters; 2002; Friedman, 2005), then it may well be that more and more project management practitioners from other sectors will follow the lead of construction management, with

236 Chapter 5: Survey development and analysis, Part 2

companies specializing in project management, either on an ‘at risk’ or agency basis. Should this evolve, advertising may well grow into an ethical and/or professional image issue. However, if the current practices in advertising advocated or tolerated in large part by PMI are any indication, a serious issue pertaining to advertising exists which is clearly harming the development/evolution of project management as a profession. That issue is the blatant advertising carried in all sorts of PMI literature and online that advocates ‘teaching to the exam’. If any organization wants to ‘protect’ the image of their credential, claims of ‘earn your PMP in five days or your money back’ are certainly not the way to go about doing so. Another example is Peter Nathan and Gerald Jone’s (2004) PMP Certification for dummies. While the Dummies series of books is generally well written, and while Nathan and Jones (2004) have produced a quality product, at least in terms of the content, the question remains whether a book of this title enhances the image of project management? If nothing else, shouldn’t this be an advertising issue which has crossed over and becoming a potential ethical violation? To quote from the relevant canons of the American Society of Civil Engineer’s code of ethics (noting that AACE, IEEE, AIA and most other professional organizations representing occupations already recognized as professions have or contain similar phrasing): Engineers shall build their professional reputation on the merit of their services and shall not compete unfairly with others. Engineers shall act in such a manner as to uphold and enhance the honor, integrity, and dignity of the engineering profession. (ASCE Code of Ethics, n.d., online)

To put the issue in perspective, one is unlikely to go to the local bookstore and find Brain surgery for dummies nor is one likely to find Commercial airline piloting for idiots, nor Trial lawyering for losers. Most professional organizations or societies include in their codes of ethics specific wording which prohibits advertising, activities or behavior which detracts from the dignity and image of their occupational specialty. Apparently, this has not been the case in the world of project management, at least up to this point.

237 Chapter 5: Survey development and analysis, Part 2

Because there are relatively few project managers who hang out their own shingles (at least at this point) (Zwerman et al., 2004), the analysis of this question was limited to comparing the various codes of ethics from those organizations which purport to represent project management practitioners to see if advertising was addressed. A literature review of ENR, Cost Engineering Magazine and the classified advertisements for most project management organizations, indicated little or no need for restrictions. The only analysis which indicated some cause for concern lay with the advertisements in PMI’s magazines for PMP exam courses. Table 5.5

Comparison of codes of ethics

Key Words phrases

AIC

AIPM ASCE INCOSE AACE CMAA IEEE

PMI

AIA ALPA Average Rank

public interest

1

5

18

2

10

1

0

1

2

1

4.1

honest

0

2

5

2

3

1

2

13

4

0

3.2

1 2

objective or truthful

0

1

7

2

2

2

0

2

3

0

1.9

3

professional reputation

3

0

3

0

1

1

0

6

2

3

1.9

4

dignified or modest

0

1

7

0

5

0

0

0

0

0

1.3

5

advertising

0

0

2

0

1

0

0

0

1

0

0.2

6

total occurrences

4

9

42

6

22

5

2

22

12

4

12.6

466

2199

406

1661

571

268

total key words in doc.

174

key words/total words

2.30%

ranking profession ?

3946 2639 1407 1373.7

1.93% 1.91% 1.48% 1.32% 0.88% 0.75% 0.56% 0.45% 0.28% 0.92%

1

2

No

?

3

4

Yes Discipline

5

6

7

8

9

10

No

No

Yes

?

Yes

Yes

The results are not conclusive one way or another. The two organizations in this listing who are most likely to ‘hang out their own shingle’ and do work on a fee for services basis are the American Society of Civil Engineers (ASCE) who ranked #3 of 10 based on key words. Contrast this against the American Institute of Architects (AIA), which ranked #9 of 10. However, only three organizations even mentioned the word ‘advertising’ in their codes of conduct/ethics – ASCE, AACE and AIA. Consider also that the Construction Management Association of America (CMAA), which specifically represents companies which provide both ‘at risk’ and ‘fee based’ construction management, ranks below average in this area and fails to even mention advertising in its code of ethics/code of conduct.

238 Chapter 5: Survey development and analysis, Part 2

5.1.12 Key question 12 Is (or should) use of the title ‘project manager’ be restricted by law? Exhibit 5.12 Key word analysis to answer key question 12 (Question details: Appendix A) Key finding

Use of title is restricted by law

Question generated by the key finding

KQ12 Is (or should) use of the title ‘project manager’ be restricted by law?

Hypothesis

As the term ‘project manager’ is so ubiquitous, there is no way at this point it could be restricted.

Survey question

Key word analysis of the term ‘project manager’

The next trait to be explored is laws to limit or otherwise restrict or control how the title or designation is used. Under the auspices of protecting the public from fraud, charlatans or impostors, the use of certain job titles or descriptors are prohibited by law. As an example, in the US, it is unlawful to use the designation ‘MD’ after your name, unless you are a licensed medical doctor. The same applies to engineers. You are not allowed to call yourself an engineer unless you have graduated from an ABET accredited engineering curriculum. Worth noting, the use of title restrictions is becoming less popular, as the ascendancy of board certifications is becoming preferred to giving preference to an entire class of practitioner. The Commonwealth Nations in particular seem to be loathe to encourage or otherwise support professional title restrictions and other forms of restraint of trade (ERAS, 2003; NSW, 2005]. The policy of the American Society of Civil Engineers provides some solid guidelines for the use of the term engineer: Use of the Term ‘Engineer’ Approved by the Committee on Professional Practice on January 17, 2004 Approved by the Board Policy Committee on March 12, 2004 Adopted by the Board of Direction on May 14, 2004 Policy The American Society of Civil Engineers (ASCE) believes that the following standards are the only basis on which any title or designation should include the term ‘engineer’. Graduation from an ABET accredited engineering program with a degree in engineering; Registration as a professional engineer or engineer-in-training under a state engineering registration law; or,

239 Chapter 5: Survey development and analysis, Part 2

An official ruling designating an individual or a group in an engineering capacity as meeting the definition of ‘Professional Engineer’ under the Taft-Hartley Act or the Fair Labor Standards Act. Only persons in one of these categories should be designated by the title ‘engineer’ or ‘professional engineer’. This policy shall not be construed to prohibit using the word ‘engineering’ as a modifier in titles such as ‘engineering assistant’, ‘engineering aide’ and ‘engineering technologist’ where the title clearly implies that the duties of the position are not those of professional engineer. ASCE further encourages registered professionals to always use their P.E. title on all professional correspondence and communication. Issue Improper use of the term ‘engineer’ is sometimes confusing or misleading to the public. Employers and employees misuse the term in titles and resumes. This misuse of the title by groups and people who are usually knowledgeable tends to diminish the value of the title which should be applied to people qualified professionally by accepted standards of education, law and/or engineering practice. Rationale There is a need within ASCE as well as within government and other organizations with practicing professional engineers to provide employee titles and/or classifications that properly identify the individual’s level of responsibility or expertise within that organization. A title such as ‘designer’ is not proper for a graduate engineer with several years of experience; ‘associate engineer’ or similar title as used by ASCE in designating professional grades is more appropriate and strongly encouraged. (American Society of Civil Engineers, online, n.d)

At this point, given the lack of any single definition of either project or project manager and given the widespread use of the term, attempting to claim jurisdiction over the name appears practically impossible. The next best approach is that taken by the North American Real Estate sector. Given that real estate agent was not possible to restrict, they took the next best approach, which was to trademark the term realtor, thus restricting the use of that term only to those who belong to the organization of realtors. Over the years, a brand image of excellence has been realized by the realtors and for over 30 years, the brand has gained in prominence and respect.

240 Chapter 5: Survey development and analysis, Part 2

5.1.13 Key question 13 Are symbolic costumes, uniforms or other identifying insignia necessary to define a profession? Exhibit 5.13 Generation of survey question 17 to answer key question 13 (Question details: Appendix A) Key finding

Symbolic costumes/uniforms

Question generated by the key finding

KQ13 Are symbolic costumes, uniforms or other identifying insignia necessary to define a profession?

Hypotheses

Costumes and symbolic uniforms or paraphernalia are necessary to define or identify a professional project manager.

Survey question

SQ17 When in a work environment, I am able to identify other project managers in my organization by the clothes they wear or some other identifiable part of their costume. (i.e. special tie, rings, headgear or other unique part of their dress or appearance)

An interesting attribute which was not very highly regarded or ranked by project managers, but one that has significance in most professions, is the wearing of symbolic costumes or uniforms. While easy to shrug off as being meaningless or irrelevant, when one walks into a hospital, there is no doubt who the doctors and nurses are. When you board a commercial airliner, the flight officers are readily distinguishable from the flight crew. In construction, the engineers traditionally wear white hard hats, and the superintendent or construction project manager often sport gold hard hats. Other examples of symbolic uniforms which are seen frequently are judges wearing black robes (and powdered wigs in most Commonwealth nations) or Canadian engineers wearing an iron ring. So while the issue of symbolic costumes or uniforms is unlikely to make a difference in determining if an occupation is a profession, once established, it serves as an important means of marking the territory and identifying oneself with a particular occupation and/or rank in that occupation. The question is whether having a distinctive PMP or other pin indicating certification by other credentialing bodies constitutes a ‘symbolic uniform’. And is the need for a uniform or costume any longer considered necessary as a means to lay claim to professional status? Consistent with Abbott (1988), given that territoriality is the nature of the professions and the organizations which represent them, then a uniform of some type becomes an extremely important non-verbal means to immediately claim territory.

241 Chapter 5: Survey development and analysis, Part 2

To test this hypothesis, survey question 17 asked whether a project manager can or cannot be identified by his uniform. The resultant responses indicated the obvious, that project managers do not have a uniform (Figure 5.22).

Figure 5.22

Results, survey question 17

Whether or not pins such as those issued by PMI indicating PMP status or AACE indicating the wearer holds their CCC/E will become the equivalent to the airline pilot captain’s wings, or the smock worn by doctors remains to be seen, but remains highly unlikely. However, as noted in previous discussions on this topic earlier in the research, the importance of a uniform to the perception of an occupation as a profession cannot and should not be underestimated. 5.1.14

Key question 14 Should project managers be licensed?

Exhibit 5.14 Generation of survey questions 37, 38 and 44 to answer key question 14 (Question details: Appendix A) Key finding

Practice limited by govt. license

Question generated by the key finding

KQ14 Should project managers be licensed?

Hypothesis

Licensing project managers will protect the consuming public.

Survey question

SQ37 I believe the primary purpose of occupational licensing of any profession is to protect the public from quacks, charlatans or incompetence. SQ38 I believe that licensing of project managers will lead to projects being completed on time, within budget and substantially fulfilling all technical requirements. SQ44 I believe occupational licensing of any type results in a monopoly and a form of restraint of trade.

242 Chapter 5: Survey development and analysis, Part 2

The question of licensing is clearly a contentious one. Economists in particular are generally not in favor of licensing. As indicated through earlier discussions, the noted economist Eliott Friedson (1994) sums it up quite effectively by stating ‘professionals differ from trade unions only in their sanctimoniousness’. The primary purpose of a license is to protect the consuming public from fakes, charlatans and frauds. So the primary question remains, will licensing deliver better project management? Perhaps the most contentious yet important attribute defining a profession is the issue of licensing. To test the sentiment of respondents, three types of questions were asked (Figures 5.23 , 5.24 , and 5.25 ). „

straightforward

„

obliquely from the perspective of whether licensing would result in projects being delivered faster or cheaper

„

a question to gauge sentiment on how project managers view licensing in general.

Figure 5.23

Results, survey question 44

Figure 5.24

Results, survey question 37

243 Chapter 5: Survey development and analysis, Part 2

Figure 5.25

Results, survey question 38

Given that responding practitioners, by a two to one margin, don’t perceive licensing negatively as a form of monopoly or restraint of trade, and believe overwhelmingly that licensing will serve to ‘weed out’ those who are incompetent, it would appear as though licensing is perceived favorably by most practitioners. However, the fact that those practitioners are mixed on their opinion that licensing will result in projects being better run presents an interesting quandary. Given that licensing is generally considered out of favour (Finnochio, Dower, Blick, & Gragnola, 1998; Cox, C. & Foster, S, 1990) and is unlikely to change in the foreseeable future (Zwerman et al., 2004), it may be unrealistic for practitioners to rely on licensing to guarantee competence in the occupation. However, the fact that South Africa has formally licensed construction project managers may well indicate that one of the most mature users of project management as a delivery system is also an early harbinger of changing attitudes. As these questions on such an important topic indicate confusion, an appropriate role of the project management professional organizations would be to educate their members on what licensing is and what it is not, and come out with a position paper on it, much along the lines of the American Society of Civil Engineers policy on licensure: Licensure All states have laws that govern the practice of civil engineering. Known as ‘Engineering Practice Acts,’ the primary purpose of this legislation is to protect the health, safety and welfare of the citizens of that state.

244 Chapter 5: Survey development and analysis, Part 2

Having an engineering license means more than just meeting a State’s minimum requirements. It means you have accepted both the technical and the ethical obligations of the engineering profession. In many states, the typical requirements to obtain your professional engineer’s license include: ABET accredited baccalaureate degree in engineering Fundamentals of Engineering (FE) examination 4 years of engineering experience under the supervision of a licensed engineer Principles and Practices (PE) examination Each state sets its own requirements for licensure and individuals seeking licensure should verify the specific requirements in the jurisdiction in which they seek licensure. (American Society of Civil Engineers: Licensure, online, n.d.)

5.1.15 Key question 15 Should project managers be required to carry professional liability insurance? Exhibit 5.15 Generation of survey questions 18 and 26 to answer key question 15 (Question details: Appendix A) Key finding

Requires professional liability insurance

Question generated by the key finding

KQ15 Should project managers be required to carry professional liability insurance?

Hypothesis

Project managers should carry professional liability insurance only if they are held responsible AND financially accountable.

Survey question

SQ18 I believe that project managers should be held financially accountable for mistakes they/their teams are responsible for when managing a project. SQ26 As a project manager, I feel I have ________________ responsibility for the decisions I make related to the execution of the Project for which I am in charge.

Of all the attributes which lead to an occupation being recognized as a profession, being held financially accountable for errors and omissions is probably the most credible (Polelle, 1999; Gawley, 2002). Yet, given the fact that project managers are rarely involved in the up front processes associated with creating projects: less than 50% of the time- the sales and marketing people are normally charged with this responsibility from the contractors perspective and from an owner’s organization perspective, it is usually senior management who conceive projects, with little or no input from the PM until such time as the project has formally been decided on. (Schoppman et al., 2006, p. 6)

is it appropriate that the PM be held financially accountable for something he/she had little or no input in defining?

245 Chapter 5: Survey development and analysis, Part 2

Furthermore, with accountability also goes control. Not withstanding the results of survey question 19, few project managers have full and total control over the resources necessary to execute projects. So without input at the early stages to determine delivery dates, cost budgets, technical specifications and methodology, and without controlling the resources necessary to execute the projects, how can any project manager be held financially accountable? To complicate the issue even more, exactly what should a project manager be held accountable for? On time? Within budget? Substantially conforming to specifications? For the purposes of the current research, as time, cost and specifications were the standard used by Standish (1994) in the Chaos Report and FMI (2006) in their Construction Survey, it will continue to be applied here, recognizing that not all agree this is correct or appropriate, especially from an owner’s perspective. Should the project manager be held accountable for the functional viability, that is, the products or deliverables of the project? Or should the project manager be held accountable for following the proper process only, regardless of the outcome? Looking to the medical profession, was the operation a success if the patient dies? And if the patient dies, should the doctor be held liable? In this instance, US, UK and Australian case law holds that as long as the doctor followed ‘generally accepted’ practices, and committed no errors of judgment or procedure, and the patient died, then he/she is not liable (Polelle, 1999; Gawley, 2002). Worth noting is the standard against which a doctor is judged is the normal and customary practice, as determined by a peer review board, not best practice, but normal and customary, and a review not by an independent body, but by peers. Thus in order to develop a profession, the key elements of having a ‘proven’ or accepted methodology, having substantial control over what is done, when it is done and how it is done (resources), and being held accountable for following the established procedures is the essence of having accountability. These are inextricably linked. Yet, the only true non-proprietary ‘methodology’ in project management is PRINCE2. Does that mean if you followed the PRINCE2 methodology without deviation and your project failed, that one would not be accountable for the failure? Further complicating this issue is the fact that the PMBOK Guide up until 2000 did not claim to be a methodology. It was only the body of knowledge. The third edition 246 Chapter 5: Survey development and analysis, Part 2

of the PMBOK Guide, however, looks more like a methodology. Assuming one did accurately follow the process outlined in the PMBOK Guide, would that be sufficient to absolve a practitioner from any liability should the project fail? And has the implied methodology outlined in the Guide ever been tested, either in practice or through simulation? Applying the basic practices found in law and medicine, new procedures in medicine are carefully tested, often on animals, before ever being tried on humans. Likewise, in the case of the legal profession, the body of case law is an ever evolving, dynamic process. The fact that any organization (be it PMI, PRINCE2 or any other entity) should develop a methodology, and place it untested and unproven in the marketplace, then imply by claiming those who subscribe to that methodology or belief are ‘professionals’, borders on misfeasance17, especially in light of the Chaos Report (Standish, 1994; FMI, 2006). The results for survey question 18 represent one of the more interesting aspects of the research (Figure 5.26).

Figure 5.26

17

Results, survey question 18

Misfeasance is defined to be ‘Performing a legal action in an improper way. This term is frequently used when a professional or public official does his job in a way that is not technically illegal, but is nevertheless mistaken or wrong. Here are some examples of misfeasance in a professional context: a lawyer who is mistaken about a deadline and files an important legal document too late, an accountant who makes unintentional errors on a client’s tax return or a doctor who writes a prescription and accidentally includes the wrong dosage’. (Nolo law dictionary)

247 Chapter 5: Survey development and analysis, Part 2

In terms of financial accountability, 60% feel that practitioners should be held financially accountable for ‘errors and omissions’, while 87% believe they have full responsibility for the decisions they make (Figure 5.27).

Figure 5.27

Results, survey question 26

While in terms of financial accountability, respondents meet the preponderance of the evidence test, they do not meet the clear and convincing evidence standard of 80%. However, responsibility for the decisions does meet that test. After analysis of this result, a flaw was detected in this line of questioning. By not defining the scope of decisions for which they were responsible, nor ascertaining whether the project manager was accountable for resources, but without ultimate authority for making decisions on the use of those resources, the question was rendered less than ideal.

248 Chapter 5: Survey development and analysis, Part 2

5.1.16

Key question 16 Given autonomy in decision making is a key attribute of a profession, do project managers have autonomy in making decisions?

Exhibit 5.16 Generation of survey questions 19, 25, 27 to answer key question 16 (Question details: Appendix A) Key finding

Autonomy in decision making

Question generated by the key finding

KQ16 Given autonomy in decision making is a key attribute of a profession, do project managers have autonomy in making decisions?

Hypothesis

Autonomy in making decisions is a required element of a profession.

Survey question

SQ19 As a project manager, I feel I have ________________ autonomy in making decisions related to the execution of the Project I am responsible for. SQ25 In fulfilment of my duties in the role of a project manager, I feel I am able to consistently exercise discretionary judgment in how the project gets executed. SQ27 People who use my services as a project manager are not concerned about how I execute the project as long as the project is successful (using their definition of success).

Survey questions 19, 25 and 27 were designed to explore how practitioners perceived their autonomy in decision making. As autonomy is generally considered to be one of the pillars of an occupation being recognized and accepted as a profession (Schein, 1976; Abbot, 1988; Haga, 1972; Polelle, 1999) this series of questions was exceptionally important (Figures 5.28, 5.29 and 5.30). Autonomy is one of the elements necessary in order to expect financial accountability of any manager (Figure 5.28). This means autonomy in determining the resources necessary as well as in determining the timing and budgets. The average score of 3.85 out of a possible 5 (77.6%) while certainly meeting the ‘preponderance of the evidence’ test (Nolo, 2006) and approaching the ‘clear and convincing evidence’ of ~80% (Tillers, 2005) is still not the kind of autonomy necessary to assume substantial financial responsibility for a project.

249 Chapter 5: Survey development and analysis, Part 2

Figure 5.28

Results, survey question 19

The reader is asked what sort of control (autonomy) would he or she require in order to be willing to assume financial responsibility for a project? 51% (preponderance of the evidence)? ~80%? (clear and convincing). Or would you expect ‘beyond a reasonable doubt’ (=>95%) (Saunders, 2003)? In response to question 25 (Figure5.29), over 85% of the respondents reported being able to exercise discretionary judgment in how their projects were executed. As in previous questions, while certainly indicating a move in the professionalization of project management, further research is necessary in order to determine better what types of discretionary judgment project managers have been granted.

Figure 5.29

Results, survey question 25

250 Chapter 5: Survey development and analysis, Part 2

Responses to question 26 (Figure 5.30) also presented an encouraging picture, with an average of 4.34 out of a possible 5.0, indicating a positive trend towards trusting project managers.

Figure 5.30

Results, survey question 26

The responses to questions 19, 25 and 27 warrant further research and confirmation. What decisions (budgetary? time? resource? other?) have been delegated to the project manager/project team to make? A look at Agile or eXtreme project management methodologies or philosophies indicates that project managers do not have control over either scope or costs. Assuming the pentagonal trade off remains valid (the theoretical algebraic relationship between scope, cost, time, quality and risk) (Wideman, 2004), the question of just how much actual control project managers have over time, costs, human resources remains an important question yet to be answered, for responsibility without authority doesn’t work (Drucker, 2001, p. 119). Compare the responses on the part of project managers against the concept of the ‘pilot in command’ principle which forms the foundation for the commercial airline pilot: Through his seat at the front of the aircraft flow the efforts of thousands of people who provide the means by which he carries out his task. However, it is an undeniable fact that: His is the final responsibility. His is the ultimate decision in any course of action.

251 Chapter 5: Survey development and analysis, Part 2

He can never be complacent. He must be humble; the elements keep him so. He must prove himself to his peers over and over again throughout his career, or seek another job. He must exude a quiet but magnetic confidence in his own ability and his aircraft. He must create an aura of efficiency and capability such that the passengers stream on and off the aircraft without even a thought about what is occurring at the front of the aircraft. Finally, he must be ready during every second of his working life to defeat the ultimate emergency he may encounter at any time. These responsibilities are recognized in the definition of his title of ‘Captain’, which means ‘in command’ and as such legally the final responsibility for the safety of the aircraft rests solely with the pilot in command. It is these professional responsibilities that not only make pilots sui generis18, but also worth every cent of their pay and conditions. (Paterson, citing Captain Dick Holt, online, n.d.)

The statement by exemplifies the fact of autonomy for airline pilots. If project managers were as autonomous in executing the projects charged to them, they should also be held legally and financially accountable if they should fail to execute them appropriately. Consistent with the theme running through this entire research, the key lies in the semantics. As was illustrated in FMI’s 2006 research, few of today’s project managers, even those in construction management, with relatively high autonomy (FMI, 2006), have anywhere near the accountability, nor the level of responsibility, accorded a commercial airline pilot. Having stated that, the core attributes necessary to establishing project management as a profession lie with the issue of autonomy and its alter ego, financial and legal accountability (Polelle, 1999; Gawley, 2002).

18

sui generis: Latin for ‘of its own kind,’ used to describe something that is unique or different

252 Chapter 5: Survey development and analysis, Part 2

5.1.17 Key question 17 Given identifying with the occupation and not with an employer is a key attribute of a profession, who do project managers identify with? Exhibit 5.17 Generation of survey question 27 to answer key question 17 (Question details: Appendix A) Key finding

Identity not with employer but profession

Question generated by the key finding

KQ17 Given Identifying with the occupation and not with an employer is a key attribute of a profession, who do project managers identify with?

Hypothesis

Project managers do not identify with their employer, but with the occupation of project management.

Survey question

SQ16 I consider project management to be my life’s work. SQ20 I relate more to being a project manager than I do to working for any particular employer SQ24 When I introduce myself to people, I would be more likely to do so by stating ‘I am a project manager who works for XYZ company’ OR ‘I work for XYZ company as a project manager’

The next attribute or trait common to most professions is a lifetime commitment. Given the investment in both time and money necessary to prepare oneself to practice in most professions is quite high, the tendency for people to choose carefully before making such an investment results in a relatively high satisfaction level as well as a willingness to stay with your chosen profession once you have ‘arrived’. This has particular relevance to project management, as it has developed a reputation of being an ‘accidental profession’. Whether the perception comes about because project management offers such a low barrier to entry, because of a lack of clear understanding by senior managers about the qualifications necessary to effectively manage projects or some other reason is unknown. However, as responses to the survey questions show, answers to questions relating to one’s identity as a project manager offer many discrepancies. A series of three questions (randomly interspersed in the survey) was specifically designed to address the issue of the ‘accidental project manager’ (Figure 5.31). As can be seen from the responses, the messages are mixed. On one hand, some 73% consider project management to be their life’s work and about the same percentage relate more to being a project manager than they do working for any particular employer (Figures 5.31 and 5.32).

253 Chapter 5: Survey development and analysis, Part 2

Figure 5.31

Results, survey question 16

Figure 5.32

Results, survey question 20

When asked a question about how they would introduce themselves, however, 57% responded in a manner indicating they relate more to a company than to their selected vocation (Figure 5.33).

Figure 5.33

Results, survey question 24

254 Chapter 5: Survey development and analysis, Part 2

5.1.18 Key question 18 Those in existing professions enjoy high community esteem. What is the community esteem of project managers? Exhibit 5.18 Generation of survey question 27 to answer key question 18 (Question details: Appendix A) Key finding

Held in high esteem by the community

Question generated by the key finding

KQ18 Those in existing professions enjoy high community esteem. What is the community esteem of project managers?

Hypothesis

Project managers are not held in the same high esteem as existing professionals.

Survey question

SQ21 As a project manager, I consider myself to be held in ______ esteem in my home (non-working) community because of the work I do as a project manager.

To paraphrase this question, would your mother be likely to be bragging to her friends and acquaintances about ‘my son/daughter’ the project manager’? If the answer to this simple question is no, then clearly, project management has yet to arrive as an honored profession. Despite Fortune magazine (Stewart, 1995) calling project management the career choice of the 90s and beyond, the hype has far exceeded any reality supported by facts. Research published by the Tasmanian (Australia) state government indicates( eGovernment, online, n.d.). Research by HCI (2001) indicates that 85-90% of projects fail to deliver on time, on budget, and to the quality of performance expected. (HCI Journal, online, 2001) KPMG’s International 2002-2003 Program Management Survey (2003) reported that in the 12 months prior to the survey, 57% of organizations surveyed internationally experienced at least one project failure. (KPMG, online, 2003) A review of studies conducted by IT Cortex (2003) found that IT projects are more likely to fail than succeed and that the larger the project the more likely it is to fail. (IT Cortex, online, n.d.) FMI, a management consulting firm and investment banker specializing in the construction sector, in their 2006 survey, discovered that only31% of construction projects finished on time and only 8% finished within budget. (Shoppmann et al., 2006)

Similar studies by Gartner Group (2003), Standish Group (1994), Engineering News Record (ENR) and other reputable sources (Lewis, 2003; CIO Magazine 2003) indicate that anywhere between 20% to 70% of projects are either canceled or ‘fail’, with failure defined to be late, over budget or not delivering substantially what the end user wanted, needed or expected. 255 Chapter 5: Survey development and analysis, Part 2

To test this hypothesis, respondents were questioned about whether they felt they were respected in their community (Figure 5.34). Note that when the survey was being designed, it was anticipated that a cross section of respondents from outside the world of project management would respond, and comparisons between the perceptions of profession would be enabled. Unfortunately a cross section of individuals failed to respond. While this thesis can report on the perception of project managers among project managers, no crosswise comparison is able to be conducted.

Figure 5.34

Results, survey question 21

While PMI in particular has done an outstanding job of ‘marketing’ project management as a career choice, in order to move beyond the hype and earn the right to be called a profession, the project delivery success rate must be substantially improved by people becoming prouder and associating more definitively with their occupation. First hand experience indicates that while working on a successful project is something one will always be proud of and remember, the rewards for someone on the project team may have little or nothing to do with the success or failure of the project and more to do with the environment in which the project was executed. It is possible to work on a project team executing a failed project and still come away with pride and satisfaction.

256 Chapter 5: Survey development and analysis, Part 2

Compare this to a medical doctor who operates on a patient with a negative outcome. A poignant example is the operation by Singapore’s Dr. Goh in June of 2003 on Iranian twins co-joined at the head: I asked and prayed about this quite intensely. At the end of doing my own post-mortem on the case, I think that looking back at the situation it was the correct thing to do. It was done with the best of intentions, and with the best available medical plan that anybody could have made. Although the outcome was the death of the twins, I think that we learnt so much from this case that it would benefit patients in future, and that it was probably an operation that carried a lot of benefits. (BBC: Hardtalk, online, n.d.)

This example illustrates yet another source of confusion in the practice of project management. If one subscribes to Fayol’s (1896-97) element of prevoyance, the strategic decisions identifying and selecting which projects should be done lie with management. Stated another way, it should not be the project manager who is held accountable for the failure (or the success) o of the project unless he/she plays an integral role in the decision making process. In the case of Dr. Goh operating on the Bijani twins, he and his team, along with the twins themselves, clearly understood the risks involved and the odds of the project succeeding (separation), including one or both of them suffering brain damage or, as happened, not surviving at all. Until or unless a project manager is involved in the Strategic decisions, he/she should only be held accountable for the planning, controlling, execution, and closing of the project. Whether the product fails or succeeds should not be on the project manager’s scorecard. Compare this also to the Aloha Airlines example concluding the case study of airline piloting. In order for project management to be enjoy high status in the community, there must be solid evidence showing that projects succeed much more frequently than they actually do. The real or perceived failure rate of projects is perhaps the single leading reason why project management has not and should not be perceived as a profession.

257 Chapter 5: Survey development and analysis, Part 2

5.1.19

Key question 19 Those in existing professions earn higher than average salaries. What is the perception of project managers in terms of the salaries they earn?

Exhibit 5.19 Generation of survey question 22 to answer key question 19 (Question details: Appendix A) Key finding

Earn higher than average compensation

Question generated by the key finding

KQ19 Those in occupations recognized as professions earn higher than average salaries. What is the perception of project managers in terms of the salaries they earn?

Hypothes1s

Project managers earn higher than average salaries.

Survey question

SQ22 Compared to other people of my age and education level, in my position as a project manager, I feel I am compensated ___________ for my services. This important attribute was confirmed /reinforced by comparing the results of AACE's annual salary survey.

Compensation is one of the more traditional measures of having achieved professional status. The response to this survey indicates 142/309 respondents (~46%) believe they earn only average compensation (Figure 3.5). And 62% feel they earn average or less compensation. While not being the primary determinant, these figures certainly don’t indicate project management has achieved professional standing. Note that as data from this question was being analyzed, it became clear that the salaries as a demographic were going to have limited value because of the multitude of situations in which the respondents found themselves working.

Figure 5.35

Results, survey question 22

258 Chapter 5: Survey development and analysis, Part 2

Assuming that Haga’s (1974) posit that money comes as a result of autonomy and the ability to intimidate clients, employers, governments is correct, however, it would seem that the results from the survey question relating to autonomy are questionable, particularly as those results do not match observations and private discussions with many global project managers, and compensation is average at best. In light of these contradictions and the failure of responses to question 22 to generate adequate data, a global survey conducted annually by AACE was interrogated as it was more representative and contained a better statistical analysis of the data than was possible from the survey instrument developed for the current research. Upon analysis, and comparing the results of the two surveys, a clear discrepancy between the results of the AACE annual study and the current research was apparent. The AACE survey data indicated above average salaries and a high level of job satisfaction (Figures 5. 36, 5.37, 5.38 and 5.39) compared with the results from the survey used in the current research, which had indicated only average compensation.

Figure 5.36

Salary by industry sector

259 Chapter 5: Survey development and analysis, Part 2

Figure 5.37

Salary by project type

Figure 5.38

Job satisfaction by salary, all respondents

Of particular interest is the data taken from the AACE Annual Survey for Project Managers only. Clearly, job satisfaction is not a function of salary, but something else. This data is also interesting in that nearly 75% of project management practitioners report they are somewhat or very satisfied with their jobs.

Figure 5.39

Job satisfaction by salary, project managers only

Compare the results from those holding the title of project manager vs. the results from the total population of respondents to AACE’s survey and it would appear as though project managers, despite getting more money, tend to be very dissatisfied and at the upper end, comparing the general population vs. project managers, 79% report being somewhat or very satisfied vs. 75% for project managers alone. Insignificant, perhaps, but it may indicate at least a certain amount of frustration or stress for which money alone is not able to compensate. 260 Chapter 5: Survey development and analysis, Part 2

Part of the problem may be that project managers, rather than being compensated on a fee for services basis, are, in fact, employees and are compensated more like nurses, teachers and airline pilots, rather than doctors, lawyers or professional engineers. However, as more doctors start to work for HMOs, and lawyers work for large firms, this distinction may be blurring. However, if Tom Osenton in Death of demand (2004) is correct, and the trend towards outsourcing continues, practitioners in all sorts of occupations may find themselves to be essentially independent contractors and not employees. If that happens, it will probably hasten the professionalization of many occupations, for when one’s reputation and future work lie in their most recent success (or failure, as the case may be), project managers, among others, will find themselves under more direct pressure to deliver successful projects, which includes the very important option of rejecting engagements which are not likely to succeed. Of all the drivers which will lead to better project management, and stand to improve the image of project management as a profession, this may well be the most significant. 5.1.20 Key question 20 A leading intrinsic attribute for a profession is the mystique which surrounds it. Does project management have mystique? Exhibit 5.20 Generation of survey question 15 to answer key question 20 (Question details: Appendix A) Key finding

Mystique (average of 15, 23 and 0) (highly ritualistic procedures)

Question generated by the key finding

KQ20 A leading intrinsic attribute for a profession is the mystique which surrounds it. Does project management have mystique?

Hypotheses

Project management does not have mystique.

Survey question

SQ23 People not familiar with project management consider what practitioners do to be mystical and/or highly ritualistic

This attribute is the first of the intrinsic traits, and perhaps one of the most intriguing. When you are sick, and go to your doctor, do you ask to see his/her credentials or CV? Do you question his/her every move? Do you haggle with him/her over the amount of time it will take for you to get better? Do you ask for an estimate of cost up front? And assuming you get one, do you arbitrarily and unilaterally tell your doctor to reduce the cost by 20% and that you want to get better in half the time he/she tells you it will take to get well again? 261 Chapter 5: Survey development and analysis, Part 2

When you get on an airplane, do you tell the pilot that because you are in a hurry, you expect him/her to land at your destination an hour early? Do you question the route she is flying? Or the altitude? Or the speed? The answers to these questions (one would hope) are of course no. When you seek out the services of a professional, you rely on their reputation, (usually passed along via word of mouth from someone you know and trust) and when you seek out their services, the estimate (if one is even provided to you) is an approximation, and based on their reputation and your comfort level (trust) in them. You accept whatever they say, both in terms of time and cost. Now, you may get a second opinion, but usually that applies more to the appropriateness of the ‘solution’ than it is to the price and/or time. And most people would laugh at the prospect of putting their next operation on Ebay. Yet that is exactly what is happening today in the world of project management. As the telecommunications and the IT sector regularly procure professional services for project management using online auctions or e-Procurement methodologies. As it seems highly unlikely that project management will ever develop the kind of ‘mystique’ where people will procure the services on a fee basis, and allow the time and to a large degree the cost, to be determined by the practitioner, is highly unlikely. To better appreciate the role that mystique plays in defining a profession, consider each April (for Americans) when you go to see your accountant or tax advisor. Because of the complexity of the tax laws, you, as the consumer, need to trust your tax professional implicitly, providing him/her with your most private information, faithfully following their advice in order to minimize the tax you have to pay. While you may have an idea of the amount, rarely do you negotiate in advance for either time or money. As can be seen in Figure 5.40, the intrinsic attributes of implicit trust, a BoK which is unique, esoteric and abstruse, as well as the cruciality are ranked #2, #3 and #6 respectively. This provides empirical evidence supporting the position of Zwerman, Thomas et al. (2004) that there is something better than the traditional trait models to define a profession. Whether this supports the ‘control’, ‘feminist’ or ‘conflict’ theories outlined by Zwerman, Thomas et al. (2004), or if it supports the findings of

262 Chapter 5: Survey development and analysis, Part 2

Haga (1974) will have to remain for subsequent research, but regardless, the evidence is clearly showing that some model other than the traditional extrinsic indicators is at work. As mystique consists of three sub-elements: „

body of knowledge which is secret, esoteric or abstruse

„

highly ritualistic procedures

„

access to knowledge limited or restricted,

I used the same results from key question 5, body of knowledge for the purposes of calculating mystique. To test the ritualistic element, the question was asked whether project management is mystical or highly ritualistic. Given that access to the knowledge of project management is definitely not limited or restricted (just look in any magazine related to project management and this will be abundantly clear), I felt this point was moot, and didn’t even measure it, simply accepted it as valid in support of the hypothesis. While it could be argued that the fact the occupation is known as the ‘accidental profession’ stands as prima facie evidence in support of the argument that project management could not possibly have mystique, the question needed to be asked.

Figure 5.40

Results, survey question 23

263 Chapter 5: Survey development and analysis, Part 2

The fact that 63% responded close to ‘not sure’ raises some question about the wording of the question about the wording, as I thought it would be a more clearly defined response. (Either a strong yes or a strong no) but that failed to happen. Analysis of the data could find no meaningful correlation between any demographic group. While it could be argued that the fact the occupation is known as the ‘Accidental Profession’ stands as prima facie evidence in support of the argument that project management could not possibly have mystique, the question needed to be asked. However, based on the BoK not being unique, esoteric, secret, abstruse or complicated, a ‘not sure’ response to the ritualistic element and the undeniable fact that access to the knowledge is not restricted, can only lead to the conclusion that project management does not have mystique. 5.1.21 Key question 21 A leading intrinsic attribute for a profession is the cruciality (criticality) of the services. Does project management have criticality? Exhibit 5.21 Generation of survey question 33 to answer key question 21 (Question details: Appendix A) Key finding

Cruciality a. immediacy of need b. Importance of need

Question generated by the key finding

KQ21 A leading intrinsic attribute for a profession is the cruciality (criticality) of the services. Does project management have criticality?

Hypothesis

Project management does have cruciality/criticality.

Survey question

SQ33 People who use the services of a project manager generally consider those services critical to the success of their project.

Cruciality is another of the intrinsic or intangible attributes. This attribute says that I need your services now and no one else will do. An example of this is your favorite hair stylist. Yes, you can use just about anyone who is available, but your favorite stylist knows exactly what you want, understands you and has established a professional bond or relationship. Examples of attempts to create this relationship abound in the corporate world, with the creation of ‘customer care’ departments and customer relationship management (CRM) software.. However, as a project, is by

264 Chapter 5: Survey development and analysis, Part 2

definition a unique, one of a kind event, the ability for a project manager to develop a long term bond, where the customer calls on the project manager for repeat business is relatively rare. Another example is the relationship built up with your family doctor. In many instances, two, three or even four generations are being looked after by the same physician. As noted previously, the very nature of projects doesn’t lend themselves to developing a long term relationship, at least not with a specific project manager. So while a client may have a fair amount of ‘brand loyalty’ to the company providing the products or services, it would be unlikely to for that loyalty to extend to a specific project manager, at least not over a long period of time. So what about a company? Using the same analogy, do you develop the same kind of loyalty to an HMO that you had for your family doctor? Professional services are very much an individual thing, based on relationships. The Chinese call this guan xi and while it plays a lesser role in Western cultures, it forms an integral part of most Eastern cultures. If you are using a ‘Big 8 less 4’ accounting firm, is your trust in the name of the firm or the individual who is responsible to provide services to you? If that individual were to leave, would you be likely to follow him to another company, or would you stick with the ‘Big 8 -4’ accounting firm? While the answer to this question is patently obvious, (isn’t one of the major tools and techniques associated with project management known as the ‘critical path method’?) the question about the perceived criticality did need to be asked. As with previous constructs, the assumption was that respondents would be coming not only from project management, but from other professions as well, providing the ability to make comparisons. As this didn’t happen, only a one sided picture can be presented (Figure 5.41).

265 Chapter 5: Survey development and analysis, Part 2

Figure 5.41

Results, survey question 33

The results, with 56% of respondents agreeing that their services are critical, still does not reconcile the ‘accidental’ project manager. (If the services of a professional practitioner were so important, how could an organization apparently randomly select someone to provide those services?) In retrospect, the results of this question are somewhat suspect. However, of the three intrinsic attributes – mystique, criticality and dénouement – probably cruciality is the strongest of the three. 5.1.22 Key question 22 A leading intrinsic attribute for a profession is the dénouement or the ability to produce positive results very quickly and without limited involvement of the consumer of the services. Does project management have dénouement? Exhibit 5.22 Generation of survey questions 19, 25 and 26 to answer key question 22 (Question details: Appendix A) Key finding

Denouement quick solutions to problems

Question generated by the key finding

KQ 22 A leading intrinsic attribute for a profession is the dénouement or the ability to produce positive results very quickly and without limited involvement of the consumer of the services. Does project management have dénouement?

Hypotheses

Project management does not have dénouement.

Survey question

SQ19 As a project manager, I feel I have ________________ autonomy in making decisions related to the execution of the Project I am responsible for. SQ25 In the fulfilment of my duties in the role of a project manager, I feel I am able to consistently exercise discretionary judgment in how the project gets executed. SQ26 As a project manager, I feel I have ________________ responsibility for the decisions I make related to the execution of the project for which I am in charge.

266 Chapter 5: Survey development and analysis, Part 2

Denouement is defined to be ‘the outcome or result of any complex situation or sequence of events’ (Merriam-Webster, 2003). In today’s world it is often exemplified by total customer care or Deming’s total customer satisfaction. In the professional world of project management, it can be illustrated through this story, a classic example of the principle of dénouement, the famous 1982 Tylenol poisoning case. The Tylenol crisis occurred in the autumn of 1982, when seven people in the Chicago area in the United States died after ingesting Extra Strength Tylenol medicine capsules which had been laced with potassium cyanide poison. This incident was the first known case of death caused by deliberate product tampering. The perpetrator has never been caught, but the incident led to reforms in the packaging of over-the-counter substances and to federal anti-tampering laws. At the request of later Chairman Joseph Chiesa, new product consultant Calle & Company rescued the brand with the invention of Tylenol Gelcaps, the first inherently tamper-proof [enrobed]capsule, recapturing the 92% of capsule segment sales lost to product tampering.

The management of Johnson & Johnson did not seek to blame, worry about qualifications, specifications or cost. Their primary consideration was reputation and the need to minimize the impact of the incident (make the pain go away and do so as quickly as possible using whatever methods deemed appropriate and necessary), using all the available talent in their organizational system. Johnson & Johnson was praised by the media at the time for its handling of the incident. While at the time of the scare the market share of Tylenol collapsed from 35% to 8%, it rebounded in less than a year, a move credited to J&J's prompt and aggressive reaction. In November it reintroduced capsules, but in a new, triple-sealed package, coupled with heavy price promotions, and within several years Tylenol had become the most popular over-the-counter analgesic in the US.

Abbot (2006), in The systems of professions elaborates on the importance of ‘using these metaphors as a means of promoting respect’ (p. 200). Through the use of storytelling such as this, linking the profession of medicine to the work of project management has proven to be an effective tool to not only build a culture within the organization, but also to pass along the ‘lessons learned’. The effectiveness of this methodology can be found in the Biblical parables. In the event outsourcing becomes more of a norm than it is now, and assuming more project managers truly do ‘hang out their shingles’, given human nature to

267 Chapter 5: Survey development and analysis, Part 2

procrastinate, the professionalization of project management may very well develop or evolve based on this model or scenario. I strongly suspect that dénouement is what our clients and managers really want from project managers. They want to give their project managers a project and then magically, within their (often unrealistic) budgets, time frames and willingness to commit resources expect the project to simply ‘happen’, with as little aggravation and involvement as possible. While the concept of dénouement is easy enough to explain through examples, it becomes very difficult to measure. Below are the actual questions as asked in the survey. Because of the difficulty in translating dénouement into a simple English word or phrase, based on the work of Freidson (1994, pp. 160, 163; 2001, p. 34), Abbot (1994, p. 163) and Haga (1974), autonomy + responsibility + discretionary judgment were taken as a means of measuring ‘dénouement’. But in fact, dénouement is more than that. Based on the example of the toothache, the results by the respondents to the autonomy question indicate practitioner, as practitioners, do not enjoy the kind of autonomy in making decisions which is necessary to meet the standards of a ‘profession’ (Figure 5.42).

Figure 5.42

Results, survey question 19

While the average results approach meeting the test of clear and decisive evidence, tipping point or Pareto’s 80% rule, the results of this question are highly suspect. Given that most knowledge workers today are employees and not independent

268 Chapter 5: Survey development and analysis, Part 2

practitioners working on a fee-for-service basis, it seems highly unlikely that management is willing to delegate decisions to project managers normally made by C- or near C-Level Senior. Freidson (1994) is especially critical of this misinterpretation of autonomy when he writes: The substance (of the professionals) work under an employee-employer relationship is both created and evaluated by the employer, and their leeway, flexibility or freedom the may have in executing their jobs is a function of the goodwill and convenience of their employers….While such professionals, as a class, often have elements of discretion in the execution of their tasks, they are hardly so significant as to warrant the term ‘autonomy’. (p. 163)

Figure 5.43

Results, survey question 25

As with the commentary above, Freidson (1994) is skeptical of claims made by ‘professionals’ who are working as employees and not on a ‘fee for services’ basis, for only they realize the cause and effect relationship between their success in the market place and the decisions they make today.

Figure 5.44

Results, survey question 26

269 Chapter 5: Survey development and analysis, Part 2

Interesting to note that the average score for responsibility is higher than that of autonomy and considerably exceeds the ‘clear and conclusive’ evidence test, it seems as though current project management practitioners feel they have more responsibility than they do autonomy. Given the difficulties in testing for dénouement are so subjective in the context of project management, (i.e. the same project executed for the same person under differing circumstances may well experience different levels of dénouement). An example of the application of absolute responsibility can best be evidenced through the principle of the ‘pilot in command’. Christopher McKenna (2006) hinted at the importance of accepting responsibility by stating in the world’s newest profession – management consulting, that ‘when the time came to reshape their professional image at the end of the 20th Century, consultants shrank from their definitive organizational challenge’ (McKenna, 2006, p. 251). What McKenna is alluding to that management consultants, while offering advice, do not shoulder the burden of failure if their advice is wrong. So it would appear as though one of the bright line themes evolving from this research is the need for project managers to be willing to accept responsibility for their projects. Some interesting anecdotal evidence can be found if we look to the key words contained in Table 5.6, and see that ‘responsibility’ only ranked 11 out of 20 in terms of frequency of occurrence in the various codes of ethics. Table 5.6

Codes of ethics sorted by frequency of words and ratio of key words to total words

Key words phrases

INCOSE ASCE

AIPM

IEEE

CMAA

AACE

AIC

AIA

PMI

ALPA

ethics or ethical

8

8

2

2

3

2

0

15

13

2

5.5

1

safety, health, welfare

2

18

3

3

1

9

0

3

0

10

4.9

2

Average Rank

public interest

2

18

5

0

1

10

1

2

1

1

4.1

3

skill, knowledge

3

6

2

1

1

5

0

11

6

4

3.9

4

honest

2

5

2

2

1

3

0

4

13

0

3.2

5

conflict of interest

3

3

1

1

3

2

0

1

18

0

3.2

6

competence

1

7

0

1

2

2

0

4

1

1

1.9

7

objective or truthful

2

7

1

0

2

2

0

3

2

0

1.9

8

professional reputation

0

3

0

0

1

1

3

2

6

3

1.9

9 10

integrity

3

4

1

0

3

1

0

5

1

0

1.8

responsibility

1

0

2

1

0

1

0

1

7

4

1.7

11

fiduciary, agent, trustee

0

6

2

0

0

5

0

0

0

0

1.3

12

kickback, bribe, gratuity

0

10

0

1

0

0

0

0

2

0

1.3

13

dignified or modest

0

7

1

0

0

5

0

0

0

0

1.3

14

confidential

0

1

0

0

1

4

1

4

2

0

1.3

15

sustainable development 0

9

0

0

0

0

0

0

0

0

0.9

16

270 Chapter 5: Survey development and analysis, Part 2

Key words corrupt

0

6

0

0

0

0

0

0

1

0

0.7

17

self or personal interest

0

1

0

0

0

0

0

0

2

0

0.3

18

whistle blower

0

0

0

0

0

0

0

0

1

0

0.1

19 20

governance

0

0

0

0

0

0

0

0

1

0

0.1

total occurrences

27

119

22

12

19

52

5

55

77

25

41.3

total key words in doc.

406

2199

466

268

571

1661

174

2639

3946

1407

1373.7

key words/total words

6.65%

5.41% 4.72% 4.48% 3.33% 3.13% 2.87% 2.08% 1.95% 1.78% 3.64%

ranking

1 2 Disciplin e Yes

profession ?

3

4

5

6

7

8

9

10

?

Yes

No

No

No

Yes

?

Yes

5.1.23 Key question 23 How does the fiduciary responsibility of project managers compare against that of established professions? Exhibit 5.23 Generation of survey questions 18, 19, 25 and 26 to answer key question 23 (Question details: Appendix A) Key finding

Fiduciary obligation to the public

Question generated by the key finding

KQ23 How does the fiduciary responsibility of project managers compare against that of established professions?

Hypothesis

Project managers do not exhibit a level of fiduciary responsibility comparable to that of established professions.

Survey questions

SQ18 As a project manager, I feel I have ________________ responsibility for the decisions I make related to the execution of the project for which I am in charge. SQ19 As a project manager, I feel I have ________________ autonomy in making decisions related to the execution of the Project I am responsible for SQ25 In the fulfilment of my duties in the role of a project manager, I feel I am able to consistently exercise discretionary judgment in how the project gets executed. SQ26 As a project manager, I feel I have ________________ responsibility for the decisions I make related to the execution of the Project for which I am in charge.

Measuring fiduciary responsibility represents a major challenge. As Zwerman (2004) pointed out, ‘project managers have never claimed that they are providing an altruistic service’ (p. 173). Yet, at the same time, Polelle (1999), Gawley (2002) and Rotunda (1997) all state in one way or another that fiduciary responsibility (which Zwerman et al. (2004) refers to as ‘norm of altruism’) is one of the ‘bright line tests’ to determine a profession from a mere occupation. Based on the work of Wideman, (2006) Kerzner, (2004) Louis (2003) and a host of others who advocate the ‘tetrad trade-off’ or ‘iron triangle’ between scope, time, cost and quality, certainly implies a fiduciary obligation of sorts, as the responsibility of the project manager is to

271 Chapter 5: Survey development and analysis, Part 2

negotiate expectations between stakeholders who have competing or often mutually exclusive needs, wants and expectations. Given that codes of ethics adopted by those organizations representing project managers do not address the issue of fiduciary responsibility directly, and the question of fiduciary responsibility is generally not understood by the general public (Polelle, 1999; Gawley, 2002), I thought it inappropriate to measure this attribute directly through a survey question. This hypothesis is grounded on the work of Polelle (1999), Gawley (2002) and Rotunda (1997) who all see the bright line test in legal parlance being the degree of fiduciary responsibility to the end user of the product or services- i.e. the consuming public. While the ‘tetrad trade-off’ implies some degree of fiduciary responsibility to a variety of stakeholders, there is not clear distinction between which one takes precedence and to whom the primary responsibility is owed. For that reason, it is argued that project managers do not meet Polelle’s bright line test (Polelle, 1999; Gawley, 2002; Rotunda, 1997). Ultimately, the ideal test for this is attribute is defined by Polelle (1999) with his statement: the judiciary should decertify a profession for malpractice purposes if the profession eliminated or diluted it’s fiduciary obligation or else turned its enforcement mechanism into a sham. (Lexis/Nexus, online, Conclusion)

As Polelle (1999), Gawley (2002) and Rotunda (1997) all link fiduciary responsibility to ethics, financial liability and autonomy, the results of survey questions addressing the key questions around these concepts have been chosen to represent the numeric value of fiduciary responsibility. Based on the example of the toothache, the results by the respondents to the autonomy question indicate practitioners do not enjoy the kind of autonomy in making decisions which is necessary to meet the standards of a ‘profession’.

272 Chapter 5: Survey development and analysis, Part 2

Figure 5.45

Results, survey question 19

While the average results approach meeting the test of clear and decisive evidence, tipping point or Pareto’s 80% rule, the results of this question are highly suspect. Given that most knowledge workers today are employees and not independent practitioners working on a fee-for-service basis, it seems highly unlikely that management is willing to delegate decisions to project managers normally made by C- or near C-level seniors. Freidson (1994) was especially critical of this misinterpretation of autonomy, writing: The substance (of the professional’s) work under an employee-employer relationship is both created and evaluated by the employer, and their leeway, flexibility or freedom the may have in executing their jobs is a function of the goodwill and convenience of their employers. (p. 161)

He goes on to conclude: While such professionals, as a class, often have elements of discretion in the execution of their tasks, they are hardly so significant as to warrant the term ‘autonomy’. (p. 163)

Figure 5.46

Results, survey question 25

273 Chapter 5: Survey development and analysis, Part 2

As with the commentary above, Freidson (1994) is skeptical of claims made by ‘professionals’ who are working as employees and not on a ‘fee for services’ basis, for only they realize the cause and effect relationship between their success in the market place and the decisions they make today.

Figure 5.47

Results, survey question 26

Interesting to note that the average score for responsibility is higher than that of autonomy and considerably exceeds the ‘clear and conclusive’ evidence test, it seems as though current project management practitioners feel they have more responsibility than they do autonomy. Given the difficulties in testing for dénouement are so subjective in the context of project management, (i.e. the same project executed for the same person under differing circumstances may well experience different levels of dénouement). An example of the application of absolute responsibility can best be evidenced through the principle of the pilot in command. Christopher McKenna hinted at the importance of accepting responsibility by stating in the World’s newest profession – management consulting, that ‘when the time came to reshape their professional image at the end of the 20th Century, consultants shrank from their definitive organizational challenge’ (McKenna, 2006, p. 251). What McKenna is alluding to is that management consultants, while offering advice, do not shoulder the burden of failure if their advice is wrong. So it would appear as though one of the bright line themes evolving from this research is the need for project managers to be willing to accept responsibility for their projects. Some interesting anecdotal evidence can be found when considering the key words in Table 5.6, where ‘responsibility’ only ranked 11 out of 20 in terms of frequency of occurrence in the various codes of ethics. 274 Chapter 5: Survey development and analysis, Part 2

5.2

Summary of key question results

As we can see from the survey results, project management is not perceived by practitioners to be a profession, but as a process, methodology or system. This leads to the conclusion that people confuse the noun, profession with the adjective and nouns, professional. We also see using three different approaches and removing bias, that practitioners consistently rated project managers lower than MBAs but higher than electricians. If nothing else, this will serve as a benchmark for future research to see if project management is gaining or losing credibility as a profession. However, while we know the rank order of project management as compared against other occupations, we cannot tell just how much more or less of a profession these other occupations are. To address this important piece of research , an attempt was made to produce a true ratio scale. While the survey did produce a Professionalization Index value for project management of 33.7%, without values from other occupations against which to compare it, the results were only partially successful. Figure 5.48 sums up graphically the three phases of the research that resulted in the outcome outlined in Table 5.7.

Figure 5.48

Three phases of the research

275 Chapter 5: Survey development and analysis, Part 2

Questions generated by the key findings (KQ) – to be addressed by a variety of test questions

KQ1 Is project management a profession?

Multiple, conflicting and confusing definitions of project management and profession abound.

SQ28 Below are 8 definitions. Please rank the definition that most represents your definition, understanding or interpretation of project management as it is practiced by you or within your organization. Mark 1 being the ‘best’ definition, 2 being the second best and so on, with 8 representing the least appropriate definition in your opinion.

Survey questions

276 Chapter 5: Survey development and analysis, Part 2

Project management is not a profession. It is a discipline within all other technical occupations.

Hypotheses based on the key findings, to be tested as part of addressing KQs

Summary of interpreted qualitative and quantitative results from survey data

Key findings (from literature research)

Table 5.7

SIGNIFICANT Project management is perceived as being a process, method or system.

Qualitative Analysis Summary

Process Ranked #1/8 Method Ranked #2/8 System Ranked #3/8 Profession Ranked #6/8

Quantitative Results from Survey Instrument Summarized

Please note that the following terms have been used in the qualitative analysis recorded in Table 5.7, which sums up the development, analysis and conclusion of the research: significant The findings from the research provided or contributed significantly to the analysis and/or conclusions. important The findings were important to the research, but the data may or may not have been useful in analysis or drawing conclusions. indeterminate The findings from the research could not be used to analyze or draw meaningful conclusions from. interesting The findings were interesting, and opened the door for further research. conflicting The data showed conflicting results which rendered any conclusions suspect. not relevant The results were not important or useable for the purposes of this research.

Questions generated by the key findings (KQ) – to be addressed by a variety of test questions

KQ2 Is there a continuum and where on that continuum of professionalization; if so, where does project management lie on the continuum?

Key findings (from literature research)

All occupations exist on a continuum

SQ 47 Rank the following list of professions in the order you perceive them to represent your professional ideal. (1 being most professional, 21 being least professional)

SQs 39, 40, 41 and 42 Listed below are pairs of occupations. From each pair, mark the circle that best represents your ideal image or impression of how much of a profession each occupation is relative to the other. if you consider them equal, mark the 0%.

SQs 30 and 31 Below is a list of 21 occupations generally considered to be ‘professions’ based upon the requirement of a license to practice? Compare each occupation against the baseline of a fresh graduate of a 4 year university, with no experience and no license, marking how much more you perceive the relative value of the occupation compared against the fresh university graduate in terms of professional image or prestige.

Survey questions

277 Chapter 5: Survey development and analysis, Part 2

Defining project management as a profession is not a yes or no answer; as an occupational specialty, project management lies on a continuum someplace between the trades and the learned professions.

Hypotheses based on the key findings, to be tested as part of addressing KQs

SIGNIFICANT Project managers are perceived to be considerably more professional than electricians but less so than MBAs and even more less so than professional engineers.

Qualitative Analysis Summary

Doctor Ranked #1/21 Airline Pilot Ranked #2/21 Lawyer Ranked #3/21 Project Manager Ranked #14/21 Bus Driver Ranked #21/21

Quantitative Results from Survey Instrument Summarized

Questions generated by the key findings (KQ) – to be addressed by a variety of test questions

KQ3 Intuitively, not all attributes are of equal value. What is the actual numeric value for each attribute by respondents?

KQ4 Is the project management body of knowledge unique?

KQ5 How much experience/apprenticeship/educat ion is associated with the term profession?

Key findings (from literature research)

Relative value or weighting of each attribute

Body of knowledge a. unique b. esoteric/ complicated/ secret

Long period of training

SQ34 How many years of education beyond high school does it take to produce a professional practiticing project manager?

SQ15 Please evaluate the following terms or phrases by how unique you consider each as they are used in project management compared to general usage

SQs45/46 Below is a list of 22 attributes normally associated with an occupation being recognized or accepted as being a ‘profession’. From the selection below, please indicate the relative weighting or importance you give to each of these attributes. You can mark any value between 0 (No importance to evaluating or determining a profession), to a maximum of 100 points (Essential attribute necessary to be considered a profession)

Survey questions

278 Chapter 5: Survey development and analysis, Part 2

A profession requires at least a 4 year degree AND 4 years of practical experience.

The project management body of knowledge is NOT unique.

Each attribute has a numeric value which can be calculated and that the sum total of the attributes will produce a ‘professional Index’

Hypotheses based on the key findings, to be tested as part of addressing KQs

SIGNIFICANT It is very clear that practioners tend to believe that a 4 year degree or better is required to be a project manager. Not quite clear and convincing evidence, but very close.

IMPORTANT While the body of knowledge did meet the preponderance of the evidence test for being unique, it did not meet the clear and convincing evidence standard. Further research indicated the research was flawed in measuring uniqueness.

SIGNIFICANT The ranking of attributes by project management practitioners should provide meaningful guidance to those organizations competing for the hearts, minds and money of practitioners.

Qualitative Analysis Summary

74.3% =>4 years

75.6% Rank PM BoK as Unique

Code of Ethics was ranked #1/22 Trust was ranked #2/22 Unique BoK was ranked #3/22 Pro Bono Work was ranked #20.22 Title Restrictions was ranked #21/22 Uniforms was ranked #22/2

Quantitative Results from Survey Instrument Summarized

KQ6 Is project management a calling?

SQ16 I consider project management to be my life’s work

279 Chapter 5: Survey development and analysis, Part 2

The term ‘accidental profession’ commonly applied to project management indicates that project management is not a profession.

CONFLICTING The results of Question #16 and 20 indicate that project managers do have a lifetime commitment, yet when asked how they introduce themselves, (SQ#24) 57% said they related more to their companies than to their job title. Given project management is known as the ‘Accidental Profession’ this needs more research.

SIGNIFICANT Respondents provided clear and convincing evidence that 3 years or more of apprenticeship should be required before a practitioner can be considered a professional.

SQ36 How many years of apprenticeship, internship or supervised work experience do you associate with the term ‘profession’?

Life time commitment

SIGNIFICANT This question very clearly supports a 3 year minimum experiential based training requirement. (Approaching beyond a reasonable doubt)

SIGNIFICANT Reponses to this question provide clear and convincing evidence that a 4 year degree is the Minimum requirement for project managers to be considered professionals.

Qualitative Analysis Summary

SQ35 How many years of work experience does it take to produce a professional project management practitioner?

Survey questions

b. apprenticeship/internship/ residency

Hypotheses based on the key findings, to be tested as part of addressing KQs SQ43 How many years of university level education do you associate with the term ‘profession”?

Questions generated by the key findings (KQ) – to be addressed by a variety of test questions

a. higher education

Key findings (from literature research)

73.0% Yes

76.3% =>3 years

90.2%=>3 years

84.8%=>4 years

Quantitative Results from Survey Instrument Summarized

Questions generated by the key findings (KQ) – to be addressed by a variety of test questions

KQ7 How do project managers perceive the practice of individual ethics?

Key findings (from literature research)

Adhering to a code of ethics

CONFLICTING The results of Question #16 and 20 indicate that project managers do have a lifetime commitment, yet when asked how they introduce themselves, (SQ#24) 57% said they related more to their companies than to their job title. Given project management is known as the ‘accidental profession’ this needs more

SQ24 When I introduce myself to people, I would be more likely to do so by stating: ‘I am a project manager who works for XYZ company’ OR ‘I work for XYZ company as a project manager’

IMPORTANT 56% of respondents claimed their clients did not care how they executed the work. Given Sarbanes Oxley has raised the potential consequences for project managers; it may well be a lead in to fiduciary responsibility

CONFLICTING The results of Question #16 and 20 indicate that project managers do have a lifetime commitment, yet when asked how they introduce themselves, (SQ#24) 57% said they related more to their companies than to their job title. Given project management is known as the ‘accidental profession’ this needs more research.

SQ20 I relate more to being a project manager than I do to working for any particular employer

SQ27 People who use my services as a project manager are not concerned about how I execute the project as long as the project is successful (using their definition of success).

Qualitative Analysis Summary

Survey questions

280 Chapter 5: Survey development and analysis, Part 2

Project managers have professional level ethics.

Hypotheses based on the key findings, to be tested as part of addressing KQs

56.1% Say their clients do not care how the achieve the goals and objectives.

57.1% would introduce themselves as project managers.

70.5% responded agree or strongly agree

Quantitative Results from Survey Instrument Summarized

Questions generated by the key findings (KQ) – to be addressed by a variety of test questions

KQ8 How do project managers compare against other professionals in terms of pro bono work?

KQ9 (original) How do organizations representing practitioners compare against other organizations in terms of ethics?

Key findings (from literature research)

Service to the public (pro bono work)

Professional association

Although responses were limited, it appears as though most of the professional organizations representing project managers do NOT aggressively enforce their codes of ethics, at least not in the context of protecting the consuming public. First hand

This was not on the regular test instrument This was answered through emails sent to the executive directors/CEOs of major professional organizations representing project managers.

SQ29 During the past year, I have donated the following number of hours performing voluntary, uncompensated community service requiring the use of my project management skills

Survey questions

281 Chapter 5: Survey development and analysis, Part 2

There is a difference between the codes of ethics/codes of conduct of organizations which are recognized as professions from those which are not

Project managers meet or exceed the total number of person hours donated by established professions.

Hypotheses based on the key findings, to be tested as part of addressing KQs

Based on the fact all organizations are claiming to have set standards, but there is

SIGNIFICANT It appears from the limited responses that codes of ethics are not enforced, and if enforced, are enforced primarily to keep control of their own members, and not to protect the consuming public. This is indicative of the professional organization functioning as a union.

INTERESTING Project managers did not measure up favorably against doctors and lawyers in terms of pro bono work. While not a direct measurement of professionalism, pro bono work speaks to the fiduciary responsibility of a professional to the consuming public in exchange for limited antitrust protection.

and financial accountability. This element bears watching as it may well further the ‘professionalization’ of project management.

Qualitative Analysis Summary

0% Confirmed enforcement for violations pertaining to the consuming public

20.2% =>40 hours (target for doctors/lawyers) Only 20.2% of project managers met or exceeded the targeted ideal for doctors/lawyers.

Quantitative Results from Survey Instrument Summarized

b. establishes acceptable standards i. performance standards ii. procedural standards (methodology)

a. enforces code of ethics

Key findings (from literature research)

This attribute was ignored for the purposes of this research, as the question of knowledge based standards (test) vs competency based standards is a study unto itself and is being researched by others.

KQ9 (substitute question) If a robust code of ethics plays a key role in determining a profession, how do various CoEs applicable to project management compare against accepted professions?

Questions generated by the key findings (KQ) – to be addressed by a variety of test questions

N/A

INDETERMINATE While there are indication that project management codes of ethics are weak in terms of focusing on the health safety and welfare of the consuming public, no correlation could be found between key words and the perceived professional image. More research needs to be done on comparing the codes of ethics to actual performance to reach more conclusive results.

no commonality or baseline yet, I arbitrarily and unilaterally gave this a value of 80%, consistent with clear and compelling evidence, but could not justify beyond a reasonable doubt, due to the questions of competency vs knowledge and methodologies vs bodies of knowledge.

experience and limited responses indicate they use the CoE primarily to control their membership.

Not a test question; investigated through key word analysis of professional society codes of ethics

Qualitative Analysis Summary

Survey questions

282 Chapter 5: Survey development and analysis, Part 2

N/A

A code of ethics adopted for project managers which clearly focuses on the health, safety and welfare of the consuming public will be a clear indication that project management is a profession.

Hypotheses based on the key findings, to be tested as part of addressing KQs

Based on key word analysis using total number of key words/total words INCOSE was ranked #1/10 ASCE was ranked #2/10 AIPM was ranked #3 of 10 PMI was ranked #9/10

Quantitative Results from Survey Instrument Summarized

Questions generated by the key findings (KQ) – to be addressed by a variety of test questions

KQ10 Do project managers publish papers in peer reviewed journals?

KQ11 Does advertising by project managers conform to other professional norms?

KQ12 Is (or should) use of the title ‘project manager’ be restricted by law?

Key findings (from literature research)

Publishing in learned journals

Advertising not permitted or restricted

Use of title is restricted by law

Key word analysis of the term ‘project manager’

N/A

SQ32 During the previous 3 years, I have written and published the following number of articles in professional journals/publications.

Survey questions

283 Chapter 5: Survey development and analysis, Part 2

As the term ‘project manager’ is so ubiquitous, there is no way at this point it could be restricted.

As few project managers work on a ‘fee for services basis’ this question was deemed irrelevant to this research..

If project management is a profession, then project managers publish in peer reviewed journals

Hypotheses based on the key findings, to be tested as part of addressing KQs

SIGNIFICANT As the definitions of ‘project manager’, ‘project manager’ or even ‘project’ have yet to be established or generally accepted, raises questions about the ability of the occupation to define boundaries or ‘turf’.

NOT RELEVANT Aside from construction management which is done primarily by firms, few individual project management practitioners provide professional services on a fee basis. As such, restrictions on advertising have little or no direct bearing on the professionalization of project management, at least at this point.

INTERESTING Project managers did not measure up favorably against doctors, lawyers and other occupations accepted as professions in terms of publishing papers.

Qualitative Analysis Summary

0% as trying to control the term at this point would be practically impossible

0% as few project managers work on a fee for services basis

20.20% Publish 1 paper per 3 years, vs 1 per year for doctors/laywers

Quantitative Results from Survey Instrument Summarized

Questions generated by the key findings (KQ) – to be addressed by a variety of test questions

KQ13 Are symbolic costumes, uniforms or other identifying insignia necessary to define a profession?

KQ14 Should project managers be licensed?

Key findings (from literature research)

Symbolic costumes/uniforms

Practice limited by govt. license

….. 72.8% of respondents do NOT believe licensing results in a restraint of trade…… SQ44 I believe occupational licensing of any type results in a monopoly and a form of restraint of trade.

While 87.10% said Yes, they believed occupation licensing will protect the consuming public…….

6.20% said Yes they were able to identify colleagues by their uniform or costume.

Quantitative Results from Survey Instrument Summarized

…. only 43% believe licensing will help deliver projects on time, within budget while substantially conforming to requirements while…

SIGNIFICANT Ultimately, licensing becomes one of the defacto criteria for an occupation to take the first preliminary steps towards becoming a profession. licensing,(limited anti-trust protection) fiduciary responsibility and financial accountability form the three primary pillars of any occupation being recognized and accepted as a profession.

SIGNIFICANT While the question of uniforms seems on the surface to be irrelevant and almost laughable, the role uniforms play in an occupation being perceived as a profession is a subtle yet serious one.

Qualitative Analysis Summary

SQ38 I believe that licensing of project managers will lead to projects being completed on time, within budget and substantially fulfilling all technical requirements.

SQ37 I believe the primary purpose of occupational licensing of any profession is to protect the public from quacks, charlatans or incompetence.

SQ17 When in a work environment, I am able to identify other project managers in my organization by the clothes they wear or some other identifiable part of their costume. (i.e. special tie, rings, headgear or other unique part of their dress or appearance)

Survey questions

284 Chapter 5: Survey development and analysis, Part 2

Licensing project managers will protect the consuming public.

Costumes and symbolic uniforms or paraphernalia are necessary to define or identify a professional project manager.

Hypotheses based on the key findings, to be tested as part of addressing KQs

Questions generated by the key findings (KQ) – to be addressed by a variety of test questions

KQ15 Should project managers be required to carry professional liability insurance?

KQ16 Given autonomy in decision making is a key attribute of a profession, do project managers have autonomy in making decisions?

Key findings (from literature research)

Requires professional liability insurance

Autonomy in decision making

SQ19 As a project manager, I feel I have ________________ autonomy in making decisions related to the execution of the Project I am responsible for.

SQ26 As a project manager, I feel I have ________________ responsibility for the decisions I make related to the execution of the project for which I am in charge.

SQ18 I believe that project managers should be held financially accountable for mistakes they/their teams are responsible for when managing a project.

Survey questions

285 Chapter 5: Survey development and analysis, Part 2

Autonomy in making decisions is a required element of a profession.

Project managers should carry professional liability insurance only if they are held responsible AND financially accountable.

Hypotheses based on the key findings, to be tested as part of addressing KQs

SIGNIFICANT Autonomy in decision making is a core element of both fiduciary responsibility and it's alter ego, financial responsibility. (How can one be held financially or morally responsible over that which there is no control?)

SIGNIFICANT Along with licensing and financial responsibility for ones errors and omissions, fiduciary responsibility to the consuming public becomes one of the defacto criteria for an occupation to earn recognition as a profession.

Qualitative Analysis Summary

On average, the respondents scored 3.85/5 or 77% that they have autonomy, while…..

…. on average, some 87% felt they had full or nearly full responsibility for the decisions they made.

Only 60.4% of respondents agreed they should be held financially accountable for their errors and omissions, while….

Quantitative Results from Survey Instrument Summarized

Questions generated by the key findings (KQ) – to be addressed by a variety of test questions

KQ17 Given Identifying with the occupation and not with an employer is a key attribute of a profession, who do project managers identify with?

Key findings (from literature research)

Identity not with employer but profession

SQ20 I relate more to being a project manager than I do to working for any particular employer

CONFLICTING The compilation of this research produced CONFLICTING results. The fact the results are in conflict does have meaning in the context of the research. It would appear as though there may be bias in the phrasing of the question.

…however, when asked about whether their clients cared about how they executed the project, the score dropped to 3.25/5 or 65%, indicating some trepidation.

CONFLICTING Along with licensing and financial responsibility for ones errors and omissions, fiduciary responsibility to the consuming public becomes one of the defacto criteria for an occupation to earn recognition as a profession.

SQ27 People who use my services as a Project Manager are not concerned about how I execute the project as long as the project is successful (using their definition of success

…77% claimed to relate more to their job as project managers,…….

While 73% said yes, project management was their life’s work, and……..

….phrased another way, 85% of respondents indicated they felt they DO have discretionary responsibility in making decisions…

CONFLICTING Trying to phrase the question another way, respondents indicated they feel they do have discretionary judgement in running their projects. However, as few have control over budgets, resources or timing, this question indicates a weakness in the wording. Perhaps within the constraints set by others they have discretion, but questionable if they have the same discretion typically ascribed to traditional professions

SQ25 In the fulfilment of my duties in the role of a Project Manager, I feel I am able to consistently exercise discretionary judgment in how the project gets executed

SQ16 I consider project management to be my life’s work

Quantitative Results from Survey Instrument Summarized

Qualitative Analysis Summary

Survey questions

286 Chapter 5: Survey development and analysis, Part 2

Project managers do not identify with their employer, but with the occupation of project management.

Hypotheses based on the key findings, to be tested as part of addressing KQs

Questions generated by the key findings (KQ) – to be addressed by a variety of test questions

KQ18 Those in existing professions enjoy high community esteem. What is the community esteem of project managers?

KQ19 Those in occupations recognized as professions earn higher than average salaries. What is the perception of project managers in terms of the salaries they earn?

Key findings (from literature research)

Held in high esteem by the community

Earn higher than average compensation

This important attribute was confirmed /reinforced by comparing the results of AACE's annual salary survey.

SQ22 Compared to other people of my age and education level, in my position as a project manager, I feel I am compensated ___________ for my services.

SQ21 As a project manager, I consider myself to be held in ______ esteem in my home (nonworking) community because of the work I do as a project manager.

SQ24 When I introduce myself to people, I would be more likely to do so by stating ‘I am a project manager who works for XYZ company’ OR ‘I work for XYZ company as a project manager’

Survey questions

287 Chapter 5: Survey development and analysis, Part 2

Project managers earn higher than average salaries.

Project managers are not held in the same high esteem as existing professionals.

Hypotheses based on the key findings, to be tested as part of addressing KQs

SIGNIFICANT From a pragmatic perspective, salary is one of the most accurate determinants of an occupation being recognized as a profession. If you are able to command a high salary in the marketplace due to demand for your particular services, then you are a professional at what you do. The danger lies if the market is artificially limited by license or other caveats.

SIGNIFICANT In terms of defining any occupation as a profession, esteem is an important one from a sociological perspective. The danger in using this as a ‘bright line’ test lies in the fact that being a perception, it is subject to manipulation and marketing hype.

Qualitative Analysis Summary

Only 38.20% consider they are compensated above average

70.40% they are held in high esteem in their community.

…..yet, when asked how they would introduce themselves, only 43% would introduce themselves as project managers,

Quantitative Results from Survey Instrument Summarized

N/A

Theattribute of highly ritualistic procedures was so patently obvious that it was given a value of zero for the purposes of the research. Not only is the body of knowledge an agglomeration coming from a multitude of sources, as most of the BoK derives from general business teachings, access is far from limited. N/A

SQ23 People not familiar with project management consider what practitioners do to be mystical and/or highly ritualistic.

Survey questions

288 Chapter 5: Survey development and analysis, Part 2

Project management does not have mystique.

KQ20 A leading intrinsic attribute for a profession is the mystique which surrounds it. Does project management have mystique?

Mystique (average of 15, 23 and 0) (highly ritualistic procedures)

Hypotheses based on the key findings, to be tested as part of addressing KQs

Questions generated by the key findings (KQ) – to be addressed by a variety of test questions

Key findings (from literature research)

IMPORTANT As shown previously, while the body of knowledge did meet the preponderance of the evidence test for being unique, it did not meet the clear and convincing evidence standard. Further research indicated the research was flawed in measuring uniqueness.

SIGNIFICANT From a semantic perspective, mystique is the element which enables an occupation to demand and receive both high salaries and respect and esteem in the community. Mystique consists of three elements: 1) The body of knowledge is difficult to master; 2) There are highly ritualistic procedures which the initiated are unable to readily follow or comprehend, and; 3) Access to the knowledge is limited, either due to the long amount of time required to obtain it, or because it is inaccessible to the average person.

Qualitative Analysis Summary

The score for highly ritualistic procedures is 0, based on the obvious.

Out of a possible 100%, respondents say project management is only 63% mystical, while 75.50% said the BoK was unique. At the same time, the score given to highly ritualistic procedures was zero, based on common sense observations. The average score for mystique looking at all three elements results in a score for mystique of only 46.2%. ((76.6% + 63% + 0%)/3)

Quantitative Results from Survey Instrument Summarized

Questions generated by the key findings (KQ) – to be addressed by a variety of test questions

KQ21 A leading intrinsic attribute for a profession is the cruciality (criticality) of the services. Does project management have criticality?

KQ22 A leading intrinsic attribute for a profession is the dénouement or the ability to produce positive results very quickly and without limited involvement of the consumer of the services. Does project management have dénouement?

Key findings (from literature research)

Cruciality a. immediacy of need b. Importance of need

Denouement quick solutions to problems. To capture the essence of Denoument, I combined the results of SQ19, SQ25 and SQ26

SQ19 As a project manager, I feel I have ________________ autonomy in making decisions related to the execution of the Project I am responsible for.

CONFLICTING This aspect of the research produced CONFLICTING or at least inconclusive results. As the dénouement is also subjective and contextual, the research as set up was flawed in not being able to capture the essence of dénouement better.

CONFLICTING This aspect of the research produced CONFLICTING or at least inconclusive results. As the cruciality is subjective and contextual, the research as set up was flawed in not being able to capture the essence of crucialty better.

IMPORTANT While the body of knowledge did meet the preponderance of the evidence test for being unique, it did not meet the clear and convincing evidence standard. Further research indicated the research was flawed in measuring uniqueness.

SQ15 Please evaluate the following terms or phrases by how unique you consider each as they are used in project management compared to general usage

SQ33 People who use the services of a project manager generally consider those services critical to the success of their project.

Qualitative Analysis Summary

Survey questions

289 Chapter 5: Survey development and analysis, Part 2

Project management does not have dénouement.

Project management does have cruciality/criticality.

Hypotheses based on the key findings, to be tested as part of addressing KQs

Respondents to SQ19 averaged 77% while respondents to SQ25 averaged 85% and 86.8% to SQ 26. Averaging all three scores, yields an 82.9% mean score for dénouement.

66.8% Mean Score 56% Agree fully Another 29% agree in part

75.6% rank PM BoK as unique

Quantitative Results from Survey Instrument Summarized

Questions generated by the key findings (KQ) – to be addressed by a variety of test questions

KQ23 How does the fiduciary responsibility of project managers compare against that of established professions?

Key findings (from literature research)

Fiduciary obligation to the public

SQ19 As a project manager, I feel I have ________________ autonomy in making decisions related to the execution of the Project I am responsible for.

SQ18 I believe that project managers should be held financially accountable for mistakes they/their teams are responsible for when managing a project

SQ26 As a project manager, I feel I have ________________ responsibility for the decisions I make related to the execution of the project for which I am in charge.

SQ25 In the fulfilment of my duties in the role of a project manager, I feel I am able to consistently exercise discretionary judgment in how the project gets executed.

Survey questions

290 Chapter 5: Survey development and analysis, Part 2

Project managers do not exhibit a level of fiduciary responsibility comparable to that of established professions.

Hypotheses based on the key findings, to be tested as part of addressing KQs

SIGNIFICANT Along with licensing and financial responsibility for ones errors and omissions, fiduciary responsibility to the consuming public becomes one of the defacto criteria for an occupation to earn recognition as a profession.

Qualitative Analysis Summary

Fiduciary obligation consists of four elements- Accountability, autonomy, responsibility and discretionary judgement.. for accountability (sq18), respondents scored only 60.4%, while for autonomy, they scored themselves 77%; discretionary judgement (sq25) got a score of 85% while sq26 (responsibility garnered 86.8%. This resulted in an average score of 77.3%.

Quantitative Results from Survey Instrument Summarized

Key findings (from literature research)

Questions generated by the key findings (KQ) – to be addressed by a variety of test questions

SQ26 As a project manager, I feel I have ________________ responsibility for the decisions I make related to the execution of the Project for which I am in charge.

SQ25 In the fulfilment of my duties in the role of a project manager, I feel I am able to consistently exercise discretionary judgment in how the project gets executed.

Survey questions

291 Chapter 5: Survey development and analysis, Part 2

Hypotheses based on the key findings, to be tested as part of addressing KQs

Qualitative Analysis Summary

Quantitative Results from Survey Instrument Summarized

CHAPTER 6

Discussion 6.1

Introduction

In Chapter 5, we explored the individual results from each of the survey questions and were able to answer the questions, Is project management a profession? (no) and If project management is not a profession, what is it? (a process or methodology). Data outlined in Chapter 5 also indicated where project management lies compared to other occupations on a continuum of professionalization of various occupations (less of a profession than being an MBA but more of a profession than being an electrician). What could not be measured was just how much more, or how much less, of a profession project management is than the occupation of electrician or the professions of medicine and law. In Chapter 6, Discussion, the results discussed throughout the thesis will be considered both qualitatively and quantitatively, and their synthesis highlighted. In Chapter 7, recommendations arising from the findings outlined in the Discussion with regard to project management will be offered to the stakeholders identified in Chapter 1. To accomplish this, the results of the survey instrument have been used to create the Professionalization Index, which can produce a true ratio scale, not only to measure how much more or less project management is a profession than the other occupations, but also to serve as a standardized approach by which to measure the standing of any occupation travelling the road to professionalization.

6.2

Summary of quantitative results

In ‘Exploring the role of formal bodies of knowledge in defining a profession – The case of project management’ (Morris et al., 2006, p. 719) the researchers identify several areas of research which this survey answers at least in part. Specifically, they ask What do the traits of project management as a profession, semi-profession or commercial profession signify to the different actors involved? Exhibit 6.1 summarizes and answers that question, at least from the perspective of project management practitioners.

292 Chapter 6: Discussion

Based on the responses to each survey question and the subsequent analysis, a numerical score was calculated for each of the 22 attributes originally extrapolated from the literature as representative of professionalization of an occupation. In analyzing the survey data, three ‘tests’ commonly used in courts of law were applied in order to assign a degree of significance to data from each individual survey question. The three tests from law are the test of: „

preponderance of the evidence (>51% but 77.5% but 95%)

Consistent with the concept of tipping points (Gladwell, 2002) and Pareto’s Law, (Pareto, 1897) it was calculated that the minimum percentage required to indicate that an occupation should be considered a profession would be 80%, or in legal parlance, ‘clear and convincing evidence’. Anything less would be to invite skepticism and doubt, even if viewed from a post-positivist perspective. While this approach is rooted in positivist thinking (the total being equal to the parts), there is simply nothing to generate any confidence that a tipping point is even close. We can see in Exhibit 6.1 that of the 22 main or primary attributes of an occupation being recognized as a profession, only one of them (dénouement) is considered to be important enough to meet the 80% or ‘clear and convincing’ evidence test. All remaining attributes only meet the ‘preponderance of the evidence’ test (yellow) or fail to even meet the preponderance test (red).

293 Chapter 6: Discussion

Exhibit 6.1 Summary of average scores from survey respondents Item

Attributes of a profession

#

Extrinsic

1

Body of Knowledge (Average of 1a + 1b)

Preponderance

Clear/ Convincing

Beyond Doubt

>51% but 77.5% but 95%

75.6%

1a

a. Unique

75.6%

1b

b. Esoteric/Complicated/Secret

75.6%

2

Long Period of Training (Average of 2A + 2B)

79.2%

2a

a. Higher Education

78.3%

2b

b. Apprenticeship/Internship/Residency

80.2%

3

Life Time Commitment

69.0%

4

Adhering to a Code of Ethics

65.0%

5

Fiduciary Obligation to the Public

0.0%

6

Service to the Public (Pro Bono Work)

20.2%

7

Professional Association (Average of 7a + 7b)

53.3%

7a

a. Enforces Code of Ethics

7b

b. Establishes Acceptable Standards

7bi

i. Performance Standards

7bii

ii. Procedural Standards

0.0% 80.0% 80.0%

8

Publishing in Learned Journals

27.9%

9

Advertising Not Permitted or Restricted

0.0%

10

Use of Title is Restricted by law

0.0%

11

Symbolic Costumes/Uniforms

6.2%

12

Practice Limited by Govt. License

52.5%

13

Requires Professional Liability Insurance

73.8%

14

Autonomy in Decision Making

74.1%

15

Identity not with Employer but Profession

69.0%

16

Held in High Esteem by the Community

70.4%

17

Earn Higher Than Average Compensation

38.2%

Intrinsic 18

Mystique (Average of 18a + 18b + 18c)

46.2%

18a

a. Body of Knowledge is Esoteric/Abstruse/Secret

75.6%

18b

b. Highly Ritualistic Procedures

63.0%

18c

c. Access to Knowledge is limited

19

0.0%

Cruciality (Average of 19a + 19b)

66.8%

19a

a. Immediacy of Need

66.8%

19b

b. Importance of Need

66.8%

20

Denouement- Quick Solutions to Problems

Does not meet preponderance of the evidence test Meets ‘preponderance of the evidence’ test Meets ‘clear and compelling’ evidence test Meets ‘beyond a reasonable doubt’ test

294 Chapter 6: Discussion

82.9%

6.3

Summary of qualitative results compared against case studies

As discussed in Chapter 3 the qualitative data relating to the professionalization of an occupation depended on how the 22 attributes derived from the literature, and considered the most salient characteristics of a profession (Exhibit 6.2) manifested themselves. Exhibit 6.2 The 22 attributes of a profession as identified during the literature research #

Attribute Extrinsic attributes

1

Professional association- defines methodology/defines standards/enforces CoE

2

Autonomy in decision making

3

Lifetime commitment/calling

4

Earn higher than average compensation

5

Publishing in learned journals

6

Long period of training including higher education

7

Subscribe to a code of ethics

8

Fiduciary obligation to public

9

Professional association defines ‘best practices’

10

Identify with occupation, not employer

11

Held in high esteem by the community

12

Apprenticeship/ internship/residency

13

Service to the public- including pro bono work

14

Practice limited by government license

15

Advertising not permitted or restricted

16

Requires professional liability insurance

17

Symbolic costumes/uniforms

18

Use of title restricted by law

19

Body of knowledge

unique/esoteric/secret (both intrinsic and extrinsic)

Intrinsic attributes 20

Cruciality- need is important/need is immediate

21

Dénouement- practitioner is trusted to produce positive results relatively quickly.

22

Mystique- highly ritualistic/access to knowledge is restricted

To assist in considering the nature of a profession from a post positivist perspective, knowing that construction project management is not considered to be a profession, although commercial aircraft piloting is, in spite of not being recognized legally as a profession, the qualitative results were compared to see if those elements could be isolated which are unique only to aircraft piloting and not construction project management or project management (Exhibit 6.3).

295 Chapter 6: Discussion

Exibit 6.3 also addresses at least in part another research question posed by Morris et al. (2006, p. 719) How important are the attributes of traditional professionalism that project managers would seem to be missing? By comparing commercial airline piloting (which is generally considered to be a profession) against construction project management we can get at least an idea of what attributes ‘count’ in today’s world vs those which are superficial or superfluous. Exhibit 6.3 Comparison between survey responses, construction project managers and airline pilots Item

Attributes of a profession

#

Extrinsic

1

Body of Knowledge

1a 1b

Project Managers (From survey Instrument)

Construction management

Commercial airline pilot

a. Unique

No

No

Yes

b. Esoteric/Complicated/Secret

No

No

Yes

2

Long Period of Training

2a

a. Higher Education

Yes

Yes

No

2b

b. Apprenticeship/Internship/Residency

No

No

Yes

Life Time Commitment

No

No

Yes

4

Adhering to a Code of Ethics

Yes

Yes

Yes

5

Fiduciary Obligation to the Public

No

No

Yes

6

Service to the Public No

No

No

3

6a

a. Pro Bono Work

7

Professional Association

7a

a. Enforces Code of Ethics

Yes

Yes

Yes

7b

b. Establishes Acceptable Standards

Yes

Yes

No

7bi

i. Performance Standards

Yes

Yes

No

7bii

ii. Procedural Standards (Methodology)

Yes

Yes

No

8

Publishing in Learned Journals

Yes

Yes

No

9

Advertising Not Permitted or Restricted

No

No

No

10

Use of Title is Restricted by law

No

No

No

11

Symbolic Costumes/Uniforms

No

No

Yes

12

Practice Limited by Govt. License

No

No

Yes

13

Requires Professional Liability Insurance

No

Yes

Yes

14

Autonomy in Decision Making

Moderate

High

Total

15

Identity not with Employer but Profession

No

No

Yes

16

Held in High Esteem by the Community

Moderate

Moderate

High

17

Earn Higher Than Average Compensation

No

No

Yes

Intrinsic 18

Mystique

18a

a. Body of Knowledge is Esoteric/Abstruse/Secret

No

No

Yes

18b

b. Highly Ritualistic Procedures

No

Some

Yes

18c

c. Access to Knowledge is limited

No

No

No

19

Cruciality (What you do for work is something I need NOW)

19a

a. Immediacy of Need

Moderate

Moderate

Yes

19b

b. Importance of Need

Moderate

Moderate

Yes

No

No

Yes

20 20a

Denouement a. Application of ‘Mysterious Principles’ for quick solutions

296 Chapter 6: Discussion

Those attributes highlighted in green are the characteristics that an occupation which is perceived to be a profession is thought to consistently exhibit, such as commercial aircraft piloting; whereas they are not exhibited by construction management. Those differences highlighted in green have been determined to be root causes, while those in yellow are viewed as being symptomatic or manifested effects rather than causative. Characteristics highlighted in blue indicate areas where commercial aircraft pilots do not do something that is common to construction project management or project management in general. Particularly worth noting is that the performance and procedural standards for commercial aircraft pilots are set by government agencies (i.e. Federal Aviation Agency (FAA) in the USA), who are independent of both the airlines and pilots unions. Interestingly, an independent organization – the Global Alliance for Project Performance Standards (GAPPS) (www.globalpmstandard.org) – has been formed to establish at least the minimum performance (competency) standards for project managers. To respond to the questions posed by Morris et al. (2006) – How important are the attributes of traditional professionalism that project management would seem to be missing? and What if anything should be done to fill the gaps? – it would certainly appear as though having an independent standard setting organization would be a gap which needs filling (p. 719). However, consistent with the Pew Task Force Report on Consumer Protection, whatever standard setting agency or entity should consist of 51% consumers (Finnochio et al., 1998, p. 60). Reviewing the key differences from Exhibit 6.3, we see the following qualitative differentiators: „

body of knowledge As has been shown in the body of this research, the body of knowledge related to project management, while certainly meeting Turner’s criteria as having a theoretical base, empirically tested and proven to be scientific and predicative, is hardly unique, esoteric or secret or mystical. On the other hand, the body of knowledge involved in aircraft piloting is unique as well as being esoteric. In addition to having a theoretical basis in the theory of flight, it is knowledge that is empirically tested by planes flying daily with few crashes being and has proven to be scientific and predicative. Consistent with Aldridge, Hugman and Labaree (as cited in Zwerman et al., 2004, p. 40), as well as Abbott (1988), control over the body of knowledge is essential to claiming professional status. 297 Chapter 6: Discussion

„

higher education Masters and PhD degrees exist for both project management and construction project management, yet one does not need even a high school diploma to obtain a commercial pilot’s license. However, for practical purposes, nearly all commercial airlines require a minimum of a Bachelor’s degree to apply for a job as a pilot, and both undergraduate and graduate level courses for pilots are offered. The official requirements to earn a pilot’s license and those required to actually obtain a job need to be taken into consideration, therefore.

„

apprenticeship/internship/residency As commercial aircraft piloting is dependent on not just a body of theoretical knowledge, but also requires a deeply practical competency, the process of ‘building hours’ is far more important than having a specific degree. All else being equal, this is one of the most important changes project management could make to increase the professionalism of the occupation. An increased focus on competency rather than merely knowledge-based certifications would enhance practice in the occupation.

„

lifetime commitment As in medicine, law and teaching, one does not see ‘accidental’ commercial airline pilots, anymore than one sees ‘accidental’ doctors or lawyers. Because of the high level of dedication necessary to obtain the specialized knowledge and build competency, a person attracted to these job specialties tends to make a long term, if not life-long, commitment to mastering the applied skills and knowledge. The commitment of the practitioners appears to be a major differentiator in the professionalization of an occupation. Like the schools of civil engineering and architecture, who gave birth to construction project management, so too we see evidence of other technical disciplines (IT, telecommunications, aeronautical engineering) offering project management courses as part of their curricula.

„

fiduciary responsibility Fiduciary responsibility is one of the more important differentiators between a mere occupation and a true profession. Fiduciary responsibility is the obligation of the practitioner to put the best interests of the client first and foremost, above those of the practitioner him/herself or of the organization employing the practitioner. This is exemplified by the principle of the pilot-in-command. Until or unless project managers can achieve this level of control and responsibility, they cannot expect their employment to be perceived as a profession.

298 Chapter 6: Discussion

„

role of professional associations For commercial pilots, the standards of performance are set by relevant government agencies. While pilot associations have considerable input into the development of the standards and methodologies, because of the global nature of commercial air transportation and the sensitivity of the general public to accidents and issues, government bodies are required to coordinate these standards and establish the minimum acceptable competencies of the pilots, who, by the very nature of aircraft piloting, may be called upon to fly through and land in jurisdictions far from where they were trained or licensed. Implicit in this is the importance of reciprocity; that is, each country must be willing to accept pilots licensed in a different country piloting a plane into their jurisdiction. For project managers, there is only one body which has embarked on developing a similar approach, which is the Global Alliance for Project Performance Standards (GAPPS). Whether this standard approach will work remains to be seen, but nonetheless, it stands as an important step in the journey towards professionalization.

„

publishing in learned journals While there are no refereed journals devoted to aircraft piloting, each and every incident involving an air disaster or even a close call is investigated and reported on, with the intent to capture lessons learned. Additionally, pilots are encouraged to file pilot reports (PIREPS) while in flight of real time conditions, such as air turbulence or icing which may negatively impact flight operations. In project management, this would be tantamount to ‘lessons learned’ being delivered in real time. So while there is less theoretical research, there is a major focus on forensic review and real time information exchange. These practices are something project management should consider carefully, especially for those wanting to increase the level of professionalism.

„

symbolic costumes/uniforms While it is unlikely that project managers will ever have a costume or uniform, this attribute should not be taken lightly. One possible way to address this is to develop a suitable competency based credential which would have a distinctive logo or pin to generate immediate recognition and respect for the holder.

„

government licensing Given that commercial aircraft piloting requires practitioners to cross through many different state, provincial or country boundaries, holding a government license makes sense, to ensure that global minimum standards of competency are

299 Chapter 6: Discussion

developed and that practitioners have met and continue to meet those standards. Although government licensing is something which is not favored, at least in the developed nations, the need for an independent organization to fulfil the same or similar role to ensure that practitioners meet some minimum levels of competency will always be necessary. As noted previously, fledgling efforts to create a global, independent set of standards for project management practitioners have been established, which seems like a step in the right direction to meet the requirement for consistent standards, provided as noted by Finnochio et al. (1998) in the Pew Task Force Report on Consumer Protection (p. 60) that significant input, participation and oversight be provided from or by consumer representation on the oversight board. „

„

„

requires professional liability insurance If a practitioner has fiduciary responsibility and somehow violates or fails to exercise this responsibility appropriately, he or she is open to litigation. In order to protect clients from insolvent practitioners, insurance is required to ensure that if a person is harmed by the actions of a practitioner, that funds to ‘make them whole’ are available. autonomy in making decisions For commercial pilots, this is the principle of the pilot-in–command, which stands as yet another key differentiator between a mere job and having your job recognized as a profession. Despite the results of the survey indicating that project managers feel they have considerable autonomy, this does not in reality seem to be the case. Clients determine the dates of performance; the clients set the fees; they determine budgets and in many cases, control over the available resources. Of all the attributes that project management seems to lack, autonomy stands as one of the major limitations. identifying with the occupation, not the employer A person who is willing to invest considerable time, money and effort to master an occupation, such as a commercial pilot, before knowing for whom he or she will work will be inclined to be loyal to the occupation rather than the employer. This attitude and behavior also reflects the nature of the present job market with its emphasis on contract work rather than devotion to a particular company.

„

intrinsic differentiators Lastly, we can see from the qualitative analysis that commercial aircraft piloting meets most if not all the important attributes related to the intrinsic or nontraditional evolution of the occupation as a profession. The body of knowledge is clearly unique, esoteric or difficult; the cruciality/criticality of the services of a

300 Chapter 6: Discussion

pilot are greater than those of a construction project manager or generic project manager; and perhaps most importantly, the average person does not pretend to know how to fly a plane, and almost certainly, without considerable training, could not possibly do so. So at least dénouement is clearly one of the most important of the intrinsic attributes that is missing in project management. To summarize the finding from Exhibit 6.3, we can see clearly that some attributes are more important than others. How practitioners perceive these attributes sets the stage to create a Professionalization Index. Consistent with the fundamentals of multiattribute modelling, by using the relative ranking of the attribute and multiplying that by the score for each of the attributes, we can develop a weighted score. I am proposing this to be the basis for turning the qualitative analysis of professionalization into a quantitative model.

6.4

Weight adjusted professionalization index

One of the primary aims from this research was to create a Professionalization Index which would serve as a standardized approach to quantifying how ‘professional’ a given occupation was at any point in time, using a true ratio scale. The need to develop the index became apparent during the literature research which showed that not only was there no ‘bright line’ test legally, but that neither economists, sociologists or semanticists could agree on a definition of the term profession. Furthermore, according to Polelle (1999), ‘Politicians preferred to stay away from such a highly charged responsibility to define which occupations are and which are not, considered professions’ (University of San Francisco Law Review, online, n.d.). Abbot (1988) based most of his theories about professional organizations around the idea that in order to be a profession, practitioners must establish the boundaries of their practice. The first step in this process is to use the body of knowledge as the basis for establishing the sphere of influence of an occupation. This was supported by Aldridge, Hugman and Labaree (cited in Zwerman et al, 2004)) who continue to argue that a ‘unique body of knowledge is the key to professional status’ (p. 40). On the other hand, Polelle, (1999), Gawley (2002) and Rotunda (2007) play down the importance of boundaries. Consistent with Freidson (1994) and MacDonald’s (1995) attempts to take a systems approach to looking at professions; consistent with Polelle’s (1999), Gawley’s (2002) and Rotunda’s (1997) search for a ‘bright line’ test; consistent with Haga (1974) and Cogan’s (1955) desire to incorporate the 301 Chapter 6: Discussion

semantic perspective, it is proposed that in the future, a weighted average model, taking the actual scores generated by the practitioners themselves, and weighted by their own valuation of each of the attributes compared against each other, should be used to produce an index of professionalization, figures from which can be used to indicate the level of professionalization reached by any particular occupation. Lacking any quantifiable approach, I adopted for this research an approach common to multi-attributed decision making. The work of Montgomery (1992) sums the approach up rather succinctly: Pure utility theory fails to address certain decision problems. Such problems arise when there exist conditions on the attributes reflecting that they are more or less worth in different situations. One example could be that a certain attribute is not worth anything before a certain utility value is reached (an apartment is worthless if it is not located within 10 kilometers from work). Another example could be that the utility of a certain attribute diminishes when its utility grows (it is more useful to come a kilometer closer to work for a person that lives two kilometers away than for a person that lives one kilometer away). In Cognitive Psychology other decision-making principles have been used as an alternative to the pure utility maximization theory. (pp. 171-172)

Montgomery (1992, cited by Brynielsson & Wallenius, n.d., online) lists the following six decision rules that can be used: „

The dominance rule means that if one strategy is better than all the other strategies in at least one attribute and at least equivalent with the other strategies in all other attributes; then this attribute should be chosen. The dominance rule seems to be obviously correct, but can only be used in some certain situations when a strategy ‘dominates’ the other strategies.

„

The conjunctive rule means that the utility in each attribute is not allowed to be below a certain threshold value that is specific for each attribute. The conjunctive rule may be especially suitable in situations where the strategies are not given in advance; but are obtained gradually. The disjunctive rule states that the utility in at least one attribute within the chosen strategy should exceed a certain threshold value that is specific for this attribute.

„

The lexicographic rule requires, unlike the previous rules, that the attributes be ranked with respect to how important they are. Making a decision means choosing the strategy that is best in the most important attribute.

302 Chapter 6: Discussion

„

Choice of the alternative with the most attractive value of a single attribute means that one chooses the strategy that has the highest utility value on a single attribute over all other attributes and strategies.

„

The addition rule requires both that attributes can be compared to each other and, unlike previous rules, that it is possible to sum together the utility values. The rule means that all the attribute utility values for a certain strategy are added together in order to choose the strategy that yields the greatest sum.

Obviously these criteria cannot be combined so that they are all fulfilled at the same time. However, it can be argued that these criteria should be used as a basis for a preference function that makes use of all these criteria to some extent. In this case, I have chosen to use the lexicographic rule which ranks the attributes in two dimensions: the actual score taken from the responses to the survey instrument and the weighting factor taken from the results of survey question 45/46 (Figure 5.3, this document) By multiplying the actual value (results from the survey questions for the 22 attributes) X the weighting factor, (which also was provided by the respondents through survey questions 45/46) the ‘addition rule’ can then be applied to compare the relative professionalization index score between any two or more occupations. This approach would provide a true zero point, meaning the ratio between any two or more occupations could be stated with certainty. Explained another way, using lexicographics and applying the additive rule, not only could the rank order of any occupation be established versus another, but how far one was from the other could also be deduced. Had this approach been completely successful, it would have been possible to measure how much more or less project management was a profession than the MBA or electrician or any other occupation covered in the research. Unfortunately, as there were insufficient respondents to the survey instrument who were not project managers (i.e respondents coming from law, medicine, commercial piloting, trades) to compare, it was only possible to calculate the value for project managers. As shown in Exhibit 6.4, the lexicographic and additive approach, with the resulting weight adjusted factor for project managers was 33.7%. Not having sufficient responses from other occupational specialties prevented the approach from

303 Chapter 6: Discussion

being validated through a comparison of the results of survey questions 30/31, 39-/42 and 47 with results obtained for other occupations. However, the approach itself remains valid and forms the basis for follow on research using multi-objective program analysis tools and techniques as the basis to validate a Professionalization Index. Exhibit 6.4 Item #

1

Weight adjusted professionalization index score

Attributes of a profession

Average PM

Extrinsic

Body of Knowledge (Average of 1a + 1b)

Weighting

Weight adjusted

Score From

Factor

Value

Survey

From Q 30-31

Col. E X Col. G

75.6%

70.7%

53.4%

1a

a. Unique

75.6%

70.7%

53.4%

1b

b. Esoteric/Complicated/Secret

75.6%

70.7%

53.4%

78.3%

53.9%

42.2%

2

Long Period of Training (Average of 2A + 2B)

2a

a. Higher Education

74.3%

58.8%

43.7%

2b

b. Apprenticeship/Internship/Residency

82.2%

53.9%

44.3%

3

Life Time Commitment

73.0%

49.6%

36.2%

4

Adhering to a Code of Ethics

65.0%

77.5%

50.4%

5

Fiduciary Obligation to the Public

62.2%

68.2%

42.4%

6

Service to the Public (Pro Bono Work)

20.2%

35.8%

7.2%

7

Professional Association (Average of 7a + 7b)

53.3%

66.3%

35.4%

0.0%

63.6%

0.0%

i. Performance Standards

80.0%

69.0%

55.2%

ii. Procedural Standards (Methodology)

7a

a. Enforces Code of Ethics

7b

b. Establishes Acceptable Standards

7bi 7bii

80.0%

69.0%

55.2%

8

Publishing in Learned Journals

20.2%

40.8%

8.2%

9

Advertising Not Permitted or Restricted

0.0%

26.1%

0.0%

10

Use of Title is Restricted by law

0.0%

49.1%

0.0%

11

Symbolic Costumes/Uniforms

6.2%

14.3%

0.9%

12

Practice Limited by Govt. License

52.5%

37.8%

19.8%

13

Requires Professional Liability Insurance

73.8%

46.8%

34.5%

14

Autonomy in Decision Making

74.1%

61.8%

45.8%

15

Identity not with Employer but Profession

69.0%

60.3%

41.6%

16

Held in High Esteem by the Community

70.4%

54.7%

38.5%

17

Earn Higher Than Average Compensation

38.2%

55.1%

21.0%

46.2%

70.7%

32.7%

a. Body of Knowledge is Esoteric/Abstruse/Secret

75.6%

70.7%

53.4%

18b

b. Highly Ritualistic Procedures

63.0%

70.7%

44.5%

18c

c. Access to Knowledge is limited

0.0%

70.7%

0.0%

Intrinsic 18 18a

19

Mystique (Average of 18a + 18b + 18c)

66.8%

67.8%

45.3%

19a

a. Immediacy of Need

66.8%

67.8%

45.3%

19b

b. Importance of Need

66.8%

67.8%

45.3%

82.9%

76.4%

63.3%

20

Cruciality (Average of 19a + 19b)

Denouement- Quick Solutions to Problems TOTAL

33.7%

304 Chapter 6: Discussion

Without more data, it is difficult to determine exactly the percentage required to confer the status of profession , but applying the previously referenced criteria, the score by project management practitioners does not even meet the ‘preponderance of the evidence’, much less approach the previously targeted ‘clear and decisive’ test of 80% Pareto, which was assumed to be a tipping point, consistent with Gladwell (2002). Applying this standard, it appears obvious that a weight adjusted score of only 33.7% doesn’t even qualify project management as meeting the preponderance of the evidence standard or test.

6.5

Considering the attributes on the whole

To conclude the assessment of the data, the various attributes are considered in the context of the summation of all the evidence: 1 Higher education vs. apprenticeship While commercial pilots are encouraged to have degrees, to become a commercial pilot and obtain work does not require a degree. however, the focus is on ‘building hours’ which implies a strong focus on demonstrated competency. 2 Lifetime commitment Research showing about 8-10% of doctors and lawyers give up their practices for whatever reasons to do something else still means that 90% or more remain as lifelong practitioners. Likewise, pilots rarely give up flying unless forced to by health restrictions. Compare this against the ‘accidental project manager’ who neither consciously seeks nor identifies with the role particularly strongly during his or her working life. 3 Fiduciary obligation to the public This was Polelle’s (1999) suggested ‘bright line’ test that courts adopt, and represents the alter ego to competency. (If one is truly competent, then he/she should not be unwilling to be held accountable by the consuming public.) 4 Symbolic uniforms While this has important implications in terms of professional identity and control, uniforms alone are unlikely to be a primary differentiator. However, as pointed out in research by Randolph, the issue of wearing a uniform is not something to be dismissed without serious consideration.

305 Chapter 6: Discussion

5 Practice limited by government license While this is one of the ‘bright line’ tests used in a court of law, as evidenced by airline cases licensing alone does not make a profession. 6 Requires professional liability insurance This is the alter-ego of fiduciary obligation. When a practitioner is competent, he/she should be willing/able to be held financially accountable for errors and omissions, but only for those errors and omissions for decisions over which he/she has direct control. 7 Autonomy in decision making This is one of the major differentiators between pilots, construction managers and project managers. The concept of ‘pilot in command’ and the implications of this authority and the inherent responsibility which goes with it are phenomenal. 8 Cruciality The services offered are important and needed immediately. While the cruciality of services provided by a commercial pilot would not likely be on par with those of an emergency room doctor, few of us voluntarily fly just for the fun of getting from point A to point B. Normally, there is some element of cruciality behind the need to fly, either in terms of importance or immediacy or both. Thus cruciality is likely to be a key differentiator. Furthermore, only a licensed pilot can fly the plane, so his or her presence is crucial. 9 Dénouement This dimension can best be described quoting Dick Holt (2000), describing the roles and responsibilities of a commercial airline pilot. He must create an aura of efficiency and capability such that the passengers stream on and off the aircraft without even a thought about what is occurring at the front of the aircraft. (Airmanship, online)

Restated, a practitioner has achieved dénouement in their craft or occupation when the person who is receiving their services remains totally aware the services are being rendered, but acts as an uninvolved bystander, trusting that the practitioner knows what he or she is doing and will do the right thing, with no questions, no challenges and no unsolicited inputs. This research started out attempting to answer two fundamental questions. 1

Is project management a profession? And if not;

2

What is it?

306 Chapter 6: Discussion

Is project management a profession? Based on both qualitative and quantitative analysis, and viewed in the context of both positivist and post-positivist perspectives, project management cannot be considered a profession. This provides quantitative support to the work of Zwerman, Thomas et al. (2004) and supports the wisdom of the recent legal ruling in the UK holding that project management is not a profession. What is it? Responses by approximately 400 global practitioners clearly indicate that project management is a process, methodology or system. That being the case, the organization most accurately reflecting this position is the International Council of Systems Engineers (INCOSE) which states: Systems Engineering is an interdisciplinary approach and means to enable the realization of successful systems. It focuses on defining customer needs and required functionality early in the development cycle, documenting requirements, then proceeding with design synthesis and system validation while considering the complete problem: operations performance test manufacturing

cost & schedule training & support disposal

Systems Engineering integrates all the disciplines and specialty groups into a team effort forming a structured development process that proceeds from concept to production to operation. Systems Engineering considers both the business and the technical needs of all customers with the goal of providing a quality product that meets the user needs.

When you consider therefore the definition of project management from Chapter 1 Project + Management The administration supervision or executive function to plan, organize, coordinate, direct and control an undertaking to achieve a particular aim or objective, with some reasonable expectation for success, through the skilful handling or use of resources and the successful organization, administration and controlling these affairs in a business-like manner, to produce results….

are there really any significant differences between the semantic definition from Chapter 1 and what INCOSE is claiming (below)? INCOSE’s Explanation- Relation to Project Management With this notion of engineering integration discussed above, it may appear that there is an inherent conflict between systems engineering and project

307 Chapter 6: Discussion

management. In practice, this conflict often leads to confusion over roles and responsibilities and, as a result, the project suffers in terms of low morale, lower productivity, and inferior quality product. In reality there should be no conflict. There is a need for both project management and systems engineering on development projects. The project manager should focus on the acquisition of resources (people, tools, facilities, funds, etc.), the protection of these resources from competing projects, and the management of these resources. He acts as the chief spokesman to upper management as to the importance and criticality of the project and how it fits into the overall strategic intent of the enterprise. The systems engineer, on the other hand, is responsible for the efficient and effective use of these resources (working closely with the program manager), in addition to making sure the system meets the needs and expectations of the client.

INCOSE further states: The International Council on Systems Engineering (INCOSE) is a not-forprofit membership organization founded in 1990. Our mission is to advance the state of the art and practice of systems engineering in industry, academia, and government by promoting interdisciplinary, scaleable approaches to produce technologically appropriate solutions that meet societal needs. Important to note is that unlike those organizations purporting to represent project management, INCOSE does NOT claim what we do is a profession, but a practice.

Again, this seems to be much more consistent with the philosophies of Plato and Hippocrates.

308 Chapter 6: Discussion

CHAPTER 7

Conclusion 7.1

Introduction

Initially, I had identified five primary objectives of this research: 1. answer the question Is project management a profession? 2. determine what project management is, if not a profession 3. ascertain where project management stands in relation to other emerging occupations claiming to be professions 4. determine what can be done, if anything, to enhance the image of the occupation 5. decide what, if anything, can be done to improve the delivery of successful projects. To start with, from a broad and diverse review of relevant literature, I identified 22 traditional and non-traditional attributes of professions. To address concerns that the traditional attributes may no longer accurately reflect the status of an occupation as being a profession, I looked to acupuncture, chiropractic and Chinese herbal medicine to see how those occupations became to be recognized as ‘professions’. The answer to that became very obvious. They earned the right to be recognized as professions by delivering consistent, positive results to the consuming public, over an extended period of time. To validate the 22 traditional and non-traditional attributes, I ‘pre-tested’ them against two occupations that I have first hand knowledge of – construction project management, which meets many of the traditional attributes of a profession but has not earned recognition as a profession equal to that of architecture or engineering, from which construction project management derived and against commercial aircraft piloting, which, despite not even requiring a high school diploma, has earned the recognition and status as a highly ranked professional occupation. This comparison led to the creation of a Professionalization Index, which would enable a quantitative analysis of any occupation based on the actual score for each of the 22 attributes (from the respondents of the survey) X the relative importance of that attribute to the occupation (also ranked by the respondents to the survey instrument)

309 Chapter 7: Conclusion

Using the 22 traditional and non-traditional attributes, I developed a web based survey instrument, designed to compare how project management and non-project management practitioners viewed project management. This survey attracted nearly 400 respondents over a nine month period, from nearly all continents and subcontinents. Using the results of this global survey as the basis and applying a variety of both qualitative and quantitative analyses, I can conclude that: 1. Project management is not yet a profession. 2. It is a process, methodology or system, and as such may never become a profession. 3. Project management is considered by practitioners to be less of a profession than being an MBA but more of a profession than being an electrician, with the ranking closer to MBA than to electrician. 4. That in order to enhance the image (‘professionalization’) of project management, it becomes imperative that, like chiropractic, acupuncture and Chinese herbal medicine, project management practitioners need to focus on improving the consistent, repeatable delivery of ‘successful’ projects. And that in order to realize more successful projects, we need to 5. Match project manager accountability with a commensurate level of authorityover time, resources and quality by enabling the project managers in our organization to make autonomous decisions in the best interests of the consuming public. The more the employer and employee contract and communicate with one another in an open and honest fashion, and the more the principles of contract are applied in accordance with agency theory (Muller & Turner), the more professional the context of project management will become and the more professional the project manager. An alliance contract would ensure that all parties take responsibility for their part in the project and in the decision making and should be the contract of choice for project managers if they aspire to professionalization. Unfortunately, because the survey instrument did not attract sufficient number of nonproject management respondents, the creation of a Professionalization Index was only partially successful. Although I was able to calculate a Professionalization Index value for project managers, I was not able to validate that value against other known professions. However, with the inclusion of the non-traditional attributes of a

310 Chapter 7: Conclusion

profession, I believe the concept of a Professionalization Index to be an appropriate means of quantifying the relative professionalization of any occupation, and would urge others to consider using it in the future. The need for some means to consistently measure the relative professionalization of an occupation becomes important, especially in the context of a ‘bright line test’ for the purposes of establishing legal accountability, and for this reason, despite not having been fully successful in testing and validating the Professionalization Index, I believe the approach I have taken in creating one represents something new and important to economists, sociologists, semanticists and legal experts alike and goes well beyond the application only to project management.

7.2

Limitations of this research

As the common quote goes, ‘hindsight is always 20:20 vision’ and in the case of this effort, it is no different. While many of the original objectives of the research were achieved, others were not when unexpected events took place or unanticipated results were generated. Some pieces of information were also perplexing, and the scope of the research did not allow their investigation. 7.2.1 Lack of significant demographic differences Few observations were of greater surprise than that fact that there were no statistically meaningful differences between any of the demographic sub-sets. I ran correlations on the data at the global level, on the question basis and then again on selected population subsets where I thought there should be some differences, but did not turn up anything which could lend insight or help in identifying any trends. This still surprises me and has left me with more questions than it answers. Another aspect of the demographics which proved to be disappointing was the fact that insufficient non-project managers responded to the survey. This limited comparisons of other occupations to project management. Despite significant efforts to seek out other professions not normally associated with project management (teachers, aircraft pilots, financiers and nurses), only 10 non-project management responses were received. There was no meaningful data generated when correlations were run on this sub-group.

311 Chapter 7: Conclusion

7.2.2 Bias in the key word analysis of the body of knowledge In setting up the survey instrument, the most frequently occurring words from the project management body of knowledge were selected to form the kernel of questions about BoK. and asked whether or not they were considered ‘unique’ to project management, expecting that the demographics would enable me to eliminate any bias or at least identify it. Because of this, I believe the score on the body of knowledge is too high and needs to be used with caution. I tried to mitigate this with secondary research on the sources of many of these terms, which supports my concern that the quantitative results from this survey contain bias, but does nothing to help arrive at the ‘right’ or appropriate value. 7.2.3 More research on the effectiveness of licensing As both the Ukraine and South Africa have implemented the licensing of construction project managers, it would have been interesting to review research on if and how licensing had been successful. Anecdotal evidence from talking with Dr. Sergei Bushyev and several of his ‘Young Crew’ students from the Ukraine indicated that the licensing processes were working, but no research supporting his opinion could be found. Reports from South Africa, which has only had licensing in place for three years as this paper is written, indicate not all is going smoothly, but, again there is no research evidence to either confirm or disclaim these observations. 7.2.4 Development of a true ratio scale While it is pleasing that an interval scale could be developed during the course of this research against which the standing of project management vis a vis other occupations could be measured, in hindsight it seems that a true zero scale could have been created. Considering the data upon completion of the study, it became apparent that doctors were consistently appearing at the top end of the scale. Therefore, instead of seeking out a baseline at the bottom, it would have been more useful to doctors at the top as zero and work backwards. 7.2.5 Data mining to research ethics questions It is difficult to set up a survey instrument to test for ethics. As demonstrated by the classic example of the lawyer who asks the defendant ‘Have you stopped beating your wife yet?’, no matter how the defendant answers, he is damned. The same

312 Chapter 7: Conclusion

analogy applies to asking about ethics. If the question is asked outright, then the respondent would be a fool to answer negatively, even if that was the honest answer. If the question is posed more subtly, the response becomes subject to a myriad interpretations. This dilemma proved very challenging and methods of dealing with it were not, on the whole, satisfactory. Of all the exciting opportunities that were stumbled upon during the research, the question linking the various codes of ethics to the real and perceived perception of an occupation as a profession seemed the most interesting. Were the research to be conducted again, more attention would be paid to the actual records of the organizations in how many ethical violations they processed and the actual nature of those violations. Experience indicated that Haga (1974) is correct in pointing out that professional organizations appear more concerned about using the organizational codes of ethics as a means of controlling their own membership than they are with protecting the consuming public from quacks, charlatans and frauds.

7.3

Reflections

7.3.1 Project management: Not a profession Applying the ‘clear and convincing evidence’ test, and looking at both extrinsic and intrinsic attributes, this current study appears to have been successful in quantifying what Zwerman et al. (2004) have established qualitatively: that project management is not now a profession. The fact that organized project management, as evidenced by the existence of professional organizations, has been around for well over 50 years, and based on the collective opinions registered by practitioners, it appears that Zwerman et al.’s (2004) finding that this occupation will most likely never become a fully-fledged profession is correct. The current study has been successful, however, in identifying where along a professionalization continuum project management dwells in relation to other occupations. 7.3.2 Observations from the literature From the literature research, came several profound revelations that, while not addressed specifically to project management, have such a direct bearing on the study that they demand inclusion in this concluding discussion.

313 Chapter 7: Conclusion

Perhaps the most important observation with regard to this current study was that of Eliot Freidson (1994), who notes that: Without sufficient time, equipment, assistance and other resources, once cannot do one’s work well and one’s freedom to employ discretion becomes meaningless. If there are not enough resources available and the work must be done in any case, it is inevitable that it can be done only by the use of short cuts and gross formulas that border upon mechanical and only marginally acceptable technique. This threatens the professional character of the work itself. (p. 210)

This statement accurately describes the nature of project management in nearly all its manifestations, and is a good indication of why project management cannot be a profession. Freidson (1994) goes on to say: Professionalism can only flourish when practitioners in an organization have a firm, but by no means absolute support, from their employer for the consequential exercise of judgment independent of their clients. (p. 211)

Unless project managers and other interested parties can change the mindset of clients who want projects done faster, cheaper, better, and the attitudes of employers who pander to unrealistic demands from clients, the prospects for ever seeing the professionalization of project management remain dim at best. This is further exacerbated by the ‘flat world’ scenario described by Friedman (2006) or the ‘98.5% factor’ of Tom Peters, (2003, p. 50) echoed by Tom Osenton (2004) and Tom Maloney (2004), where most employment will part time in a contingent work force, and project management will not be a profession but a way of life. There will no longer be jobs; everyone will be working on projects (Casey, 1995; Sabel, 1995). Despite this being an academic research project, as a successful project management consultant and teacher, it would be remiss of me not to conclude with some observations that may guide the activities and thinking of all categories of major stakeholders identified earlier in the thesis. This also is consistent with the question posed by Morris et al. (2006, p. 719) What if anything, should be done to fill the gaps?

314 Chapter 7: Conclusion

7.3.3

For those stakeholders who believe project management is NOT a profession

ANALYSIS OF THE RESULTS

Just because project management is not a profession does not mean the world is no longer going to need professional managers capable of initiating, planning, executing controlling and closing projects. The words of Roland Rotunda (1997) have special meaning for project management: The only fruitful use of the term profession today relates to individuals, not groups. (Lexus-Nexus)

This opinion has been backed up by research by the American Bar Association. It is further supported by Freidson (1994), who, in predicting the future of professionalism, concludes: If we are to be mere passive employees, without a strong organized voice in the allocation of resources that are essential for doing good work, we will find it difficult to remain committed to doing good work. If we are to play the role of merely providing whatever is demanded by consumers and authorized by those who pay for it, we will find it difficult to preserve a sense of the value of our schooled judgment. If we are to be merely loyal servants of the interests of our employers or of our own businesses, then we will have difficulty sustaining any independent commitment to serving the good of both individual clients and the public. And if we are to be required to work within an ultimately mechanical, albeit permissive standard established and enforced by professionals who act as our administrating and cognitive superiors, we will have to forsake the communal or collegiate principles that are distinctive of the professional mode of organizing work. (p. 215)

Given the words of Rotunda (1997) and Friedson, (1994), practitioners have no need to worry about whether project management is or is not recognized as a profession. What is more important is that the truly dedicated practitioner focuses on being a professional in creed, word and action. FOLLOW ON RESEARCH TOPICS: FOR THOSE WHO BELIEVE PROJECT MANAGEMENT IS NOT A PROFESSION

To understand the future role of project managers, the work of John Sterman (2003) at MIT’s Sloan School should be considered, along with that of the Santa Fe Institute. Furthermore, the work of several members of the project management community is worth examining. Terry Williams (2002) with his ‘Modeling of complex projects’, as well as Schlomo Globerson (2005), Avraham Shtube (2005) and Johathan Bard

315 Chapter 7: Conclusion

(2005), are all exploring the future of project management, featuring the use of simulation. And the very essence of simulation is that no two models will be exactly the same. This means that instead of the approaches taken by PRINCE2 and AACE’s Total Cost Management Framework, there will be many possible processes, each one tailored to meet a specific project. Analogous to a pilot being required to take so many hours of simulated flight training, future research needs to be done to explore professionalism in the context of simulated project management. In support of this, simulation training should be incorporated into all educational programs and into the knowledge based examinations. 7.3.4 For those who believe project management IS a profession There will remain individuals who believe that project management is a profession in spite of the research reported here. It behooves them to strengthen the practices’s standing and professionalism. Michael Polelle (1999) has proposed a ‘bright line’ incentive, built around fiduciary responsibility as being the true differentiator between occupations being merely businesses and those seeking justification in claiming to be a profession. He states: In conclusion, the fundamental and implied bargain between a free society and its professions is defined by effective codes of fiduciary ethics that rise above the minimal ethical norms of other occupations in that society. (University of San Francisco Law Review, online)

He goes on to explain further that: In return for allowing professions the special privileges of self policing and peer review, society rightfully demands something in return. That ‘something’ is by vigorously policing our own for fiduciary violations of express codes of ethics, the professions relieve the state (court systems) from having to deal with the nuances of problems best addressed within the profession. (University of San Francisco Law Review, online)

Summarized, Polelle (1999) sees a true professional as a guardian: interposed between a state, ever inclined to extend its centralized power on one hand, while ensuring that the consumer of the goods and services provided by the professional is protected on the other, which results in more freedom for everyone in the society. (University of San Francisco Law Review, online)

316 Chapter 7: Conclusion

This same theme is addressed by Freidson (1994) in his description of a ‘professional as a trusted servant’, where he states: professionalism entails commitment to a particular body of knowledge and skill, both for it’s own sake and for the use to which it is put, which is to say, commitment to preserve, refine and elaborate that knowledge and skill, to do good work, and, where it has application to worldly problems, to perform it well for the benefit of others – to do Good Works. (pp. 208-210)

For those who wish to enhance the professional image of project management focus might best be directed to ‘doing good works’. An example of this is the American Society of Civil Engineers committing to zero tolerance on corruption in construction projects or their commitment to sustainable development. Attention to ‘doing good works’ would include incorporating appropriate elements of the millennium development goals into a professional organization’s mission statement and/or code of conduct/code of ethics and attempting to realising them. Like Polelle, Freidson has developed several ‘bright line tests’ which he proposes organizations employ to see if their employees have the discretion to qualify as being true professionals or whether they are they merely well educated mechanics or technicians. Freidson (1994) asks: „

Can employed professionals undertake activities which violate established organizational policies which they (professional) believe to be against the best interests of their client or the public?

„

Does the fiduciary relationship professionals have with their employer supersede the fiduciary relationship with the client or with the public at large?

„

Is it corporate policy to substitute the discretionary judgment of the consumer over that of the professional?

„

Does the professional have sufficient time, equipment, assistance and other resources to perform the work up to professional standards?

„

Is there a non-hierarchical method of regulating the execution of work based not on standardized methodologies but on peer reviewed ‘best practices’? (p. 212)

To the questions developed by Freidson, another can be proposed: Would you, as an individual, be willing to put your own savings and professional reputation on the line for the project you are currently working on?

317 Chapter 7: Conclusion

For those individuals committed to raising the professional image of project management, these are the areas which, if appropriately addressed, are most likely to raise the real or perceived professional image of project management. When the point has been reached at which a clear and unequivocal ‘yes’ can be answered in response to each of the above questions, then project management can claim to be a profession. To conclude this recommendation, from Christopher McKenna (2006) is supplied. In his current book The world’s newest profession – Management consulting in the 20th century, McKenna notes: ‘The world’s newest profession will have to wait until the next (21st) Century…’ (p. 251). It would seem that Project Management too will have to wait until the 21st Century to realize any dreams for the professionalization of what it is we do, and that like Chiropractors, Acupuncturists and Chinese Herbal Medicine, the success of professionalizing project management will hinge directly on our ability to deliver value to the end user, consistently and repeatedly. FOLLOW ON RESEARCH FOR THOSE WHO BELIEVE PROJECT MANAGEMENT IS A PROFESSION

In the development of this research, a simple ‘key word’ analysis of various codes of ethics/codes of conduct was undertaken. There is considerable value to be gained by looking at key words or key phrases to determine the relative sophistication of a collection of practitioners and their relative positions on a professionalization index, like that developed in this study. While the results of the limited key word analysis proved inconclusive in the current study, more sophisticated software and a focus on key words as an aspect of a research project might generate some very interesting results with regard to the way professions operate and see themselves. Consistent with the concepts of capability maturity models, the examination of key words in a context of semiotics seems ripe for further exploration and analysis. Such an examination may well prove to be an important element measuring the capability or maturity of an organization. Conducting key word or key phrase analysis from large numbers of significant documents, when placed in context of the ‘bright line tests’ of professions could potentially produce results that could or should play a prominent role in the crafting of vision, mission and value statements, as well as codes of conduct/codes of ethics.

318 Chapter 7: Conclusion

Another important area of research for those who believe project management to be a profession lies with measuring the success of projects. While the Standish (2004), Gartner (2005) FMI (2005) and more recently, Parity Group (2007)research all point to less than exemplary success rates, what should we be measuring? Is ‘on time, within budget, conforming to specifications while delivering the functionality which was desired’ reasonable and sufficient? More research needs to be done on defining what can ‘reasonably’ be expected, assuming the five ‘bright line tests’ identified by Freidson (1994) are answered affirmatively. 7.3.5 For those who believe it doesn’t matter Individuals who believe the professional image of project management doesn’t matter would appear to be either lying to themselves, to have a hidden agenda or to be passing through project management as a step on a career path to someplace else. For those who see project management as a small element in a long career path, its level of professionalism should actually matter very much. Assuming Tom Peters (2003), Tom Osenton (2004), Tom Maloney (2005) and Tom Friedman (2006) are even close to correct in their collective predictions about the future or work, project management is going to become far more pervasive than it is now as a delivery system of choice. Freidson (1994) notes: we are living in a time when many are looking closely at the professions, their costs and benefits and when a variety of methods are being considered to make their services more accessible to those who need them and to provide assurance that they are reliable and well considered. (p. 214)

For any individuals claiming the level of professionalization in project management doesn’t matter, but who speak for or represent professional organizations, the question is: Why, then, do so many organizations purporting to represent project management make unsubstantiated claims that it is already a profession, when there is no empirical evidence supporting these claims? Those representing the various organizations intent on representing the ‘hearts, minds and money’ of those practicing project management, need to look more closely at the work of Haga (1974) and Abbott (1988).

319 Chapter 7: Conclusion

In 1974, Haga was especially forthright (almost brutally so) as he wrote: The utility of distinguishing true professions from merely would be professions can be appreciated when one looks at why so many occupations pursue the elusive status of profession. Here are the chief things that occupational groups seek when they undertake professionalization: –

Above all else, they want autonomy- the freedom to carry out their jobs as they see fit and appropriate;



They want recognition of themselves, not based upon the name of their employer, but based upon their occupational identity;



They want the power to determine who is ‘in’ their group and who is ‘out’. They want to establish a monopoly over their work, freeing it from outside influence. They do not want to share the occupations work ideology;



They want the power to discipline ‘wayward’ colleagues who do not share the occupations work ideology;



Autonomy is the key and the key to autonomy is intimidation – of clients; of employees; of members; of anyone who stands to threaten the occupation’s autonomy. (p. 7)

Freidson (1994) states: Professionalism is under attack today….When leaders of any profession invoke ethics and the values of professionalism critics declare it a self serving ideology that masks the reality of naked self interest….(p. 169)

Abbot (1988) was also rather blunt in his restatement of Theodore Caplow’s theory about the role of professional organizations, which he says ‘set up a code of ethics to, amongst other things, reduce internal competition’ (p. 11). Abbot (1988) goes on to further discuss the social and cultural controls: A profession is not prevented from founding a national association, because others exist. It can create schools, journals, ethics case as well, but it cannot occupy a jurisdiction without either finding one vacant or fighting for one. (p. 86)

Implicit in this is a commercial interest by the association itself in ‘global domination’. Especially in the developing nations, first hand experience has shown that professional organizations are being used by some, with the explicit or implicit knowledge of the leadership, as a front for business interests. The fact that organizations purporting to represent the practitioners of project management are

320 Chapter 7: Conclusion

perceived to be doing so ‘for the money’ destroys the efforts of those who truly want to professionalize project management, by losing the focus described earlier as ‘Doing Good Works’. For this reason, I would urge any organizations which have amassed money far in excess of one years operating capital, to turn the remaining money back to the consuming public in the form of funding for projects in support of the millennium development goals. FOLLOW ON RESEARCH

There is a burning need for further research on how the various professional organizations representing the practitioners of project management are being run. As most of them hold not for profit status, yet many of them are in fact, big businesses, there are ethical questions which need to be explored in greater detail. There are also questions about how much money is necessary and appropriate for a not for profit professional organization to carry forward in cash or near cash instruments. Other areas for research on the role of the professional organization are whether or not the organization is obligated to reflect the values of its members/owners? Can an occupational specialty cum profession have one set of beliefs while the organization which represents it has another?

321 Chapter 7: Conclusion

References Abbott, A. (1988). The system of profession: An essay on the division of expert labor. Chicago: University of Chicago Press. Air Line Pilots Association, Int’l. (n.d.). A strong voice for airline pilots. Retrieved May 27, 2007, from http://www.alpa.org/?tabid=139. Air Line Pilots Association, Int’l. (n.d.). ALPA’s vision statement, strategic goals & initiatives. Retrieved May 27, 2007, from http://www.alpa.org/?tabid=2170. Air Line Pilots Association, Int’l. (n.d.). Code of ethics. Retrieved May 27, 2007, from http://www.alpa.org/?tabid=270. Air Line Pilots Association, Int’l. (n.d.). Mission statement. Retrieved May 27 2007, from http://www.alpa.org/?tabid=188. Aircraft Owners and Pilots Association (AOPA). Advocacy Website. Retrieved September 6, 2006, from http://www.aopa.org/advocacy/ American College of Clinical Engineering (n.d.). Clinical engineer (defined). Retrieved May 27, 2007, from http://www.accenet.org/default.asp?page=about, §ion=definition. American Institute of Architects (AIA). (n.d.). AIA contract documents. Retrieved September 6, 2006, from http://www.aia.org/docs_default. American Institute of Architects (AIA). (n.d.). History of the American Institute of Architects. (n.d.). Retrieved August 31, 2006, from http://www.aia.org/about_history. American Institute of Architects (AIA). (n.d.). The role of ethics in sustaining the profession: More than just words on paper. Retrieved August 31, 2006, from http://www.aia.org/about_antitrust. American Institute of Constructors (AIC). (n.d.). Code of ethics. Retrieved September 6, 2006, from http://www.aicnet.org/about/code_of_ethics.asp. American Institute of Constructors (AIC). (n.d.). Mission statement. Retrieved September 6, 2006, from http://www.aicnet.org/about/mission_statement.asp. American Medical Association. (n.d.). Community service projects. Anecdotal information regarding planning community service projects by the AMA Retrieved September 6, 2006, from http://www.ama-assn.org/ama/pub/category/8650.html. American Society of Civil Engineers (ASCE) Publications (n.d.). Engineers Joint Contract Documents Committee (EJCDC) contract documents. Retrieved September 6, 2006, from http://ascestore.aip.org/shop.do?cID=4. American Society of Civil Engineers (ASCE). (14 May 2004). Policy on Use of the Title Engineer. Retrieved May 27, 2007 from http://www.asce.org/pressroom/news/policy_details.cfm?hdlid=29. American Society of Civil Engineers (ASCE). Code of Ethics. Retrieved September 6, 2006, from https://www.asce.org/inside/codeofethics.cfm.

322 References

American Society of Civil Engineers (ASCE). Policies related to the Code of Ethics. Retrieved May 27, 2007 from http://www.asce.org/pressroom/news/policy_details.cfm?hdlid=29. American Society of Civil Engineers (ASCE). Policy on Licensure. Retrieved September 6, 2006, from http://www.asce.org/professional/licensure/ American Society of Civil Engineers (ASCE). Zero Tolerance Policy. Retrieved September 6, 2006, from http://www.asce.org/professional/zero.cfm. AskCaptainLim. (n.d.). Airborne with the Captain. Retrieved May 27, 2007, from http://www.askcaptainlim.com/ashowsafe1.htm#How%20safe%20is%20flying. Associated General Contractors of America (AGC). (n.d.). About contract documents. Retrieved September 6, 2006, from: http://www.agc.org/page.ww?section=Contract+Documents& name=About+Contract+Documents. Association for the Advancement of Cost Engineering International. (2004). Annual salary survey 2004. Retrieved February 4 2006, from http://www.aacei.org/ resources/salarysurveys.shtml. Association for the Advancement of Cost Engineering International (AACE). (n.d.). Canon of ethics. Retrieved September 6, 2006, from http://www.aacei.org/ membership/about/CanonEthics.shtml Barabasi, A. L. (2003, April 29). Linked: How everything is connected to everything else and what it means. Plume. Reissue edition. Belgum, D. (2001, July 16). Medical association reports rise in physician discontent. Los Angeles Business Journal. Retrieved May 23, 2006, from findarticles.com/p/ articles/mi_m5072/is_29_23/ai_76738359. Bianco, D. J., & Moran, A. (Eds.). (1996). Professional and occupational licensing directory. USA: Gales Publishing. Blair, R. D., & Rubin, S. (Eds.). (1980). Regulating the professions: A public policy symposium. Lexington, MA: Lexington Press. Blueprint for improving the civil justice system 12. (1992). Polelle: American Bar Association Working Group on Civil Justice. Bond, M. H. (1996). Chinese values. In M. H. Bond (Ed.), The handbook of Chinese psychology (pp. 208-226). Hong Kong: Oxford University Press. Bond, M. H., & Hwang, K. K. (1986). The social psychology of the Chinese people. In M. H. Bond (Ed.), The psychology of the Chinese people (pp.213-66). Oxford University Press, Hong Kong. British Broadcasting Company. Hardtalk. Retrieved September 6, 2006, from http://news.bbc.co.uk/1/hi/health/3149786.stm.

323 References

Brynielsson, J., & Wallenius, K. (2003 December). A toolbox for multi-attribute decision-making. Report number: TRITA-NA-0307. Stockholm, Sweden: Department of Numerical Analysis and Computer Science (NADA), Royal Institute of Technology. Retrieved May 7, 2007, from http://www.nada.kth.se/~joel/TRITANA-0307.pdf. Brynielsson, J., & Wallenius, K. (2003, December). A toolbox for multi-attribute decision-making. Report number: TRITA-NA-0307. Retrieved 7 May 2007, from http://www.nada.kth.se/~joel/TRITA-NA-0307.pdf Accessed 07 May 07. Carnegie Foundation. Initiative on the Doctorate. Retrieved May 27, 2007, from http://www.carnegiefoundation.org/CID/ Carroll, S. L., & Gaston, R. J. (1983). Occupational licensing and the quality of service: An overview. Law and Human Behavior, 7, 139-146. Casey, C. (1995). Work, self and society in post industrialism London. London: Routledge. Chan, W. M., & Kumaraswamy, M. M. (1999). Forecasting construction durations for public housing: A Hong Kong perspective. Building and Environment, 34(5), 633646. Chaos report. (1994). Retrieved September 12, 2006, from Standish Group Website: http://www.standishgroup.com/sample_research/chaos_1994_1.php Chicago Tylenol murders. Wikipedia. Retrieved 10 November 2007, from http://en.wikipedia.org/wiki/1982_Chicago_Tylenol_murders. Cleland, D. I. (1999). Project management: Strategic design and implementation (3rd ed.). New York: McGraw-Hill. Code of ethics. (n.d.). Retrieved September 6 2006, from Royal Chartered Institute of Building Website: http://www.ciob.org.uk/ciob/ Code of ethics. (n.d.). Retrieved September 6 2006, from Royal Institution of Chartered Quantity Surveyors Website: from http://www.rics.org/ Cogen, M. L. (1955). The problem of defining a profession. Annals of the American Academy of Political and Social Science, 297, 105-115. Cohen, P., & Sayer, B. (2001). Avoiding software development failure. HCI Journal. Retrieved September 12, 2006, from http://www.hci.com.au/hcisite2/journal/Avoiding%20software%20development%20f ailure.htm . Computer Sciences Accreditation Board (CSAB). (Last updated 28 May 1997). Defining the computer science professions. CSAB White Paper. Retrieved February 4, 2006, from http://www.csab.org/comp_sci_profession.html. Concise Oxford English Dictionary. (2004). Oxford: Oxford University Press Conrad D. A., & Sheldon, G. G. (1982). The effects of legal constraints on dental care prices. Inquiry, 19, 51-67.

324 References

Construction Industry Institute. (online, n.d.). CII Best Practices. Retrieved June 17, 2007, from http://www.constructioninstitute.org/scriptcontent/bp.cfm?section=aboutcii. Construction Management Association of America. (online, n.d.). Code of professional ethics for the construction manager. Retrieved May 27, 2007, from http://cmaanet.org/ethics.php. Construction Management Association of America. What is Construction Management? Retrieved May 27, 2007, from http://cmaanet.org/cm_is.php. Construction Specifications Institute. (n.d.). Professional development. Retrieved September 6 2006, from http://www.csinet.org/s_csi/sec.asp?TRACKID =&CID=6&DID=6. Cooper, K. C. (2000). Effective competency modeling and reporting (1st ed.). New York: AMACOM/American Management Association. Covey, S. (2004). The 7 habits of highly effective people. London: Free Press Cox, C., & Foster, S. (October 1991). The costs and benefits of occupational licensing. Report prepared for the US Federal Trade Commission. US Bureau of Economics, Federal Trade Commission. Crane, D. (2000). Fashion and its social agendas: Class, gender, and identity in clothing. Chicago: University Of Chicago Press. Crawford, L. H. (2002). Profiling the competent project manager. In D. P.Slevin, D. I. Cleland, & J. K. Pintok (Eds.), The frontiers of project management research. Newton Square, PA: Project Management Institute. Crawford, L. H. (2005). Senior management perceptions of project management competence. International Journal of Project Management 23(1), 7-16. CSI masterformat. (n.d.). Retrieved May 27 2007, from Construction Specifications Institute Website: http://www.csinet.org/s_csi/sec_bookstore.asp?TRACKID=& CID=22&DID=22. Cullen, J. B. (1978). The structure of professionalism: A quantitative examination. New York, NY: Petrocelli Books. Curling, D. H. (online, n.d.). The (accidental). profession of project management. Retrieved May 6, 2007, from http://www.maxwideman.com/guests/globalization/profession.htm D’Agostino, D. (online, n.d.). Cincinnati City Beat. Retrieved September 6, 2006, from http://www.citybeat.com/2000-04-27/cover.shtml. Davis, M. J., & Elliston, F. (1986). Ethics and the legal profession. New York, NY: Prometheus Books. Deming, J. E. (2000). Out of the crisis. Boston: MIT Press. Donaldson, T. (1995). The stakeholder theory of the corporation: Concepts, evidence, and implications. Lee E. Preston Academy of Management Review, 20(1), pp. 65-91.

325 References

Donaldson, T., & Dunfee, T.W. (1999). Ties that bind: A social contracts approach to business ethics. Boston: Harvard Business School Press. Donaldson, T., & Preston, L. F. (1995). The stakeholder theory of the corporation: Concepts, evidence, and implications. The Academy of Management Review, 20(1), 65-91. Drucker, P. (1954). The practice of management. New York, NY: Collins. Drucker, P. (1974). Management tasks, responsibilities and practices (reprint ed.). New York, NY: Collins. Drucker, P. (1999). Management challenges for the 21st century (1st ed.). New York, NY: Collins. Drucker, P. (2001). The essential Drucker: In one volume the best of sixty years of Peter Drucker’s essential writings on management (1st ed.). New York, NY: Collins. Drucker, P. (2003). On the profession of management. New York, NY: Collins. Dunfee, T. W., Smith, N. C., & Ross, W. T. Jr. (1999). Social contracts and marketing ethics. Journal of Marketing, 63(3), 14-32. Ehrenberg, S., & Valentine, S. (n.d.). Second restatement of torts. Retrieved September 2 2006, from Kent Law website: http://www.kentlaw.edu/academics/lrw/ tutorials/restate.htm. Emmanuel, L., & Baker, R. (2000). The efficacy of professional ethics: The AMA code of ethics in historical and current perspective. The Hastings Center Report, 30(4), S13-S17. Employee Retirement Income Security Act 1974 [Pub.L. 93-406, 88 Stat. 829, September 2 1974] Retrieved June 17, 2007 from www.dol.gov/ebsa/compliance_assistance.html. Encarta Electronic Encyclopaedia. Software last updated May, 2007. Eras Ltd. (May 2003). Licensing and the UK engineering profession. Report for the Engineering and Technology Board. London: Eras. Establishing the fiduciary responsibilities required under Employee Retirement Income Security Act of 1974 (ERISA). as the baseline or benchmark against which to judge profession fiduciary responsibility. (n.d.). Retrieved September 9, 2006, from US Department of Labor Website: http://www.dol.gov/dol/topic/health-plans/erisa.htm. Feldman, R., & Begun, J. W. (1985). The welfare cost of quality changes due to professional regulation. The Journal of Industrial Economics, 34(1), 17-32. Finnochio, L. J, Dower, C. M., Blick, N. T., & Gragnola, C.M. and the Task Force on Health Care Workforce Regulation. (1998). Strengthening consumer protection: Priorities for health care workforce regulation. Pew Health Care Commission Report. San Francisco California: Pew Research Centre. Forman, E. H., Thomas, L., Saaty, M. S., & Waldron, R. (1983). Expert choice, decision support software. McLean, Virginia: Self-published.

326 References

Forrester, J. W. (1988). Designing social and managerial systems. Working paper (D4006-1). System Dynamics Group, Sloan School. Cambridge, Massachusetts: Massachusetts Institute of Technology, pp. 1-10. Frame, J. (2002). The new project management: Tools for an age of rapid change, complexity, and other business realities, (2nd ed.). New York: Wiley, & Sons. Frazier, I., & Rodriguez, B., (2006). Growth and collaboration for a project management profession. Conference paper, presented at the PMI-PMSA Conference, Midrand, Gauteng, Johannesburg, South Africa, May 30- Jun 1, 2006. Freidson, E. (1994). Professionalism reborn: Theory, philosophy and policy. Chicago: University of Chicago Press. Freidson, E. (2001). Professionalism: The third logic. Chicago: University of Chicago Press. Friedman, T. L. (2005). The world is flat: A brief history of the twenty-first century. New York: Farrar, Straus and Giroux. Friedson, E. (1988). Professional powers: A study of the institutionalization of formal knowledge (reprinted.). Chicago: University of Chicago Press. Garner, B. A., & Black, H. C. (1999). Black’s law dictionary (8th ed.). West Publishing Company. Gawley, C. J. (2002). Protecting professionals from competition: The necessity of a limited antitrust exemption for professionals. South Dakota Law Review, SDL Rev 233. Gibb, C.L. (2006, December). Learning in Health and Social Care, 5(4), 169-180. Glawell, M. (2002). The tipping point: How little things can make a big difference (reprinted.). Boston: Back Bay Books. Gnabasik, M. (2002). Smart choices: Selecting and administering a safe 401(k) plan. Chicago: Blue Prairie Group, p. 29. Goldman, A. I. (1999). Knowledge in a social world. Oxford: Oxford University Press. Goode, W. J. (1969). The theoretical limits of professionalism in the semi professions and their organizations. Edited by A. Etzioni. New York: New York Free Press. Grace, D., & Cohen, S. (2001). Business ethics (2nd ed.). South Melbourne: Oxford University Press. Greenspun, Philip. What does it take to become an airline pilot? (n.d.). Retrieved May 27, 2007, from http://philip.greenspun.com/bboard/q-and-a-fetchmsg?msg_id=000tiQ&topic_id=21&topic=Aviation. Haas-Wilson, D. (1986). The effect of commercial practice restrictions: The case of optometry. Journal of Law and Economics, 29, 165-186. Haga, W. J. (1974, September). Perils of professionalism. Management Quarterly, pp. 310. Hardin, G. (1968). The tragedies of the commons. Science, 162, 1243-1248. Retrieved September 6, 2006, from http://www.dieoff.org/page95.htm.

327 References

Hardtalk. (online, n.d.). British Broadcasting Company. Retrieved April 15, 2006, from http://news.bbc.co.uk/1/hi/health/3149786.stm. Healey, K. (1973). The effect of licensure on clinical laboratory effectiveness. Doctoral dissertation. University of California, Los Angeles. Hippocratic Oath. Wikipedia. Retrieved May 27, 2007, http://en.wikipedia.org/wiki/Hippocratic_Oath. Holen, A. S. (1965). Effects of professional licensing arrangements on interstate labor mobility and resource allocation. Journal of Political Economy, 72, 492-498. Holmes, R. (Lord), & Watts, P. (online, n.d.). Making good business sense. Cited in Corporate social responsibility – What does it mean? (online, n.d.). Retrieved May 27, 2007, from http://www.mallenbaker.net/csr/CSRfiles/definition.html. Hughes, E.C. (1963). Professions. Daedalus 92, 655-68. International Council of Systems Engineers. (n.d.). What is systems engineering? Retrieved May 27, 2007, from http://www.incose.org/practice/whatissystemseng.aspx. IT Cortex. (2003). Failure rate – Statistics over IT projects failure rate. Retrieved June 7, 2007, from http://www.it-cortex.com/Stat_Failure_Rate.htm. Jamal, K., & Bowie, D. (1995). Theoretical considerations for a meaningful code of professional ethics. Journal of Business Ethics, 14, 703-714. Juran, J. M. (1988). Juran’s quality control handbook (4th ed.). Mcgraw-Hill (Tx). Kerzner, H. (2003). Project management: A systems approach to planning, scheduling, and controlling (8th edition). New York: John Wiley. Kimball, B. A. (1995). The true professional ideal in America: A history. New York: Bowman and Littlefield. Klesius, M. (2003, December). The future of flying. National Geographic online. Retrieved May 27, 2007, from http://magma.nationalgeographic.com/ngm/0312/feature1/index.html?fs=www7.nati onalgeographic.com . Kliem, R. L. (2006, February). Managing the Sarbanes Oxley Project. Retrieved March 23, 2006, from http://www.projectmagazine.com/v5i4/sox1.html February, 2006 . KPMG. (2003). KPMG’s International 2002-2003 Programme Management Survey. Retrieved September 12, 2006, from.http://www.kpmg.com.au/content/Services/Services/Audit_and_Risk_Advisory /Information_Risk_Management/docs/irmprm_pm-survey2003.pdf. Krause, E. A. (1996). Death of the guilds: Professions, states and the advance of capitalism, 1930 to present. New Haven: Yale University Press. Kwoka, J. E. (2003). Antitrust revolution: Economics, competition, and policy. Oxford: Oxford University Press. Labor Management Relations Act 29 USC152(12). 1995. Retrieved May 27, 2007, from http://law.justia.com/us/codes/title29/29usc152.html.

328 References

Lardent, E. F. (2000). Making the business case for pro bono. Law Firm Pro Bono Project. Georgetown: The Pro Bono Institute, Georgetown University Law Center. Retrieved June 17, 2007, from http://www.abanet.org/rppt/meetings_cle/2005/spring/rp/HotTips/Lardent.pdf. Lee, D. P., & Philip A. R. (2005). Re-visiting Fayol: anticipating contemporary management. British Journal of Management, 16(3), 175. Lee, H.C. (2003, July 8). Update: Operation on Iranian Twins Ladan and Laleh Bijani. Singapore Straights Times. Retrieved September 15, 2006, from http://www.straitstimes.com/topstories/story/0,4386,198588,00.html. Lewis, B. (2003). The 70-percent failure. Retrieved January 18 2006, from InfoWorld Website: http://archive.infoworld.com/articles/op/xml/01/10/29/011029/ opsurvival.xml Lewisch, P. (1991). The political economy of barriers to entry: The example of the amendment for taxicab regulation in Austria. In W. Weigel (Ed.), Economic analysis of law: A collection of applications (pp. 222-234). Vienna: Österreichischer Wirtschaftsverlag. Li, Z., Cheng, K., Wang, Y., Hiltz, S. R., & Turoff, M. (2001). Thurston’s Law of Comparative Judgment for group support. In J. DeGross, D. Strong, & D. Straub (Eds.), Proceedings of the Americas Conference on Information Systems (AMCIS). August 2-5, 2001. Boston, Massachusetts, USA. Light, M., & Hotle, M. (2003). The emergent PMO: Projects, programs and portfolios, Gartner Inc. project portfolio management software market trends. Conference presentation at the Project Portfolio Management 2006 Conference. December 13-15, 2006, Wuzhou Guest House Shenzhen, China. Luthans, F. (1976). Introduction to management: A contingent approach. New York: McGraw Hill. MacDonald, K. A. (1995). The sociology of the professions. Boston: Sage Publications. Maister, D. H. (1999). True professionalism: The courage to care about your career, your people and your clients. Boston: Simon and Schuster. Malone, T. W. (2004). The future of work. Boston: Harvard Business School Press. Marco, D. (2000). Building and managing the meta data repository: A full life-cycle guide. New York: Wiley. Marco, D. (2004). Universal meta data models. New York: Wiley. Marutello, F. (1981, Fall). The semantic definition of a profession. Southern Review of Public Administration Journal, 247-257. [Currently titled the American Review of Public Administration Journal.] Mascarenhas, A. (2006, September 7). Down by law, with the black dog. Sydney Morning Herald. Retrieved September 7, 2006, from http://www.smh.com.au/ Matter of Freeman, 34 NY2d 1, 9-10 [1974] Maynard, A. (n.d.). Trust me! I’m a doctor! Retrieved June 13 2007, from http://www.chere.uts.edu.au/pdf/maynard.pdf. 329 References

McCraw, T. K. (2007). Prophet of innovation: Joseph Schumpeter and creative destruction. New York: Belknap Press. McKenna, C. (2006). The world’s newest profession: Management consultancy in the 20th century. London: Cambridge Press. Merriam Webster’s third new international dictionary. Unabridged. Electronic version. Software updated 27 May, 2007. Millenium development goals defined. Retrieved February 12, 2007, from http://www.un.org/millenniumgoals/ Müller, R. & Turner, J. R. (2005). The impact of principal–agent relationship and contract type on communication between project owner and manager. International Journal of Project Management 23, 398-403. Modelski, G. (n.d.). The evolutionary world politics homepage: Kondratieff Waves. Retrieved June 17, 2007, from http://faculty.washington.edu/modelski/IPEKWAVE.html. Modern history sourcebook. (n.d.). Frederick W. Taylor: The principles of scientific management, 1911. Retrieved June 17, 2007, from http://www.fordham.edu/halsall/mod/1911taylor.html. [Copyright free source: Frederick W. Taylor, The Principles of Scientific Management (New York: Harper Bros., 1911): 5-29]. Montag, E. D. (n.d.). Empirical formula for creating error bars for the method of paired comparison. Retrieved September 6 2006, from Rochester Institute of Technology, Website: http://www.cis.rit.edu/people/faculty/montag Montgomery, H. (1992). Decision-making (in Swedish). In L.G. Lundh, H. Montgomery, & Y. Wærn (Eds.), Cognitive psychology (pp. 171-188). Stockholm: Studentlitteratur. Morgan, T. D., & Rotunda, R. (2004). Selected standards on professional responsibility, including California and New York rules. New York: Foundation Press. Morris, P. W. G, Crawford, L., Hodgson, D., Shepherd, M., Thomas, J. (2006). Exploring the role of formal bodies of knowledge in defining a profession – The case of project management. International Journal of Project Management, 24, 710-721. Morris, P. W. G., & Hough, G. H. (1987). The anatomy of major projects. Chichester: Wiley and Sons. Muris, T. J., & Clarkson, K.W. (1980). Occupational licensure and regulation, 107-41, (S. Rottenberg, ed., American Enterprise Institute 1980) Muris, T. J., Clarkson, K.W, & Martin, D. L. (1980). Will the sun set on occupational licensing? In S. Rottenberg (Ed.), Occupational licensing and regulation (pp. 107141). Washington, DC: American Enterprise Institute. Naritomi, J. Soares, R.R., Asuncao, J.J. (2007). Rent seeking and the unveiling of ‘de facto’ institutions: Development and colonial heritage within Brazil. 8th Annual Meeting of the LACEA Political Economy Group, Cartagena, 2007. Nathan, P., & Jones, G. (2003). PMP Exam Certification for Dummies. New York: Wiley Publishing.

330 References

Nelson, C., Lawrence, L., Triano, J., Bronfort, G., Perle, S., Metz, R., Hegetschweiler, K., & LaBrot, T. (2005). Chiropractic as spine care: A model for the profession. Chiropractic, & Osteopathy Journal 13, 9. Retrieved June 7, 2007, from http://www.chiroandosteo.com/content/13/1/9. Northern Michigan University. (2006). Course description: Contract management. Retrieved August 31, 2006, from http://www.nmu.edu/tas/ConsBachLead.htm Odone, A., Odone, M., & Odone, L. (n.d.). Lorenzo’s oil. Retrieved May 27 2007, from Myelin Project, http://www.myelin.org/aboutlorenzo.htm Olsen, P. A. (1983 September). Credentialism as monopoly, class war, and socialization scheme: Some historical reflections on modern ways of determining who can do a job. Law and Human Behaviour, 7(2/3), 291-299. Online Course Catalog. (n.d.). Retrieved May 27 2007, from Embry Riddle University Website: http://www.erau.edu/er/degrees/index.html. Organisation for Economic Co-operation and Development (OECD). (online, n.d.). For a better world economy. Retrieved May 27, 2007, from http://www.oecd.org/home/0,2987,en_2649_201185_1_1_1_1_1,00.html. Osenton, T. (2004). The death of demand. New York: Prentice Hall. Pareto, Vilfredo. (n.d.). Vilfredo Pareto guide. Retrieved September 10, 2006, from University of Melbourne Website: http://www.economics.unimelb.edu.au/ rdixon/pareto.html Parson, T. (1968). Sociological theory and modern society. Social Forces, 47(1), 90-91 Patterson, A. (n.d.). The profession of airline pilot. Retrieved May 27 2007, from airmanshiponline.com Website: http://www.airmanshiponline.com/fall99-2/05The% 20Airline%20Captain.htm Paul, C. (1984). Physician licensure legislation and the quality of medical care. Atlantic Economic Journal, 12, 18-30. Peters, T. (2003). Re-imagine. London: DK Publishing. Pinto, J., & Kharbanda, O. (1995, March-April). Lessons for an accidental profession. Project Management Business Horizons, 26-28. Piore, M., & Charles F. S. (1984). The second industrial divide. New York: Basic Books. Pollele, M. J. (1999, Winter). Who’s on first and what’s a professional? University of San Francisco Law Review. Pound, R. (1953). The lawyer from antiquity to modern times. St. Paul, Michegan: West Publishing Co. Professional Management Institute. PMI Annual Financial Report 2005. (2006). PM Network Magazine. Project Management Institute (2000). Project Management Institute policy governance manual. Version 5.1. Paragraph 1.1. Project Management Institute. (2004). Guide to the project management body of knowledge. Newton Square, PA: Project Management Institute.

331 References

Reish, F. (2005, July). The fiduciary responsibility to be responsible. Retrieved May 27 2007, from, reish.com Website: http://www.reish.com/publications/article_detail.cfm ?ARTICLEID=533 Richardson, G. P., & Pugh, A (1981). System dynamics: Modeling with DYNAMO. Portland: Productivity Press. Richardson, G., & Sterman, J. (Eds.). (1996). System dynamics ’96. Proceedings of the 1996 International System Dynamics Conference, Cambridge, Massachusetts, July 1996. Cambridge, MA: System Dynamics Society. Rosenbaum, D. B., & Rubin, D. K. (2001). The nation’s C-schools. Engineering NewsRecord, 247(18), 26-37. Rotunda, R. (1995). Professional responsibility (6th edition). Black Letter Series. St Paul: West Group, MN Royal Chartered Institute of Building. Retrieved September 6, 2006, from http://www.ciob.org.uk/ciob/ Royal Chartered Institute of Building. (n.d.). Requirements for certification. Retrieved September 6, 2006, from http://www.ciob.org.uk/ciob/ Royal Institution of Chartered Quantity Surveyors. (n.d.).History of the organization. Retrieved September 6 2006, from from http://www.rics.org/ Royal Institution of Chartered Quantity Surveyors. (n.d.).Requirements for certification. Retrieved September 6, 2006, from http://www.rics.org/ Sabel, C. (1995). The new division of labor: Emerging forms of work organization in international perspectives. Berlin: Walter De Gruyter. Sarbanes-Oxley Act 2002 (Pub. L. No. 107-204, 116 Stat. 745. [Also known as the Public Company Accounting Reform and Investor Protection Act 2002. Commonly called SOX]. Saunders, D.H. (2005). Quantifying reasonable doubt: A proposed solution to an equal protection problem. bepress Legal Series. Paper 881. Retrieved May 27, 2007 from http://law.bepress.com/cgi/viewcontent.cgi?article=4389&context=expresso. Schein, E. H. (1973). Professional education: Some new directions by The Journal of Higher Education, 44(4), 322-324. Schoppman, G., & Warner, P. (2006). First annual project management survey, 2006. Schumpeter, J. A. (1975; orig pub. 1942). Capitalism, socialism and democracy. New York: Harper. Shepard, L. (1978). Licensing restrictions and the cost of dental care. Journal of Law and Economics, 21, 187-201. Shimberg, B., Esser, B. F., & Kruger, D.H. (1972). Occupational licensing: Practices and policies. New York: Public Affairs Press. Shtub, A., Bard, J. B., & Globerson, S. (2005). Project management processes, methodologies and economics (2nd ed.). New York: Pearson, Prentice Hall. Simon, R. J. (1970). Beyond a reasonable doubt: An experimental attempt at quantification. Journal of Applied Behavioral Science, 6(2), 203-209.

332 References

Simon, W. S. (2004). The Prudent Investor Act: A guide to understanding, fiduciary focus: We ain’t got no fiduciary duties. Morningstar. (Online Advisor Edition). Retrieved May 27, 2007, from http://advisor.morningstar.com/articles/doc.asp?docId=3548. Simon, W. S. (2005, Autumn). Illuminating the broad range requirements of ERISA section 404(c). with the language of modern portfolio theory found in the uniform Prudent Investor Act. Journal of Pension Benefits, 87-90. Slayton, P., & Trebilcock, M. J. (Eds.). (1978). The professions and public policy. Toronto: University of Toronto Press. Smothers, D., & Smothers, T. (n.d.). The streets of Laredo. Retrieved September 2 2006, from Everything 2 Website: http://everything2.com/index.pl?node_id=953261 Spaulding, N. W. (2003). Reinterpreting professional identify. University of Colorado Law Review, 74(Winter). Spencer, L. M., & Spencer, S. M. (1993). Competence at work: Models for superior performance. New York: Wiley. Standish Group. (1994). Chaos Report. Retrieved September 10, 2006, from http://www.projectsmart.co.uk/docs/chaos_report.pdf. Sterman, J. D. (2000). Business dynamics: Systems thinking and modeling in a complex world. New York: McGraw Hill. Stewart, T. (1995, March). Project management: Career choice of the 90’s and beyond. Fortune Magazine. Taurasi, E. M. (2006, July 28). Boston’s big dig: One of engineering’s biggest mistakes? Retrieved May 27 2007, from Design News Website: http://www.designnews.com/ article/CA6357443.html The Law Society (2002). International pro bono work by solicitors’ firms: A database of work undertaken by major firms. Retrieved June 17, 2007, from http://www.lawsociety.org.uk/influencinglaw/internationlaw/probonowork.law. I Thomas, D. (1995). The stakeholder theory of the corporation: Concepts, evidence, and implications. Lee E. Preston Academy of Management Review, 20(1), 65-91. Thurstone, L. L. (1927). A law of comparative judgment. Psychological Review, 34, 273286. Tillers, P. (n.d.). Should courts use numbers to describe standards of persuasion in legal proceedings? Tillerstiller’s Blogspot. Retrieved September 10, 2006, from http://tillerstillers.blogspot.com/2005_05_15_tillerstillers_archive.html from 10. Tillers, P., & Gottfried, J. (2006, August 9). United States v. Copeland: A collateral attack on the legal maxim that proof beyond a reasonable doubt is unquantifiable? Cardozo Legal Studies Research Paper No. 160. Available at SSRN: http://ssrn.com/abstract=923480. Turner, J.R. and Müller, R. (2003). On the nature of the project as a temporary organization. International Journal of Project Management 21, 7.

333 References

US Census Bureau. (2005). Median household income, 2005. Retrieved November 4, 2006, from Website: http://www.census.gov/hhes/www/income/faq.html US Federal Aviation Agency. (n.d.). IASA Model Aviation Regulatory Document. Retrieved May 27, 2007, from http://www.faa.gov/safety/programs_initiatives/oversight/iasa/model_aviation/ US Federal Aviation Agency. (n.d.). US Federal Aviation Agency Regulations. Retrieved May 27, 2007, from http://ecfr.gpoaccess.gov/cgi/t/text/textidx?&c=ecfr&tpl=/ecfrbrowse/Title14/14tab_02.tpl. US Federal Aviation Agency. (n.d.). US Federal Aviation Agency Rules, Regulations and Policies. Retrieved May 27, 2007, from http://www.faa.gov/regulations_policies/ Vollmer, H. M., & Mills, D. L. (1965). Some comments on the professionalization of everyone? The American Journal of Sociology, 70(4), 480-481. Welch, A., Conklin, D., & Firth, R. (n.d.). In search of excellence, what makes a winning project team? A white paper from Parity. Retrieved May 31, 2007, from http://www.parity.net/documents/pdf/whitepaper/Parity-PPM_WhitePaper.pdf. Wideman, M. (n.d.) Come with questions, leave with answers: Welcome to my project management site. Retrieved September 9, 2006, from http://www.maxwideman.com/index.htm. Wideman, M. (n.d.). Come with questions, leave with answers: Standards for very large projects. Retrieved September 9, 2006, from http://www.maxwideman.com/musings/largeprojects.htm. Wideman, M. (n.d.). Comparative glossary of project management terms v3.1: Project draw to project boundary. Retrieved September 3, 2006, from http://www.maxwideman.com/pmglossary/PMG_P09.htm. Wideman, M. (n.d.). Comparative glossary of project management terms v3.1: Project manual to project procedures. Retrieved May 27, 2007, from http://www.maxwideman.com/pmglossary/PMG_P13.htm. Wikipedia. (n.d.) Duty of care. Retrieved May 27, 2007, from http://en.wikipedia.org/wiki/Duty_of_care. Wikipedia. (n.d.) Life skills. Retrieved June 17, 2007, from http://en.wikipedia.org/wiki/Life_skills Wilensky, H. (1964). Professionalization of everything. American Journal of Management, 70, 137-48. Williams, T. (2002). Modelling Complex Projects. New York: John Wiley and Sons Ltd. Winter, M., & Smith, C. (2006). Rethinking project management. Report to the Engineering and Physical Sciences Research Council (EPSRC). Manchester: EPSRC. Wood, D. J. (1991). Corporate social performance revisited. The Academy of Management Review, 16(4), 691-718. Young, D. S. (2000). Occupational licensing. Washington DC: Cato Institute.

334 References

Younkins, E.W. (2006, February 12). Jean-Baptiste Say's law of markets: A

fundamental, conceptual integration. Le Québécois Libre. (Online journal). Retrieved 27 May, 2007, from http://www.quebecoislibre.org/06/060212-4.htm.

Yourdon, E. (2003). Death march: The complete software developer’s guide to surviving ‘mission impossible’ projects (2nd ed.). New York: Prentice Hall. Zwerman, B. L., Thomas, J. L. et al. (2004). Professionalization of project management: Exploring the past to map the future. Newton Square, PA: Project Management Institute.

335 References

Appendices

Appendix A: Survey instrument 411 responses Privacy Statement The purpose of this survey is to collect data for Paul Giammalvo, CDT, PMP, CCE, MScPM, PhD Candidate to conduct research to evaluate whether Project Management is a stand-alone profession, and if not, what is it? This research is being conducted under the supervision of Dr. Christophe N. Bredillet, IPMA Level A, CCE, Director of Project/Program Management, ESC-Lille University, This comprehensive survey will take approximately one hour to complete. You must complete it in one sitting. You can not save your results and log back at a later time. For those expecting to claim PDU/CEU's, you will also be expected to read the survey results and reflect on them. You have three options to participate: 1) You can participate completely anonymously. As the demographic information is an essential part of this research, if you do NOT want your demographic information linked to your name, then choose this option, which will preclude you from claiming either PDU's or the T-Shirt OR receiving the results via email. 2) You can participate and earn 2 Professional Development Units (PDU’s) or .2 Continuing Education Units (CEU’s); (NOTE: For those collecting PDU's for PMI, they must be recorded and collected under Category 2-SDL) The breakdown of the PDU's is: 1 hour or PDU to actually participate in the survey, and; 1 hour or PDU to read and reflect on it as part of your self directed learning. 3) You can participate and receive a complimentary "Project Management- Art? Science? Or Bull....." T-Shirt; Should you choose to participate under Options 2 (PDU’s) or 3 (T-Shirt), it will be necessary for us to collect your name and physical mailing address from you. You will also have the option to receive “soft copy” of the results of the survey via email. Our online survey asks respondents for contact information, financial information, and demographic information. This information is used in the aggregate exclusively. The data collected is used for the stated purpose only and is not sold, leased or used for any other purposes. The research will not disclose information about individual respondents or share any personal information with outside parties. Chart Wizard

I Agree

Percen tage

Respo nses

100.0

405

Total responses:

1 /27

405

2) You have three options to participate: Anonymously- If you choose this option, do NOT provide any personal contact information at all. Choosing this option will preclude you from receiving PDU/CEU's, TShirt OR the results of the survey and analysis via email. Should you choose the Anonymous option, DO fill out the demographics, as that is a key element of the research. PDU's/CEU's- If you choose this option, you must provide a mailing address in order to receive a Certificate of Participation. For PMI Members wishing to claim the PDU's towards their recertification, you must do so under Category 2-SDL. You may use the online self reporting form: http://www.pmi.org/info/PDC_CCRReportingForms.asp. However, you will receive a Certificate of Participation, serving as evidence that you did participate in the full survey, which is worth 1.0 PDU. The claim for the second PDU will need to come from your reading and reflecting on the results of the survey. (Estimated to be ~1 hour or 1.0 PDU's) These PDU's will NOT be able to be claimed until AFTER the release of the Survey Results in late 3Q 2005. As the Demographics is an essential part of the research, should you choose this option, I will be able to link your demographics to your name, so make the selection carefully. This information will not be shared with anyone, and will only be used in the aggregate. T-Shirt- Should you choose this option you will need to provide me with your size and physical mailing address. As the Demographics is an essential part of the research, should you choose this option, I will be able to link your demographics to your name, so make the selection carefully. This information will not be shared with anyone, and will only be used in the aggregate. Chart Wizard

Percen tage

Respo nses

I want to participate anonymously

6.8

26

I want to participate and receive Professional Development Units (PDU’s) or Continue Education Units (CEU's))

46.3

177

I want to participate and receive a T-Shirt.

46.9

179

Total responses:

382

3) If you choose to earn 2 Professional Development Units (PDU’s) or .2 Continuing Education Units (CEU’s) or if you choose to earn a T Shirt then please provide your name and physical mailing address. Example if you choose to earn PDU/CEU: SEND ME A PDU/CEU CERTIFICATE Name : John Doe Address : Orchard 60A Orchard Road Singapore 238890

2 /27

NOTE: Should you choose this option, while you will receive a certificate attesting to your participation, that will only be valid for the estimated time to fill in the survey, 1.0 hours. To earn the second hour or PDU, will require you to read and interpret the results. To claim the full 2.0 PDU's for PMI will require you to do so under Category 2-Self Directed Learning. To access this form, point your browser to: http://www.pmi.org/info/PDC_CCRReportingForms.asp. Example if you choose to receive the T-Shirt: SEND ME A (pick your size) SMALL, MEDIUM, LARGE, EXTRA LARGE, XX LARGE, TSHIRT Name : John Doe Address : Orchard 60A Orchard Road Singapore 238890

Please provide a valid address and write down "SEND ME A PDU/CEU CERTIFICATE" or "SEND ME A T-SHIRT" with your preferred "SIZE", so we know your choice. 4) Do you also want to receive the results of the survey? As the Demographics is an essential part of the research, should you choose this option, I will be able to link your demographics to your name, so make the selection carefully. This information will not be shared with anyone, and will only be used in the aggregate. IF you do NOT want your demographic linked to your name, go back and select the "Anonymous" option.

Yes, send me the results by email

Chart Wizard Percen tage

Respo nses

100.0

371

Total responses:

371

5) If you want to receive the results of the survey, then please provide your valid email address. As the Demographics is an essential part of the research, should you choose this option, I will be able to link your demographics to your name, so make the selection carefully. This information will not be shared with anyone, and will only be used in the aggregate. IF you do NOT want your demographic linked to your name, go back and select the "Anonymous" option.

(The last five responses are given) - [email protected] - [email protected] - [email protected] - [email protected] - [email protected]

3 /27

6) Age :

Chart Wizard

Percen tage

Respo nses

19-24

3.5

14

25-30

17.1

69

31-36

27.0

109

37-42

19.8

80

43-48

15.1

61

49-54

8.2

33

55-60

6.4

26

61-66

1.5

6

>66

1.5

6

Total responses:

7) Sex:

404

Chart Wizard

Percen tage

Respo nses

Female

23.9

96

Male

76.1

306

Total responses:

8) Education Level:

402

Chart Wizard

Percen tage

Respo nses

High School

6.7

27

Bachelors Degree

45.3

182

Masters Degree

43.3

174

PhD

4.7

19

Total responses:

402

9) Certifications: IF you do not currently hold a Professional Certification or Credential, SKIP THIS QUESTION. If you DO hold MORE THAN ONE Professional Certification, select the "Other" option and list them. OR, if you prefer to choose only a single credential, select the one having the most real or perceived value to you.

Chart Wizard

4 /27

Percen tage

Respo nses

PE

3.4

9

PMP

65.3

171

CCE

1.1

3

RegPM

2.3

6

IPMA Level 1

0.8

2

Other

27.1

71

Total responses:

262

Click to view all OTHER text responses

10) Years PM Experience:

Chart Wizard

Percen tage

Respo nses

35

1.3 Total responses:

11) Salary Range (in Thousands USD):

5 399

Chart Wizard

Percen tage

Respo nses

100

15.9

64

Total responses:

5 /27

402

12) What best describes the industry you work in?

Chart Wizard

Percen tage

Respo nses

General Construction

7.4

30

Oil, Gas, Petrochemical or Natural Resources

6.7

27

Telecommunications

12.7

51

IT/IS

24.8

100

Pharmaceuticals

1.7

7

Management Services

3.0

12

Banking

4.0

16

Consulting

15.6

63

Other

24.1

97

Total responses:

403

Click to view all OTHER text responses

13) What best describes your JOB TITLE?

Chart Wizard

Percen tage

Respo nses

Project Manager

51.4

207

Functional Manager

5.5

22

Cost Estimator/Scheduler

2.7

11

Finance/Accounting

0.5

2

Sales and Marketing

2.5

10

Training, Mentoring or Consulting

7.7

31

Human Resources

0.5

2

Engineering/Technical Support

6.5

26

Procurement/Purchasing/Expediti ng

0.5

2

Other

22.3

90

Total responses: Click to view all OTHER text responses

6 /27

403

14) What best describes the part of the world you originate from?

Chart Wizard

Percen tage

Respo nses

North America

26.8

108

Central/South America

5.5

22

European Union

8.4

34

Russia

0.7

3

Middle East

2.5

10

Africa

16.4

66

39.7

160

Asia Pacific

Total responses:

403

15) -IF YOU DO NOT HOLD THE JOB TITLE OF PROJECT MANAGER OR FUNCTION IN THE ROLE OF A PROJECT MANGAGER, SKIP THIS QUESTIONPlease evaluate the following terms or phrases by how UNIQUE you consider each as they are used in PROJECT MANAGEMENT compared to General Usage : Wizard

Chart

1 More Commonly Use For General Business

2

3 Used Equally by Both

4

5 Unique ONLY to Project Management

Responses

Average Score

a. Stakeholde r

28 (9.03%)

15 (4.84%)

169 (54.52%)

81 (26.13%)

17 (5.48%)

310

3.14 / 5 (62.80%)

b. Work Breakdown Structure

2 (0.64%)

3 (0.96%)

44 (14.15%)

83 (26.69%)

179 (57.56%)

311

4.40 / 5 (88.00%)

c. Earned Value

8 (2.59%)

12 (3.88%)

71 (22.98%)

72 (23.30%)

146 (47.25%)

309

4.09 / 5 (81.80%)

d. Team Building

23 (7.42%)

35 (11.29%)

209 (67.42%)

22 (7.10%)

21 (6.77%)

310

2.95 / 5 (59.00%)

e. Risk Analysis

13 (4.18%)

21 (6.75%)

185 (59.49%)

71 (22.83%)

21 (6.75%)

311

3.21 / 5 (64.20%)

f. Cost Estimating

8 (2.59%)

22 (7.12%)

207 (66.99%)

51 (16.50%)

21 (6.80%)

309

3.18 / 5 (63.60%)

g. Effective Communic ations

15 (4.84%)

31 (10.00%)

241 (77.74%)

15 (4.84%)

8 (2.58%)

310

2.90 / 5 (58.00%)

h. Actual Cost of Work Performed

3 (0.96%)

15 (4.82%)

68 (21.86%)

90 (28.94%)

135 (43.41%)

311

4.09 / 5 (81.80%)

i. Activity

12 (3.90%)

14 (4.55%)

172 (55.84%)

71 (23.05%)

39 (12.66%)

308

3.36 / 5 (67.20%)

j. Administrat ive Closure

10 (3.23%)

28 (9.03%)

75 (24.19%)

98 (31.61%)

99 (31.94%)

310

3.80 / 5 (76.00%)

k. Baseline

3 (0.97%)

7 (2.27%)

92 (29.87%)

87 (28.25%)

119 (38.64%)

308

4.01 / 5

7 /27

Cost Budget

(80.20%)

l. Cost Control

7 (2.25%)

22 (7.07%)

226 (72.67%)

31 (9.97%)

25 (8.04%)

311

3.14 / 5 (62.80%)

m. Deliverable s

7 (2.28%)

8 (2.61%)

145 (47.23%)

88 (28.66%)

59 (19.22%)

307

3.60 / 5 (72.00%)

n. Duration Estimate

2 (0.65%)

3 (0.97%)

87 (28.16%)

109 (35.28%)

108 (34.95%)

309

4.03 / 5 (80.60%)

o. Finish to Finish Relationshi p

1 (0.32%)

4 (1.30%)

28 (9.09%)

50 (16.23%)

225 (73.05%)

308

4.60 / 5 (92.00%)

p. Integrated Change Control

2 (0.65%)

7 (2.29%)

52 (16.99%)

88 (28.76%)

157 (51.31%)

306

4.28 / 5 (85.60%)

q. Free Float

5 (1.63%)

7 (2.28%)

35 (11.40%)

51 (16.61%)

209 (68.08%)

307

4.47 / 5 (89.40%)

r. Network Logic

5 (1.63%)

8 (2.61%)

51 (16.67%)

82 (26.80%)

160 (52.29%)

306

4.25 / 5 (85.00%)

s. Organizatio nal Breakdown Struct

10 (3.26%)

26 (8.47%)

138 (44.95%)

66 (21.50%)

67 (21.82%)

307

3.50 / 5 (70.00%)

t. Performanc e Reporting

12 (3.92%)

18 (5.88%)

211 (68.95%)

40 (13.07%)

25 (8.17%)

306

3.16 / 5 (63.20%)

u. Project Plan

4 (1.29%)

4 (1.29%)

91 (29.45%)

79 (25.57%)

131 (42.39%)

309

4.06 / 5 (81.20%)

v. Quantitativ e Risk Analysis

5 (1.62%)

16 (5.19%)

114 (37.01%)

101 (32.79%)

72 (23.38%)

308

3.71 / 5 (74.20%)

w. Resource Leveling

2 (0.65%)

6 (1.95%)

55 (17.92%)

73 (23.78%)

171 (55.70%)

307

4.32 / 5 (86.40%)

x. Schedule Baseline

2 (0.65%)

5 (1.63%)

44 (14.33%)

87 (28.34%)

169 (55.05%)

307

4.36 / 5 (87.20%)

y. Target Completion Date

3 (0.97%)

7 (2.27%)

166 (53.90%)

61 (19.81%)

71 (23.05%)

308

3.62 / 5 (72.40%)

z. Work Package

4 (1.31%)

7 (2.29%)

71 (23.20%)

95 (31.05%)

129 (42.16%)

306

4.10 / 5 (82.00%) 3.78 / 5 (75.60%)

8 /27

16) -IF YOU DO NOT HOLD THE JOB TITLE OF PROJECT MANAGER OR FUNCTION IN THE ROLE OF A PROJECT MANGAGER, SKIP THIS QUESTIONI consider Project Management to be my life’s work :

Chart Wizard

Percen tage

Respo nses

Yes

73.0%

227

No

27.0%

84

Total responses:

311

17) -IF YOU DO NOT HOLD THE JOB TITLE OF PROJECT MANAGER OR FUNCTION IN THE ROLE OF A PROJECT MANGAGER, SKIP THIS QUESTIONWhen in a work environment, I am able to identify other project managers in my organization by the clothes they wear or some other identifiable part of their costume. (i.e. special tie, rings, headgear or other unique part of their dress or appearance): Chart Wizard

Yes No

Percen tage

Respo nses

6.2%

19

93.8%

289

Total responses:

308

18) -IF YOU DO NOT HOLD THE JOB TITLE OF PROJECT MANAGER OR FUNCTION IN THE ROLE OF A PROJECT MANGAGER, SKIP THIS QUESTIONI believe that Project Managers should be held financially accountable for mistakes they/their teams are responsible for when managing a project :

Yes No

Chart Wizard

Percen tage

Respo nses

60.4%

186

39.6%

122

Total responses:

9 /27

308

19) -IF YOU DO NOT HOLD THE JOB TITLE OF PROJECT MANAGER OR FUNCTION IN THE ROLE OF A PROJECT MANGAGER, SKIP THIS QUESTIONAs a Project Manager, I feel I have ________________ autonomy in making decisions related to the execution of the Project I am responsible for :

Chart Wizard

1 No

2

3 Limited

4

5 Full

Responses

Average Score

1 (0.32%)

7 (2.27%)

80 (25.97%)

169 (54.87%)

51 (16.56%)

308

3.85 / 5 (77.00%)

Chart

3.85 / 5 (77.00%)

20) -IF YOU DO NOT HOLD THE JOB TITLE OF PROJECT MANAGER OR FUNCTION IN THE ROLE OF A PROJECT MANGAGER, SKIP THIS QUESTIONI relate more to being a project manager than I do to working for any particular employer : 1 Strongly Disagree 6 (1.95%)

Chart Wizard 2

3 Not Sure

32 (10.39%) 53 (17.21%)

4

5 Strongly Agree

Responses

Average Score

127 (41.23%)

90 (29.22%)

308

3.85 / 5 (77.00%)

Chart

3.85 / 5 (77.00%)

21) -IF YOU DO NOT HOLD THE JOB TITLE OF PROJECT MANAGER OR FUNCTION IN THE ROLE OF A PROJECT MANGAGER, SKIP THIS QUESTIONAs a Project Manager, I consider myself to be held in ______ esteem in my home (nonChart

working) community because of the work I do as a Project Manager : Wizard 1 Low

2

3 Moderate

4

5 High

Responses

Average Score

6 (1.94%)

16 (5.16%)

139 (44.84%)

110 (35.48%)

39 (12.58%)

310

3.52 / 5 (70.40%)

Chart

3.52 / 5 (70.40%)

22) -IF YOU DO NOT HOLD THE JOB TITLE OF PROJECT MANAGER OR FUNCTION IN THE ROLE OF A PROJECT MANGAGER, SKIP THIS QUESTIONCompared to other people of my age and education level, in my position as a project manager, I feel I am compensated ___________ for my services :

10 /27

Chart Wizard

1 Below Average

2

3 Average

4

5 Above Average

Responses

Average Score

19 (6.15%)

30 (9.71%)

142 (45.95%)

92 (29.77%)

26 (8.41%)

309

3.25 / 5 (65.00%)

Chart

3.25 / 5 (65.00%)

23) -IF YOU DO NOT HOLD THE JOB TITLE OF PROJECT MANAGER OR FUNCTION IN THE ROLE OF A PROJECT MANGAGER, SKIP THIS QUESTIONPeople not familiar with Project Management consider what practitioners do to be mystical and/or highly ritualistic.

Chart Wizard

1 Strongly Disagree

2

3 Not Sure

4

5 Strongly Agree

Responses

Average Score

20 (6.49%)

65 (21.10%)

103 (33.44%)

90 (29.22%)

30 (9.74%)

308

3.15 / 5 (63.00%)

Chart

3.15 / 5 (63.00%)

24) -IF YOU DO NOT HOLD THE JOB TITLE OF PROJECT MANAGER OR FUNCTION IN THE ROLE OF A PROJECT MANGAGER, SKIP THIS QUESTIONWhen I introduce myself to people, I would be more likely to do so by stating : Chart Wizard

Percen tage

Respo nses

I am a project manager who works for XYZ company.

42.9

132

I work for XYZ company as a project manager.

57.1

176

Total responses:

308

25) -IF YOU DO NOT HOLD THE JOB TITLE OF PROJECT MANAGER OR FUNCTION IN THE ROLE OF A PROJECT MANGAGER, SKIP THIS QUESTIONIn the fulfillment of my duties in the role of a Project Manager, I feel I am able to consistently exercise discretionary judgment in how the project gets executed : Chart Wizard

Percen tage

Respo nses

Yes

85.0%

261

No

15.0%

46

Total responses:

11 /27

307

26) -IF YOU DO NOT HOLD THE JOB TITLE OF PROJECT MANAGER OR FUNCTION IN THE ROLE OF A PROJECT MANGAGER, SKIP THIS QUESTIONAs a Project Manager, I feel I have ________________ responsibility for the decisions I Chart

make related to the execution of the Project for which I am in charge : Wizard 1 No

2

3 Partial

4

5 Full

Responses

Average Score

0 (0.00%)

1 (0.32%)

39 (12.66%)

121 (39.29%)

147 (47.73%)

308

4.34 / 5 (86.80%)

Chart

4.34 / 5 (86.80%)

27) -IF YOU DO NOT HOLD THE JOB TITLE OF PROJECT MANAGER OR FUNCTION IN THE ROLE OF A PROJECT MANGAGER, SKIP THIS QUESTIONPeople who use my services as a Project Manager are not concerned about how I execute the project as long as the project is successful (using their definition of Chart Wizard

success) : 1 Disagree

2

41 (13.23%) 78 (25.16%)

3 Not Sure

4

5 Agree

Responses

Average Score

17 (5.48%)

110 (35.48%)

64 (20.65%)

310

3.25 / 5 (65.00%)

Chart

3.25 / 5 (65.00%)

28) Below are 8 definitions. Please rank the definition that MOST represents your definition, understanding or interpretation of PROJECT MANAGEMENT as it is practiced by you or within your organization. Mark 1 being the “best” definition, 2 being the second best and so on, with 8 representing the least appropriate definition in your opinion. NOTE. Please give a twice.

different

rank for each option. Do not use the same number

Chart Wizard Average Score

Respons es

“a subject, field or duties executed within my existing occupation or profession”

4.57 / 8

400

“a way of doing something or carrying something out, according to a plan”

2.95 / 8

401

“to produce something in a particular or standardized way”

4.33 / 8

399

“the job by which a person earns a living”

6.83 / 8

399

“an occupation that requires long and specialized education or training”

5.20 / 8

398

“activity(ies) that somebody does regularly for pay

6.89 / 8

399

“a method or set of procedures for achieving or producing something”

3.04 / 8

400

“a series of actions directed towards achieving a specific aim or objective”

1.99 / 8

400

1.99 / 8

12 /27

29) During the past year, I have donated the following number of hours performing voluntary, uncompensated community service requiring the use of my Project Management skills.

Chart Wizard

Percen tage

Respo nses

0

26.9

108

1-20

36.7

147

21-40

16.2

65

41-60

5.0

20

>60

15.2 Total responses:

13 /27

61 401

11 (2.79%)

61 (15.48%)

4 (1.02%)

Fresh University Graduate Certified Public Accountant

Fresh University 83 Graduate (21.07%) Commercial Tractor Trailer Truck Driver

10 (2.54%)

6 (1.52%)

2 10%

Fresh University Graduate Commercial Airline Pilot

1 Equal

69 (17.51%)

16 (4.06%)

7 (1.78%)

3 20%

51 (12.94%)

15 (3.81%)

22 (5.58%)

4 30%

38 (9.64%)

25 (6.35%)

7 (1.78%)

5 40%

7 60%

17 (4.31%)

36 (9.14%)

30 (7.61%)

14 /27

33 (8.38%)

27 (6.85%)

27 (6.85%)

6 50%

13 (3.30%)

47 (11.93%)

43 (10.91%)

8 70%

11 (2.79%)

87 (22.08%)

85 (21.57%)

9 80%

10 (2.54%)

43 (10.91%)

46 (11.68%)

10 90%

Chart Wizard

8 (2.03%)

83 (21.07%)

111 (28.17%)

11 100%

394

394

394

Responses

3.84 / 11 (34.91%)

8.06 / 11 (73.27%)

8.46 / 11 (76.91%)

Average Score

EXAMPLE: Comparing a fresh graduate of a 4 year University against a Candlestick Maker, by marking the “40%” box, you are saying that in your opinion, the Candlestick Maker is perceived by you to be forty percent more of a profession than the fresh graduate (out of a possible maximum of 100%).

30) Below is a list of 21 occupations generally considered to be “Professions” based upon the requirement of a license to practice. Compare each occupation against the baseline of a fresh graduate of a 4 year university, with no experience and no license, marking how much more you perceive the RELATIVE VALUE of the occupation compared against the fresh university graduate in terms of PROFESSIONAL IMAGE or PRESTIGE.

11 (2.79%)

39 (9.92%)

8 (2.05%)

45 (11.48%)

36 (9.18%)

9 (2.28%)

Fresh University Graduate MBA/Busines s Consultant

Fresh University Graduate Librarian

Fresh University Graduate Civil Engineer/Lan d Surveyor

Fresh University Graduate Plumber

Fresh University Graduate Electrician

Fresh University Graduate Dentist

5 (1.27%)

28 (7.14%)

39 (9.95%)

10 (2.56%)

33 (8.40%)

7 (1.78%)

59 (14.94%)

12 (3.05%)

52 (13.27%)

53 (13.52%)

15 (3.84%)

61 (15.52%)

26 (6.60%)

70 (17.72%)

17 (4.31%)

47 (11.99%)

46 (11.73%)

20 (5.12%)

41 (10.43%)

19 (4.82%)

48 (12.15%)

18 (4.57%)

58 (14.80%)

59 (15.05%)

30 (7.67%)

48 (12.21%)

25 (6.35%)

52 (13.16%)

35 (8.88%)

52 (13.27%)

52 (13.27%)

47 (12.02%)

60 (15.27%)

46 (11.68%)

35 (8.86%)

21 (5.33%)

43 (10.97%)

29 (7.40%)

46 (11.76%)

42 (10.69%)

45 (11.42%)

22 (5.57%)

45 (11.42%)

30 (7.65%)

31 (7.91%)

48 (12.28%)

25 (6.36%)

55 (13.96%)

12 (3.04%)

70 (17.77%)

13 (3.32%)

10 (2.55%)

42 (10.74%)

11 (2.80%)

55 (13.96%)

5 (1.27%)

97 (24.62%)

13 (3.32%)

12 (3.06%)

46 (11.76%)

9 (2.29%)

46 (11.68%)

5 (1.27%)

6.25 / 11 (56.82%)

65 (16.50%)

20 (5.10%)

16 (4.08%)

79 (20.20%)

24 (6.11%)

59 (14.97%)

12 (3.04%)

394

392

392

391

393

394

395

8.33 / 11 (75.73%)

5.18 / 11 (47.09%)

4.85 / 11 (44.09%)

7.49 / 11 (68.09%)

5.02 / 11 (45.64%)

7.42 / 11 (67.45%)

3.87 / 11 (35.18%)

15 /27

31) CONTINUED- Below is a list of 21 occupations generally considered to be “Professions” based upon the requirement of a license to practice. Compare each occupation against the baseline of a fresh graduate of a 4 year university, with no experience and no license, marking how much more you perceive the RELATIVE VALUE of the occupation compared against the fresh university graduate in terms of PROFESSIONAL IMAGE or PRESTIGE.

75 (18.99%)

Fresh University Graduate Barber/Cosm etologist

6 (1.53%)

4 (1.02%)

11 (2.81%)

5 (1.28%)

4 (1.02%)

Fresh University Graduate Lawyer

Fresh University Graduate Medical Doctor

Fresh University Graduate High School Principal

Fresh University Graduate Constructio n Manager

Fresh University Graduate Project Manager

1 Equal

9 (2.30%)

10 (2.55%)

18 (4.59%)

6 (1.53%)

12 (3.06%)

2 10%

9 (2.30%)

18 (4.59%)

19 (4.85%)

9 (2.30%)

8 (2.04%)

3 20%

17 (4.34%)

27 (6.89%)

31 (7.91%)

7 (1.79%)

11 (2.81%)

4 30%

14 (3.57%)

33 (8.42%)

21 (5.36%)

4 (1.02%)

12 (3.06%)

5 40%

7 60%

51 (13.01%)

62 (15.82%)

51 (13.01%)

17 (4.35%)

22 (5.61%)

16 /27

48 (12.24%)

53 (13.52%)

60 (15.31%)

20 (5.12%)

45 (11.48%)

6 50%

75 (19.13%)

70 (17.86%)

56 (14.29%)

33 (8.44%)

42 (10.71%)

8 70%

63 (16.07%)

58 (14.80%)

63 (16.07%)

48 (12.28%)

76 (19.39%)

9 80%

54 (13.78%)

32 (8.16%)

32 (8.16%)

84 (21.48%)

57 (14.54%)

10 90%

Chart Wizard

48 (12.24%)

24 (6.12%)

30 (7.65%)

159 (40.66%)

101 (25.77%)

11 100%

392

392

392

391

392

Responses

7.82 / 11 (71.09%)

7.07 / 11 (64.27%)

6.93 / 11 (63.00%)

9.25 / 11 (84.09%)

8.38 / 11 (76.18%)

Average Score

EXAMPLE: Comparing a fresh graduate of a 4 year University against a Candlestick Maker, by marking the “40%” box, you are saying that in your opinion, the Candlestick Maker is perceived by you to be forty percent more of a profession than the fresh graduate (out of a possible maximum of 100%).

111 (28.39%)

18 (4.62%)

11 (2.81%)

42 (10.74%)

58 (14.83%)

6 (1.54%)

Fresh University Graduate School Bus Driver

Fresh University Graduate Computer/ Software Engineer

Fresh University Graduate Registered Nurse

Fresh University Graduate Real Estate Broker/App raiser

Fresh University Graduate Insurance Agent

Fresh University Graduate Electrical/T elecommun ications Engineer

18 (4.63%)

48 (12.28%)

42 (10.74%)

22 (5.61%)

12 (3.08%)

72 (18.41%)

20 (5.14%)

62 (15.86%)

48 (12.28%)

31 (7.91%)

36 (9.23%)

66 (16.88%)

35 (9.00%)

54 (13.81%)

44 (11.25%)

47 (11.99%)

34 (8.72%)

43 (11.00%)

44 (11.31%)

51 (13.04%)

57 (14.58%)

40 (10.20%)

38 (9.74%)

26 (6.65%)

38 (9.77%)

28 (7.16%)

31 (7.93%)

44 (11.22%)

57 (14.62%)

22 (5.63%)

17 /27

51 (13.11%)

44 (11.25%)

58 (14.83%)

52 (13.27%)

51 (13.08%)

22 (5.63%)

50 (12.85%)

15 (3.84%)

21 (5.37%)

47 (11.99%)

46 (11.79%)

6 (1.53%)

42 (10.80%)

6 (1.53%)

12 (3.07%)

23 (5.87%)

24 (6.15%)

8 (2.05%)

29 (7.46%)

9 (2.30%)

7 (1.79%)

30 (7.65%)

15 (3.85%)

6 (1.53%)

6.52 / 11 (59.27%)

56 (14.40%)

16 (4.09%)

29 (7.42%)

45 (11.48%)

59 (15.13%)

9 (2.30%)

389

391

391

392

390

391

6.88 / 11 (62.55%)

4.33 / 11 (39.36%)

4.92 / 11 (44.73%)

6.37 / 11 (57.91%)

6.37 / 11 (57.91%)

3.37 / 11 (30.64%)

32) During the previous 3 years, I have written and published the following number of articles in professional journals/publications.

Chart Wizard

Percen tage

Respo nses

0

72.1

287

1

13.6

54

2

5.3

21

3

2.3

9

>3

6.8

27

Total responses:

398

33) People who use the services of a Project Manager generally consider those services critical to the success of their project.

Chart Wizard

1 Agree

2

3 Not Sure

4

5 Disagree

Responses

Average Score

223 (56.31%)

117 (29.55%)

29 (7.32%)

22 (5.56%)

5 (1.26%)

396

1.66 / 5 (33.20%)

Chart

1.66 / 5 (33.20%)

34) How many years formal EDUCATION beyond High School do you believe it takes to produce a competent practitioner, worthy of the title of “Professional Project Manager” ?

Chart Wizard

Percen tage

Respo nses

2

11.4

45

3

14.4

57

4

26.3

104

5

16.9

67

>5

31.1

123

Total responses:

18 /27

396

35) How many years working EXPERIENCE do you believe it takes to produce a competent practitioner, worthy of the title of “Project Manager” ?

Chart Wizard Percen tage

Respo nses

0.8

3

2-3

9.1

36

3-4

20.2

80

4-5

21.5

85

48.5

192

0-1

>5

Total responses:

396

36) How many years of apprenticeship, internship or supervised work experience (Residency) do you think of when you see the word “Profession”?

Chart Wizard Percen tage

Respo nses

2

23.7

94

3

25.9

103

4

20.2

80

5

12.1

48

>5

18.1

72

Total responses:

397

37) I believe the primary purpose of occupational licensing of any profession is to protect the public from quacks, charlatans or incompetence .

Chart Wizard Percen tage

Respo nses

Yes

87.1%

345

No

12.9%

51

Total responses:

396

38) I believe that licensing of Project Managers will lead to projects being completed on time, within budget and substantially fulfilling all technical requirements . Wizard

Yes No

Percen tage

Respo nses

43.0%

169

57.0%

224

Total responses:

19 /27

Chart

393

Real Estate 2 (0.52%)

2 (0.52%)

29 (7.49%)

Medical Doctor Airline Pilot

10 (2.58%)

3 (0.78%)

Real Estate 1 (0.26%) Agent Building Contractor

15 (3.86%)

35 (9.00%)

2 (0.52%)

Medical Doctor Lawyer

1 (0.26%)

Accountant - Lawyer

62 (15.94%)

101 (26.17%)

19 (4.88%)

Medical Doctor Accountant

2 (0.51%)

2 80%

Lawyer 31 (8.03%) Real Estate Agent

3 (0.77%)

Plumber Airline Pilot

1 100%

4 (1.03%)

38 (9.82%)

6 (1.55%)

37 (9.51%)

77 (19.95%)

10 (2.58%)

41 (10.54%)

3 (0.77%)

3 60%

8 (2.07%)

56 (14.47%)

11 (2.85%)

46 (11.83%)

65 (16.84%)

13 (3.36%)

56 (14.40%)

10 (2.57%)

4 40%

5 (1.29%)

36 (9.30%)

27 (6.99%)

42 (10.80%)

35 (9.07%)

13 (3.36%)

35 (9.00%)

10 (2.57%)

5 20%

42

7 20%

46

11 (2.84%)

67 (17.36%)

2 (0.51%)

8 (2.07%)

48 (12.40%)

5 (1.29%)

22 (5.66%)

20 /27

186 (48.06%)

141 (36.53%)

200 (51.41%)

46 (11.92%)

196 (50.65%)

139 (35.73%)

62 (15.94%)

6 Equal

67

11 (2.84%)

50 (12.95%)

5 (1.29%)

4 (1.04%)

44 (11.37%)

6 (1.54%)

38 (9.77%)

8 40%

94

2 (0.52%)

53 (13.73%)

4 (1.03%)

7 (1.81%)

33 (8.53%)

11 (2.83%)

61 (15.68%)

9 60%

70

5 (1.29%)

23 (5.96%)

1 (0.26%)

9 (2.33%)

21 (5.43%)

10 (2.57%)

119 (30.59%)

10 80%

11 100%

47

3 (0.78%)

4 (1.04%)

2 (0.51%)

3 (0.78%)

6 (1.55%)

5 (1.29%)

59 (15.17%)

Chart Wizard

387

387

386

389

386

387

389

389

Responses

8.39 / 11

5.09 / 11 (46.27%)

6.92 / 11 (62.91%)

4.92 / 11 (44.73%)

3.75 / 11 (34.09%)

6.69 / 11 (60.82%)

4.72 / 11 (42.91%)

8.54 / 11 (77.64%)

Average Score

EXAMPLE- Based on the selection in the sample answer below, you are saying that in your opinion, a Baker is perceived by you to be 80% more of a profession than that of a Candlestick Maker

39) Listed below are pairs of occupations. From each pair, mark the circle that best represents your ideal IMAGE or IMPRESSION of how much of a PROFESSION each occupation is relative to the other. If you consider them EQUAL, mark the 0%.

64 (16.54%)

61 (15.76%)

56 (14.47%)

90 (23.20%)

32 (8.27%)

58 (14.95%)

45 (11.63%)

66 (17.01%)

5 (1.29%)

8 (2.06%)

7 (1.82%)

1 (0.26%)

(11.89%)

6 (1.55%)

1 (0.26%)

8 (2.08%)

3 (0.77%)

(17.31%)

6 (1.55%)

3 (0.77%)

7 (1.82%)

6 (1.55%)

(18.09%)

5.31 / 11 (48.27%)

1 (0.26%)

2 (0.52%)

5 (1.30%)

4 (1.03%)

(24.29%)

1 (0.26%)

0 (0.00%)

4 (1.04%)

4 (1.03%)

(12.14%)

387

388

385

388

3.32 / 11 (30.18%)

3.97 / 11 (36.09%)

4.56 / 11 (41.45%)

2.82 / 11 (25.64%)

(76.27%)

21 /27

Chart Wizard

EXAMPLE- Based on the selection in the sample answer below, you are saying that in your opinion, a Baker is perceived by you to be 80% more of a profession than that of a Candlestick Maker

40) CONTINUED- Listed below are pairs of occupations. From each pair, mark the circle that best represents your ideal IMAGE or IMPRESSION of how much of a PROFESSION each occupation is relative to the other. If you consider them EQUAL, mark the 0%.

110 (28.42%)

83 (21.39%)

103 (26.75%)

Medical Doctor Building Contractor

55 (14.18%)

58 (15.06%)

22 (5.67%)

74 (19.22%)

Accountant - Real Estate Agent

57 (14.81%)

46 (11.95%)

16 (4.16%)

Medical Doctor Project Manager

19 (4.90%)

28 (7.22%)

34 (8.76%)

109 (28.09%)

116 Medical (29.90%) Doctor Real Estate Agent

64 (16.49%)

(10.85%)

Agent Engineer

10 (2.56%)

7 (1.79%)

1 (0.26%)

19 (4.86%)

2 (0.51%)

1 (0.26%)

2 (0.51%)

18 (4.62%)

3 (0.78%)

5 (1.29%)

0 (0.00%)

Medical Doctor Engineer

Accountant - Building Contractor

Real Estate Agent Electrician

Accountant - Plumber

Accountant - Airline Pilot

Plumber Project Manager

Building Contractor Engineer

Engineer Electrician

Real Estate Agent Plumber

Building Contractor Electrician

Accountant - Project Manager

1 100%

4 40%

5 20% 147 (37.60%)

6 Equal

15 (3.86%)

14 (3.60%)

32 (8.23%)

105 (26.99%)

55 (14.10%) 69 (17.69%) 81 (20.77%) 85 (21.79%)

47 (12.02%) 79 (20.20%) 66 (16.88%)

3 60%

10 (2.57%)

6 (1.54%)

11 (2.83%)

11 (2.83%)

6 (1.54%)

21 (5.40%)

15 (3.86%)

10 (2.56%)

31 (7.97%)

8 (2.05%)

10 (2.57%)

9 (2.33%)

13 (3.33%)

20 (5.14%)

21 (5.43%)

60 (15.50%)

27 (6.92%)

38 (9.74%)

47 (12.08%) 52 (13.37%)

27 (6.98%)

18 (4.62%)

2 (0.51%)

8 40%

10 (2.56%)

6 (1.53%)

34 (8.74%)

11 (2.82%)

4 (1.02%)

9 60%

34 (8.72%)

12 (3.08%)

12 (3.08%)

28 (7.24%)

16 (4.10%)

37 (9.49%)

5 (1.29%)

8 (2.07%)

3 (0.77%)

35 (9.00%)

14 (3.59%)

2 (0.51%)

4 (1.03%)

3 (0.77%)

18 (4.63%)

6.08 / 11 (55.27%)

47 (12.05%) 42 (10.77%)

51 (13.11%) 41 (10.54%)

33 (8.46%)

14 (3.60%)

1 (0.26%)

4 (1.03%)

3 (0.77%)

1 (0.26%)

11 100%

52 (13.33%) 82 (21.03%) 98 (25.13%) 50 (12.82%)

68 (17.57%) 47 (12.14%)

11 (2.82%)

22 /27

131 (33.59%)

144 (37.02%)

112 (28.94%)

3 (0.77%)

13 (3.34%)

4 (1.03%)

1 (0.26%)

10 80%

42 (10.80%) 55 (14.14%) 51 (13.11%) 41 (10.54%)

9 (2.30%)

83 (21.34%) 83 (21.34%)

25 (6.41%)

3 (0.77%)

7 20%

86 (22.11%) 53 (13.62%) 87 (22.37%) 67 (17.22%)

45 (11.54%)

112 (28.79%)

47 (12.05%) 72 (18.46%) 72 (18.46%) 54 (13.85%) 82 (21.03%)

5 (1.29%)

6 (1.54%)

9 (2.31%)

53 (13.55%) 60 (15.35%) 75 (19.18%) 83 (21.23%) 72 (18.41%)

5 (1.29%)

32 (8.21%)

31 (7.93%)

2 80%

390

389

387

390

389

390

389

391

389

390

391

Responses

6.79 / 11 (61.73%)

5.81 / 11 (52.82%)

6.18 / 11 (56.18%)

4.51 / 11 (41.00%)

7.44 / 11 (67.64%)

8.45 / 11 (76.82%)

6.99 / 11 (63.55%)

4.32 / 11 (39.27%)

6.75 / 11 (61.36%)

4.92 / 11 (44.73%)

4.69 / 11 (42.64%)

Average Score

32 (8.21%)

1 (0.26%)

2 (0.51%)

2 (0.51%)

1 (0.26%)

4 (1.03%)

30 (7.71%)

39 (10.03%)

1 (0.26%)

Lawyer Building Contractor

Accountant - Engineer

Electrician Airline Pilot

Lawyer Engineer

Electrician Plumber

Building Contractor - Airline Pilot

Lawyer Electrician

Lawyer Plumber

Engineer Airline Pilot

1 100%

9 (2.31%)

67 (17.22%)

61 (15.68%)

3 (0.78%)

9 (2.31%)

12 (3.08%)

3 (0.77%)

1 (0.26%)

70 (17.95%)

2 80%

20 (5.14%)

91 (23.39%)

75 (19.28%)

8 (2.07%)

9 (2.31%)

27 (6.92%)

4 (1.03%)

13 (3.35%)

84 (21.54%)

3 60%

39 (10.03%)

66 (16.97%)

85 (21.85%)

6 (1.55%)

29 (7.44%)

47 (12.05%)

14 (3.60%)

26 (6.70%)

62 (15.90%)

4 40%

31 (7.97%)

46 (11.83%)

47 (12.08%)

12 (3.10%)

44 (11.28%)

51 (13.08%)

7 (1.80%)

26 (6.70%)

53 (13.59%)

5 20%

7 20%

53 (13.62%)

14 (3.60%)

14 (3.60%)

40 (10.34%)

9 (2.31%)

30 (7.69%)

52 (13.37%)

45 (11.60%)

9 (2.31%)

23 /27

130 (33.42%)

45 (11.57%)

49 (12.60%)

55 (14.21%)

280 (71.79%)

176 (45.13%)

56 (14.40%)

180 (46.39%)

53 (13.59%)

6 Equal

48 (12.34%)

7 (1.80%)

9 (2.31%)

81 (20.93%)

7 (1.79%)

19 (4.87%)

67 (17.22%)

48 (12.37%)

9 (2.31%)

8 40%

36 (9.25%)

8 (2.06%)

13 (3.34%)

68 (17.57%)

1 (0.26%)

18 (4.62%)

71 (18.25%)

35 (9.02%)

12 (3.08%)

9 60%

15 (3.86%)

3 (0.77%)

2 (0.51%)

85 (21.96%)

0 (0.00%)

4 (1.03%)

81 (20.82%)

11 (2.84%)

3 (0.77%)

10 80%

11 100%

7 (1.80%)

3 (0.77%)

4 (1.03%)

25 (6.46%)

1 (0.26%)

4 (1.03%)

32 (8.23%)

2 (0.52%)

3 (0.77%)

Chart Wizard

389

389

389

387

390

390

389

388

390

Response s

6.37 / 11 (57.91%)

3.85 / 11 (35.00%)

4.11 / 11 (37.36%)

8.04 / 11 (73.09%)

5.64 / 11 (51.27%)

5.68 / 11 (51.64%)

8.09 / 11 (73.55%)

6.45 / 11 (58.64%)

4.00 / 11 (36.36%)

Average Score

EXAMPLE- Based on the selection in the sample answer below, you are saying that in your opinion, a Baker is perceived by you to be 80% more of a profession than that of a Candlestick Maker

41) CONTINUED- Listed below are pairs of occupations. From each pair, mark the circle that best represents your ideal IMAGE or IMPRESSION of how much of a PROFESSION each occupation is relative to the other. If you consider them EQUAL, mark the 0%.

1 (0.26%)

Engineer Project Manager

7 (1.81%)

9 (2.33%)

13 (3.36%)

49 (12.69%) 32 (8.27%)

69 (17.88%) 39 (10.08%)

73 (18.91%) 133 (34.37%)

126 (32.64%) 42 (10.85%)

30 (7.77%) 51 (13.18%)

14 (3.63%)

24 (6.20%)

2 (0.52%)

6 (1.55%)

0 (0.00%)

5.81 / 11 (52.82%)

39 (10.08%)

6 (1.55%)

387

386

6.55 / 11 (59.55%)

5.09 / 11 (46.27%)

3 (0.77%)

3 (0.77%)

1 (0.26%)

Lawyer - Project Manager

Real Estate Agent - Airline Pilot

Building Contractor Project Manager

60 (15.31%)

1 (0.26%)

2 (0.51%)

3 (0.77%)

3 (0.77%)

17 (4.36%) 34 (8.72%)

11 (2.81%) 24 (6.12%)

Accountant Electrician

3 60%

4 40%

5 20%

61 (15.64%)

87 (22.19%)

122 (31.28%)

88 (22.45%)

132 (33.67%)

6 Equal

7 (1.79%)

11 (2.81%)

24 /27

91 (23.21%)

10 (2.56%) 11 (2.82%) 29 (7.44%)

54 (13.85%)

79 (20.15%)

19 (4.85%) 24 (6.12%) 34 (8.67%) 39 (9.95%)

4 (1.02%)

2 80%

Lawyer - Airline Pilot

1 100%

9 60%

10 80%

9 (2.30%)

3 (0.77%)

65 (16.58%)

82 (20.92%)

71 (18.21%)

55 (14.03%)

79 (20.26%)

54 (13.78%)

91 (23.33%)

27 (6.92%) 21 (5.38%) 28 (7.18%) 15 (3.85%) 45 (11.54%)

11 100%

22 (5.61%)

46 (11.79%)

8 (2.05%)

1 (0.26%)

33 (8.42%) 31 (7.91%) 20 (5.10%) 10 (2.55%)

8 40%

18 (4.59%) 12 (3.06%)

46 (11.73%)

7 20%

Chart Wizard

392

390

390

392

392

Response s

7.73 / 11 (70.27%)

8.45 / 11 (76.82%)

5.74 / 11 (52.18%)

4.75 / 11 (43.18%)

6.15 / 11 (55.91%)

Average Score

EXAMPLE- Based on the selection in the sample answer below, you are saying that in your opinion, a Baker is perceived by you to be 80% more of a profession than that of a Candlestick Maker

42) CONTINUED- Listed below are pairs of occupations. From each pair, mark the circle that best represents your ideal IMAGE or IMPRESSION of how much of a PROFESSION each occupation is relative to the other. If you consider them EQUAL, mark the 0%.

8 (2.07%)

Building Contractor - Plumber

23 (5.88%)

0 (0.00%)

76 (19.44%)

0 (0.00%)

103 (26.55%)

4 (1.02%)

Engineer Plumber

Electrician Project Manager

Medical Doctor Electrician

Real Estate Agent - Project Manager

Medical Doctor Plumber

Airline Pilot Project Manager

53 (13.66%)

4 (1.02%)

76 (19.44%)

7 (1.79%)

73 (18.67%)

28 (7.14%) 32 (8.16%)

124 (31.96%)

3 (0.77%)

122 (31.20%)

5 (1.28%)

57 (14.58%)

62 (15.82%)

39 (10.05%)

4 (1.02%)

41 (10.49%)

9 (2.30%)

94 (24.04%) 49 (12.53%)

63 (16.11%)

3 (0.77%)

46 (11.76%)

9 (2.30%)

63 (16.07%)

25 /27

104 (26.53%)

15 (3.87%) 38 (9.79%)

2 (0.52%)

67 (17.09%)

4 (1.02%)

74 (18.93%)

8 (2.05%)

5 (1.29%)

85 (21.68%)

5 (1.28%)

86 (21.99%)

8 (2.05%)

1 (0.26%)

85 (21.68%)

2 (0.51%)

56 (14.32%)

2 (0.51%)

4 (1.03%)

59 (15.05%)

3 (0.77%)

37 (9.46%)

0 (0.00%)

5.91 / 11 (53.73%)

24 (6.12%) 27 (6.89%) 21 (5.36%) 17 (4.34%) 10 (2.55%)

4 (1.03%)

11 (2.81%) 39 (9.95%) 35 (8.93%)

20 (5.12%) 39 (9.97%)

22 (5.63%)

54 (13.81%)

392

388

392

391

391

391

5.60 / 11 (50.91%)

2.87 / 11 (26.09%)

8.59 / 11 (78.09%)

3.06 / 11 (27.82%)

7.99 / 11 (72.64%)

4.09 / 11 (37.18%)

43) How many years of formal (university level) education do you think of when you see the word “Profession”?

Chart Wizard

2

Percen tage

Respo nses

5.3

21

3

9.9

39

4

44.2

174

5

18.8

74

>5

21.8 Total responses:

86 394

44) I believe occupational licensing of any type results in a monopoly and a form of restraint of trade.

Chart Wizard

Yes No

Percen tage

Respo nses

27.4%

107

72.6%

284

Total responses:

391

45) Below is a list of 11 attributes normally associated with an occupation being recognized or accepted as being a “Profession”. From the selection below, please indicate the relative weighting or importance you give to each of these attributes. You can mark any value between 0 (No importance to evaluating or determining a profession), to a maximum of 100 points (Essential attribute necessary to be considered a profession)

Click to view ALL responses

46) (Continued from #45) Below is the next list of 11 attributes normally associated with an occupation being recognized or accepted as being a “Profession”. From the selection below, please indicate the relative weighting or importance you give to each of these attributes. You can mark any value between 0 (No importance to evaluating or determining a profession), to a maximum of 100 points (Essential attribute necessary to be considered a profession)

Click to view ALL responses

26 /27

47) Rank the following list of professions in the order you perceive them to represent your professional ideal. (1 being most professional, 21 being least professional) NOTE. Please give a different rank for each question. Do not use the same number twice.

Chart Wizard

Average Score

Respon ses

Aircraft Pilots (Commercial Jet)

5.48 / 21

392

Accountant (Certified Public)

7.05 / 21

391

Commercial Tractor Trailer Truck Driver

16.39 / 21

386

Barber/Cosmetologists

16.75 / 21

387

MBA’s/Business Consultants

8.77 / 21

388

Librarians

13.77 / 21

387

Civil Engineers/Land Surveyors

8.45 / 21

388

Dentists

5.54 / 21

390

Lawyers

6.52 / 21

391

Physicians (Medical Doctors)

3.41 / 21

390

High School Principal

11.48 / 21

389

Construction Managers

10.12 / 21

388

Project Managers

7.36 / 21

388

School Bus Drivers

16.99 / 21

384

Computer/Software Engineers

9.99 / 21

388

Electricians

13.17 / 21

385

Plumbers

14.44 / 21

385

Registered Nurses

10.03 / 21

388

Real Estate Brokers/Appraisers

15.48 / 21

384

Insurance Agents

15.79 / 21

384

Electrical/Telecommunication Engineers

9.69 / 21

387

9.69 / 21

27 /27

Appendix B: Codes of ethics INCOSE Code of Ethics This Code is concerned with how certain fundamental imperatives apply to one's conduct as an engineering professional. These imperatives are expressed in a general form to emphasize that principles which apply to engineering ethics are derived from more general ethical principles. It is understood that some words and phrases in a code of ethics are subject to varying interpretations, and that any ethical principle may conflict with other ethical principles in specific situations. Questions related to ethical conflicts can best be answered by thoughtful consideration of fundamental principles, rather than reliance on detailed regulations. Preamble Engineering is a profession that requires its practitioners to be well educated and knowledgeable. Systems Engineering, in particular, is a unique discipline in that 1) it is highly integrative, spanning elements of many activities, 2) often provides representation of stakeholders' interests other than employer or client, and 3) operates in largely international arenas where value systems, beliefs and customs vary widely. The practice of Systems Engineering can result in significant social and environmental benefits, but only if unintended and undesired effects are considered and mitigated. Fundamental Principles Systems Engineers uphold and advance the integrity, honor and dignity of the engineering profession by: • Being honest and impartial; • Maintaining the highest levels of integrity and keeping abreast of the knowledge of their disciplines; • Striving to increase the competence and prestige of the engineering profession; and • Supporting the educational institutions, the professional societies and technical societies of their disciplines.

Fundamental Duties to Society and Public Infrastructure • Guard the public interest and protect the environment, safety and welfare of those affected by engineering activities and technological artifacts. • Accept responsibility for your actions and engineering results, including being open to ethical scrutiny and assessment. • Proactively mitigate unsafe practice. • Manage risk using knowledge granted by a whole system viewpoint and understanding of systemic interfaces. • Promote the understanding, implementation, and acceptance of prudent Systems Engineering measures .

Rules of Practice • Act legally, honorably, honestly, justly, and responsibly. • Respect, protect, and preserve the intellectual properties of others. • Honor all legal contracts and agreements. • Treat all constituents fairly. • Give prudent advice. Be truthful, objective, and maintain your professional and technical integrity.

• • •

Provide diligent and competent services to the best of your ability. Respect the trust and the privileges granted to you. Avoid conflicts of interest and the appearance thereof.

ASCE Code of Ethics Fundamental Principles2 Engineers uphold and advance the integrity, honor and dignity of the engineering profession by: 1. using their knowledge and skill for the enhancement of human welfare and the environment; 2. being honest and impartial and serving with fidelity the public, their employers and clients; 3. striving to increase the competence and prestige of the engineering profession; and 4. supporting the professional and technical societies of their disciplines. Fundamental Canons 1. Engineers shall hold paramount the safety, health and welfare of the public and shall strive to comply with the principles of sustainable development3 in the performance of their professional duties. 2. Engineers shall perform services only in areas of their competence. 3. Engineers shall issue public statements only in an objective and truthful manner. 4. Engineers shall act in professional matters for each employer or client as faithful agents or trustees, and shall avoid conflicts of interest. 5. Engineers shall build their professional reputation on the merit of their services and shall not compete unfairly with others. 6. Engineers shall act in such a manner as to uphold and enhance the honor, integrity, and dignity of the engineering profession and shall act with zero-tolerance for bribery, fraud, and corruption. 7. Engineers shall continue their professional development throughout their careers, and shall provide opportunities for the professional development of those engineers under their supervision. Guidelines to Practice Under the Fundamental Canons of Ethics CANON 1. Engineers shall hold paramount the safety, health and welfare of the public and shall strive to comply with the principles of sustainable development in the performance of their professional duties. a. Engineers shall recognize that the lives, safety, health and welfare of the general public are dependent upon engineering judgments, decisions and practices incorporated into structures, machines, products, processes and devices. b. Engineers shall approve or seal only those design documents, reviewed or prepared by them, which are determined to be safe for public health and welfare in conformity with accepted engineering standards. c. Engineers whose professional judgment is overruled under circumstances where the safety, health and welfare of the public are endangered, or the principles of sustainable development ignored, shall inform their clients or employers of the possible consequences.

d. Engineers who have knowledge or reason to believe that another person or firm may be in violation of any of the provisions of Canon 1 shall present such information to the proper authority in writing and shall cooperate with the proper authority in furnishing such further information or assistance as may be required. e. Engineers should seek opportunities to be of constructive service in civic affairs and work for the advancement of the safety, health and well-being of their communities, and the protection of the environment through the practice of sustainable development. f. Engineers should be committed to improving the environment by adherence to the principles of sustainable development so as to enhance the quality of life of the general public. CANON 2. Engineers shall perform services only in areas of their competence. a. Engineers shall undertake to perform engineering assignments only when qualified by education or experience in the technical field of engineering involved. b. Engineers may accept an assignment requiring education or experience outside of their own fields of competence, provided their services are restricted to those phases of the project in which they are qualified. All other phases of such project shall be performed by qualified associates, consultants, or employees. c. Engineers shall not affix their signatures or seals to any engineering plan or document dealing with subject matter in which they lack competence by virtue of education or experience or to any such plan or document not reviewed or prepared under their supervisory control. CANON 3. Engineers shall issue public statements only in an objective and truthful manner. a. Engineers should endeavor to extend the public knowledge of engineering and sustainable development, and shall not participate in the dissemination of untrue, unfair or exaggerated statements regarding engineering. b. Engineers shall be objective and truthful in professional reports, statements, or testimony. They shall include all relevant and pertinent information in such reports, statements, or testimony. c. Engineers, when serving as expert witnesses, shall express an engineering opinion only when it is founded upon adequate knowledge of the facts, upon a background of technical competence, and upon honest conviction. d. Engineers shall issue no statements, criticisms, or arguments on engineering matters which are inspired or paid for by interested parties, unless they indicate on whose behalf the statements are made. e. Engineers shall be dignified and modest in explaining their work and merit, and will avoid any act tending to promote their own interests at the expense of the integrity, honor and dignity of the profession CANON 4. Engineers shall act in professional matters for each employer or client as faithful agents or trustees, and shall avoid conflicts of interest. a. Engineers shall avoid all known or potential conflicts of interest with their employers or clients and shall promptly inform their employers or clients of any business association, interests, or circumstances which could influence their judgment or the quality of their services.

b. Engineers shall not accept compensation from more than one party for services on the same project, or for services pertaining to the same project, unless the circumstances are fully disclosed to and agreed to, by all interested parties. c. Engineers shall not solicit or accept gratuities, directly or indirectly, from contractors, their agents, or other parties dealing with their clients or employers in connection with work for which they are responsible. d. Engineers in public service as members, advisors, or employees of a governmental body or department shall not participate in considerations or actions with respect to services solicited or provided by them or their organization in private or public engineering practice. e. Engineers shall advise their employers or clients when, as a result of their studies, they believe a project will not be successful. f. Engineers shall not use confidential information coming to them in the course of their assignments as a means of making personal profit if such action is adverse to the interests of their clients, employers or the public. g. Engineers shall not accept professional employment outside of their regular work or interest without the knowledge of their employers. CANON 5. Engineers shall build their professional reputation on the merit of their services and shall not compete unfairly with others. a. Engineers shall not give, solicit or receive either directly or indirectly, any political contribution, gratuity, or unlawful consideration in order to secure work, exclusive of securing salaried positions through employment agencies. b. Engineers should negotiate contracts for professional services fairly and on the basis of demonstrated competence and qualifications for the type of professional service required. c. Engineers may request, propose or accept professional commissions on a contingent basis only under circumstances in which their professional judgments would not be compromised. d. Engineers shall not falsify or permit misrepresentation of their academic or professional qualifications or experience. e. Engineers shall give proper credit for engineering work to those to whom credit is due, and shall recognize the proprietary interests of others. Whenever possible, they shall name the person or persons who may be responsible for designs, inventions, writings or other accomplishments. f. Engineers may advertise professional services in a way that does not contain misleading language or is in any other manner derogatory to the dignity of the profession. Examples of permissible advertising are as follows: o Professional cards in recognized, dignified publications, and listings in rosters or directories published by responsible organizations, provided that the cards or listings are consistent in size and content and are in a section of the publication regularly devoted to such professional cards. o Brochures which factually describe experience, facilities, personnel and capacity to render service, providing they are not misleading with respect to the engineer's participation in projects described. o Display advertising in recognized dignified business and professional publications, providing it is factual and is not misleading with respect to the engineer's extent of participation in projects described. o A statement of the engineers' names or the name of the firm and statement of the type of service posted on projects for which they render services.

Preparation or authorization of descriptive articles for the lay or technical press, which are factual and dignified. Such articles shall not imply anything more than direct participation in the project described. o Permission by engineers for their names to be used in commercial advertisements, such as may be published by contractors, material suppliers, etc., only by means of a modest, dignified no4tation acknowledging the engineers' participation in the project described. Such permission shall not include public endorsement of proprietary products. g. Engineers shall not maliciously or falsely, directly or indirectly, injure the professional reputation, prospects, practice or employment of another engineer or indiscriminately criticize another's work. h. Engineers shall not use equipment, supplies, laboratory or office facilities of their employers to carry on outside private practice without the consent of their employers. o

CANON 6. Engineers shall act in such a manner as to uphold and enhance the honor, integrity, and dignity of the engineering profession and shall act with zero-tolerance for bribery, fraud, and corruption. a. Engineers shall not knowingly engage in business or professional practices of a fraudulent, dishonest or unethical nature. b. Engineers shall be scrupulously honest in their control and spending of monies, and promote effective use of resources through open, honest and impartial service with fidelity to the public, employers, associates and clients. c. Engineers shall act with zero-tolerance for bribery, fraud, and corruption in all engineering or construction activities in which they are engaged. d. Engineers should be especially vigilant to maintain appropriate ethical behavior where payments of gratuities or bribes are institutionalized practices. e. Engineers should strive for transparency in the procurement and execution of projects. Transparency includes disclosure of names, addresses, purposes, and fees or commissions paid for all agents facilitating projects. f. Engineers should encourage the use of certifications specifying zero-tolerance for bribery, fraud, and corruption in all contracts. CANON 7. Engineers shall continue their professional development throughout their careers, and shall provide opportunities for the professional development of those engineers under their supervision. a. Engineers should keep current in their specialty fields by engaging in professional practice, participating in continuing education courses, reading in the technical literature, and attending professional meetings and seminars. b. Engineers should encourage their engineering employees to become registered at the earliest possible date. c. Engineers should encourage engineering employees to attend and present papers at professional and technical society meetings. d. Engineers shall uphold the principle of mutually satisfying relationships between employers and employees with respect to terms of employment including professional grade descriptions, salary ranges, and fringe benefits. 1. As adopted September 2, 1914; amended November 10, 1996; and most recently July, 2006. 2. The American Society of Civil Engineers adopted THE FUNDAMENTAL PRINCIPLES OF THE abet Code of Ethics of Engineers as accepted by the

Accreditation Board for Engineering and Technology, Inc. (ABET). (By ASCE Board of Direction action April 12-14, 1975) 3. In November 1996, the ASCE Board of Direction adopted the following definition of Sustainable Development: "Sustainable Development is the challenge of meeting human needs for natural resources, industrial products, energy, food, transportation, shelter, and effective waste management while conserving and protecting environmental quality and the natural resource base essential for future development." 4. As adopted September 2, 1914; amended November 10, 1996; and most recently amended July, 22-23, 2006. Guidelines and Policies Encouraging "Zero Tolerance" for Bribery1 ASCE, in cooperation with others, has begun a journey that will lead over time to the development of principles of professional conduct that will help reduce corruption in all its forms, such as fraud, bribery and other unethical and illegal practices in the engineering and construction industry. Only with openness and transparency in the procurement and delivery of engineering services can resources be efficiently allocated for their intended purpose. As a direct result, with the additional financial resources available, sustainable development and the welfare of the world's population can improve and the quality of life for people everywhere be achieved. The American Society of Civil Engineers is joining the consulting engineering profession, the construction industry, major lenders, the legal profession, and organizations committed to the battle against worldwide corruption. To meet this challenge, the Society pledges the development of principles and over time adoption of guidelines and policies for ethical professional practice by members of engineering societies in the U.S. and worldwide. These guidelines and policies will include zero tolerance for bribery and promote effective programs for reporting unethical behavior. The Society will consider the best practices adopted by the engineering and construction industry in developing or benchmarking anti-bribery programs. ASCE, supported by the Task Committee on Global Principles for Professional Conduct, will provide -- through its media outreach and forums of leaders, and other avenues -- a global communications platform for the issue. Our work will be performed in an open and transparent manner to encourage good faith discussion and negotiations leading to principles and policies that other societies could adopt.

AIPM Code of Ethics Preamble Project Managers, in the pursuit of their profession, affect the quality of life for all people in society. It is therefore vital that project managers conduct their work in an ethical manner to earn and maintain the confidence of team members, colleagues, employees, employers, clients and the public. Article 1 Project managers shall actively maintain high standards of personal and professional conduct and: (a) accept responsibility for their advice, actions and omissions; (b) undertake projects and accept responsibility only if qualified by training or experience, or after full disclosure to their employers or clients of pertinent qualifications;

(c) maintain their professional skills at 'state-of-the-art' level and recognize the importance of continued personal development and education; (d) advance the integrity and prestige of the profession by practicing in a dignified manner; (e) support the Australian Institute of Project Management by actively participating and encouraging colleagues and co-workers to participate; (f) obey the laws of the country in which work is being performed, and (g) be aware and respectful of the culture of other countries in which work is being performed. Article 2 In their work, project managers shall: (a) provide the necessary project leadership to promote maximum productivity while striving to minimize costs; (b) apply state-of-the-art project management tools and techniques to ensure quality, cost and time objectives in the project plan are met; (c) treat all project team members, colleagues and co-workers fairly, regardless of race, religion, sex, age or national origin; (d) seek, accept and offer honest criticism of work and properly credit the contribution of others; and (e) assist project team members, colleagues and co-workers in their professional development. Article 3 In their relations with their employers and clients, project managers shall: (a) act as faithful agents or trustees for their employers and clients in professional or business matters; (b) keep information on the business affairs or technical processes of an employer or client in confidence at all times until such information is properly released; (c) inform their employers, clients, professional societies or public agencies of which they are members, or to which they may make any presentations, of any circumstance that could lead to a conflict of interest; (d) neither give or accept, directly or indirectly, any gift, payment or service of more than nominal value to or from those having business relationships with their employers or clients; and (e) be honest and realistic in reporting project quality, cost and time. Article 4 In fulfilling their responsibilities to the community, project managers shall: (a) protect the safety, health and welfare of the public and speak out against any abuse which has an affect on the public interest; and (b) seek to extend public knowledge and appreciation of the project management profession and its achievements.

IEEE Code of Ethics We, the members of the IEEE, in recognition of the importance of our technologies in affecting the quality of life throughout the world, and in accepting a personal obligation to our profession, its members and the communities we serve, do hereby commit ourselves to the highest ethical and professional conduct and agree: 1. to accept responsibility in making decisions consistent with the safety, health and welfare of the public, and to disclose promptly factors that might endanger the public or the environment; 2. to avoid real or perceived conflicts of interest whenever possible, and to disclose them to affected parties when they do exist; 3. to be honest and realistic in stating claims or estimates based on available data; 4. to reject bribery in all its forms; 5. to improve the understanding of technology, its appropriate application, and potential consequences; 6. to maintain and improve our technical competence and to undertake technological tasks for others only if qualified by training or experience, or after full disclosure of pertinent limitations; 7. to seek, accept, and offer honest criticism of technical work, to acknowledge and correct errors, and to credit properly the contributions of others; 8. to treat fairly all persons regardless of such factors as race, religion, gender, disability, age, or national origin; 9. to avoid injuring others, their property, reputation, or employment by false or malicious action; 10. to assist colleagues and co-workers in their professional development and to support them in following this code of ethics. Approved by the IEEE Board of Directors February 2006

CMAA Code of Ethics

Since 1982, the Construction Management Association of America (CMAA) has taken a leadership role in regard to critical issues impacting the construction and program management industry, including the setting of ethical standards of practice for the Professional Construction Manager. The Board of Directors of CMAA has adopted the following Code of Professional Ethics of the Construction Manager (CODE) which apply to CMAA members in performance of their services as Construction and Program Managers. This Code applies to the individuals and to organizations who are members of CMAA.

All members of the Construction Management Association of America commit to conduct themselves and their practice of Construction and Program Management in accordance with the Code of Professional Ethics of the Construction Manager. As a professional engaged in the business of providing construction and program management services, and as a member of CMAA, I agree to conduct myself and my business in accordance with the following: 1. Client Service. I will serve my clients with honesty, integrity, candor, and objectivity. I will provide my services with competence, using reasonable care, skill and diligence consistent with the interests of my client and the applicable standard of care. 2. Representation of Qualifications and Availability. I will only accept assignments for which I am qualified by my education, training, professional experience and technical competence, and I will assign staff to projects in accordance with their qualifications and commensurate with the services to be provided, and I will only make representations concerning my qualifications and availability which are truthful and accurate. 3. Standards of Practice. I will furnish my services in a manner consistent with the established and accepted standards of the profession and with the laws and regulations which govern its practice. 4. Fair Competition. I will represent my project experience accurately to my prospective clients and offer services and staff that I am capable of delivering. I will develop my professional reputation on the basis of my direct experience and service provided, and I will only engage in fair competition for assignments. 5. Conflicts of Interest. I will endeavor to avoid conflicts of interest; and will disclose conflicts which in my opinion may impair my objectivity or integrity. 6. Fair Compensation. I will negotiate fairly and openly with my clients in establishing a basis for compensation, and I will charge fees and expenses that are reasonable and commensurate with the services to be provided and the responsibilities and risks to be assumed. 7. Release of Information. I will only make statements that are truthful, and I will keep information and records confidential when appropriate and protect the proprietary interests of my clients and professional colleagues. 8. Public Welfare. I will not discriminate in the performance of my Services on the basis of race, religion, national origin, age, disability, or sexual orientation. I will not knowingly violate any law, statute, or regulation in the performance of my professional services. 9. Professional Development. I will continue to develop my professional knowledge and competency as Construction Manager, and I will contribute to the advancement of the construction and program management practice as a profession by fostering research and education and through the encouragement of fellow practitioners. 10. Integrity of the Profession. I will avoid actions which promote my own self-interest at the expense of the profession, and I will uphold the standards of the construction management profession with honor and dignity.

AACE Code of Ethics ƒ ƒ ƒ ƒ ƒ

Introduction I. Relations With the Public II. Relations With Employers and Clients III. Relations With Other Professionals IV. Standards of Professional Performance

Introduction The AACE member, to uphold and advance the honor and dignity of Cost Engineering and the Cost Management profession and in keeping with the high standards of ethical conduct will (1) be honest and impartial and will serve employer, clients, and the public with devotion; (2) strive to increase the competence and prestige of their profession; and (3) will apply knowledge and skill to advance human welfare. I. Relations With the Public A. Members will hold paramount the safety, health, and welfare of the public, including that of future generations. B. Members will endeavor to extend public knowledge and appreciation of cost engineering and cost management and its achievements, and will oppose any untrue, unsupported, or exaggerated statements regarding cost engineering and cost management. C. Members will be dignified and modest, ever upholding the honor and dignity of their profession, and will refrain from self-laudatory advertising. D. Members will express an opinion on a cost engineering or cost management subject only when it is founded on adequate knowledge and honest conviction. E. On cost engineering or cost management matters, members will issue no statements, criticisms, or arguments that are inspired or paid for by an interested party or parties, unless they preface their comments by identifying themselves, by disclosing the identities of the party or parties on whose behalf they are speaking, and by revealing the existence of any pecuniary interest they may have in matters under discussion. F. Members will approve or seal only those documents, reviewed or prepared by them, which are determined to be safe for public health and welfare in conformity with accepted cost engineering, cost management and economic standards. G. Members whose judgment is overruled under circumstances where the safety, health, and welfare of the public are endangered shall inform their clients or employers of the possible consequences. H. Members will work through professional societies to encourage and support others who follow these concepts. I. Members will work only with those who follow these concepts. J. Members shall be objective and truthful in professional reports, statements, or testimony. They shall include all relevant and pertinent information in such reports, statements, and testimony. II. Relations With Employers and Clients A. Members will act in all matters as a faithful agent or trustee for each employer or client. B. Members will act fairly and justly toward vendors and contractors and will not accept any commissions or allowances from vendors or contractors, directly or indirectly. C. Members will inform their employer or client of financial interest in any potential vendor or contractor, or in any invention, machine, or apparatus that is involved in a project or work for either employer or client. Members will not allow such interest to affect any decisions regarding cost engineering or cost management services that they may be called upon to perform.

D. When, as a result of their studies, members believe a project(s) will not be successful, or if their cost engineering and cost management or economic judgment is overruled, they shall so advise their employer or client. E. Members will undertake only those cost engineering and cost management assignments for which they are qualified. Members will engage or advise their employers or clients to engage specialists whenever their employer’s or client’s interests are served best by such an arrangement. Members will cooperate fully with specialists so engaged. F. Members shall treat information coming to them in the course of their assignments as confidential and shall not use such information as a means of making personal profit if such action is adverse to the interests of their clients, their employers, or the public. 1. Members will not disclose confidential information concerning the business affairs or technical processes of any present or former employer or client or bidder under evaluation, without consent, unless required by law.

2. Members shall not reveal confidential information or finding of any commission or board of which they are members, unless required by law. 3. Members shall not duplicate for others, without express permission of the client(s), designs, calculations, sketches, etc., supplied to them by clients. 4. Members shall not use confidential information coming to them in the course of their assignments as a means of making personal profit if such action is adverse to the interests of their clients, employers, or the public. G. Members will not accept compensation—financial or otherwise—from more than one party for the same service, or for other services pertaining to the same work, without the consent of all interested parties. H. Employed members will engage in supplementary employment or consulting practice only with the consent of their employer. I. Members shall not use equipment, supplies, laboratory, or office facilities of their employers to carry on outside private practice without the consent of their employers. J. Members shall not solicit a contract from a governmental body on which a principal officer or employee of their organization serves as a member. K. The member shall act with fairness and justice to all parties when administering a construction (or other) contract. L. Before undertaking work for others in which the member may make improvements, plans, designs, inventions, or records that may justify copyrights or patents, the member shall enter into a positive agreement regarding the rights of respective parties. M. Members shall admit and accept their own errors when proven wrong and refrain from distorting or altering the facts to justify their decisions. N. Members shall not attempt to attract an employee from another employer by false or misleading representations. O. Members shall act in professional matters for each employer or client as faithful agents or trustees and shall avoid conflicts of interest. 1. Members shall avoid all known or potential conflicts of interest with their employers or clients and shall promptly inform their employers or clients of any business association, interests, or circumstances that could influence their judgment or the quality of their services.

2. Members shall not solicit or accept gratuities, directly or indirectly, from contractors, their agents, or other parties dealing with their clients or employers in connection with work for which they are responsible. III. Relations With Other Professionals

A. Members will take care that credit for cost engineering and cost management work is given to those to whom credit is properly due. B. Members will provide prospective employees with complete information on working conditions and their proposed status of employment. After employment begins, they will keep the employee informed of any changes in status and working conditions. C. Members will uphold the principle of appropriate and adequate compensation for those engaged in cost engineering and cost management work, including those in subordinate capacities. D. Members will endeavor to provide opportunity for the professional development and advancement of individuals in their employ or under their supervision. E. Members will not attempt to supplant other cost engineers or cost management professionals in a particular employment after becoming aware that definite steps have been taken toward the others’ employment or after they have been employed. F. Members shall not maliciously or falsely, directly or indirectly, injure the professional reputation, prospects, practice, or employment of another, nor shall they indiscriminately criticize another’s work. Proof that another cost professional has been unethical, illegal, or unfair in his/her practice shall be cause for advising the proper authority. G. Members will not compete unfairly with other cost professionals. H. Members will cooperate in advancing the cost engineering and cost management profession by interchanging information and experience with other cost professionals and students, by contributing to public communication media and to cost engineering, cost management and scientific societies and schools. I. Members will not request, propose, or accept professional commissions on a contingent basis under circumstances that compromise their professional judgments. J. Members will not falsify or permit misrepresentation of their own or their associates’ academic or professional qualifications. They shall not misrepresent or exaggerate their degrees or responsibility in or for the subject matter of prior assignments. Brochures or other presentations incident to the solicitation of employment shall not misrepresent pertinent facts concerning employers, employees, associates, joint ventures, accomplishments, or membership in technical societies. K. Members will prepare articles for the lay or technical press that are only factual, dignified, and free from ostentatious or laudatory implications. Such articles shall not imply credit to the cost professionals for other than their direct participation in the work described unless credit is given to others for their share of the work. L. Members will not campaign, solicit support, or other wise coerce other cost professionals to support their candidacy or the candidacy of a colleague for elective office in a technical association. IV. Standards of Professional Performance

A. Members shall be dignified and modest in explaining their work and merit and will avoid any act tending to promote their own interests at the expense of the integrity, honor, and dignity of the profession. B. Members, when serving as expert witnesses, shall express a cost engineering and cost management opinion only when it is founded upon adequate knowledge of the facts, upon a background of technical competence, and upon honest conviction.

C. Members shall continue their professional development throughout their careers and shall provide opportunities for the professional development of those cost professionals under their supervision. 1. Members should keep current in their specialty fields by engaging in professional practice, participating in continuing education courses, reading in the technical literature, and attending professional meetings and seminars. 2. Members should encourage their cost engineering and cost management employees to become certified at the earliest possible date. 3. Members should encourage their cost engineering and cost management employees to attend and present papers at professional and technical society meetings. 4. Members shall uphold the principle of mutually satisfying relationships between employers and employees with respect to terms of employment including professional grade descriptions, salary ranges, and fringe benefits.

AIC Code of Ethics I. A member shall have full regard to the public interest in fulfilling his or her responsibilities to the employer or client. II. A member shall not engage in any deceptive practice, or in any practice which creates an unfair advantage for the member or another. III. A member shall not maliciously or recklessly injure or attempt to injure, whether directly or indirectly, the professional reputation of others. IV. A member shall ensure that when providing a service which includes advice, such advice shall be fair and unbiased. V. A member shall not divulge to any person, firm, or company, information of a confidential nature acquired during the course of professional activities. VI. A member shall carry out responsibilities in accordance with current professional practice, so far as it lies within his or her power. VII. A member shall keep informed of new thought and development in the construction process appropriate to the type and level of his or her responsibilities and shall support research and the educational processes associated with the construction profession.

PMI Code of Ethics 1.0 Introduction This Code of Professional Conduct is based on the ethical behaviors that PMI members identified as being most important to them in their professional roles. Those key values are: responsibility, respect, fairness and honesty. These values define professionalism in the practice of project management. This Code employs these values to describe the ideal ethical behaviors that we, as practitioners of project management and volunteers in PMI aspire to, both in the professional workplace and in the volunteer environment. The Code also sets forth behaviors that are unacceptable for professionals and unacceptable for PMI volunteers. Compliance with those sections of the Code describing unacceptable behaviors will be enforced by PMI through its Ethics Enforcement Process, as described [citation to be inserted]. 1.1 Purpose of the Code of Professional Conduct The Primary purpose of the Code is to promote a fair business environment for project management. That is an environment in which the guidelines for professional activity are clear, understood, and embraced. The Code also protects the practitioner of project management from pressure to compromise professional or personal standards by establishing a profession-wide understanding of appropriate behavior. Finally, the Code establishes expectations for individuals in their roles as PMI volunteers both when working on global projects, such as standards development, and when fulfilling a leadership role in PMI and its component organizations. 1.2 The Ethical Values that Support this Code Professionalism is one of PMI’s core values as defined in the Strategic Plan. The components of “professionalism” are further explained as “competence, ethical behavior, and honesty.” In member surveys and focus groups the most important ethical behaviors are represented by the following ethical values: • Responsibility – being accountable for our actions both as to developing our competencies as project manager practitioners and in conducting ourselves in the workplace and as volunteers. • Respect – conduct towards others that promotes an environment of cooperation and trust. • Fairness - conduct towards others that shows we make decisions on impartial and objective criteria for the common good and not on personal favoritism or personal selfinterest. • Honesty - speaking the truth and acting in a truthful manner, not withholding information that would mislead others regarding the truth. 2.0 Professionalism in the Workplace. 2.1 The standard will apply to the following people: “Practitioner” which is defined as an individual who is engaged in a professional or management activity that contributes to the execution of a project, portfolio or programs management. The term “project” for purposes of this Code includes a single project, portfolios and programs. The Code applies to the Practitioners who are PMI members (regardless of whether they have obtained a PMI certification or not) and Non-members who hold PMI certification(s). Comment: In the past PMI had separate ethics standards for members and for certificants. Stakeholders who contributed input to develop this code agreed that having several codes

was confusing. Therefore, this Code is applicable to both PMI members and individuals who have been certified by PMI, regardless of their membership in the Institute. 2.2. Professional Conduct in the Workplace 2.2.1 As Practitioners, we are responsible. • We only undertake activities for which we are competent to perform; • We are accountable for the roles to which we have been assigned, ensuring that we understand our authority and the boundaries of our role. • We avoid conducting ourselves in a manner that interferes with or inhibits the legitimate roles and responsibilities of others. • When we are assigned to an activity to which we do not reasonably believe that we are competent to perform or to which we are unclear as to our authority, responsibility or the boundaries, we seek direction from the appropriate stakeholder. Comment: Projects are a team effort. An individual practitioner, who fails to take responsibility for personal conduct on a project, is jeopardizing the success of the entire project. As stated in this section that personal responsibility includes ensuring that the practitioner is competent to perform the assigned project activities. The PMBOK® Guide defines “competency” as “ The skills and capacity required to complete a project activity.” (PMBOK® Guide (Third Edition) at 2.1.3) When a practitioner has been assigned to a task that the individual does not feel qualified or is unsure as to the nature, scope and authority regarding the activity, the Practitioner has a duty to inform the appropriate stakeholder of those questions and seek an appropriate resolution. If the stakeholder is unwilling to provide an adequate answer to those questions, the practitioner should then take those concerns to higher levels of management until the issues are resolved. 2.2.2 As practitioners, we treat other individuals with respect and avoid conduct that is discourteous or abusive and that may jeopardize the success of the project. Likewise, a practitioner does not reciprocate when experiencing disrespectful conduct. Comment: The PMBOK® Guide states that “By understanding the sentiments of project team members, anticipating their actions, acknowledging their concerns and following up on their issues, the project management team can greatly reduce problems and increase cooperation.” (PMBOK® Guide at 9.3.2) In surveying stakeholders in the process of developing this Code, lack of respect was identified as a significant problem. A practitioner who fails to act in a respectful manner towards others, is jeopardizing the success of the project. 2.2.3 As practitioners, we treat individuals with whom we work with fairness: • We provide access to information to all who have a legitimate need for the information, without violating any duty of confidentiality. • We do not abuse our authority by favoring specific individuals based on personal friendship, personal gain or other reasons that are not based on the success of the project. 2.2.4 Practitioners shall be honest and not withhold the truth. Practitioners will not knowingly: • Make false statements regarding project activities for which he has accountability. • Fail to disclose facts that are necessary for the superiors and other stakeholders to know in order to access the true status of the project and the likelihood that the project will be performed in accordance with the project plan.

• Offer any statement of fact that while not false but in the totality of the project is intended to mislead stakeholders as to the true status of the project. Comment: Practitioner must develop trust by accurately providing information concerning the status of the project activities for which he is responsible. Half-truths and non-disclosures intended to mislead stakeholders as to progress of a project are as unprofessional as affirmatively making misrepresentations. 2.2.5 If we become aware that others are engaging in activities which are intended to misrepresent or hide the truth from the customer or other stakeholder, we will report the facts relating to the non-disclosure to appropriate management and, if necessary, to those who are impacted by the conduct. Comment: Customers and other stakeholders must have trust in the integrity of the practitioner. Since the corporate scandals of the early 2000’s, many organizations have, adopted “ Whistleblower” policies to protect employees who reveal the truth about illegal or unethical activities. Governments have also adopted legislation to protect employees who come forward with the truth. In line with these trends, if a practitioner should report any illegal or untruthful conduct to management. If no satisfaction is achieved, the practitioner should ideally to report the conduct to those whom the conduct will adversely impact, such as the customer. 2.2.6 The Practitioner recognizes, avoids and discloses conflicts of interest. Comment: The Practitioner owes a duty of loyalty to the customer and the Practitioner’s employer. The Practitioner also has loyalty to his own family and self. There is nothing wrong with having multiple loyalties. It is unavoidable. However, a potential “conflict of interest” occurs when the Practitioner is responsible for decisions that involved conflicting needs and expectations of those to whom the Practitioner has a duty of loyalty. A Practitioner recognizes these divergent interests and avoids making decisions that would create an actual conflict of interest by making the stakeholders aware of the conflict and cooperating with the stakeholders in finding ways to mitigate the conflict by either abstaining from the decision or advising the client of the potential conflict. 2.3 Prohibited Behaviors in the Workplace. 2.3.1 We will not engage in any illegal behavior, including theft, fraud, corruption, bribery, kickbacks and embezzlement. When there is a conflict between the laws of the country in which the project is being conducted and the laws of the country in which we live, we will obey the laws of country, which sets the highest standard of conduct. 2.3.2 We will not engage in, or allow our subordinates to engage in, abusive behavior towards our co-workers and other persons in the work place. Abusive conduct includes, screaming at co-workers, harassment by words or conduct and sexual harassment. 2.3.3 We will not engage in hiring based on favoritism, payments or other considerations other than the best interest for the success of the project. 2.3.4 Conflict of interest. We will not participate in a project activity when we have a conflict of interest or there is an appearance of a conflict of interest, unless: • Full disclosure of the conflict or apparent conflict has been made to all stakeholders and the stakeholders have in writing agreed to the practitioner continuing in that role; or • A plan has been developed in writing to mitigate any conflict of interest or potential conflict of interest, including the delegation of authority for particular activities to other non-conflicted team members, and that plan has been approved by all stakeholders. 2.3.5 Dishonesty and withholding the truth. When we are aware that our employer is engaging in activities, which are intended to misrepresent or hide the truth from the customer

or ultimate stakeholder, we have an obligation to report the facts relating to the dishonest action to the next level of management, and if no resolution is achieved, to the highest governing body of our employer, such as its Board of Directors. If the activity is illegal, we must comply with the law and make disclosures as the law may require. Comment: While the practitioner should ideally do whatever is necessary to correct illegal or other behavior, at some point the matter rises above the responsibility of the individual to that of the overall organization. Therefore, the practitioner has an obligation to ensure that management is aware of the issue. If necessary, this may mean raising the matter with the top level of management, including the Board of Directors. Moreover, where there is a more stringent duty of disclosure imposed by law other professional requirements. The practitioner should follow these requirements as well. Section 2.2.5, above, states the inspirational goal that the practitioner will advise those impacted by the conduct if no satisfactory resolution is achieved through reporting the conduct to management. 3.0 Professionalism in Volunteer Efforts in the Project Management Organizations 3.1. Applicability of this Standard Section 3 of this Code applies to all “Volunteers” participating in PMI sponsored activities, whether they are members or not. “PMI sponsored activities” includes, but is not limited to, participation on a Member Advisory Group, activities engaged in under the auspices of a chartered PMI component organization (whether it is in a leadership role in the component or another type of component educational activity or event) and working on other volunteer activities such as a standards development team. The term “organization” is used below to include both PMI and its components. 3.2 Professional Conduct as Volunteers 3.2.1. Responsibility. We place the good of the profession above considerations of personal gain. Comment: Individuals join professional organizations for numerous valid reasons such as the betterment of the profession, the creation of tools and knowledge that will benefit all participants in the profession. Valid motivations also include personal considerations, such as networking, career advancement and promotion of the individual’s own work. While those personal motivations are valid, individuals should not allow personal considerations to influence the outcome of volunteer work so that personal gain overrides the good of the profession. 3.2.2 Respect: We show respect for others with whom we are working on volunteer activities. Comment: Respect allows volunteers to work together with effectiveness and enthusiasm. People are valued for their unique contributions. Volunteer work is about giving beyond self. The Volunteer considers other people’s time valuable and trusts other volunteers to deliver on their commitments and provides constructive criticism when it is required. 3.2.3 Fairness: We treat others fairly and do not allow personal favoritism to influence their treatment. Comment: Fairness must govern professional associations—it provides the incentive for greater effort. Rules must be applied equally to everyone. Leadership positions are available to everyone with appropriate skill and experience. Volunteer leaders assign project opportunities based on merit without regard for personal relationships and share the credit for success. Equally, constructive criticism is shared with everyone. 3.2.4 Honesty: We are honest in all volunteer dealings. Comment: Shared honesty in the association builds trust among members and trust of leadership. In turn, trust allows groups of people to achieve results far beyond the best

efforts of untrusting individuals. Honest volunteers give accurate financial reports and truthful assessments of association projects. They disclose conflicts of interest and apparent conflicts of interest. 3.3. Unacceptable behaviors for PMI volunteers 3.3.1. We will follow the rules established by the organization for participation in the Volunteer activity, including by-laws, charters and other governing documents. Comment: It is the duty of every Volunteer to be familiar with the rules by which the Volunteer activity is governed. This includes the affirmative duty to inquire in situations where the Volunteer is unclear of the rules. 3.3.2 We will not engage in conduct that is disrespectful or abusive of the rights of others, including: • Volunteer work is expected to take place in a civil, safe environment. Applying leverage, misusing confidential information, or a threatening posture to sway the direction of a decision is not at all appropriate for a professional organization. • Undermining authority of the leadership by talking behind other’s backs or by spreading false statements or by taking actions calculated to damage the Volunteer activity. • Attacking another Volunteer, PMI staff or other persons engaged in the Volunteer activity with name calling, verbal, emotional, or physical abuse. 3.3.3 Lying and Dishonesty. We will not commit any dishonest acts while engaging in Volunteer activities, including the following: • We are responsible for submitting accurate and timely documentation, such as expense reports, disclosure forms, and conflict of interest forms. Comment: Submitting inappropriate expense reports or expenses without receipts raises questions about the credibility of the person or group submitting the report. Approving such a report is clearly in violation of the rules governing travel. • We will not knowingly, or without reasonable belief, make false ethics accusations against others in the PMI organization. Comment: The issue of “reasonable” belief is an objective standard as to what a person of reasonable mind, acting in good faith, would consider to be a possible ethics violation. The term “possible” is used, because an individual does not have to know with absolute certainty that there was a violation, only that upon a plain reading of the text of the Code, there is a reasonable question of whether an ethics violation has occurred. It is for the PMI ethics processes to determine whether the conduct actually constitutes a violation. • We will not fail to disclose to the organization the full truth about our relationships with vendors or suppliers. Comment: There is nothing wrong with an appropriate relationship with a vendor as long as that relationship does not damage PMI, the component or its members with exaggerated pricing and incomplete or low-quality services. Hiding such a relationship is not appropriate regardless of the quality of service, because the organization has the right to know that such relationships exist and to consider that information when selecting vendors. The failure to disclose may also imply that harm is being done and that the relationship is inappropriate. Such conduct may also constitute a conflict of interest. • Falsely accusing leaders to remove them from leadership. 3.3.4 We will not take the property of the association and use it for the Volunteer’s own personal benefit, including:

• Taking a business opportunity that was made available to the organization and that we discovered in our position as a Volunteer and using that business opportunity for our own self-interest. Comment: Individuals may come upon a business opportunity that was presented to the organization, but which the volunteer sees as a possibility for the volunteer’s own business advancement. If the volunteer became aware of the business activity independently of the individual’s volunteer activities, then the volunteer may pursue it. If the activity was one that came to the individual as an opportunity for the organization, then the individual cannot take the opportunity from the organization. • Using the organization’s intellectual property, such as books, reports, trademarks and other works commissioned and owned by the organization without permission and attribution. Comment: The organization creates intellectual property by the work of its members for the benefit of its members and for the advancement of the profession. Such intellectual property has value that results from its content and the organization’s name. Individuals who attempt to use intellectual property for their own benefit are not only doing so at the expense of the members, but also threaten the reputation of the Institute and confuse the public as to the source of the goods and services. Individuals have a duty to check beforehand to understand their rights before reprinting and selling material they created but sold to PMI. Such misappropriation includes, for example, setting up a website that has the name of a PMI component in the URL, and using educational materials that were developed by a component without getting the component’s permission and without giving attribution to the source. 3.3.5 Conflict of interest. We will not participate in a Volunteer activity when we have a conflict of interest or there is an appearance of a conflict of interest, unless: • Full disclosure of the conflict or apparent conflict has been made to the organization and the organization has agreed in writing to our volunteer continuing in that role; • A plan has been developed in writing to remove potential negative impact of the conflict of interest or potential conflict of interest, including the delegation of authority for particular activities to other non-conflicted Volunteers, and that plan has been approved by the organization. Comment: Whenever a person is in a conflict of interest or an apparent conflict of interest, the individual has a duty to inform the organization of the conflict and allow the organization, free of any influence by the individual, to determine what remedial action is needed to ensure that the conflict does not adversely affect the organization. The remedy may be to resign from office where the conflict is pervasive or to withdraw from the particular decision being made, where the conflict is narrow and relates only to one issue. 3.3.6 Unfair Behavior. As Volunteers we will not engage in any unfair behaviors towards others, including: • Exhibiting favoritism towards any person or group to the detriment of others based on personal friendship or other personal interest. Comment: Volunteers, including component leaders and others who are entrusted with authority by the organization are required to act fairly when making work assignments, approving travel, and offering opportunities for advancement. It is important to take into account the skills, enthusiasm, and years of service when making choices for volunteer opportunities and carefully balancing people’s abilities and needs.

• Use a position of leadership in a volunteer activity to promote one’s own company or services. Comment: Such conduct is both a conflict of interest and an abuse of authority. Promoting self-interest in a volunteer role damages the image and the professional credibility of the organization. Decisions made by leaders who are looking for personal gain are adverse to the needs of the organization and the profession. • Discrimination based on gender, race, religion, nationality, or sexual preference. Comment: The PMI environment at its best is a cooperative organization. A PMI member or volunteer must not deny opportunities to others for advancement or travel based on gender, race, religion, nationality, or sexual preference. Volunteers must not withhold information or work around fellow volunteers who exhibit these differences. • Failure to apply the rules of the organization fairly and equally to all for whom the rules were intended to apply. Comment: Allowing some to bend the rules is an elementary leadership mistake. If the rules were not applied to all for whom they were intended, members would become cynical about the quality of leadership, and the organization’s governance processes. Leaders must hold themselves to higher standards of behavior, applying their own rules justly and evenly among themselves or face the possibility of losing the moral support of the Volunteers and the larger society. • Engaging in the selection of vendors for the organization where preferences are given to a particular vendor based on consideration other than the best interest of the organization. Comment: Giving preferential treatment to some vendors by hiring them without a bidding process or by using a favorite vendor exclusively when there are other vendors as competent, effective, and efficient is short-sighted for the long-term good of the organization. If the favorite vendor provides lower quality or higher prices compared to his competition, then PMI’s resources are being abused in the short term and the long term. This could also be considered a conflict of interest, especially when the favorite vendor is related to the buyer or is a friend, or if the buyer and the vendor have an off-the-books arrangement for returning some of the fee to the buyer. 4.0 Professionalism in the Certification Process. 4.1 Applicability of this Section. This Section 4 is applicable to all persons who apply to commence the certification process with PMI, by their agreement in their application to abide by the Code (‘Applicants”) and to all individuals who have received and currently maintain a certification from PMI (“Certificants”). 4.2 Prohibited Conduct Related to Certification. 4.2.1 Certificants and applicants shall not disclose the content of the certification examination to anyone. Comment: For purposes of this section, the term “content” includes the exact questions on the examination or any paraphrase of the question, answers to a question, including both the substance of the answer or the lettered or numbered identifier used on the examination to identify the answer or any other information directly related to the content of an examination, that would assist a person taking the examination to get a better score.

4.2.2 Certificants and applicants shall disclose to the Institute any violation they encounter of other certificants disclosing examination content to third parties. 4.2.3 Certificants and applicants shall disclose to the Institute any attempt by educational providers to obtain from them the content of the examination. 4.2.4 Certificants and applicants will be honest and accurate in providing information regarding their experience in project management, including, but not limited to answering questions on application forms for the certification. Comment: The public’s trust in individuals holding a certification and in the validity of the certification is undermined when individuals who do not have the appropriate experience or knowledge obtain the certification by other means. It is in the best interest of all certificants, as well as the public that certificants have achieved the certification on individual knowledge and experience. 4.2.5 Certificants will accurately report all Professional Development Units (PDUs) and will report to PMI any instance where a provider of educational services is offering PDUs that do not accurately reflect the hours required to actually complete the work.

ALPA Code of Ethics Code of Ethics An Air Line Pilot will keep uppermost in his mind that the safety, comfort, and wellbeing of the passengers who entrust their lives to him are his first and greatest responsibility. • He will never permit external pressures or personal desires to influence his judgment, nor will he knowingly do anything that could jeopardize flight safety. • He will remember that an act of omission can be as hazardous as a deliberate act of commission, and he will not neglect any detail that contributes to the safety of his flight, or perform any operation in a negligent or careless manner. • Consistent with flight safety, he will at all times operate his aircraft in a manner that will contribute to the comfort, peace of mind, and well-being of his passengers, instilling in them trust in him and the airline he represents. • Once he has discharged his primary responsibility for the safety and comfort of his passengers, he will remember that they depend upon him to do all possible to deliver them to their destination at the scheduled time. • If disaster should strike, he will take whatever action he deems necessary to protect the lives of his passengers and crew. An Air Line Pilot will faithfully discharge the duty he owes the airline that employs him and whose salary makes possible his way of life. • He will do all within his powers to operate his aircraft efficiently and on schedule in a manner that will not cause damage or unnecessary maintenance. • He will respect the officers, directors, and supervisors of his airline, remembering that respect does not entail subservience. • He will faithfully obey all lawful directives given by his supervisors, but will insist and, if necessary, refuse to obey any directives that, in his considered judgment, are not lawful or will adversely affect flight safety. He will remember that in the final analysis the responsibility for safe completion of the flight rests upon his shoulders. • He will not knowingly falsify any log or record, nor will he condone such action by other

crew members. • He will remember that a full month’s salary demands a full and fair month’s work. On his days off, he will not engage in any occupation or activity that will diminish his efficiency or bring discredit to his profession. • He will realize that he represents the airline to all who meet him and will at all times keep his personal appearance and conduct above reproach. • He will give his airline, its officers, directors, and supervisors the full loyalty that is their due, and will refrain from speaking ill of them. If he feels it necessary to reveal and correct conditions that are not conducive to safe operations and harmonious relations, he will direct his criticism to the proper authorities within ALPA. • He will hold his airline’s business secrets in confidence, and will take care that they are not improperly revealed. An Air Line Pilot will accept the responsibilities as well as the rewards of command and will at all times so conduct himself both on duty and off as to instill and merit the confidence and respect of his crew, his fellow employees, and his associates within the profession. • He will know and understand the duties of each member of his crew. If in command, he will be firm but fair, explicit yet tolerant of deviations that do not affect the safe and orderly completion of the flight. He will be efficient yet relaxed, so that the duties of the crew may be carried out in a harmonious manner. • If in command, he will expect efficient performance of each crew member’s duties, yet he will overlook small discrepancies and refrain from unnecessary and destructive criticism, so that the crew member will retain his self-respect and cooperative attitude. A frank discussion of minor matters of technique and performance after the flight will create goodwill and a desire to be helpful, whereas sharp criticism and peremptory orders at the moment will result only in the breakdown of morale and an inefficient, halting performance of future duties. • An Air Line Pilot will remember that his is a profession heavily dependent on training during regular operations and, if in command, will afford his flight crew members every reasonable opportunity, consistent with safety and efficiency, to learn and practice. He will endeavor to instill in his crew a sense of pride and responsibility. In making reports on the work and conduct of his crew members, he will avoid personal prejudices, make his reports factual and his criticisms constructive so that actions taken as a result of his reports will improve the knowledge and skill of his crew members, rather than bring discredit, endanger their livelihood, and threaten their standing in the profession. • While in command, the Air Line Pilot will be mindful of the welfare of his crew. He will see to it that his crew are properly lodged and cared for, particularly during unusual operating conditions. When cancellations result in deadheading, he will ensure that proper arrangements are made for the transportation of his crew before he takes care of himself. An Air Line Pilot will conduct his affairs with other members of the profession and with ALPA in such a manner as to bring credit to the profession and ALPA as well as to himself. • He will not falsely or maliciously injure the professional reputation, prospects, or job security of another pilot, yet if he knows of professional incompetence or conduct detrimental to the profession or to ALPA, he will not shrink from revealing this to the proper authorities within ALPA, so that the weak member may be brought up to the standards demanded, or ALPA and the profession alike may be rid of one unworthy to share its rewards. • He will conduct his affairs with ALPA and its members in accordance with the rules laid down in the Constitution and By-Laws of ALPA and with the policies and interpretations

promulgated there from. Whenever possible, he will attend all meetings of ALPA open to him and will take an active part in its activities and in meetings of other groups calculated to improve air safety and the standing of the profession. • An Air Line Pilot shall refrain from any action whereby, for his personal benefit or gain, he take advantage of the confidence reposed in him by his fellow members. If he is called upon to represent ALPA in any dispute, he will do so to the best of his ability, fairly and fearlessly, relying on the influence and power of ALPA to protect him. • He will regard himself as a debtor to his profession and ALPA, and will dedicate himself to their advancement. He will cooperate in the upholding of the profession by exchanging information and experience with his fellow pilots and by actively contributing to the work of professional groups and the technical press. An Air Line Pilot the honor of his profession is dear, and he will remember that his own character and conduct reflect honor or dishonor upon the profession. • He will be a good citizen of his country, state, and community, taking an active part in their affairs, especially those dealing with the improvement of aviation facilities and the enhancement of air safety. • He will conduct all his affairs in a manner that reflects credit on himself and his profession. • He will remember that to his neighbors, friends, and acquaintances he represents both the profession and ALPA, and that his actions represent to them the conduct and character of all members of the profession and ALPA. • He will realize that nothing more certainly fosters prejudices against and deprives the profession of its high public esteem and confidence than do breaches in the use of alcohol. • He will not publish articles, give interviews, or permit his name to be used in any manner likely to bring discredit to another pilot, the airline industry, the profession, or ALPA. • He will continue to keep abreast of aviation developments so that his skill and judgment, which heavily depend on such knowledge, may be of the highest order. Having Endeavored to his utmost to faithfully fulfill the obligations of the ALPA Code of Ethics and Canons for the Guidance of Air Line Pilots, a pilot may consider himself worthy to be called…an AIRLINE PILOT.

AIA Code of Ethics 2004 Code of Ethics & Professional Conduct Preamble Members of The American Institute of Architects are dedicated to the highest standards of professionalism, integrity, and competence. This Code of Ethics and Professional Conduct states guidelines for the conduct of Members in fulfilling those obligations. The Code is arranged in three tiers of statements:

Canons, Ethical Standards, and Rules of Conduct: � Canons are broad principles of conduct. � Ethical Standards (E.S.) are more specific goals toward which Members should aspire in professional performance and behavior. � Rules of Conduct (Rule) are mandatory; violation of a Rule is grounds for disciplinary action by the Institute. Rules of Conduct, in some instances, implement more than one Canon or Ethical Standard.

The Code applies to the professional activities of all classes of Members, wherever they occur. It addresses responsibilities to the public, which the profession serves and enriches; to the clients and users of architecture and in the building industries, who help to shape the built environment; and to the art and science of architecture, that continuum of knowledge and creation which is the heritage and legacy of the profession. Commentary is provided for some of the Rules of Conduct. That commentary is meant to clarify or elaborate the intent of the rule. The commentary is not part of the Code. Enforcement will be determined by application of the Rules of Conduct alone; the commentary will assist those seeking to conform their conduct to the Code and those charged with its enforcement. Statement in Compliance With Antitrust Law The following practices are not, in themselves, unethical, unprofessional, or contrary to any policy of The American Institute of Architects or any of its components: (1) submitting, at any time, competitive bids or price quotations, including in circumstances where price is the sole or principal consideration in the selection of an architect; (2) providing discounts; or (3) providing free services. Individual architects or architecture firms, acting alone and not on behalf of the Institute or any of its components, are free to decide for themselves whether or not to engage in any of these practices. Antitrust law permits the Institute, its components, or Members to advocate legislative or other government policies or actions relating to these practices. Finally, architects should continue to

consult with state laws or regulations governing the practice of architecture. CANON I General Obligations Members should maintain and advance their knowledge of the art and science of architecture, respect the body of architectural accomplishment, contribute to its growth, thoughtfully consider the social and environmental impact of their professional activities, and exercise learned and uncompromised professional judgment. E.S. 1.1 Knowledge and Skill: Members should strive to improve their professional knowledge and skill. Rule In practicing architecture, 1.101 Members shall demonstrate a consistent pattern of reasonable care and competence, and shall apply the technical knowledge and skill which is ordinarily applied by architects of good standing practicing in the same locality. Commentary: By requiring a "consistent pattern" of adherence to the common law standard of competence, this rule allows for discipline of a Member who more than infrequently does not achieve that standard. Isolated instances of minor lapses would not provide the basis for discipline. E.S. 1.2 Standards of Excellence: Members should continually seek to raise the standards of aesthetic excellence, architectural education, research, training, and practice. E.S. 1.3 Natural and Cultural Heritage: Members should respect and help conserve their natural and cultural heritage while striving to improve the environment and the quality of life within it. E.S. 1.4 Human Rights: Members should uphold human rights in all their professional endeavors. Rule Members shall not discriminate in 1.401 their professional activities on the basis of race, religion, gender, national origin, age, disability, or sexual orientation. 1

2004CODEOFETHICSANDP ROFESSIONALCONDUCT E.S. 1.5 Allied Arts & Industries: Members should promote allied arts and contribute to the knowledge and capability of the building industries as a whole. CANON II Obligations to the Public Members should embrace the spirit and letter of the law governing their professional affairs and should promote and serve the public interest in their personal and professional activities. E.S. 2.1 Conduct: Members should uphold the law in the conduct of their professional activities. Rule Members shall not, in the conduct 2.101 of their professional practice, knowingly violate the law. Commentary: The violation of any law, local, state or federal, occurring in the conduct of a Member's professional practice, is made the basis for discipline by this rule. This includes the federal Copyright Act, which prohibits copying architectural works without the permission of the copyright owner: Allegations of violations of this rule must be based on an independent finding of a violation of the law by a court of competent jurisdiction or an administrative or regulatory body. Rule Members shall neither offer nor 2.102 make any payment or gift to a public official with the intent of influencing the official's judgment in connection with an existing or prospective project in which the Members are interested. Commentary: This rule does not prohibit campaign contributions made in conformity with applicable campaign financing laws. Rule Members serving in a public 2.103 capacity shall not accept payments or gifts which are intended to influence their judgment. Rule Members shall not engage in 2.104 conduct involving fraud or wanton disregard of the rights of others. Commentary: This rule addresses serious misconduct whether or not related to a

Member's professional practice. When an alleged violation of this rule is based on a violation of a law, or of fraud, then its proof must be based on an independent finding of a violation of the law or a finding of fraud by a court of competent jurisdiction or an administrative or regulatory body. Rule If, in the course of their work 2.105 on a project, the Members become aware of a decision taken by their employer or client which violates any law or regulation and which will, in the Members’ judgment, materially affect adversely the safety to the public of the finished project, the Members shall: (a) advise their employer or client against the decision, (b) refuse to consent to the decision, and (c) report the decision to the local building inspector or other public official charged with the enforcement of the applicable laws and regulations, unless the Members are able to cause the matter to be satisfactorily resolved by other means. Commentary: This rule extends only to violations of the building laws that threaten the public safety. The obligation under this rule applies only to the safety of the finished project, an obligation coextensive with the usual undertaking of an architect. Rule Members shall not counsel or 2.106 assist a client in conduct that the architect knows, or reasonably should know, is fraudulent or illegal. E.S. 2.2 Public Interest Services: Members should render public interest professional services and encourage their employees to render such services. E.S. 2.3 Civic Responsibility: Members should be involved in civic activities as citizens and professionals, and should strive to improve public appreciation and understanding of architecture and the functions and responsibilities of architects.

Rule Members making public 2.301 statements on architectural issues shall disclose when they are being compensated for making such statements or when they have an economic interest in the issue. CANON III Obligations to the Client Members should serve their clients competently and in a professional manner, and should exercise unprejudiced and unbiased judgment when performing all professional services. E.S. 3.1 Competence: Members should serve their clients in a timely and competent manner. Rule In performing professional 3.101 services, Members shall take into account applicable laws and regulations. Members may rely on the advice of other qualified persons as to the intent and meaning of such regulations. Rule Members shall undertake to 3.102 perform professional services only when they, together with those whom they may engage as consultants, are qualified by education, training, or experience in the specific technical areas involved. Commentary: This rule is meant to ensure that Members not undertake projects that are beyond their professional capacity. Members venturing into areas that require expertise they do not possess may obtain that expertise by additional education, training, or through the retention of consultants with the necessary expertise. Rule Members shall not materially 3.103 alter the scope or objectives of a project without the client's consent. E.S. 3.2 Conflict of Interest: Members should avoid conflicts of interest in their professional practices and fully disclose all unavoidable conflicts as they arise. 2 2004CODEOFETHICSANDP ROFESSIONALCONDUCT Rule A Member shall not render 3.201 professional services if the Member's professional judgment

could be affected by responsibilities to another project or person, or by the Member's own interests, unless all those who rely on the Member's judgment consent after full disclosure. Commentary: This rule is intended to embrace the full range of situations that may present a Member with a conflict between his interests or responsibilities and the interest of others. Those who are entitled to disclosure may include a client, owner, employer, contractor, or others who rely on or are affected by the Member's professional decisions. A Member who cannot appropriately communicate about a conflict directly with an affected person must take steps to ensure that disclosure is made by other means. Rule When acting by agreement of the 3.202 parties as the independent interpreter of building contract documents and the judge of contract performance, Members shall render decisions impartially. Commentary: This rule applies when the Member, though paid by the owner and owing the owner loyalty, is nonetheless required to act with impartiality in fulfilling the architect’s professional responsibilities. E.S. 3.3 Candor and Truthfulness: Members should be candid and truthful in their professional communications and keep their clients reasonably informed about the clients’ projects. Rule Members shall not intentionally 3.301 or recklessly mislead existing or prospective clients about the results that can be achieved through the use of the Members’ services, nor shall the Members state that they can achieve results by means that violate applicable law or this Code. Commentary: This rule is meant to preclude dishonest, reckless, or illegal representations by a Member either in the course of soliciting a client or during performance. E.S. 3.4 Confidentiality: Members should safeguard the trust placed in them

by their clients. Rule Members shall not knowingly 3.401 disclose information that would adversely affect their client or that they have been asked to maintain in confidence, except as other wise allowed or required by this Code or applicable law. Commentary: To encourage the full and open exchange of information necessary for a successful professional relationship, Members must recognize and respect the sensitive nature of confidential client communications. Because the law does not recognize an architect-client privilege, however, the rule permits a Member to reveal a confidence when a failure to do so would be unlawful or contrary to another ethical duty imposed by this Code. CANON IV Obligations to the Profession Members should uphold the integrity and dignity of the profession. E.S. 4.1 Honesty and Fairness: Members should pursue their professional activities with honesty and fairness. Rule Members having substantial 4.101 information which leads to a reasonable belief that another Member has committed a violation of this Code which raises a serious question as to that Member’s honesty, trustworthiness, or fitness as a Member, shall file a complaint with the National Ethics Council. Commentary: Often, only an architect can recognize that the behavior of another architect poses a serious question as to that other's professional integrity. In those circumstances, the duty to the professional's calling requires that a complaint be filed. In most jurisdictions, a complaint that invokes professional standards is protected from a libel or slander action if the complaint was made in good faith. If in doubt, a Member should seek counsel before reporting on another under this rule. Rule Members shall not sign or seal 4.102 drawings, specifications, reports, or other professional work for

which they do not have responsible control. Commentary: Responsible control means the degree of knowledge and supervision ordinarily required by the professional standard of care. With respect to the work of licensed consultants, Members may sign or seal such work if they have reviewed it, coordinated its preparation, or intend to be responsible for its adequacy. Rule Members speaking in their 4.103 professional capacity shall not knowingly make false statements of material fact. Commentary: This rule applies to statements in all professional contexts, including applications for licensure and AIA membership. E.S. 4.2 Dignity and Integrity: Members should strive, through their actions, to promote the dignity and integrity of the profession, and to ensure that their representatives and employees conform their conduct to this Code. Rule Members shall not make 4.201 misleading, deceptive, or false statements or claims about their professional qualifications, experience, or performance and shall accurately state the scope and nature of their responsibilities in connection with work for which they are claiming credit. Commentary: This rule is meant to prevent Members from claiming or implying credit for work which they did not do, misleading others, and denying other participants in a project their proper share of credit. Rule Members shall make reasonable 4.202 efforts to ensure that those over whom they have supervisory authority conform their conduct to this Code. Commentary: What constitutes "reasonable efforts" under this rule is a common sense matter. As it makes sense to ensure that those over whom the architect exercises supervision be made generally aware of the Code, it can also make sense

to bring a particular provision to the attention of a particular employee when a situation is present which might give rise to violation. 3 2004CODEOFETHICSANDP ROFESSIONALCONDUCT CANON V Obligations to Colleagues Members should respect the rights and acknowledge the professional aspirations and contributions of their colleagues. E.S. 5.1 Professional Environment: Members should provide their associates and employees with a suitable working environment, compensate them fairly, and facilitate their professional development. E.S. 5.2 Intern and Professional Development: Members should recognize and fulfill their obligation to nurture fellow professionals as they progress through all stages of their career, beginning with professional education in the academy, progressing through internship and continuing throughout their career. E.S. 5.3 Professional Recognition: Members should build their professional reputation on the merits of their own service and performance and should recognize and give credit to others for the professional work they have performed. Rule Members shall recognize and 5.301 respect the professional contributions of their employees, employers, professional colleagues, and business associates. Rule Members leaving a firm shall not, 5.302 without the permission of their employer or partner, take designs, drawings, data, reports, notes, or other materials relating to the firm’s work, whether or not performed by the Member. Rule A Member shall not unreasonably 5.303 withhold permission from a departing employee or partner to

take copies of designs, drawings, data, reports, notes, or other materials relating to work performed by the employee or partner that are not confidential. Commentary: A Member may impose reasonable conditions, such as the payment of copying costs, on the right of departing persons to take copies of their work. RULES OF APPLICATION, ENFORCEMENT, AND AMENDMENT Application The Code of Ethics and Professional Conduct applies to the professional activities of all members of the AIA. Enforcement The Bylaws of the Institute state procedures for the enforcement of the Code of Ethics and Professional Conduct. Such procedures provide that: (1) Enforcement of the Code is administered through a National Ethics Council, appointed by the AIA Board of Directors. (2) Formal charges are filed directly with the National Ethics Council by Members, components, or anyone directly aggrieved by the conduct of the Members. (3) Penalties that may be imposed by the National Ethics Council are: (a) Admonition (b) Censure (c) Suspension of membership for a period of time (d) Termination of membership (4) Appeal procedures are available. (5) All proceedings are confidential, as is the imposition of an admonishment; however, all other penalties shall be made public. Enforcement of Rules 4.101 and 4.202 refer to and support enforcement of other Rules. A violation of Rules 4.101 or 4.202 cannot be established without proof of a pertinent violation of at least one other Rule. Amendment The Code of Ethics and Professional Conduct may be amended by the convention of the Institute under the same

procedures as are necessary to amend the Institute's Bylaws. The Code may also be amended by the AIA Board of Directors upon a two-thirds vote of the entire Board.

*2004 Edition. This copy of the Code of Ethics is current as of September 2004. Contact the General Counsel's Office for further information at (202) 626-7311. 4

1 ASCE Zero Tolerance Policy http://www.asce.org/professional/zero.cfm

(Raw Results)

Professionalization Continuum- Question 30-31, 39-42 & 47 Compared Appendix C Thurstone's Law of Comparative Judgement

Thurstone’s Law of Comparative Judgment as applied to Questions 39-42

Professionalization Continuum- Question 30-31, 39-42 & 47 Compared

Professionalization Continuum- Question 39-42 Rank Ordered

Unbiased Ranking of PM’s Per Thurstone’s Law of Comparative Judgment

Professionalization Continuum- Question 30-31, 39-42 & 47 Compared